You are on page 1of 213

Study the notes and cases in the text book (“Land Titles, Deeds, Liens and Dealings”) from

pages 1
to 82.

Read and digest/dissect the following cases:

1. Quimson v. Suarez, GR 21381, April 5, 1924


2. Revilla v. Galindez, GR L-9940, March 30, 1960
3. Doronio v. Doronio, GR 169454, December 27, 2007
4. Castillo v. Excutin, GR 171056, March 13, 2009
5. Legarda v. Saleeby, GR L-8936, October 2, 1915
6. Rufloe v. Burgos, GR 143573, January 30, 2009
7. Cariño v. Insular Government, GR 2746, Dec. 6, 1906
8. Cruz v. Secretary of DENR, GR 135385, Dec. 6, 2000
9. Malabanan v. Republic, GR 179987, April 29, 2009
10. Amunategui v. Director of Forestry, GR L-27873, Nov. 29, 1983
11. Republic v. IAC, GR L-75042, November 29, 1988
12. Santulan v. Executive Secretary, GR L-28021, Dec. 15, 1977
13. Chavez v. PEA, GR 133250, July 9, 2002
14. Government of the Phil. Islands v. Abural, GR 14167, Aug. 14, 1919
15. Morandarte v. CA, GR 123586, Aug. 12, 2004

Write your case digests on yellow pad (one case per sheet), append a cover sheet with your name,
section and table of contents, and place them neatly in a sliding folder.

G.R. No. L-21381 April 5, 1924

SANTIAGO QUIMSON, plaintiff-appellee,


vs.
PABLO SUAREZ, defendant-appellant.

Camus and Delgado and Jose Serapio for appellant.


Jose Bernabe for appellee.

OSTRAND, J.:

This is an action of forcible entry and detainer, the plaintiff alleging that he is entitled to the
possession of the land in question under a lease from its owner and that the defendant is wrongfully
withholding it from him. The defendant maintains that he is lawfully in possession under another lease
of an earlier date from the same owner.

It appears from the evidence that on February 28, 1917, the deceased Pablo Tecson leased a large
fishpond situated in the municipality of Orani, Province of Bataan, to one David Luna for the term of
ten years at an annual rent of P900. Two months later Luna, with the consent of Tecson, assigned
the lease to Pablo Suarez, the defendant herein. Shortly afterwards Tecson applied for the
registration of the land under Act No. 496, but died before the final disposal of the case and the final
decree and corresponding certificate of title were issued in the names of Esperanza Tongco de Trias,
as administratrix of Tecson's estate, and Maximiana Tongco, his widow, in equal shares. Neither in
the final decree nor in the certificate of title was any mention made of the lease in favor of Suarez.

Esperanza Tongco de Trias appears to have been succeeded as administratrix by Maximiana Tongco
and the latter as such administratrix, and as guardian of the heirs of Tecson, and also in her own
behalf, granted a lease of the land to the plaintiff Quimson. The lease is dated May 23, 1920, and is
for the term of six years from May 1, 1921, the rent for the entire term being fixed at P6,250, payable
at the time of the execution of the lease. This lease was at once entered by memorandum upon the
certificate of title for the land. At the time of the execution of the lease the plaintiff knew that the
defendant Suarez was in possession of the land as a tenant of Maximiana Tongco, by whom he had
been told that the term of Suarez' lease expired on March 1, 1921, and there is no evidence showing
that he had notice of the fact that the term extended beyond that date.

On November 15, 1920, Suarez brought an action in the Court of First Instance against Quimson and
Maximiana Tongco to have Quimson's lease set aside. As far as the record shows the action may still
be pending.

On May 1, 1921, Quimson made a formal demand upon Suarez for the surrender of the possession
and the demand not being complied with, the present action was brought in the court of the justice of
the peace of Orani, the complaint being filed May 6, 1921. The defendant objected formally to the
jurisdiction of the court on the ground that the action was in reality not one of forcible entry and
unlawful detainer, and therefore not within the jurisdiction of the justice of the peace. The court
overruled the objection, took cognizance of the case, and rendered judgment in favor of the plaintiff
ordering the defendant to vacate the land and to pay damages in the sum of P280.

The defendant appealed to the Court of First Instance where he appeared specially and presented a
motion for the dismissal of the complaint on the same grounds as those urged in support of his
objection to the jurisdiction of the justice of the peace. The motion was denied.

The case was placed on the calendar for June 22, 1922, but was continued on motion of the
defendant. It was again set for trial on August 11, 1922, and plaintiff appeared accompanied by his
counsel; the defendant also appeared and presented another motion for a continuance on the ground
that his counsel had a case for trial in another court and was unable to appear in the present case on
that date. The court denied the motion and proceeded with the trial, at which trial the defendant
testified in his own behalf. The court thereupon rendered judgment in favor of the plaintiff for the
possession of the land and the sum of P590 in damages, with costs, and the case is now before us
upon appeal by the defendant from that judgment.

The appellant presents three assignments of error, viz.:

(a) The trial court erred in overstepping its discretionary powers by denying the petition for
postponement of the trial filed by the defendant on August 9, 1922.

(b) The trial court erred in overruling the motion for dismissal presented by the defendant on
the ground that the Court of First Instance of Bataan had no jurisdiction to take cognizance of
this case on appeal.

(c) The trial court erred in holding that the right of possession of the defendant to the land
described in paragraph 1 of the complaint was extinguished on April 30, 1921; and in
adjudging that the contract of lease, Exhibit 4 of the defendant, is null and void for not having
been noted on the corresponding certificate of title.

(1) There is no merit in the first assignment of error. The defendant had already been granted
a continuance of the case and failed to give plaintiff's counsel due notice of the second motion
for a continuance, thus causing the latter the expense and trouble of going from Manila to
Balanga. Under the circumstances, the denial of the second motion certainly did not constitute
abuse of discretion.

(2) Neither can the second assignment of error be sustained. The defendant argues that the
action is not one of forcible entry and detainer as defined in section 80 of the Code of Civil
Procedure. The pertinent part of that section reads as follows:

Anyone deprived of the possession of any land or building by force, intimidation, threat, strategy, or
stealth, and any landlord, vendor, vendee, or other person against whom the possession of any land
or building is unlawfully withheld after the expiration or determination of the right to hold possession,
by virtue of any contract, express or implied, and the legal representatives or assigns of any such
landlord, vendor, vendee, or other person, shall at any time within one year after such unlawful
deprivation or withholding of possession be entitled, as against the person or persons unlawfully
withholding or depriving of possession, or against any person or persons claiming under them, to
restitution of the land, building, and premises possession of which is unlawfully withheld, together with
damages and costs. . . .

The complaint alleges that the plaintiff has been entitled to the possession of the land since May,
1921; that the defendant's right of possession expired April 30, 1921; and that the defendant is
illegally withholding the possession from the plaintiff. These allegations bring the case squarely within
the section quoted and, if so, the justice of the peace had original jurisdiction. The authorities cited by
the defendant in support of his contention are so clearly inapplicable to this case that a discussion of
them seems unnecessary.

(3) The third assignment of error relates to the principal question in the case, i.e., the
determination of the legal effect of the two leases and of their relative priority. As we have
already stated, the land in question is registered under Act No. 496. The plaintiff's lease is duly
entered upon the certificate of title; the defendant's lease, though prior in date, has never been
registered, nor is any intimation of its existence to be found upon the certificate of title. At the
time the plaintiff entered into his contract of lease, he had knowledge of the fact that the
defendant was in physical possession of the land and if the land were unregistered this would
be sufficient to put him upon inquiry and charge him with constructive notice of the defendant's
rights.

But here the land is registered and in regard to such lands as unrecorded lease operates only as a
contract between the parties and does not affect the rights of third parties in the absence of fraud on
their part. Inasmuch as sales, mortgages and leases stand on the same footing in this respect, the
following quotation from Niblack on the Torrens System, p. 222, citing Independent Lumber Co. vs.
Gardiner (3 Sask., 140), is in point:

The rule relative to the protection which will be afforded by a new registration is that a
purchaser for value will be protected in his registered interests unless actual and moral fraud
on his part is to be inferred from the circumstances under which he obtained them. A person
taking a mortgage from a registered owner of land is not affected by notice of an unregistered
interest in another person, whereby such person is the owner of an undivided one-half of the
land, but if the mortgagor, when he executed it, expressly told the mortgagee that he owned
only an undivided one-half of the land, that he only intended to mortgage his one-half, and that
he intended to exempt the interest of his cotenant, the mortgagee is guilty of fraud against the
unregistered owner in attempting to enforce the mortgage against the whole land. Fraud on the
part of a vendor in acquiring his title cannot affect the statutory protection and indefeasibility of
title given to a registered purchaser for value, who had no part in or knowledge of the fraud.
This rule is the same as in case of original registration.
One of the principal features of the Torrens System of registration is that all incumbrances on the land
or special estates therein shall be shown, or, at least, intimated upon the certificate of title and a
person dealing with the owner of the registered land is not bound to go behind the certificate and
inquire into transactions, the existence of which is not there intimated. There being in the present
case not indication on the certificate of title of the existence of the defendant's lease, and the
certificate therefore showing a clear title and right of possession in favor of the lessor, the plaintiff had
a perfect right to rely on the lessor's statement that defendant's right of possession terminated on
April 30, 1921, and was not bound to make further inquiries. He can, therefore, not be charged with
fraud neither actual nor constructive.

The disadvantages of adopting the rule suggested by the appellant would far outweigh the
advantages, would be out of harmony with the underlying principles of the Torrens System of
registration and would tend to impair the value of registered titles.

The judgment appealed from is therefore affirmed, with the costs against the appellant. So ordered.

Araullo, C.J., Johnson, Avanceña and Romualdez, JJ., concur.


Street, J., dissents.

Separate Opinions

JOHNS, J., dissenting:

I dissent.

At the time the plaintiff took his lease, he personally knew that the defendant was in the actual
physical possession of the property, and had been for a number of years. The fact that the plaintiff
had personal knowledge of such fact was sufficient to put him upon notice of defendant's rights, and
to legally charge him with knowledge of any information, which he might received upon inquiry from
the defendant as to the tenure of his lease.

The only object of the registration law is to give a party notice of the rights of third parties, and if he
has personal knowledge of such rights, or such notice as to put him upon inquiry, as in this case, he
is not in a position to say that he took the lease without knowledge of defendant's rights.

Upon that ground, I dissent.

G.R. No. L-9940 March 30, 1960

AVELINO REVILLA and ELENA FAJARDO, plaintiffs-appellants,


vs.
GODOFREDO GALINDEZ, defendant-appellee.

Vergara and Associates for appellants.


Lauro O. Sansano for appellee.
GUTIERREZ DAVID, J.:

In the complaint they filed in the Court of First Instance of Nueva Ecija, Avelino Revilla, and Elena
Fajardo are seeking to recover the possession of a parcel of land, now known as Lot No. 659-A,
subdivision plan Psd-28954, but which used to be the southwestern portion of Lot No. 659 of the
cadastral survey of Rizal, Nueva Ecija. This latter lot was formerly registered in the name of Alipio
Gasmeña as per T.C.T. No. 7454 (Exhibit 4). On May 18, 1938, Alipio Gasmeña donated to Florencio
Gasmeña said southwestern portion of the lot, with an area of 1.8144 hectares (Exhibit 5). The
donation was duly annotated on the certificates of title. On May 21, 1938, Florencio Gasmeña
mortgaged his unsegregated portion to Godofredo Galindez, defendant-appellee herein, for the sum
of P350.00 (Exhibit 8); and on October 5, 1938 sold it outright to defendant-appellee (Exhibit 7). The
mortgage was registered and a memorandum thereof entered on T.C.T. No. 7454, but the
subsequent sale was never registered. However, from the date of the mortgage, defendant-appellee
had been in possession of the property.

Several years after Florencio Gasmeña's death in 1941, the portion which he had conveyed to
defendant-appellee was segregated and designated as Lot No. 659-A. On August 28, 1950, T.C.T.
No. NT-7782, covering Lot No. 659-A, was issued in the name of the already deceased Florencio
Gasmeña. This certificate of title carried no annotation of the registered mortgage in favor of
defendant-appellee. On September 20, 1950, the widow and heirs of Florencio Gasmeña executed a
deed of extrajudicial partition with sale (Exhibit B) wherein they declared that on November 15, 1941,
Florencio Gasmeña died intestate, without debts and possessed solely of Lot No. 659-A, which lot
they adjudicated to themselves and then sold for P2,000.00 to plaintiff-appellants. Before they
purchased the land, plaintiff-appellants had examined Florencio Gasmeña's title and had found no
encumbrance noted thereon. The deed of extrajudicial partition with sale was registered, so T.C.T.
No. NT-7782 in the name of Florencio Gasmeña was cancelled, and in lieu thereof T.C.T. No. NT-
7938 was issued to plaintiffs-appellants subject to the provisions of Sections 4 and 5 of Rule 74,
Rules of Court. Plaintiffs-appellants attempted to take possession of the land, but defendant-
appellee's overseer informed them that defendant-appellee had previously purchased it from
Florencio Gasmeña. Defendant-appellee's refusal to relinquish possession of the lot prompted
plaintiffs-appellants to commence this action.

The lower court, after trial, rendered judgment declaring the deed of extrajudicial partition with sale
(Exhibit B) null and void and ordering the Register of Deeds to cancel T.C.T. No. NT-7938.

Raising questions purely of law, plaintiffs have appealed directly to this Court.

We have here a case of registered land which had been sold to two different persons. Where the
same immovable property is sold to different vendees, the property shall belong to the one who first
recorded it in the Registry of Deeds (Article 1544, N.C.C., Article 1473, O.C.C.). This rule covers all
kinds of immovables, including land, and makes no distinction as to whether the immovable is
registered or not. But in so far as registered land is concerned said rule is in perfect accord with the
Land Registration Act, Section 50 of which provides that no deed, mortgage, lease or other voluntary
instrument except a will, purporting to convey or affect registered land shall take effect as a
conveyance or bind the land until its registration.

The first sale was in favor of defendant-appellee, and never registered, although from the time the
land was mortgaged to him up to the present, he has been in uninterrupted possession of the land.
But since we are dealing with registered land, title to which is imprescriptible (Section 46, Act 496),
defendant-appellee certainly cannot claim title by acquisitive prescription. To successfully bind the
land, he should have registered the sale in his favor. By reason of his failure to do so, the sale
operated only as a contract between him and the vendor, Florencio Gasmeña, and as evidence of
authority to the Register of Deeds to make registration (Section 50, Act 496; Buzon vs. Licauco, 13
Phil., 354; Worcester vs. Ocampo, 34 Phil., 646; Fidelity and Surety Co. vs. Conegero, 41 Phil., 396).
Florencio Gasmeña and his heirs were bound to respect the contract, but innocent third persons
cannot be affected thereby. From the time Florencio Gasmeña acquired the questioned lot up to the
time of his death (and even for nine years thereafter), there was an annotation on Alipio Gasmeña's
certificate of title to the effect that said portion of land had been conveyed to Florencio Gasmeña.
Hence, from the viewpoint of third persons, Florencio Gasmeña was still the owner even after the
unregistered sale to defendant-appellee. And it follows that if Florencio Gasmeña had been of a mind
to sell the land, he could have subsequently sold it and could still have passed good title to an
innocent purchaser for value, subject, of course, to the registered mortgage in favor of defendant-
appellee.

One of the main features of the Torrens system of registration is that all encumbrances on the land or
special estates therein shall be shown, or at least intimated upon the certificate of title and a person
dealing with the owner of the registered land is not bound to go behind the certificate and inquire into
transactions, the existence of which is not there intimated (Quimson vs. Suarez, 45 Phil., 901); but he
is only charged with notice of the burdens on the property which are noted on the face of the register
or on the certificate of title (Anderson and Co. vs. Garcia, 64 Phil., 506). The Torrens system seeks to
insure the efficacy and conclusiveness of the certificate of title. To enhance this aim we thus have the
provision that every person receiving a certificate of title in pursuance of a degree of registration, and
every subsequent purchaser of registered land who takes certificate of title for value in good faith
shall hold the same free of all incumbrances except those noted on said certificate (Section 39, Act
496).

Unquestionably, the sale in favor of plaintiffs-appellants was a registered one, and a certificate of title
was issued to them. The point of inquiry is whether they are purchasers in good faith. It is true that
before they bought the lot they had first examined the certificate of title of Florencio Gasmeña and
had found it absolutely clean in the sense that there was no annotation of any encumbrance. But one
fact stands out just as clearly. Plaintiffs-appellants did not buy the land from the registered owner,
Florencio Gasmeña. They bought it from his heirs.

The law protects to a greater degree a purchaser who buys from the registered owner himself.
Corollarily, it requires a higher degree of prudence from one who buys from a person who is not the
registered owner, although the land object of the transaction is registered. While one who buys from
the registered owner does not need to look behind the certificate of title, one who buys from one who
is not the registered owner is expected to examine not only the certificate of title but all factual
circumstances necessary for him to determine if there are any flaws in the title of the transferor, or in
his capacity to transfer the land.

This Court has consistently applied the stricter rule when it comes to deciding the issue of good faith
of one who buys from one who is not the registered owner, but who exhibits a certificate of title.

This is well illustrated in Veloso and Rosales vs. La Urbana and Del Mar (58 Phil., 681) where this
Court declared that one who deals with the supposed attorney in fact of a registered owner, has the
duty of ascertaining the genuineness of the deed purporting to be a power of attorney, and that
should he fail to determine whether such attorney in fact has the power to dispose of the land, then
he must suffer the consequences and damages resulting from the transaction. One who intends to
purchase registered land, must first make sure that the persons selling it to him is the person named
as owner in the certificate of title, and not an impostor. If he should neglect to ascertain the identity of
the seller, the law will not protect him, should such seller turn out to be an impostor (De Lara and De
Guzman vs. Ayroso, 95 Phil., 185; 50 Off. Gaz., No. 10, 4838). Where a person buys land not from
the registered owner but from one whose right to the land has been merely annotated on the
certificate of title, and such purchaser merely had his deed of sale annotated on the certificate of title,
he is not considered a "subsequent purchaser of registered land who takes certificate of title for value
and in good faith and who is protected against any encumbrance except those noted on said
certificate" (Mirasol vs. Gerochi, 93 Phil., 480).

The problem before us finds a parallel in the case of Mari vs. Bonilla (83 Phil., 137; 46 Off. Gaz.,
4258), where a question also arose respecting the vendees' good faith. In said case, the property was
still in the name of the already deceased Casimiro Evangelista at the time the vendees purchased it
from one of Casimiro's son. This son, Deogracias Evangelista, posing as sole heir of the deceased,
had adjudicated the land to himself and had then sold it to the vendees without first acquiring a
certificate of title in his name. It turned out later that the deceased had other heirs who sought to
recover the property from the vendees. In resolving the question presented, this Court declared that:

Good faith affords protection only to purchasers for value from the registered owner.
Deogracias Evangelista, defendants' grantor, was not a registered owner. The land was and
still is registered in the name of Casimiro Evangelista. In no way does the certificate state that
Deogracias owned the land; consequently defendants cannot summon to their aid the theory of
indefeasibility of Torrens Title. There is nothing in the certificate and in the circumstances of
the transaction which warrant them in supposing that they need not look beyond the title. If
anything, it would have put them on their guard, cautioned them to ascertain and verify that the
vendor was the only heir of his father, that there was no debt, and that the latter was the sole
owner of the parcel of land.

The above pronouncement may well be made here. Plaintiffs-appellants did not buy the lot from the
registered owner. Thus they were bound at their peril to investigate their transferors' right to sell the
property. Ordinary prudence called for a scrutiny of the deed of extrajudicial partition with sale (Exhibit
B) as well as the transfer certificate in the name of Florencio Gasmeña. Such an examination would
have inevitably revealed to plaintiffs-appellants that Florencio Gasmeña's heirs allowed almost a
decade to pass before they attempted to adjudicate the lot unto themselves, and, more important,
that Florencio Gasmeña had been dead nine years before the lot was segregated from Lot No. 659
and T.C.T. No. NT-7782 issued in his name. With these facts on hand, any prospective buyer of the
land would have examined the previous transfer certificate of title in the name of Alipio Gasmeña,
T.C.T. No. 7454, and would thus have discovered that at no time during his life was Florencio
Gasrmeña ever the registered owner of said portion of land. The memorandum of conveyance to him
in the certificate of title of Alipio Gasmeña served merely as a notice to third parties of the fact that
said portion had been transferred to Florencio Gasmeña (Section 58, Act No. 496, as amended by
Act No. 4029), but it did not have the same effect as a certificate of title issued to Florencio Gasmeña
himself. The doctrine therefore in the Bonilla case applies with more force herein for neither plaintiffs-
appellants transferors nor the latter's predecessor was ever the registered owner of the lot.

An examination of Alipio Gasmeña's certificate of title would likewise have yielded the fact that said
portion of land had been mortgaged to defendant-appellee. All these circumstances were sufficient to
warn plaintiffs-appellants that their transferors did not have clean title to the land. Their failure to
make the investigations required by the circumstances constitutes lack of good faith. They cannot
now plead ignorance because they had before them facts which would have reasonably impelled an
ordinarily prudent buyer to make an inquiry and to exercise due care.

A purchaser cannot close his eyes to facts which should put a reasonable man upon his guard,
and then claim that he acted in good faith under the belief that there was no defect in the title
of the vendor. His mere refusal to believe that such defect exists, or his willful closing of his
eyes to the possibility of the existence of a defect in his vendor's title, will not make him an
innocent purchaser for value, if it afterwards develops that the title was in fact defective, and it
appears that he had such notice of the defect as would have led to its discovery had he acted
with that measure of precaution which may reasonably be required of a prudent man in like
situation. (Leung Yee vs. F. L. Strong Machinery Co., and Williamson, 37 Phil., 644).

Not being purchasers in good faith, plaintiffs-appellants are clearly not entitled to the rights of a
registered owner.

Neither of the vendees having registered their respective sales in good faith, their right to the property
must be determined by the priority of possession. Where the same immovable property was sold to
two different persons neither of whom recorded the transfer in good faith, ownership shall pertain to
the person who in good faith was first in the possession (Article 1544, N.C.C.; Article 1473, O.C.C.).
The lot, therefore properly belongs to defendant-appellee.

Having arrived at the foregoing conclusions, we find it unnecessary to discuss the other points raised
by the parties.

Wherefore, the appealed judgment is hereby affirmed with costs against appellants.

Paras, C.J., Bengzon, Montemayor, Bautista Angelo, Labrador, Concepcion, Reyes, J.B.L. and
Barrera, JJ., concur.

THE HEIRS OF MARCELINO DORONIO, NAMELY: REGINA AND FLORA, BOTH SURNAMED
DORONIO,Petitioners,
vs.
HEIRS OF FORTUNATO DORONIO, NAMELY: TRINIDAD ROSALINA DORONIO-BALMES,
MODING DORONIO, FLORENTINA DORONIO, AND ANICETA ALCANTARA-
MANALO, Respondents.

DECISION

REYES, R.T., J.:

For Our review on certiorari is the Decision1 of the Court of Appeals (CA) reversing that2 of the
Regional Trial Court (RTC), Branch 45, Anonas, Urdaneta City, Pangasinan, in an action for
reconveyance and damages. The CA declared respondents as rightful owners of one-half of the
subject property and directed petitioners to execute a registerable document conveying the same to
respondents.

The Facts

Spouses Simeon Doronio and Cornelia Gante, now both deceased, were the registered owners of a
parcel of land located at Barangay Cabalitaan, Asingan, Pangasinan covered by Original Certificate
of Title (OCT) No. 352.3 The courts below described it as follows:

Un terreno (Lote 1018), situada en el municipio de Asingan, Linda por el NE; con propriedad de
Gabriel Bernardino; con el SE con propriedad de Zacarias Najorda y Alejandro Najorda; por el SO
con propriedad de Geminiano Mendoza y por el NO con el camino para Villasis; midiendo una
extension superficial mil ciento cincuenta y dos metros cuadrados.4
The spouses had children but the records fail to disclose their number. It is clear, however, that
Marcelino Doronio and Fortunato Doronio, now both deceased, were among them and that the
parties in this case are their heirs. Petitioners are the heirs of Marcelino Doronio, while respondents
are the heirs of Fortunato Doronio.

On April 24, 1919, a private deed of donation propter nuptias 5 was executed by spouses Simeon
Doronio and Cornelia Gante in favor of Marcelino Doronio and the latter’s wife, Veronica Pico. One of
the properties subject of said deed of donation is the one that it described as follows:

Fourth – A piece of residential land located in the barrio of Cabalitian but we did not measure it, the
area is bounded on the north by Gabriel Bernardino; on the east by Fortunato Doronio; on the south
by Geminiano Mendoza and on the west by a road to Villasis. Constructed on said land is a house of
light materials – also a part of the dowry. Value …200.00.6

It appears that the property described in the deed of donation is the one covered by OCT No. 352.
However, there is a significant discrepancy with respect to the identity of the owner of adjacent
property at the eastern side. Based on OCT No. 352, the adjacent owners are Zacarias Najorda and
Alejandro Najorda, whereas based on the deed of donation, the owner of the adjacent property is
Fortunato Doronio. Furthermore, said deed of donation remained a private document as it was never
notarized.7

Both parties have been occupying the subject land for several decades 8 although they have different
theories regarding its present ownership. According to petitioners, they are now the owners of the
entire property in view of the private deed of donation propter nuptias in favor of their predecessors,
Marcelino Doronio and Veronica Pico.

Respondents, on the other hand, claim that only half of the property was actually incorporated in the
said deed of donation because it stated that Fortunato Doronio, instead of Zacarias Najorda and
Alejandro Najorda, is the owner of the adjacent property at the eastern side. Respondents posit that
the donors respected and segregated the possession of Fortunato Doronio of the eastern half of the
land. They are the ones who have been possessing said land occupied by their predecessor,
Fortunato Doronio.

Eager to obtain the entire property, the heirs of Marcelino Doronio and Veronica Pico filed, on
January 11, 1993, before the RTC in Urdaneta, Pangasinan a petition "For the Registration of a
Private Deed of Donation"9 docketed as Petition Case No. U-920. No respondents were named in the
said petition10 although notices of hearing were posted on the bulletin boards of Barangay Cabalitaan,
Municipalities of Asingan and Lingayen.11

During the hearings, no one interposed an objection to the petition.12 After the RTC ordered a general
default,13the petition was eventually granted on September 22, 1993. This led to the registration of the
deed of donation, cancellation of OCT No. 352 and issuance of a new Transfer Certificate of Title
(TCT) No. 44481 in the names of Marcelino Doronio and Veronica Pico.14 Thus, the entire property
was titled in the names of petitioners’ predecessors.

On April 28, 1994, the heirs of Fortunato Doronio filed a pleading before the RTC in the form of a
petition in the same Petition Case No. U-920. The petition was for the reconsideration of the decision
of the RTC that ordered the registration of the subject deed of donation. It was prayed in the petition
that an order be issued declaring null and void the registration of the private deed of donation and
that TCT No. 44481 be cancelled. However, the petition was dismissed on May 13, 1994 on the
ground that the decision in Petition Case No. U-920 had already become final as it was not appealed.
Determined to remain in their possessed property, respondent heirs of Fortunato Doronio (as
plaintiffs) filed an action for reconveyance and damages with prayer for preliminary
injunction15 against petitioner heirs of Marcelino Doronio (as defendants) before the RTC, Branch 45,
Anonas, Urdaneta City, Pangasinan. Respondents contended, among others, that the subject land is
different from what was donated as the descriptions of the property under OCT No. 352 and under
the private deed of donation were different. They posited that spouses Simeon Doronio and Cornelia
Gante intended to donate only one-half of the property.

During the pre-trial conference, the parties stipulated, among others, that the property was originally
covered by OCT No. 352 which was cancelled by TCT No. 44481. They also agreed that the issues
are: (1) whether or not there was a variation in the description of the property subject of the private
deed of donation and OCT No. 352; (2) whether or not respondents had acquired one-half of the
property covered by OCT No. 352 by acquisitive prescription; (3) whether or not the transfer of the
whole property covered by OCT No. 352 on the basis of the registration of the private deed of
donation notwithstanding the discrepancy in the description is valid; (4) whether or not respondents
are entitled to damages; and (5) whether or not TCT No. 44481 is valid. 16

RTC Decision

After due proceedings, the RTC ruled in favor of petitioner heirs of Marcelino Doronio (defendants). It
concluded that the parties admitted the identity of the land which they all occupy;17 that a title once
registered under the torrens system cannot be defeated by adverse, open and notorious possession
or by prescription;18 that the deed of donation in consideration of the marriage of the parents of
petitioners is valid, hence, it led to the eventual issuance of TCT No. 44481 in the names of said
parents;19 and that respondent heirs of Fortunato Doronio (plaintiffs) are not entitled to damages as
they are not the rightful owners of the portion of the property they are claiming.20

The RTC disposed of the case, thus:

WHEREFORE, premises considered, the Court hereby renders judgment DISMISSING the herein
Complaint filed by plaintiffs against defendants.21

Disagreeing with the judgment of the RTC, respondents appealed to the CA. They argued that the
trial court erred in not finding that respondents’ predecessor-in-interest acquired one-half of the
property covered by OCT No. 352 by tradition and/or intestate succession; that the deed of donation
dated April 26, 1919 was null and void; that assuming that the deed of donation was valid, only one-
half of the property was actually donated to Marcelino Doronio and Veronica Pico; and that
respondents acquired ownership of the other half portion of the property by acquisitive prescription. 22

CA Disposition

In a Decision dated January 26, 2005, the CA reversed the RTC decision with the following
disposition:

WHEREFORE, the assailed Decision dated June 28, 2002 is REVERSED and SET ASIDE. Declaring
the appellants as rightful owners of one-half of the property now covered by TCT No. 44481, the
appellees are hereby directed to execute a registerable document conveying the same to appellants.

SO ORDERED.23

The appellate court determined that "(t)he intention to donate half of the disputed property to
appellees’ predecessors can be gleaned from the disparity of technical descriptions appearing in the
title (OCT No. 352) of spouses Simeon Doronio and Cornelia Gante and in the deed of donation
propter nuptias executed on April 24, 1919 in favor of appellees’ predecessors." 24

The CA based its conclusion on the disparity of the following technical descriptions of the property
under OCT No. 352 and the deed of donation, to wit:

The court below described the property covered by OCT No. 352 as follows:

"Un terreno (Lote 1018), situada en el municipio de Asingan, Linda por el NE; con propriedad de
Gabriel Bernardino; con el SE con propriedad de Zacarias Najorda y Alejandro Najorda; por el SO
con propriedad de Geminiano Mendoza y por el NO con el camino para Villasis; midiendo una
extension superficial mil ciento cincuenta y dos metros cuadrados."

On the other hand, the property donated to appellees’ predecessors was described in the deed of
donation as:

"Fourth – A piece of residential land located in the barrio of Cabalitian but we did not measure it, the
area is bounded on the north by Gabriel Bernardino; on the east by Fortunato Doronio; on the south
by Geminiano Mendoza and on the west by a road to Villasis. Constructed on said land is a house of
light materials – also a part of the dowry. Value …200.00."25 (Emphasis ours)

Taking note "that the boundaries of the lot donated to Marcelino Doronio and Veronica Pico differ
from the boundaries of the land owned by spouses Simeon Doronio and Cornelia Gante," the CA
concluded that spouses Simeon Doronio and Cornelia Gante donated only half of the property
covered by OCT No. 352.26

Regarding the allegation of petitioners that OCT No. 352 is inadmissible in evidence, the CA pointed
out that, "while the OCT is written in the Spanish language, this document already forms part of the
records of this case for failure of appellees to interpose a timely objection when it was offered as
evidence in the proceedings a quo. It is a well-settled rule that any objection to the admissibility of
such evidence not raised will be considered waived and said evidence will have to form part of the
records of the case as competent and admitted evidence."27

The CA likewise ruled that the donation of the entire property in favor of petitioners’ predecessors is
invalid on the ground that it impairs the legitime of respondents’ predecessor, Fortunato Doronio. On
this aspect, the CA reasoned out:

Moreover, We find the donation of the entire property in favor of appellees’ predecessors invalid as it
impairs the legitime of appellants’ predecessor. Article 961 of the Civil Code is explicit. "In default of
testamentary heirs, the law vests the inheritance, x x x, in the legitimate x x x relatives of the
deceased, x x x." As Spouses Simeon Doronio and Cornelia Gante died intestate, their property shall
pass to their lawful heirs, namely: Fortunato and Marcelino Doronio. Donating the entire property to
Marcelino Doronio and Veronica Pico and excluding another heir, Fortunato, tantamounts to divesting
the latter of his rightful share in his parents’ inheritance. Besides, a person’s prerogative to make
donations is subject to certain limitations, one of which is that he cannot give by donation more than
what he can give by will (Article 752, Civil Code). If he does, so much of what is donated as exceeds
what he can give by will is deemed inofficious and the donation is reducible to the extent of such
excess.28

Petitioners were not pleased with the decision of the CA. Hence, this petition under Rule 45.

Issues
Petitioners now contend that the CA erred in:

1. DECLARING ADMISSIBILITY OF THE ORIGINAL CERTIFICATE OF TITLE NO. 352


DESPITE OF LACK OF TRANSLATION THEREOF.

2. (RULING THAT) ONLY HALF OF THE DISPUTED PROPERTY WAS DONATED TO THE
PREDECESSORS-IN-INTEREST OF THE HEREIN APPELLANTS.

3. (ITS) DECLARATION THAT THE DONATION PROPTER NUPTIAS IS INNOFICIOUS, IS


PREMATURE, AND THUS IT IS ILLEGAL AND UNPROCEDURAL.29

Our Ruling

OCT No. 352 in Spanish Although Not


Translated into English or Filipino Is
Admissible For Lack of Timely Objection

Petitioners fault the CA for admitting OCT No. 352 in evidence on the ground that it is written in
Spanish language. They posit that "(d)ocumentary evidence in an unofficial language shall not be
admitted as evidence, unless accompanied with a translation into English or Filipino."30

The argument is untenable. The requirement that documents written in an unofficial language must
be accompanied with a translation in English or Filipino as a prerequisite for its admission in evidence
must be insisted upon by the parties at the trial to enable the court, where a translation has been
impugned as incorrect, to decide the issue.31 Where such document, not so accompanied with a
translation in English or Filipino, is offered in evidence and not objected to, either by the parties or the
court, it must be presumed that the language in which the document is written is understood by all,
and the document is admissible in evidence.32

Moreover, Section 36, Rule 132 of the Revised Rules of Evidence provides:

SECTION 36. Objection. – Objection to evidence offered orally must be made immediately after the
offer is made.

Objection to a question propounded in the course of the oral examination of a witness shall be made
as soon as the grounds therefor shall become reasonably apparent.

An offer of evidence in writing shall be objected to within three (3) days after notice of the offer unless
a different period is allowed by the court.

In any case, the grounds for the objections must be specified. (Emphasis ours)

Since petitioners did not object to the offer of said documentary evidence on time, it is now too late in
the day for them to question its admissibility. The rule is that evidence not objected may be deemed
admitted and may be validly considered by the court in arriving at its judgment.33 This is true even if
by its nature, the evidence is inadmissible and would have surely been rejected if it had been
challenged at the proper time.34

As a matter of fact, instead of objecting, petitioners admitted the contents of Exhibit "A," that is, OCT
No. 352 in their comment35 on respondents’ formal offer of documentary evidence. In the said
comment, petitioners alleged, among others, that "Exhibits A, B, C, D, E, F and G, are admitted but
not for the purpose they are offered because these exhibits being public and official documents are
the best evidence of that they contain and not for what a party would like it to prove." 36 Said evidence
was admitted by the RTC.37 Once admitted without objection, even though not admissible under an
objection, We are not inclined now to reject it.38 Consequently, the evidence that was not objected to
became property of the case, and all parties to the case are considered amenable to any favorable or
unfavorable effects resulting from the said evidence.39

Issues on Impairment of Legitime


Should Be Threshed Out in a Special
Proceeding, Not in Civil Action for
Reconveyance and Damages

On the other hand, petitioners are correct in alleging that the issue regarding the impairment of
legitime of Fortunato Doronio must be resolved in an action for the settlement of estates of spouses
Simeon Doronio and Cornelia Gante. It may not be passed upon in an action for reconveyance and
damages. A probate court, in the exercise of its limited jurisdiction, is the best forum to ventilate and
adjudge the issue of impairment of legitime as well as other related matters involving the settlement
of estate.40

An action for reconveyance with damages is a civil action, whereas matters relating to settlement of
the estate of a deceased person such as advancement of property made by the decedent, partake of
the nature of a special proceeding. Special proceedings require the application of specific rules as
provided for in the Rules of Court.41

As explained by the Court in Natcher v. Court of Appeals:42

Section 3, Rule 1 of the 1997 Rules of Civil Procedure defines civil action and special proceedings, in
this wise:

x x x a) A civil action is one by which a party sues another for the enforcement or protection of a right,
or the prevention or redress of a wrong.

A civil action may either be ordinary or special. Both are governed by the rules for ordinary civil
actions, subject to specific rules prescribed for a special civil action.

xxxx

c) A special proceeding is a remedy by which a party seeks to establish a status, a right or a


particular fact.

As could be gleaned from the foregoing, there lies a marked distinction between an action and a
special proceeding. An action is a formal demand of one’s right in a court of justice in the manner
prescribed by the court or by the law. It is the method of applying legal remedies according to definite
established rules. The term "special proceeding" may be defined as an application or proceeding to
establish the status or right of a party, or a particular fact. Usually, in special proceedings, no formal
pleadings are required unless the statute expressly so provides. In special proceedings, the remedy
is granted generally upon an application or motion.

Citing American Jurisprudence, a noted authority in Remedial Law expounds further:

It may accordingly be stated generally that actions include those proceedings which are instituted and
prosecuted according to the ordinary rules and provisions relating to actions at law or suits in equity,
and that special proceedings include those proceedings which are not ordinary in this sense, but is
instituted and prosecuted according to some special mode as in the case of proceedings commenced
without summons and prosecuted without regular pleadings, which are characteristics of ordinary
actions x x x. A special proceeding must therefore be in the nature of a distinct and independent
proceeding for particular relief, such as may be instituted independently of a pending action, by
petition or motion upon notice.

Applying these principles, an action for reconveyance and annulment of title with damages is a civil
action, whereas matters relating to settlement of the estate of a deceased person such as
advancement of property made by the decedent, partake of the nature of a special proceeding, which
concomitantly requires the application of specific rules as provided for in the Rules of Court.

Clearly, matters which involve settlement and distribution of the estate of the decedent fall within the
exclusive province of the probate court in the exercise of its limited jurisdiction.

Thus, under Section 2, Rule 90 of the Rules of Court, questions as to advancement made or alleged
to have been made by the deceased to any heir may be heard and determined by the court having
jurisdiction of the estate proceedings, and the final order of the court thereon shall be binding on the
person raising the questions and on the heir.

While it may be true that the Rules used the word "may," it is nevertheless clear that the same
provision contemplates a probate court when it speaks of the "court having jurisdiction of the estate
proceedings."

Corollarily, the Regional Trial Court in the instant case, acting in its general jurisdiction, is devoid of
authority to render an adjudication and resolve the issue of advancement of the real property in favor
of herein petitioner Natcher, inasmuch as Civil Case No. 71075 for reconveyance and annulment of
title with damages is not, to our mind, the proper vehicle to thresh out said question. Moreover, under
the present circumstances, the RTC of Manila, Branch 55, was not properly constituted as a probate
court so as to validly pass upon the question of advancement made by the decedent Graciano Del
Rosario to his wife, herein petitioner Natcher.

We likewise find merit in petitioners’ contention that before any conclusion about the legal share due
to a compulsory heir may be reached, it is necessary that certain steps be taken first. 43 The net estate
of the decedent must be ascertained, by deducting all payable obligations and charges from the value
of the property owned by the deceased at the time of his death; then, all donations subject to collation
would be added to it. With the partible estate thus determined, the legitime of the compulsory heir or
heirs can be established; and only then can it be ascertained whether or not a donation had
prejudiced the legitimes.44

Declaration of Validity of Donation


Can Be Challenged by an Interested
Party Not Impleaded in Petition for
Quieting of Title or Declaratory Relief
or Where There is No Res Judicata.
Moreover, This Court Can Consider
a Factual Matter or Unassigned Error
in the Interest of Substantial Justice.

Nevertheless, petitioners cannot preclude the determination of validity of the deed of donation on the
ground that (1) it has been impliedly admitted by respondents; (2) it has already been determined with
finality by the RTC in Petition Case No. U-920; or (3) the only issue in an action for reconveyance is
who has a better right over the land.45

The validity of the private deed of donation propter nuptias in favor of petitioners’ predecessors was
one of the issues in this case before the lower courts. The pre-trial order46 of the RTC stated that one
of the issues before it is "(w)hether or not the transfer of the whole property covered by OCT No. 352
on the basis of the private deed of donation notwithstanding the discrepancy in the description is
valid." Before the CA, one of the errors assigned by respondents is that "THE TRIAL COURT ERRED
IN NOT FINDING THAT THE PRIVATE DEED OF DONATION DATED APRIL 26, 1919 WAS NULL
AND VOID."47

The issue of the validity of donation is likewise brought to Us by petitioners as they stated in their
Memorandum48that one of the issues to be resolved is regarding the alleged fact that "THE
HONORABLE COURT OF APPEALS ERRED IN FINDING THE DONATION INVALID." We are thus
poised to inspect the deed of donation and to determine its validity.

We cannot agree with petitioners’ contention that respondents may no longer question the validity of
the deed of donation on the ground that they already impliedly admitted it. Under the provisions of the
Civil Code, a void contract is inexistent from the beginning. The right to set up the defense of its
illegality cannot be waived.49 The right to set up the nullity of a void or non-existent contract is not
limited to the parties as in the case of annullable or voidable contracts; it is extended to third persons
who are directly affected by the contract.50

Consequently, although respondents are not parties in the deed of donation, they can set up its nullity
because they are directly affected by the same.51 The subject of the deed being the land they are
occupying, its enforcement will definitely affect them.

Petitioners cannot also use the finality of the RTC decision in Petition Case No. U-92052 as a shield
against the verification of the validity of the deed of donation. According to petitioners, the said final
decision is one for quieting of title.53 In other words, it is a case for declaratory relief under Rule 64
(now Rule 63) of the Rules of Court, which provides:

SECTION 1. Who may file petition. – Any person interested under a deed, will, contract or other
written instrument, or whose rights are affected by a statute, executive order or regulation, or
ordinance, may, before breach or violation thereof, bring an action to determine any question of
construction or validity arising under the instrument or statute and for a declaration of his rights or
duties thereunder.

An action for the reformation of an instrument, to quiet title to real property or remove clouds
therefrom, or to consolidate ownership under Article 1607 of the Civil Code, may be brought under
this rule.

SECTION 2. Parties. – All persons shall be made parties who have or claim any interest which would
be affected by the declaration; and no declaration shall, except as otherwise provided in these rules,
prejudice the rights of persons not parties to the action. (Emphasis ours)

However, respondents were not made parties in the said Petition Case No. U-920.1âwphi1 Worse,
instead of issuing summons to interested parties, the RTC merely allowed the posting of notices on
the bulletin boards of Barangay Cabalitaan, Municipalities of Asingan and Lingayen, Pangasinan. As
pointed out by the CA, citing the ruling of the RTC:
x x x In the said case or Petition No. U-920, notices were posted on the bulletin boards of barangay
Cabalitaan, Municipalities of Asingan and Lingayen, Pangasinan, so that there was a notice to the
whole world and during the initial hearing and/or hearings, no one interposed objection thereto. 54

Suits to quiet title are not technically suits in rem, nor are they, strictly speaking, in personam, but
being against the person in respect of the res, these proceedings are characterized as quasi in
rem.55 The judgment in such proceedings is conclusive only between the parties. 56 Thus, respondents
are not bound by the decision in Petition Case No. U-920 as they were not made parties in the said
case.

The rules on quieting of title57 expressly provide that any declaration in a suit to quiet title shall not
prejudice persons who are not parties to the action.

That respondents filed a subsequent pleading58 in the same Petition Case No. U-920 after the
decision there had become final did not change the fact that said decision became final without their
being impleaded in the case. Said subsequent pleading was dismissed on the ground of finality of the
decision.59

Thus, the RTC totally failed to give respondents their day in court. As a result, they cannot be bound
by its orders. Generally accepted is the principle that no man shall be affected by any proceeding to
which he is a stranger, and strangers to a case are not bound by judgment rendered by the court. 60

Moreover, for the principle of res judicata to apply, the following must be present: (1) a decision on
the merits; (2) by a court of competent jurisdiction; (3) the decision is final; and (4) the two actions
involve identical parties, subject matter and causes of action. 61 The fourth element is not present in
this case. The parties are not identical because respondents were not impleaded in Petition Case No.
U-920. While the subject matter may be the same property covered by OCT No. 352, the causes of
action are different. Petition Case No. U-920 is an action for declaratory relief while the case below is
for recovery of property.

We are not persuaded by petitioners’ posture that the only issue in this action for reconveyance is
who has a better right over the land; and that the validity of the deed of donation is beside the
point.62 It is precisely the validity and enforceability of the deed of donation that is the determining
factor in resolving the issue of who has a better right over the property. Moreover, notwithstanding
procedural lapses as to the appropriateness of the remedies prayed for in the petition filed before Us,
this Court can brush aside the technicalities in the interest of justice. In some instances, this Court
even suspended its own rules and excepted a case from their operation whenever the higher
interests of justice so demanded.63

Moreover, although respondents did not directly raise the issue of validity of the deed of donation at
the commencement of the case before the trial court, it was stipulated64 by the parties during the pre-
trial conference. In any event, this Court has authority to inquire into any question necessary in
arriving at a just decision of a case before it.65 Though not specifically questioned by the parties,
additional issues may also be included, if deemed important for substantial justice to be rendered.66

Furthermore, this Court has held that although a factual issue is not squarely raised below, still in the
interest of substantial justice, this Court is not prevented from considering a pivotal factual matter.
The Supreme Court is clothed with ample authority to review palpable errors not assigned as such if it
finds that their consideration is necessary in arriving at a just decision.67

A rudimentary doctrine on appealed cases is that this Court is clothed with ample authority to review
matters, even if they are not assigned as errors on appeal, if it finds that their consideration is
necessary at arriving at a just decision of the case.68 Also, an unassigned error closely related to an
error properly assigned or upon which the determination of the question raised by the error properly
assigned is dependent, will be considered by the appellate court notwithstanding the failure to assign
it as an error.69

Donation Propter Nuptias of Real


Property Made in a Private Instrument
Before the New Civil Code Took Effect
on August 30, 1950 is Void

We now focus on the crux of the petition, which is the validity of the deed of donation.1avvphi1 It is
settled that only laws existing at the time of the execution of a contract are applicable to it and not the
later statutes, unless the latter are specifically intended to have retroactive effect. 70 Accordingly, the
Old Civil Code applies in this case as the donation propter nuptias was executed in 1919, while the
New Civil Code took effect only on August 30, 1950.

Under the Old Civil Code, donations propter nuptias must be made in a public instrument in which the
property donated must be specifically described.71 Article 1328 of the Old Civil Code provides that
gifts propter nuptias are governed by the rules established in Title 2 of Book 3 of the same Code.
Article 633 of that title provides that the gift of real property, in order to be valid, must appear in a
public document.72 It is settled that a donation of real estate propter nuptias is void unless made by
public instrument.73

In the instant case, the donation propter nuptias did not become valid. Neither did it create any right
because it was not made in a public instrument.74 Hence, it conveyed no title to the land in question to
petitioners’ predecessors.

Logically, then, the cancellation of OCT No. 352 and the issuance of a new TCT No. 44481 in favor of
petitioners’ predecessors have no legal basis. The title to the subject property should, therefore, be
restored to its original owners under OCT No. 352.

Direct reconveyance to any of the parties is not possible as it has not yet been determined in a proper
proceeding who among the heirs of spouses Simeon Doronio and Cornelia Gante is entitled to it. It is
still unproven whether or not the parties are the only ones entitled to the properties of spouses
Simeon Doronio and Cornelia Gante. As earlier intimated, there are still things to be done before the
legal share of all the heirs can be properly adjudicated.75

Titled Property Cannot Be Acquired


By Another By Adverse Possession
or Extinctive Prescription

Likewise, the claim of respondents that they became owners of the property by acquisitive
prescription has no merit. Truth to tell, respondents cannot successfully invoke the argument of
extinctive prescription. They cannot be deemed the owners by acquisitive prescription of the portion
of the property they have been possessing. The reason is that the property was covered by OCT No.
352. A title once registered under the torrens system cannot be defeated even by adverse, open and
notorious possession; neither can it be defeated by prescription.76 It is notice to the whole world and
as such all persons are bound by it and no one can plead ignorance of the registration.77

The torrens system is intended to guarantee the integrity and conclusiveness of the certificate of
registration, but it cannot be used for the perpetration of fraud against the real owner of the registered
land.78 The system merely confirms ownership and does not create it. Certainly, it cannot be used to
divest the lawful owner of his title for the purpose of transferring it to another who has not acquired it
by any of the modes allowed or recognized by law. It cannot be used to protect a usurper from the
true owner, nor can it be used as a shield for the commission of fraud; neither does it permit one to
enrich himself at the expense of another.79 Where such an illegal transfer is made, as in the case at
bar, the law presumes that no registration has been made and so retains title in the real owner of the
land.80

Although We confirm here the invalidity of the deed of donation and of its resulting TCT No. 44481,
the controversy between the parties is yet to be fully settled. The issues as to who truly are the
present owners of the property and what is the extent of their ownership remain unresolved. The
same may be properly threshed out in the settlement of the estates of the registered owners of the
property, namely: spouses Simeon Doronio and Cornelia Gante.

WHEREFORE, the appealed Decision is REVERSED AND SET ASIDE. A new one is entered:

(1) Declaring the private deed of donation propter nuptias in favor of petitioners’ predecessors
NULL AND VOID; and

(2) Ordering the Register of Deeds of Pangasinan to:

(a) CANCEL Transfer Certificate of Title No. 44481 in the names of Marcelino Doronio
and Veronica Pico; and

(b) RESTORE Original Certificate of Title No. 352 in the names of its original owners,
spouses Simeon Doronio and Cornelia Gante.

SO ORDERED.

DINAH C. CASTILLO, Petitioner,


vs.
ANTONIO M. ESCUTIN, AQUILINA A. MISTAS, MARIETTA L. LINATOC, AND THE HONORABLE
COURT OF APPEALS, Respondents.

DECISION

CHICO-NAZARIO, J.:

Before this Court is a Petition for Review on Certiorari 1 under Rule 45 of the Rules of Court filed by
petitioner Dinah C. Castillo seeking the reversal and setting aside of the Decision, 2 dated 18 October
2005, of the Court of Appeals in CA-G.R. SP No. 90533, as well as the Resolution,3 dated 11 January
2006 of the same court denying reconsideration of its afore-mentioned Decision. The Court of
Appeals, in its assailed Decision, affirmed the Joint Resolution 4 dated 28 April 2004 and Joint
Order5 dated 20 June 2005 of the Office of the Deputy Ombudsman for Luzon in OMB-L-A-03-0573-F
and OMB-L-C-03-0728-F, dismissing petitioner Dinah C. Castillo’s complaint for grave misconduct
and violation of Section 3(e) of Republic Act No. 3019, the Anti-Graft and Corrupt Practices Act, as
amended, against respondent public officers Antonio M. Escutin (Escutin), Aquilina A. Mistas (Mistas)
and Marietta L. Linatoc (Linatoc), together with private individuals Lauro S. Leviste II (Leviste) and
Benedicto L. Orense (Orense).
Petitioner is a judgment creditor of a certain Raquel K. Moratilla (Raquel), married to Roel
Buenaventura. In the course of her search for properties to satisfy the judgment in her favor,
petitioner discovered that Raquel, her mother Urbana Kalaw (Urbana), and sister Perla K. Moratilla
(Perla), co-owned Lot 13713, a parcel of land consisting of 15,000 square meters, situated at Brgy.
Bugtongnapulo, Lipa City, Batangas, and covered by Tax Declaration No. 00449.

Petitioner set about verifying the ownership of Lot 13713. She was able to secure an Order6 dated 4
March 1999 issued by Secretary Horacio R. Morales, Jr. of the Department of Agrarian Reform (DAR)
approving the application of Summit Point Golf & Country Club, Inc. for conversion of several
agricultural landholdings, including Lot 13713 owned by "Perla K. Mortilla, et al." and covered by Tax
Declaration No. 00449, to residential, commercial, and recreational uses. She was also able to get
from the Office of the City Assessor, Lipa City, a Certification7 stating that Lot 13713, covered by Tax
Declaration No. 00554-A, was in the name of co-owners Raquel, Urbana, and Perla; and a certified
true copy of Tax Declaration No. 00554-A itself.8 Lastly, the Register of Deeds of Lipa City issued a
Certification9 attesting that Lot 13713 in the name of co-owners Raquel, Urbana, and Perla, was not
covered by a certificate of title, whether judicial or patent, or subject to the issuance of a Certificate of
Land Ownership Award or patent under the Comprehensive Agrarian Reform Program.

Only thereafter did petitioner proceed to levy on execution Lot 13713, and the public auction sale of
the same was scheduled on 14 May 2002. Sometime in May 2002, before the scheduled public
auction sale, petitioner learned that Lot 13713 was inside the Summit Point Golf and Country Club
Subdivision owned by Summit Point Realty and Development Corporation (Summit Realty). She
immediately went to the Makati City office of Summit Realty to meet with its Vice President, Orense.
However, she claimed that Orense did not show her any document to prove ownership of Lot 13713
by Summit Realty, and even threatened her that the owners of Summit Realty, the Leviste family, was
too powerful and influential for petitioner to tangle with.

The public auction sale pushed through on 14 May 2002, and petitioner bought Raquel’s 1/3 pro-
indiviso share in Lot 13713.

On 4 June 2002, petitioner had the following documents, on her acquisition of Raquel’s 1/3 pro-
indiviso share in Lot 13713, recorded in the Primary Entry Book and Registration Book of the Register
of Deeds of Lipa City in accordance with Act No. 334410: (a) Notice of Levy;11 (b) Certificate of
Sale;12 (c) Affidavit of Publication;13 and (d) Writ of Execution.14

Subsequently, petitioner was issued by the City Assessor of Lipa City Tax Declaration No. 00942-
A,15 indicating that she owned 5,000 square meters of Lot 13713, while Urbana and Perla owned the
other 10,000 square meters.

When petitioner attempted to pay real estate taxes for her 5,000-square-meter share in Lot 13713,
she was shocked to find out that, without giving her notice, her Tax Declaration No. 00942-A was
cancelled. Lot 13713 was said to be encompassed in and overlapping with the 105,648 square meter
parcel of land known as Lot 1-B, covered by Transfer Certificate of Title (TCT) No. 12964216 and Tax
Declaration No. 00949-A,17 both in the name of Francisco Catigbac (Catigbac). The reverse side of
TCT No. 129642 bore three entries, reflecting the supposed sale of Lot 1-B to Summit Realty, to wit:

ENTRY NO. 184894: SPECIAL POWER OF ATTORNEY: In favor of LEONARDO YAGIN: For
purposes more particularly stipulated in the contract ratified before Atty. Ernesto M. Vergara of
Lipa City as per Doc. No. 639; Page No. 29; Book No. LXXVI; Series of 1976.

Date of instrument – 2-6-1976


Date of inscription – 6-26-2002 at 11:20 a.m.

ENTRY NO. 185833: SALE IN FAVOR OF SUMMIT POINT REALTY & DEVELOPMENT
CORP: –

ENTRY NO. 185834: BIR CLEARANCE: – Of the parcel of land described in this cert. of title is
hereby sold and cancelled TCT No. 134609(SN-6672938) Vol. 671-A, having been issued by
virtue of the aforesaid instrument ratified before Perfecto L. Dimayuga, Notary Public for
Makati City as per Doc. No. 148; Page 31, Book No. LXVII, Series of 2002.

Date of instrument: July 22, 2002

Date of inscription: July 25, 2002 at 2:30 P.M.18

On 25 July 2002, at 2:30 p.m., TCT No. 129642 in the name of Catigbac was cancelled and TCT No.
T-134609 in the name of Summit Realty was issued in its place.

The foregoing incidents prompted petitioner to file a Complaint Affidavit19 before the Office of the
Deputy Ombudsman for Luzon charging several public officers and private individuals as follows:

32. I respectfully charge that on or about the months of June 2002 and July 2002 and onwards in Lipa
City, Atty. Antonio M. [Escutin], the Register of Deeds of Lipa City[;] Aquilina A. Mistas, the Local
Assessment Operations Officer III of the City Assessor’s Office of Lipa City[;] Marietta Linatoc,
Records Clerk, Office of the City Assessor of Lipa City, who are public officers and acting in concert
and conspiring with Lauro S. Leviste II and Benedicto L. Orense, Executive Vice-President and Vice-
President, respectively[,] of Summit Point Realty and Development Corporation x x x while in the
discharge of their administrative functions did then and there unlawfully, through evident bad faith,
gross inexcusable negligence and with manifest partiality towards Summit caused me injury in the
sum of ₱20,000,000.00 by cancelling my TD #00942-A in the Office of the City Assessor of Lipa City
and instead issuing in the name of Francisco Catigbac TC #00949-A when aforesaid personalities
well knew that TCT No. 129642 was already cancelled and therefore not legally entitled to a new tax
declaration thereby manifestly favoring Summit Point Realty and Development Corporation who now
appears to be the successor-in-interest of Francisco Catigbac, all to my damage and
prejudice.20 (Emphasis ours.)

Petitioner’s Complaint Affidavit gave rise to simultaneous administrative and preliminary (criminal)
investigations, docketed as OMB-L-A-03-0573-F and OMB-L-C-03-0728-F, respectively.

Petitioner pointed out several irregularities in the circumstances surrounding the alleged sale of Lot 1-
B to Summit Realty and in the documents evidencing the same.

The supposed Deed of Absolute Sale in favor of Summit Realty executed on 22 July 2002 by
Leonardo Yagin (Yagin), as Catigbac’s attorney-in-fact, appeared to be a "one-way street." It did not
express the desire of Summit Realty, as vendee, to purchase Lot 1-B or indicate its consent and
conformity to the terms of the Deed. No representative of Summit Realty signed the left margin of
each and every page of said Deed. It also did not appear from the Deed that a representative of
Summit Realty presented himself before the Notary Public who notarized the said document. The Tax
Identification Numbers of Yagin, as vendor, and Summit Realty, as vendee, were not stated in the
Deed.

Petitioner also averred that, being a corporation, Summit Realty could only act through its Board of
Directors. However, when the Deed of Absolute Sale of Lot 1-B was presented for recording before
the Register of Deeds, it was not accompanied by a Secretary’s Certificate attesting to the existence
of a Board Resolution which authorized said purchase by Summit Realty. There was no entry
regarding such a Secretary’s Certificate and/or Board Resolution, whether on TCT No. 129642 or
TCT No. T-134609. A Secretary’s Certificate eventually surfaced, but it was executed only on 30 July
2002, five days after TCT No. T-134609 in the name of Summit Realty was already issued.

The Deed of Absolute Sale was presented before and recorded by the Register of Deeds of Lipa City
on 25 July 2002 at 2:30 p.m., at exactly the same date and time TCT No. T-134609 was issued to
Summit Realty. Petitioner theorizes that for this to happen, TCT No. T-134609 was already prepared
and ready even before the presentation for recording of the Deed of Absolute Sale before the
Register of Deeds.

Moreover, Catigbac had long been dead and buried. The agency Catigbac supposedly executed in
favor of Yagin was extinguished by Catigbac’s death. Thus, petitioner argued, Yagin no longer had
authority to execute on 22 July 2002 the Deed of Absolute Sale of Lot 1-B in favor of Summit Realty,
making the said Deed null and void ab initio.

Petitioner asserted that Summit Realty was well-aware of Catigbac’s death, having acknowledged the
same in LRC Case No. 00-0376, the Petition for Issuance of New Owner’s Duplicate of TCT No. 181
In Lieu of Lost One, filed by Summit Realty before the Regional Trial Court (RTC) of Lipa City. During
the ex parte presentation of evidence in the latter part of 2000, Orense testified on behalf of Summit
Realty that Catigbac’s property used to form part of a bigger parcel of land, Lot 1 of Plan Psu-12014,
measuring 132,975 square meters, covered by TCT No. 181 in the name of Catigbac; after
Catigbac’s death, Lot 1 was informally subdivided into several parts among his heirs and/or
successors-in-interest, some of whom again transferred their shares to other persons; Summit Realty
separately bought subdivided parts of Lot 181 from their respective owners, with a consolidated area
of 105,648 square meters, and identified as Lot 1-B after survey; despite the subdivision and transfer
of ownership of Lot 1, TCT No. 181 covering the same was never cancelled; and the owner’s
duplicate of TCT No. 181 was lost and the fact of such loss was annotated at the back of the original
copy of TCT No. 181 with the Registry of Deeds. Subsequently, in an Order21 dated 3 January 2001,
the RTC granted the Petition in LRC Case No. 00-0376 and directed the issuance of a new owner’s
duplicate of TCT No. 181 in the name of Catigbac, under the same terms and condition as in its
original form.

Petitioner further cast doubt on the acts undertaken by Summit Realty in connection with Catigbac’s
property, purportedly without legal personality and capacity. The Special Power of Attorney dated 6
February 1976 granted Yagin the right to sue on behalf of Catigbac, yet it was Summit Realty which
instituted LRC Case No. 00-0376, and Yagin had no participation at all in said case. Likewise, it was
not Yagin, but Orense, who, through a letter22 dated 27 June 2001, requested the cancellation of TCT
No. 181 covering Lot 1 and the issuance of a new certificate of title for Lot 1-B. Hence, it was
Orense’s request which resulted in the issuance of TCT No. 129642 in the name of Catigbac, later
cancelled and replaced by TCT No. T-134609 in the name of Summit Realty.

Lastly, petitioner questioned why, despite the cancellation of TCT No. 129642 in the name of
Catigbac and the issuance in its place of TCT No. T-134609 in the name of Summit Realty, it was the
former cancelled title which was used as basis for canceling petitioner’s Tax Declaration No. 00942-
A. Tax Declaration No. 00949-A was thus still issued in the name of Catigbac, instead of Summit
Realty.

Piecing everything together, petitioner recounted in her Complaint Affidavit the alleged scheme
perpetrated against her and the involvement therein of each of the conspirators:
28. Summit Point Realty and Development Corporation went into action right after I paid Orense a
visit sometime May 2002. Summit resurrected from the grave. (sic) Francisco Catigbac whom they
knew to be long dead to face possible litigation. This is the height of malice and bad faith on the part
of Summit through its Lauro Leviste II, the Executive Vice President and Benedicto Orense, the Vice
President. I had only in my favor a tax declaration to show my interest and ownership over the 5, 000
sq.m. of the subject parcel of land. Evidently, Leviste and Orense came to the desperate conclusion
that they needed a TCT which is a far better title than any tax declaration.

Both then methodically commenced their evil and illegal scheme by causing on June 26, 2002 at
11:20 a.m. the inscription with the Register of Deeds of Lipa City of a purported Special Power of
Attorney in favor of Leonardo Yagin (Annex "I"). Next, the Deed of Absolute Sale (Annex "J") was
made the following month in order to make it appear that Yagin unilaterally sold to Summit the subject
parcel of land purportedly belonging to Francisco Catigbac. Since the latter was already dead and
realizing that the agency was already extinguished, Annex "J" was not signed or executed by Leviste
or Orense. This fact however did not deter the two from securing a BIR clearance on July 25, 2002.
Also, on this same day, July 25, 2002, Annex "J" was presented to Atty. [Escutin] at 2:30 p.m.
simultaneously, at exactly the same time of 2:30 p.m. TCT No. T-134609 in Summit’s name was
issued by Atty. [Escutin] WITHOUT benefit of the submission of the necessary documentation such
as the Board Resolution, DAR Clearance, Revenue Tax Receipts for documentary stamps, real
property tax clearance, proof of payment of transfer tax, tax declaration, articles of incorporation, SEC
certification, license to sell and/or certificate of registration by HLURB, etc. Without the total and
lightning speed cooperation of Atty. [Escutin] to close his eyes to the total absence of said vital
documents, the desperately needed TCT to erase my interest and ownership would not have come
into existence. Atty. [Escutin] had indeed acted in concert and in conspiracy with Leviste and Orense
in producing Annex "H" and Annex "K".

29. Thereafter, Leviste and Orense utilized the already cancelled TCT No. 129642 in the name of
Francisco Catigbac to be the basis in seeking the cancellation of TD #00942A in my name (Annex
"F"). The Tax Mapping Division of the Office of City Assessor of Lipa City opined that my 5,000 sq.m.
was (sic) part and parcel of the 105,648 sq.m. covered by TCT No. 129642. A photocopy of the
Certification from said division is hereto marked and attached as Annex "P", hereof. Aquilina Mistas,
the Local Assessment Operations Officer III of the Office of the City Assessor of Lipa City then
conveniently caused the disappearance of my Notice of Levy and other supporting documents which
she had personally received from me on March 13, 2002. For her part of the conspiracy likewise,
Marietta Linatoc, Records Clerk, forthwith cancelled by TD#00942-A and in lieu thereof she issued
TD #00949-A in the name of Francisco Catigbac. I dare say so because Mistas and Linatoc were
presented a cancelled TCT as basis for obliterating my 5,000 sq.m. The fact of cancellation is clearly
stated on the posterior side of TCT No. 129642. Both can read. But the two nevertheless proceeded
with dispatch in canceling my TD, though they had ample time and opportunity to reject the request of
Summit who is not even the registered owner appearing on TCT No. 129642. Francisco Catigbac
could not have been in front of Mistas and Linatoc because he was already six feet below the ground.
Mistas and Linatoc could have demanded presentation of the document authorizing Summit in
requesting for the cancellation of my TD. Also, they could have demanded from Summit any
document transferring my interest and ownership in favor of a third party. Or, at least, they could have
annotated in Tax Declaration No. 00949-A the fact that I bought my 5,000 sq.m. from a public auction
sale duly conducted by the court sheriff. Alternatively, Linatoc and Mistas should have advised
Summit to the effect that since they already appear to be the owners of the subject parcel of land, the
new tax declaration should bear their name instead. Mistas and Linatoc indeed conspired with
Summit in the illegal and unwarranted cancellation of my TD and in covering up the behind-the-
scenes activities of Summit by making it appear that it was Francisco Catigbac who caused the
cancellation. Even Leonardo Yagin, the alleged attorney-in-fact did not appear before Mistas and
Linatoc. Yagin could not have appeared because he is rumored to be long dead. The aforementioned
acts of the two benefitted (sic) Summit through their manifest partiality, evident bad faith and/or gross
inexcusable negligence. Perhaps, there is some truth to the rumor that Yagin is dead because he
does not even have a TIN in the questioned Deed of Absolute Sale. If indeed Yagin is already dead
or inexistent[,] the allged payment of the purchase price of P5,282,400.00 on July 25, 2002 is a mere
product of the fertile imagination of Orense and Leviste.1avvphi1.zw+ To dispute this assertion[,] the
live body of Leonardo Yagin must be presented by Orense and Leviste. 23

After filing her Affidavit Complaint, petitioner attempted to have the Sheriff’s Deed of Final
Sale/Conveyance of her 5,000 square meter pro-indiviso share in Lot 13713 registered with the
Register of Deeds of Lipa City. She also sought the annotation of her Affidavit of Adverse Claim on
the said 5,000 square meters on TCT No. T-134609 of Summit Realty.

Escutin, the Register of Deeds of Lipa City, relying on the finding of Examiner Juanita H. Sta. Ana
(Sta. Ana), refused to have the Sheriff’s Deed of Final Sale/Conveyance registered, since:

The Sheriff’s Deed of Final Sale/Conveyance is a Mode of Transfers (sic) ownership in favor of the
Plaintiff, [Dinah] C. Castillo, (sic) However[,] it happen (sic) that the presented Tax Declaration [No.]
00942-A is already transfer (sic) in the name of the said [Dinah] C. Castillo, therefore[,] the
registration of Sheriff (sic) Final Sale is no longer necessary.24

Escutin likewise denied petitioner’s request to have her Affidavit of Adverse Claim annotated on TCT
No. T-134609 on the following grounds:

1. The claimants (sic) rights or interest is not adverse to the registered owner. The registered
owner is Summit Point Realty and Development Corporation under Transfer Certificate of Title
No. T-134609 of the Registry of Deeds for Lipa City.

2. The records of the Registry reveals that the source of the rights or interest of the adverse
claimant is by virtue of a Levy on Execution by the Regional Trial Court Fourth Judicial Region,
Branch 30, San Pablo City, in Civil Case No. SP-4489 (1996), [Dinah] C. Castillo vs. Raquel
Buenaventura. The registered owner, Summit Point Realty and Development Corporation nor
its predecessor-in-interest are not the judgment debtor or a party in the said case. Simply
stated, there is no privity of contract between them (Consulta No. 1044 and 1119). If ever, her
adverse claim is against Raquel Buenaventura, the judgment debtor who holds no title over the
property.25

Escutin did mention, however, that petitioner may elevate en consulta to the Land Registration
Authority (LRA) the denial of her request for registration of the Sheriff’s Deed of Final
Sale/Conveyance and annotation of her adverse claim on TCT No. T-134609. This petitioner did on 3
July 2003.

While her Consulta was pending before the LRA, petitioner filed a Supplemental Complaint
Affidavit26 and a Second Supplemental Complaint Affidavit27 with the Office of the Deputy
Ombudsman for Luzon, bringing to its attention the aforementioned developments. In her Second
Supplemental Complaint Affidavit, petitioner prayed that Sta. Ana be included as a co-respondent in
OMB-L-A-03-0573-F and OMB-L-C-03-0728-F, averring that the latter’s actuation deprived petitioner
of a factual basis for securing a new title in her favor over her 5,000 square meter pro-indiviso share
in Lot 13713, because the public auction sale of the said property to her could never become final
without the registration of the Sheriff’s Deed.

The persons charged in OMB-L-A-03-0573-F and OMB-L-C-03-0728-F filed their respective Counter-
Affidavits.
Respondent Escutin clarified in his Counter Affidavit that TCT No. T-134609 reflected the same date
and time of entry of the Deed of Absolute Sale between Yagin (as Catigbac’s attorney-in-fact) and
Summit Realty, i.e., 25 July 2002 at 2:30 p.m., in accordance with Section 56 28 of Presidential Decree
No. 1529, otherwise known as the Property Registration Decree. He emphasized that his duty as
Register of Deeds to register the Deed of Absolute Sale presented before him was purely ministerial.
If the document was legal and in due form, and there was nothing mutilated or irregular on its face,
the Register of Deeds had no authority to inquire into its intrinsic validity based upon proofs aliunde. It
was not true that he allowed the registration of the Deed of Absolute Sale notwithstanding the
absence of the required documents supporting the application for registration thereof. On the
contrary, all the required documents such as the DAR Clearance, Bureau of Internal Revenue (BIR)
Certificate Authorizing Registration (CAR), Real Property Tax, Transfer Tax, Secretary’s Certificate
and Articles of Incorporation of Summit Realty were submitted. While it was true that the Secretary’s
Certificate did not accompany the Deed of Absolute Sale upon the presentation of the latter for
registration, Section 117 of the Property Registration Decree gives the party seeking registration five
days to comply with the rest of the requirements; and only if the party should still fail to submit the
same would it result in the denial of the registration. The License to Sell and the Housing and Land
Use Regulatory Board Registration of Summit Realty are only required when a subdivision project is
presented for registration. The use of TINs in certain documents is a BIR requirement. The BIR itself
did not require from Yagin as vendor his TIN in the Deed of Absolute Sale, and issued the CAR even
in the absence thereof. The Register of Deeds, therefore, was only bound by the CAR. As to the
Certification earlier issued by the Register of Deeds of Lipa City attesting that Lot 13713 in the name
of co-owners Raquel, Urbana, and Perla, was not covered by any certificate of title, Escutin explained
that the Register of Deeds was not technically equipped to determine whether a cadastral lot number
was within a titled property or not. Lastly, Escutin denied conspiring or participating in the cancellation
of petitioner’s Tax Declaration No. 00942-A for, as Register of Deeds, he was not concerned with the
issuance (or cancellation) of tax declarations.

Respondent Mistas, the Assistant City Assessor for Administration of the Office of the City Assessor,
Lipa City, disputed petitioner’s allegations that she personally received from petitioner copies of the
Notice of Levy and other supporting documents, and that she caused the disappearance thereof.
Although she admitted that said documents were shown to her by petitioner, she referred petitioner to
the Receiving Clerk, Lynie Reyes, who accordingly received the same. Mistas maintained that she
was not the custodian of records of the Office and she should not be held responsible for the missing
documents. She opined that petitioner’s documents could have been among those misplaced or
destroyed when the Office of the City Assessor was flooded with water leaking from the toilet of the
Office of the City Mayor. As Assistant City Assessor for Administration, Mistas identified her main
function to be the control and management of all phases of administrative matters and support. She
had no hand in the cancellation of petitioner’s Tax Declaration No. 00942-A, and the issuance of
Catigbac’s Tax Declaration No. 00949-A for such function pertained to another division over which
she did not exercise authority. Thus, it was also not within her function or authority to demand the
presentation of certain documents to support the cancellation of petitioner’s Tax Declaration No.
00942-A or to cause the annotation of petitioner’s interest on Catigbac’s Tax Declaration No. 00949-
A.

Respondent Linatoc averred that as Local Assessment Operation Officer II of the Office of the City
Assessor, Lipa City, she was in charge of safekeeping and updating the North District Records. With
respect to the transfer of a tax declaration from one name to another, her duty was limited only to the
act of preparing the new tax declaration and assigning it a number, in lieu of the cancelled tax
declaration. It was a purely ministerial duty. She had no authority to demand the presentation of any
document or question the validity of the transfer. Neither was it within her jurisdiction to determine
whether petitioner’s interest should have been annotated on Catigbac’s Tax Declaration No. 00949-A.
Examining the documents presented in support of the transfer of the tax declaration to another’s
name was a function belonging to other divisions of the Office of the City Assessors. The flow of
work, the same as in any other ordinary transaction, mandated her to cancel petitioner’s Tax
Declaration No. 00942-A, and to prepare and release Catigbac’s Tax Declaration No. 00949-A after
the transfer had been reviewed and approved by other divisions of the Office. It was also not true that
TCT No. 129642 in the name of Catigbac was already cancelled when it was presented before the
Office of the City Assessors; the photocopy of said certificate of title with the Office bore no mark of
cancellation.

Leviste and Orense, the private individuals charged with the respondent public officers, admitted that
they were corporate officers of Summit Realty. They related that Summit Realty bought a parcel of
land measuring 105,648 square meters, later identified as Lot 1-B, previously included in TCT No.
181, then specifically covered by TCT No. 129642, both in the name of Catigbac. As a result of such
purchase, ownership of Lot 1-B was transferred from Catigbac to Summit Realty. Summit Realty had
every reason to believe in good faith that said property was indeed owned by Catigbac on the basis of
the latter’s certificate of title over the same. Catigbac’s right as registered owner of Lot 1-B under TCT
No. 181/No. 129642, was superior to petitioner’s, which was based on a mere tax declaration. Leviste
and Orense rebutted petitioner’s assertion that the Deed of Absolute Sale between Yagin, as
Catigbac’s attorney-in-fact, and Summit Realty was a "one-way street." The Deed was actually signed
on the left margin by both Yagin and the representative of Summit Realty. The inadvertent failure of
the representative of Summit Realty to sign the last page of the Deed and of both parties to indicate
their TINs therein did not invalidate the sale, especially since the Deed was signed by witnesses
attesting to its due execution. Questions as regards the scope of Catigbac’s Special Power of
Attorney in favor of Yagin and the effectivity of the same after Catigbac’s death can only be raised in
an action directly attacking the title of Summit Realty over Lot 1-B, and not in an administrative case
and/or preliminary investigation before the Ombudsman, which constituted a collateral attack against
said title. Leviste and Orense further explained that since the owner’s duplicate of TCT No. 181 was
lost and was judicially ordered replaced only on 3 January 2001, entries/inscriptions were necessarily
made thereon after said date. As to Orense’s failure to show petitioner any document proving
ownership of Lot 1-B by Summit Realty when the latter paid him a visit, it was not due to the lack of
such documents, but because of petitioner’s failure to establish her right to peruse the same. Orense
also denied ever threatening petitioner during their meeting. Finally, according to Leviste and Orense,
petitioner’s allegations were based on mere conjectures and unsupported by evidence. That
particular acts were done or not done by certain public officials was already beyond the control of
Leviste and Orense, and just because they benefited from these acts did not mean that they had a
hand in the commission or omission of said public officials.

After more exchange of pleadings, OMB-L-A-03-0573-F and OMB-L-C-03-0728-F were finally


submitted for resolution.

In a Joint Resolution29 dated 28 April 2004, the Office of the Deputy Ombudsman for Luzon gave
more credence to respondent Escutin’s defenses, as opposed to petitioner’s charges against him:

Going to the charges against respondent Escutin, he convincingly explained that he allowed the
registration of the allegedly defective Deed of Sale because he, as Register of Deeds, has no power
to look into the intrinsic validity [of] the contract presented to him for registration, owing to the
ministerial character of his function. Moreover, as sufficiently explained by said respondent, all the
documents required for the registration of the Deed of Sale were submitted by the applicant.

We likewise find said respondent’s explanation satisfactory that Section 56 of P.D. 1529 mandates
that the TCT bear the date of registration of the instrument on which the said TCT’s issuance was
based. It is for this reason that TCT 134609 bears the same date and time as the registration of the
Deed of Absolute Sale, which deed served as basis for its issuance.
As to his denial to register [herein petitioner’s] Affidavit of Adverse Claim and Sheriff’s Certificate of
Final Sale, through the issuance by the Registry of Deeds Examiner Juanita H. Sta. Ana, of the 29
June 2003 Order denying registration thereof, such matter had been raised by herein [petitioner] in a
letter-consulta to the Administrator of the Land Registration Authority (LRA) on 03 July 2003. As the
criminal and administrative charges respecting this issue is premised, in part, on a matter still pending
with the LRA, we find it premature to make a finding on the same.

It is for the same reason that we deny the motion contained in the Second Supplemental Complaint
Affidavit praying for the inclusion, as additional respondent, of Juanita H. Sta. Ana, who is impleaded
solely on the basis of having signed, by authority of Escutin, the 29 July 2003 Order of denial of
[petitioner’s] application for registration.

Finally, respondent Escutin was able to successfully demonstrate, through Consulta 2103 dated 25
July 1994, wherein the denial of registration by the Examiner of the Registry of Deeds of Quezon City
was upheld by the LRA Administrator, that the (sic) it was practice in the different Registries that
Examiners are given authority by the Register to sign letters of denial. 30

The Office of the Deputy Ombudsman for Luzon declared in the same Joint Resolution that there was
no basis to hold respondents Mistas and Linatoc administratively or criminally liable:

In this respect, this Office notes that while [herein petitioner] alleges that Aquilina Mistas caused the
disappearance of the Notice of Levy and other supporting documents received from [petitioner] on 13
March 2003 when she applied for the issuance of a Tax Declaration in her favor, she did not present
her receiving copy thereof showing that it was Mistas who received said documents from her. Neither
did she show that Mistas is the employee responsible for record safekeeping.

Next, we find, as convincingly answered, the allegation that respondent Marietta Linatoc cancelled
Tax Declaration No. 00942-A and issued Tax Declaration 00949-Q (sic) on the basis of a cancelled
Transfer Certificate of Title upon the behest of Summit [Realty], which was not the registered owner
of the property.

Respondent Linatoc, meeting squarely [petitioner’s] allegation, admits having physically cancelled
Tax Declaration No. 00942-A and having prepared a new declaration covering the same property in
Catigbac’s [name], as mandated by the flow of work in the City Assessor’s Office. However, she
denies having the authority or discretion to evaluate the correctness and sufficiency of the documents
supporting the application for the issuance of the Tax Declaration, arguing that her official function is
limited to the physical preparation of a new tax declaration, the assignment of a new tax declaration
number and the cancellation of the old tax declaration, after the application had passed the other
divisions of the City Assessor’s Office.

Verily, [petitioner] failed to establish that respondent Mistas and Linatoc, are the ones officially
designated to receive applications for issuance of Tax Declaration, evaluate the sufficiency of the
documents supporting such applications, and on the basis of the foregoing recommend or order the
cancellation of an existing Tax Declaration and direct the annotation of any fact affecting the property
and direct the issuance of a new tax declaration covering the same property.

In fact, there is even a discrepancy as to the official designation of said respondents. While
[petitioner] impleads Mistas, in her capacity as Local Assessment Officer, and Linatoc, in her capacity
as Records Clerk, Mistas, in her counter-affidavit, alleges a different designation, i.e., Assistant City
Assessor for Administration, while Linatoc claims to be the Local Assessment Operation Officer II of
the City Assessor’s Office.
With the scope of work of said respondents not having been neatly defined by [petitioner], this Office
cannot make a definitive determination of their liability for Grave Misconduct and violation of Section
3(e) of R.A. No. 3019, which charges both relate to the performance or discharge of Mistas’ and
Linatoc’s official duties.31

Neither did the Office of the Deputy Ombudsman for Luzon find any probable cause to criminally
charge private individuals Leviste and Orense for the following reasons:

Anent private respondents, with the alleged conspiracy to unlawfully cause the transfer of the title of
[herein petitioner’s] property to Summit sufficiently explained by respondent Register of Deeds, such
allegation against private respondents loses a legal leg to stand on.1avvphi.zw+

Inasmuch as [petitioner] was not able to sufficiently outline the official functions of respondents Mistas
and Linatoc to pin down their specific accountabilities, the imputation that private respondent (sic)
conspired with said public respondents respecting the cancellation of Tax Declaration No. 00942-A is
likewise stripped of any factual and legal bases.32

As to whether petitioner was indeed unlawfully deprived of her 5,000 square meter property, which
issue comprised the very premise of OMB-L-A-03-0573-F and OMB-L-C-03-0728-F, the Office of the
Deputy Ombudsman for Luzon ruled that such matter was not within its jurisdiction and should be
raised in a civil action before the courts of justice.

In the end, the Office of the Ombudsman decreed:

WHEREFORE premises considered, it is respectfully recommended that : (1) the administrative case
against public respondents ANTONIO M. ESCUTIN, AQUILINA A. MISTAS and MARIETA L.
LINATOC be DISMISSED, for lack of substantial evidence; and (2) the criminal case against the
same respondents including private respondent LAURO S. LEVISTE II and BENEDICTO L.
ORENSE, be DISMISSED, for lack of probable cause.33

In a Joint Order34 dated 20 June 2005, the Office of the Deputy Ombudsman for Luzon denied
petitioner’s Motion for Reconsideration.

The Office of the Deputy Ombudsman for Luzon, in its Joint Order, took notice of the Resolution
dated 17 December 2002 of the LRA in Consulta No. 3483, which involved circumstances similar to
those in petitioner’s case. The LRA distinguished between two systems of land registration: one is the
Torrens system for registered lands under the Property Registration Decree, and the other is the
system of registration for unregistered land under Act No. 3344 (now Section 113 of the Property
Registration Decree). These systems are separate and distinct from each other. For documents
involving registered lands, the same should be recorded under the Property Registration Decree. The
registration, therefore, of an instrument under the wrong system produces no legal effect. Since it
appeared that in Consulta No. 3483, the registration of the Kasulatan ng Sanglaan, the Certificate of
Sale and the Affidavit of Consolidation was made under Act No. 3344, it did not produce any legal
effect on the disputed property, because the said property was already titled when the
aforementioned documents were executed and presented for registration, and their registration
should have been made under the Property Registration Decree.

Furthermore, the Office of the Deputy Ombudsman for Luzon, in the same Joint Order, took into
account petitioner’s withdrawal of her appeal en consulta before the LRA of the denial by the Register
of Deeds of her request for registration of the Sheriff’s Deed of Final Sale/Conveyance and Affidavit
of Adverse Claim, which prompted the LRA Administrator to declare the consulta moot and academic.
For want of a categorical declaration on the registerability of petitioner’s documents from the LRA, the
competent authority to rule on the said matter, there could be no basis for a finding that respondent
public officers could be held administratively or criminally liable for the acts imputed to them.

Petitioner sought recourse from the Court of Appeals by filing a Petition for Review under Rule 43 of
the Rules of Court challenging the 28 April 2004 Joint Resolution and 20 June 2005 Joint Order of the
Office of the Deputy Ombudsman for Luzon.35 The appeal was docketed as CA-G.R. SP No.
90533.1avvphi1

The Court of Appeals promulgated its Decision36 on 18 October 2005, also finding no reason to
administratively or criminally charge respondents. Essentially, the appellate court adjudged that
petitioner can not impute corrupt motives to respondents’ acts:

Without evidence showing that respondents received any gift, money or other pay-off or that they
were induced by offers of such, the Court cannot impute any taint of direct corruption in the
questioned acts of respondents. Thus, any indication of intent to violate the laws or of flagrant
disregard of established rule may be negated by respondents’ honest belief that their acts were
sanctioned under the provisions of existing law and regulations. Such is the situation in the case at
bar. Respondent Register of Deeds acted in the honest belief that the agency recognized by the court
in LRC Case No. 00-0376 between the registered owner Francisco Catigbac and Leonardo Yagin
subsisted with respect to the conveyance or sale of Lot 1 to Summit as the vendee, and that the
Special Power of Attorney and Deed of Absolute Sale presented as evidence during said proceedings
are valid and binding. Hence, respondent Escutin was justified in believing that there is no legal
infirmity or defect in registering the documents and proceeding with the transfer of title of Lot 1 in the
name of the new owner Summit. On the other hand, respondent Linatoc could not be held
administratively liable for effecting the cancellation in the course of ordinary flow of work in the City
Assessor’s Office after the documents have undergone the necessary evaluation and verification by
her superiors.37

The Court of Appeals referred to the consistent policy of the Supreme Court not to interfere with the
exercise by the Ombudsman of his investigatory power. If the Ombudsman, using professional
judgment, finds the case dismissible, the Court shall respect such findings, unless clothed with grave
abuse of discretion. The appellate court pronounced that there was no grave abuse of discretion on
the part of the Office of the Deputy Ombudsman for Luzon in dismissing petitioner’s Complaint
Affidavit against respondents.

Hence, the dispositive portion of the Decision of the Court of Appeals reads:

WHEREFORE, premises considered, the present petition is hereby DISMISSED for lack of merit. The
challenged Joint Resolution dated April 28, 2004 and Joint Order dated June 20, 2005 in OMB-L-A-
03-0573-F and OMB-L-C-03-0728-F are hereby AFFIRMED.38

In its Resolution dated 11 January 2006, the Court of Appeals denied petitioner’s Motion for
Reconsideration for failing to present new matter which the appellate court had not already
considered in its earlier Decision.

Petitioner now comes before this Court via the instant Petition for Review on Certiorari, with the
following assignment of errors:

I.

THE HONORABLE COURT OF APPEALS PATENTLY ERRED IN AFFIRMING THE


CANCELLATION OF THE TAX DECLARATION 00942 OF PETITIONER IN VIOLATION OF
SECTION 109 OF PRESIDENTIAL DECREE 1529, OTHERWISE KNOWN AS THE PROPERTY
REGISTRATION ACT (sic);

II.

THE HONORABLE COURT OF APPEALS PATENTLY ERRED IN RULING THAT RESPONDENTS


COULD NOT BE HELD ADMINISTRATIVELY LIABLE FOR UNDULY FAVORING SUMMIT TO THE
DAMAGE AND PREJUDICE OF PETITIONER.39

The Petition at bar is without merit.

As to the first issue, petitioner invokes Section 109 of the Property, Registration Decree which
provides:

SEC. 109. Notice and replacement of lost duplicate certificate. – In case of loss or theft of an owner’s
duplicate certificate of title, due notice under oath shall be sent by the owner or by someone in his
behalf to the Register of Deeds of the province or city where the land lies as soon as the loss or theft
is discovered. If a duplicate certificate is lost or destroyed, or cannot be produced by a person
applying for the entry of a new certificate to him or for the registration of any new instrument, a sworn
statement of the fact of such loss or destruction may be filed by the registered owner or other person
in interest and registered.

Upon the petition of the registered owner or other person in interest, the court may, after notice and
due hearing, direct the issuance of a new duplicate certificate, which shall contain a memorandum of
the fact that it is issued in place of the lost duplicate certificate, but shall in all respects be entitled to
like faith and credit as the original duplicate, and shall thereafter be regarded as such for all purposes
of this decree.

Petitioner argues that the RTC, in LRC Case No. 00-0376, only ordered the issuance of a new
owner’s duplicate of TCT No. 181 in lieu of the lost one. However, respondents did not only issue a
new owner’s duplicate of TCT No. 181, but also cancelled petitioner’s Tax Declaration No. 00942-A
and issued in its place Tax Declaration No. 00949-A in the name of Catigbac. Respondents did not
even annotate petitioner’s existing right over 5,000 square meters of Lot 1-B or notify petitioner of the
cancellation of her Tax Declaration No. 00942-A. Petitioner maintains that a new owner’s duplicate of
title is not a mode of acquiring ownership, nor is it a mode of losing one. Under Section 109 of the
Property Registration Decree, the new duplicate of title was issued only to replace the old; it cannot
cancel existing titles.

Petitioner’s position on this issue rests on extremely tenuous arguments and befuddled reasoning.

Before anything else, the Court must clarify that a title is different from a certificate of title. Title is
generally defined as the lawful cause or ground of possessing that which is ours. It is that which is the
foundation of ownership of property, real or personal.40 Title, therefore, may be defined briefly as that
which constitutes a just cause of exclusive possession, or which is the foundation of ownership of
property.41 Certificate of title, on the other hand, is a mere evidence of ownership; it is not the title to
the land itself.42 Under the Torrens system, a certificate of title may be an Original Certificate of Title,
which constitutes a true copy of the decree of registration; or a Transfer Certificate of Title, issued
subsequent to the original registration.

Summit Realty acquired its title to Lot 1-B, not from the issuance of the new owner’s duplicate of TCT
No. 181, but from its purchase of the same from Yagin, the attorney-in-fact of Catigbac, the registered
owner of the said property. Summit Realty merely sought the issuance of a new owner’s duplicate of
TCT No. 181 in the name of Catigbac so that it could accordingly register thereon the sale in its favor
of a substantial portion of Lot 1 covered by said certificate, later identified as Lot 1-B. Catigbac’s title
to Lot 1-B passed on by sale to Summit Realty, giving the latter the right to seek the separation of the
said portion from the rest of Lot 1 and the issuance of a certificate of title specifically covering the
same. This resulted in the issuance of TCT No. 129642 in the name of Catigbac, covering Lot 1-B,
which was subsequently cancelled and replaced by TCT No. T-134609 in the name of Summit Realty.

Petitioner’s reliance on Section 109 of the Property Registration Decree is totally misplaced. It
provides for the requirements for the issuance of a lost duplicate certificate of title. It cannot, in any
way, be related to the cancellation of petitioner’s tax declaration.

The cancellation of petitioner’s Tax Declaration No. 00942-A was not because of the issuance of a
new owner’s duplicate of TCT No. 181, but of the fact that Lot 1-B, which encompassed the 5,000
square meters petitioner lays claim to, was already covered by TCT No. 181 (and subsequently by
TCT No. 129642) in the name of Catigbac. A certificate of title issued is an absolute and indefeasible
evidence of ownership of the property in favor of the person whose name appears therein. It is
binding and conclusive upon the whole world.43 All persons must take notice, and no one can plead
ignorance of the registration.44 Therefore, upon presentation of TCT No. 129642, the Office of the City
Assessor must recognize the ownership of Lot 1-B by Catigbac and issue in his name a tax
declaration for the said property. And since Lot 1-B is already covered by a tax declaration in the
name of Catigbac, accordingly, any other tax declaration for the same property or portion thereof in
the name of another person, not supported by any certificate of title, such that of petitioner, must be
cancelled; otherwise, the City Assessor would be twice collecting a realty tax from different persons
on one and the same property.

As between Catigbac’s title, covered by a certificate of title, and petitioner’s title, evidenced only by a
tax declaration, the former is evidently far superior and is, in the absence of any other certificate of
title to the same property, conclusive and indefeasible as to Catigbac’s ownership of Lot 1-B.
Catigbac’s certificate of title is binding upon the whole world, including respondent public officers and
even petitioner herself. Time and again, the Court has ruled that tax declarations and corresponding
tax receipts cannot be used to prove title to or ownership of a real property inasmuch as they are not
conclusive evidence of the same.45 Petitioner acquired her title to the 5,000 square meter property
from Raquel, her judgment debtor who, it is important to note, likewise only had a tax declaration to
evidence her title. In addition, the Court of Appeals aptly observed that, "[c]uriously, as to how and
when petitioner’s alleged predecessor-in-interest, Raquel K. Moratilla and her supposed co-owners
acquired portions of Lot 1 described as Lot 13713 stated in TD No. 00449, petitioner had so far
remained utterly silent."46

Petitioner’s allegations of defects or irregularities in the sale of Lot 1-B to Summit Realty by Yagin, as
Catigbac’s attorney-in-fact, are beyond the jurisdiction of the Office of the Deputy Ombudsman for
Luzon to consider. It must be remembered that Summit Realty had already acquired a certificate of
title, TCT No. T-134609, in its name over Lot 1-B, which constitutes conclusive and indefeasible
evidence of its ownership of the said property and, thus, cannot be collaterally attacked in the
administrative and preliminary investigations conducted by the Office of the Ombudsman for Luzon.
Section 48 of the Property Registration Decree categorically provides that a certificate of title shall not
be subject to collateral attack. It cannot be altered, modified, or cancelled except in a direct
proceeding in accordance with law. For this same reason, the Court has no jurisdiction to grant
petitioner’s prayer in the instant Petition for the cancellation of TCT No. T-134609 in the name of
Summit Realty.

Which now brings the Court to the second issue raised by petitioner on the administrative liability of
respondents.
Before the Court proceeds to tackle this issue, it establishes that petitioner’s Complaint Affidavit
before the Office of the Ombudsman for Luzon gave rise to two charges: (1) OMB-L-A-03-0573-F
involved the administrative charge for Gross Misconduct against respondent public officers; and (2)
OMB-L-C-03-0728-F concerned the criminal charge for violation of Section 3(e) of the Anti-Graft and
Corrupt Practices Act47 against respondent public officers and private individuals Leviste and Orense.
The Office of the Deputy Ombudsman for Luzon, affirmed by the Court of Appeals, dismissed both
charges. In the Petition at bar, petitioner only assails the dismissal of the administrative charge for
grave misconduct against respondent public officers. Since petitioner did not raise as an issue herein
the dismissal by the Office of the Deputy Ombudsman for Luzon, affirmed by the Court of Appeals, of
the criminal charge against respondent public officers for violation of Section 3(e) of the Anti-Graft
and Corrupt Practices Act, the same became final and executory. 48

In Domingo v. Quimson,49 the Court adopted the well-written report and recommendation of its Clerk
of Court on the administrative matter then pending and involving the charge of gross or serious
misconduct:

"Under Section 36, par. (b) [1] of PD No. 807, otherwise known as the Civil Service Decree of the
Philippines, 'misconduct' is a ground for disciplinary action. And under MC No. 8, S. 1970, issued by
the Civil Service Commission on July 28, 1970, which sets the 'Guidelines in the Application of
Penalties in Administrative Cases and other Matters Relative Thereto,' the administrative offense of
'grave misconduct' carries with it the maximum penalty of dismissal from the service (Sec. IV-C[3],
MC No. 8, S. 1970). But the term 'misconduct' as an administrative offense has a well defined
meaning. It was defined in Amosco vs. Judge Magno, Adm. Mat. No. 439-MJ, Res. September 30,
1976, as referring 'to a transgression of some established and definite rule of action, more
particularly, unlawful behavior or gross negligence by the public officer.' It is a misconduct 'such as
affects the performance of his duties as an officer and not such only as effects his character as a
private individual.' In the recent case of Oao vs. Pabato, etc., Adm. Mat. No. 782-MJ, Res. July 29,
1977, the Court defined 'serious misconduct' as follows:

‘Hence, even assuming that the dismissal of the case is erroneous, this would be merely an error of
judgment and not serious misconduct. The term `serious misconduct’ is a transgression of some
established and definite rule of action more particularly, unlawful behavior of gross negligence by the
magistrate. It implies a wrongful intention and not a mere error of judgment. For serious misconduct
to exist, there must be reliable evidence showing that the judicial acts complained of were corrupt or
inspired by intention to violate the law, or were a persistent disregard of well-known legal rules. We
have previously ruled that negligence and ignorance on the part of a judge are inexcusable if they
imply a manifest injustice which cannot be explained by a reasonable interpretation. This is not so in
the case at bar.’" (Italics supplied.)

To reiterate, for grave misconduct to exist, there must be reliable evidence showing that the acts
complained of were corrupt or inspired by an intention to violate the law, or were a persistent
disregard of well-known legal rules. Both the Office of the Deputy Ombudsman for Luzon and the
Court of Appeals found that there was no sufficient evidence to substantiate petitioner’s charge of
grave misconduct against respondents. For this Court to reverse the rulings of the Office of the
Deputy Ombudsman for Luzon and the Court of Appeals, it must necessarily review the evidence
presented by the parties and decide on a question of fact. Once it is clear that the issue invites a
review of the evidence presented, the question posed is one of fact. 50

Factual issues are not cognizable by this Court in a Petition for Review under Rule 45 of the Rules of
Court. In order to resolve this issue, the Court would necessarily have to look into the probative value
of the evidence presented in the proceedings below. It is not the function of the Court to reexamine or
reevaluate the evidence all over again. This Court is not a trier of facts, its jurisdiction in these cases
being limited to reviewing only errors of law that may have been committed by the lower courts or
administrative bodies performing quasi-judicial functions. It should be emphasized that findings made
by an administrative body, which has acquired expertise, are accorded not only respect but even
finality by the Court. In administrative proceedings, the quantum of evidence required is only
substantial.51

Absent a clear showing of grave abuse of discretion, the Court shall not disturb findings of fact. The
Court cannot weigh once more the evidence submitted, not only before the Ombudsman, but also
before the Court of Appeals. Under Section 27 of Republic Act No. 6770, findings of fact by the
Ombudsman are conclusive, as long as they are supported by substantial evidence.52 Substantial
evidence is the amount of relevant evidence which a reasonable mind might accept as adequate to
justify a conclusion.53

The Court finds no reason to disturb the finding of the Office of the Deputy Ombudsman for Luzon
and the Court of Appeals that respondents did not commit gross misconduct. Evident from the 28
April 2004 Joint Resolution of the former and the 18 October 2005 Decision of the latter is that they
arrived at such findings only after a meticulous consideration of the evidence submitted by the
parties.

Respondents were able to clearly describe their official functions and to convincingly explain that they
had only acted in accordance therewith in their dealings with petitioner and/or her documents.
Respondents also enjoy in their favor the presumption of regularity in the performance of their official
duty. The burden of proving otherwise by substantial evidence falls on petitioner, who failed to
discharge the same.

From the very beginning, petitioner was unable to identify correctly the positions held by respondents
Mistas and Linatoc at the Office of the City Assessor. How then could she even assert that a
particular action was within or without their jurisdiction to perform? While it may be true that petitioner
should have at least been notified that her Tax Declaration No. 00942-A was being cancelled, she
was not able to establish that such would be the responsibility of respondents Mistas or Linatoc.
Moreover, petitioner did not present statutory, regulatory, or procedural basis for her insistence that
respondents should have done or not done a particular act. A perfect example was her assertion that
respondents Mistas and Linatoc should have annotated her interest on Tax Declaration No. 00949-A
in the name of Catigbac. However, she failed to cite any law or rule which authorizes or recognizes
the annotation of an adverse interest on a tax declaration. Finally, absent any reliable evidence,
petitioner’s charge that respondents conspired with one another and with corporate officers of Summit
Realty is nothing more than speculation, surmise, or conjecture. Just because the acts of
respondents were consistently favorable to Summit Realty does not mean that there was a concerted
effort to cause petitioner prejudice. Respondents’ actions were only consistent with the recognition of
the title of Catigbac over Lot 1-B, transferred by sale to Summit Realty, registered under the Torrens
system, and accordingly evidenced by certificates of title.

WHEREFORE, premises considered, the instant Petition for Review is hereby DENIED. The Decision
dated 18 October 2005 and Resolution dated 11 January 2006 of the Court of Appeals in CA-G.R. SP
No. 90533 are hereby AFFIRMED in toto. Costs against the petitioner Dinah C. Castillo.

SO ORDERED.

G.R. No. L-8936 October 2, 1915


CONSUELO LEGARDA, with her husband MAURO PRIETO, plaintiffs-appellants,
vs.
N.M. SALEEBY, defendant-appellee.

Singson, Ledesma and Lim for appellants.


D.R. Williams for appellee.

JOHNSON, J.:

From the record the following facts appear:

First. That the plaintiffs and the defendant occupy, as owners, adjoining lots in the district of Ermita in
the city of Manila.

Second. That there exists and has existed a number of years a stone wall between the said lots. Said
wall is located on the lot of the plaintiffs.

Third. That the plaintiffs, on the 2d day of March, 1906, presented a petition in the Court of Land
Registration for the registration of their lot. After a consideration of said petition the court, on the 25th
day of October, 1906, decreed that the title of the plaintiffs should be registered and issued to them
the original certificate provided for under the torrens system. Said registration and certificate included
the wall.

Fourth. Later the predecessor of the defendant presented a petition in the Court of Land Registration
for the registration of the lot now occupied by him. On the 25th day of March, 1912, the court decreed
the registration of said title and issued the original certificate provided for under the torrens system.
The description of the lot given in the petition of the defendant also included said wall.

Fifth. Several months later (the 13th day of December, 1912) the plaintiffs discovered that the wall
which had been included in the certificate granted to them had also been included in the certificate
granted to the defendant .They immediately presented a petition in the Court of Land Registration for
an adjustment and correction of the error committed by including said wall in the registered title of
each of said parties. The lower court however, without notice to the defendant, denied said petition
upon the theory that, during the pendency of the petition for the registration of the defendant's land,
they failed to make any objection to the registration of said lot, including the wall, in the name of the
defendant.

Sixth. That the land occupied by t he wall is registered in the name of each of the owners of the
adjoining lots. The wall is not a joint wall.

Under these facts, who is the owner of the wall and the land occupied by it?

The decision of the lower court is based upon the theory that the action for the registration of the lot of
the defendant was a judicial proceeding and that the judgment or decree was binding upon all parties
who did not appear and oppose it. In other words, by reason of the fact that the plaintiffs had not
opposed the registration of that part of the lot on which the wall was situate they had lost it, even
though it had been theretofore registered in their name. Granting that theory to be correct one, and
granting even that the wall and the land occupied by it, in fact, belonged to the defendant and his
predecessors, then the same theory should be applied to the defendant himself. Applying that theory
to him, he had already lost whatever right he had therein, by permitting the plaintiffs to have the same
registered in their name, more than six years before. Having thus lost hid right, may he be permitted
to regain it by simply including it in a petition for registration? The plaintiffs having secured the
registration of their lot, including the wall, were they obliged to constantly be on the alert and to watch
all the proceedings in the land court to see that some one else was not having all, or a portion of the
same, registered? If that question is to be answered in the affirmative, then the whole scheme and
purpose of the torrens system of land registration must fail. The real purpose of that system is to quiet
title to land; to put a stop forever to any question of the legality of the title, except claims which were
noted at the time of registration, in the certificate, or which may arise subsequent thereto. That being
the purpose of the law, it would seem that once a title is registered the owner may rest secure,
without the necessity of waiting in the portals of the court, or sitting in the "mirador de su casa," to
avoid the possibility of losing his land. Of course, it can not be denied that the proceeding for the
registration of land under the torrens system is judicial (Escueta vs. .Director of Lands, 16 Phil. Rep.,
482). It is clothed with all the forms of an action and the result is final and binding upon all the world. It
is an action in rem. (Escueta vs. Director of Lands (supra); Grey Alba vs. De la Cruz, 17 Phil. rep., 49
Roxas vs. Enriquez, 29 Phil. Rep., 31; Tyler vs. Judges, 175 Mass., 51 American Land Co. vs. Zeiss,
219 U.S., 47.)

While the proceeding is judicial, it involves more in its consequences than does an ordinary action. All
the world are parties, including the government. After the registration is complete and final and there
exists no fraud, there are no innocent third parties who may claim an interest. The rights of all the
world are foreclosed by the decree of registration. The government itself assumes the burden of
giving notice to all parties. To permit persons who are parties in the registration proceeding (and they
are all the world) to again litigate the same questions, and to again cast doubt upon the validity of the
registered title, would destroy the very purpose and intent of the law. The registration, under the
torrens system, does not give the owner any better title than he had. If he does not already have a
perfect title, he can not have it registered. Fee simple titles only may be registered. The certificate of
registration accumulates in open document a precise and correct statement of the exact status of the
fee held by its owner. The certificate, in the absence of fraud, is the evidence of title and shows
exactly the real interest of its owner. The title once registered, with very few exceptions, should not
thereafter be impugned, altered, changed, modified, enlarged, or diminished, except in some direct
proceeding permitted by law. Otherwise all security in registered titles would be lost. A registered title
can not be altered, modified, enlarged, or diminished in a collateral proceeding and not even by a
direct proceeding, after the lapse of the period prescribed by law.

For the difficulty involved in the present case the Act (No. 496) providing for the registration of titles
under the torrens system affords us no remedy. There is no provision in said Act giving the parties
relief under conditions like the present. There is nothing in the Act which indicates who should be the
owner of land which has been registered in the name of two different persons.

The rule, we think, is well settled that the decree ordering the registration of a particular parcel of land
is a bar to future litigation over the same between the same parties .In view of the fact that all the
world are parties, it must follow that future litigation over the title is forever barred; there can be no
persons who are not parties to the action. This, we think, is the rule, except as to rights which are
noted in the certificate or which arise subsequently, and with certain other exceptions which need not
be dismissed at present. A title once registered can not be defeated, even by an adverse, open, and
notorious possession. Registered title under the torrens system can not be defeated by prescription
(section 46, Act No. 496). The title, once registered, is notice to the world. All persons must take
notice. No one can plead ignorance of the registration.

The question, who is the owner of land registered in the name of two different persons, has been
presented to the courts in other jurisdictions. In some jurisdictions, where the "torrens" system has
been adopted, the difficulty has been settled by express statutory provision. In others it has been
settled by the courts. Hogg, in his excellent discussion of the "Australian Torrens System," at page
823, says: "The general rule is that in the case of two certificates of title, purporting to include the
same land, the earlier in date prevails, whether the land comprised in the latter certificate be wholly,
or only in part, comprised in the earlier certificate. (Oelkers vs. Merry, 2 Q.S.C.R., 193;
Miller vs. Davy, 7 N.Z.R., 155; Lloyd vs. Myfield, 7 A.L.T. (V.) 48; Stevens vs. Williams, 12 V.L. R.,
152; Register of Titles, vs. Esperance Land Co., 1 W.A.R., 118.)" Hogg adds however that, "if it can
be very clearly ascertained by the ordinary rules of construction relating to written documents, that the
inclusion of the land in the certificate of title of prior date is a mistake, the mistake may be rectified by
holding the latter of the two certificates of title to be conclusive." (See Hogg on the "Australian torrens
System," supra, and cases cited. See also the excellent work of Niblack in his "Analysis of the
Torrens System," page 99.) Niblack, in discussing the general question, said: "Where two certificates
purport to include the same land the earlier in date prevails. ... In successive registrations, where
more than one certificate is issued in respect of a particular estate or interest in land, the person
claiming under the prior certificates is entitled to the estate or interest; and that person is deemed to
hold under the prior certificate who is the holder of, or whose claim is derived directly or indirectly
from the person who was the holder of the earliest certificate issued in respect thereof. While the acts
in this country do not expressly cover the case of the issue of two certificates for the same land, they
provide that a registered owner shall hold the title, and the effect of this undoubtedly is that where two
certificates purport to include the same registered land, the holder of the earlier one continues to hold
the title" (p. 237).

Section 38 of Act No. 496, provides that; "It (the decree of registration) shall be conclusive upon and
against all persons, including the Insular Government and all the branches thereof, whether
mentioned by name in the application, notice, or citation, or included in the general description "To all
whom it may concern." Such decree shall not be opened by reason of the absence, infancy, or other
disability of any person affected thereby, nor by any proceeding in any court for reversing judgments
or decrees; subject, however, to the right of any person deprived of land or of any estate or interest
therein by decree of registration obtained by fraud to file in the Court of Land Registration a petition
for review within one year after entry of the decree (of registration), provided no innocent purchaser
for value has acquired an interest.

It will be noted, from said section, that the "decree of registration" shall not be opened,
for any reason, in any court, except for fraud, and not even for fraud, after the lapse of one year. If
then the decree of registration can not be opened for any reason, except for fraud, in a direct
proceeding for that purpose, may such decree be opened or set aside in a collateral proceeding by
including a portion of the land in a subsequent certificate or decree of registration? We do not believe
the law contemplated that a person could be deprived of his registered title in that way.

We have in this jurisdiction a general statutory provision which governs the right of the ownership of
land when the same is registered in the ordinary registry in the name of two persons. Article 1473 of
the Civil Code provides, among other things, that when one piece of real property had been sold to
two different persons it shall belong to the person acquiring it, who first inscribes it in the registry. This
rule, of course, presupposes that each of the vendees or purchasers has acquired title to the land.
The real ownership in such a case depends upon priority of registration. While we do not now decide
that the general provisions of the Civil Code are applicable to the Land Registration Act, even though
we see no objection thereto, yet we think, in the absence of other express provisions, they should
have a persuasive influence in adopting a rule for governing the effect of a double registration under
said Act. Adopting the rule which we believe to be more in consonance with the purposes and the real
intent of the torrens system, we are of the opinion and so decree that in case land has been
registered under the Land Registration Act in the name of two different persons, the earlier in date
shall prevail.
In reaching the above conclusion, we have not overlooked the forceful argument of the appellee. He
says, among other things; "When Prieto et al. were served with notice of the application of Teus (the
predecessor of the defendant) they became defendants in a proceeding wherein he, Teus, was
seeking to foreclose their right, and that of orders, to the parcel of land described in his application.
Through their failure to appear and contest his right thereto, and the subsequent entry of a default
judgment against them, they became irrevocably bound by the decree adjudicating such land to Teus.
They had their day in court and can not set up their own omission as ground for impugning the validity
of a judgment duly entered by a court of competent jurisdiction. To decide otherwise would be to hold
that lands with torrens titles are above the law and beyond the jurisdiction of the courts".

As was said above, the primary and fundamental purpose of the torrens system is to quiet title. If the
holder of a certificate cannot rest secure in this registered title then the purpose of the law is
defeated. If those dealing with registered land cannot rely upon the certificate, then nothing has been
gained by the registration and the expense incurred thereby has been in vain. If the holder may lose a
strip of his registered land by the method adopted in the present case, he may lose it all. Suppose
within the six years which elapsed after the plaintiff had secured their title, they had mortgaged or
sold their right, what would be the position or right of the mortgagee or vendee? That mistakes are
bound to occur cannot be denied, and sometimes the damage done thereby is irreparable. It is the
duty of the courts to adjust the rights of the parties under such circumstances so as to minimize such
damages, taking into consideration al of the conditions and the diligence of the respective parties to
avoid them. In the present case, the appellee was the first negligent (granting that he was the real
owner, and if he was not the real owner he can not complain) in not opposing the registration in the
name of the appellants. He was a party-defendant in an action for the registration of the lot in
question, in the name of the appellants, in 1906. "Through his failure to appear and to oppose such
registration, and the subsequent entry of a default judgment against him, he became irrevocably
bound by the decree adjudicating such land to the appellants. He had his day in court and should not
be permitted to set up his own omissions as the ground for impugning the validity of a judgment duly
entered by a court of competent jurisdiction." Granting that he was the owner of the land upon which
the wall is located, his failure to oppose the registration of the same in the name of the appellants, in
the absence of fraud, forever closes his mouth against impugning the validity of that judgment. There
is no more reason why the doctrine invoked by the appellee should be applied to the appellants than
to him.

We have decided, in case of double registration under the Land Registration Act, that the owner of
the earliest certificate is the owner of the land. That is the rule between original parties. May this rule
be applied to successive vendees of the owners of such certificates? Suppose that one or the other of
the parties, before the error is discovered, transfers his original certificate to an "innocent purchaser."
The general rule is that the vendee of land has no greater right, title, or interest than his vendor; that
he acquires the right which his vendor had, only. Under that rule the vendee of the earlier certificate
would be the owner as against the vendee of the owner of the later certificate.

We find statutory provisions which, upon first reading, seem to cast some doubt upon the rule that the
vendee acquires the interest of the vendor only. Sections 38, 55, and 112 of Act No. 496 indicate that
the vendee may acquire rights and be protected against defenses which the vendor would not. Said
sections speak of available rights in favor of third parties which are cut off by virtue of the sale of the
land to an "innocent purchaser." That is to say, persons who had had a right or interest in land
wrongfully included in an original certificate would be unable to enforce such rights against an
"innocent purchaser," by virtue of the provisions of said sections. In the present case Teus had his
land, including the wall, registered in his name. He subsequently sold the same to the appellee. Is the
appellee an "innocent purchaser," as that phrase is used in said sections? May those who have been
deprived of their land by reason of a mistake in the original certificate in favor of Teus be deprived of
their right to the same, by virtue of the sale by him to the appellee? Suppose the appellants had sold
their lot, including the wall, to an "innocent purchaser," would such purchaser be included in the
phrase "innocent purchaser," as the same is used in said sections? Under these examples there
would be two innocent purchasers of the same land, is said sections are to be applied .Which of the
two innocent purchasers, if they are both to be regarded as innocent purchasers, should be protected
under the provisions of said sections? These questions indicate the difficulty with which we are met in
giving meaning and effect to the phrase "innocent purchaser," in said sections.

May the purchaser of land which has been included in a "second original certificate" ever be regarded
as an "innocent purchaser," as against the rights or interest of the owner of the first original certificate,
his heirs, assigns, or vendee? The first original certificate is recorded in the public registry. It is never
issued until it is recorded. The record notice to all the world. All persons are charged with the
knowledge of what it contains. All persons dealing with the land so recorded, or any portion of it, must
be charged with notice of whatever it contains. The purchaser is charged with notice of every fact
shown by the record and is presumed to know every fact which the record discloses .This rule is so
well established that it is scarcely necessary to cite authorities in its support (Northwestern National
Bank vs. Freeman, 171 U.S., 620, 629; Delvin on Real Estate, sections 710, 710 [a]).

When a conveyance has been properly recorded such record is constructive notice of its contents
and all interests, legal and equitable, included therein. (Grandin vs. Anderson, 15 Ohio State, 286,
289; Orvis vs. Newell, 17 Conn., 97; Buchanan vs. Intentional Bank, 78 Ill., 500; Youngs vs. Wilson,
27 N.Y., 351; McCabe vs. Grey, 20 Cal., 509; Montefiore vs. Browne, 7 House of Lords Cases, 341.)

Under the rule of notice, it is presumed that the purchaser has examined every instrument of record
affecting the title. Such presumption is irrebutable. He is charged with notice of every fact shown by
the record and is presumed to know every fact which an examination of the record would have
disclosed. This presumption cannot be overcome by proof of innocence or good faith. Otherwise the
very purpose and object of the law requiring a record would be destroyed. Such presumption cannot
be defeated by proof of want of knowledge of what the record contains any more than one may be
permitted to show that he was ignorant of the provisions of the law. The rule that all persons must
take notice of the facts which the public record contains is a rule of law. The rule must be absolute.
Any variation would lead to endless confusion and useless litigation.

While there is no statutory provision in force here requiring that original deeds of conveyance of real
property be recorded, yet there is a rule requiring mortgages to be recorded. (Arts. 1875 and 606 of
the Civil Code.) The record of a mortgage is indispensable to its validity. (Art .1875.) In the face of
that statute would the courts allow a mortgage to be valid which had not been recorded, upon the
plea of ignorance of the statutory provision, when third parties were interested? May a purchaser of
land, subsequent to the recorded mortgage, plead ignorance of its existence, and by reason of such
ignorance have the land released from such lien? Could a purchaser of land, after the recorded
mortgage, be relieved from the mortgage lien by the plea that he was a bona fide purchaser? May
there be a bona fide purchaser of said land, bona fide in the sense that he had no knowledge of the
existence of the mortgage? We believe the rule that all persons must take notice of what the public
record contains in just as obligatory upon all persons as the rule that all men must know the law; that
no one can plead ignorance of the law. The fact that all men know the law is contrary to the
presumption. The conduct of men, at times, shows clearly that they do not know the law. The rule,
however, is mandatory and obligatory, notwithstanding. It would be just as logical to allow the defense
of ignorance of the existence and contents of a public record.

In view, therefore, of the foregoing rules of law, may the purchaser of land from the owner of the
second original certificate be an "innocent purchaser," when a part or all of such land had theretofore
been registered in the name of another, not the vendor? We are of the opinion that said sections 38,
55, and 112 should not be applied to such purchasers. We do not believe that the phrase "innocent
purchaser should be applied to such a purchaser. He cannot be regarded as an "innocent purchaser"
because of the facts contained in the record of the first original certificate. The rule should not be
applied to the purchaser of a parcel of land the vendor of which is not the owner of the original
certificate, or his successors. He, in nonsense, can be an "innocent purchaser" of the portion of the
land included in another earlier original certificate. The rule of notice of what the record contains
precludes the idea of innocence. By reason of the prior registry there cannot be an innocent
purchaser of land included in a prior original certificate and in a name other than that of the vendor, or
his successors. In order to minimize the difficulties we think this is the safe rule to establish. We
believe the phrase "innocent purchaser," used in said sections, should be limited only to cases where
unregistered land has been wrongfully included in a certificate under the torrens system. When land
is once brought under the torrens system, the record of the original certificate and all subsequent
transfers thereof is notice to all the world. That being the rule, could Teus even regarded as the
holder in good fifth of that part of the land included in his certificate of the appellants? We think not.
Suppose, for example, that Teus had never had his lot registered under the torrens system. Suppose
he had sold his lot to the appellee and had included in his deed of transfer the very strip of land now
in question. Could his vendee be regarded as an "innocent purchaser" of said strip? Would his
vendee be an "innocent purchaser" of said strip? Certainly not. The record of the original certificate of
the appellants precludes the possibility. Has the appellee gained any right by reason of the
registration of the strip of land in the name of his vendor? Applying the rule of notice resulting from
the record of the title of the appellants, the question must be answered in the negative. We are of the
opinion that these rules are more in harmony with the purpose of Act No. 496 than the rule contended
for by the appellee. We believe that the purchaser from the owner of the later certificate, and his
successors, should be required to resort to his vendor for damages, in case of a mistake like the
present, rather than to molest the holder of the first certificate who has been guilty of no negligence.
The holder of the first original certificate and his successors should be permitted to rest secure in their
title, against one who had acquired rights in conflict therewith and who had full and complete
knowledge of their rights. The purchaser of land included in the second original certificate, by reason
of the facts contained in the public record and the knowledge with which he is charged and by reason
of his negligence, should suffer the loss, if any, resulting from such purchase, rather than he who has
obtained the first certificate and who was innocent of any act of negligence.

The foregoing decision does not solve, nor pretend to solve, all the difficulties resulting from double
registration under the torrens system and the subsequent transfer of the land. Neither do we now
attempt to decide the effect of the former registration in the ordinary registry upon the registration
under the torrens system. We are inclined to the view, without deciding it, that the record under the
torrens system, supersede all other registries. If that view is correct then it will be sufficient, in dealing
with land registered and recorded alone. Once land is registered and recorded under the torrens
system, that record alone can be examined for the purpose of ascertaining the real status of the title
to the land.

It would be seen to a just and equitable rule, when two persons have acquired equal rights in the
same thing, to hold that the one who acquired it first and who has complied with all the requirements
of the law should be protected.

In view of our conclusions, above stated, the judgment of the lower court should be and is hereby
revoked. The record is hereby returned to the court now having and exercising the jurisdiction
heretofore exercised by the land court, with direction to make such orders and decrees in the
premises as may correct the error heretofore made in including the land in the second original
certificate issued in favor of the predecessor of the appellee, as well as in all other duplicate
certificates issued.

Without any findings as to costs, it is so ordered.


Arellano, C.J., Torrens, and Araullo, JJ., concur.

G.R. No. 143573 January 30, 2009

ADORACION ROSALES RUFLOE, ALFREDO RUFLOE and RODRIGO RUFLOE, Petitioners,


vs.
LEONARDA BURGOS, ANITA BURGOS, ANGELITO BURGOS, AMY BURGOS, ELVIRA DELOS
REYES and JULIAN C. TUBIG, Respondents.

DECISION

LEONARDO-DE CASTRO, J.:

Under consideration is this petition for review under Rule 45 of the Rules of Court seeking the
reversal and setting aside of the Decision1 dated January 17, 2000 of the Court of Appeals (CA) in
CA-G.R. CV. No. 49939, and its Resolution2 dated June 9, 2000, denying petitioners’ motion for
reconsideration.

The assailed decision reversed and set aside the February 10, 1995 decision3 of the Regional Trial
Court (RTC) at Muntinlupa, Metro Manila, Branch 276,4 in its Civil Case No. 90-359, an action for
Declaration of Nullity of Contract and Cancellation of Transfer Certificate of Titles and Damages,
commenced by the petitioners against herein respondents.

The factual antecedents are as follows:

Petitioner Adoracion Rufloe is the wife of Angel Rufloe, now deceased, while co-petitioners Alfredo
and Rodrigo are their children. During the marriage of Adoracion and Angel, they acquired a 371-
square meter parcel of land located at Barangay Bagbagan, Muntinlupa, and covered by Transfer
Certificate of Title (TCT) No. 406851 which is the subject of the present controversy.

Sometime in 1978, respondent Elvira Delos Reyes forged the signatures of Adoracion and Angel in a
Deed of Sale dated September 8, 1978 to make it appear that the disputed property was sold to her
by the spouses Rufloe. On the basis of the said deed of sale, Delos Reyes succeeded in obtaining a
title in her name, TCT No. S-74933.

Thus, in November 1979, the Rufloes filed a complaint for damages against Delos Reyes with the
RTC of Pasay City alleging that the Deed of Sale was falsified as the signatures appearing thereon
were forged because Angel Rufloe died in 1974, which was four (4) years before the alleged sale in
favor of Delos Reyes. The complaint was docketed as Civil Case No. M-7690.5 They also filed a
notice of adverse claim on November 5, 1979.

On December 4, 1984, during the pendency of Civil Case No. M-7690, Delos Reyes sold the subject
property to respondent siblings Anita, Angelina, Angelito and Amy (Burgos siblings). A new title, TCT
No. 135860, was then issued in their names.

On December 12, 1985, the Burgos siblings, in turn, sold the same property to their aunt, Leonarda
Burgos. However, the sale in favor of Leonarda was not registered. Thus, no title was issued in her
name. The subject property remained in the name of the Burgos siblings who also continued paying
the real estate taxes thereon.

On February 6, 1989, the RTC of Pasay City, Branch 108, 6 rendered its decision in Civil Case No. M-
7690 declaring that the Deed of Sale in favor of Delos Reyes was falsified as the signatures of the
spouses Rufloe had been forged. The trial court ruled that Delos Reyes did not acquire ownership
over the subject property. Said decision had become final and executory.

Such was the state of things when, on February 8, 1990, in the RTC of Muntinlupa, the Rufloes filed
their complaint for Declaration of Nullity of Contract and Cancellation of Transfer Certificate of Titles
against respondents Leonarda and the Burgos siblings, and Delos Reyes. In their complaint,
docketed as Civil Case No. 90-359, the Rufloes basically alleged that inasmuch as the Deed of Sale
in favor of Delos Reyes was falsified, no valid title was ever conveyed to the Burgos siblings. 7 The
Burgos siblings executed a simulated deed of sale in favor of Leonarda knowing fully well that their
title was a nullity.

In their common "Answer," respondents maintained that they bought the property in good faith after
they were shown a genuine copy of the title of the disputed property by Delos Reyes. They also
insisted that they were innocent purchasers in good faith and for value. 8

On February 10, 1995, the trial court rendered a decision declaring that Leonarda and the Burgos
siblings were not innocent purchasers for value and did not have a better right to the property in
question than the true and legal owners, the Rufloes. The trial court also held that the subsequent
conveyance of the disputed property to Leonarda by the Burgos siblings was simulated to make it
appear that Leonarda was a buyer in good faith. The trial court then directed the Register of Deeds of
Makati, Rizal to reinstate the title of the spouses Rufloe, and to cancel all other titles subsequent to
the said title particularly TCT No. S-74933 issued to Delos Reyes and TCT No. 135860 issued to the
Burgos siblings.9

Respondents interposed an appeal to the CA, whereat the appellate recourse was docketed as CA-
G.R. CV. No. 49939.

As stated at the threshold hereof, the CA, in its decision dated January 17, 2000, reversed and set
aside that of the trial court, declaring in the process that respondents were purchasers in good faith
and for value. In so ruling, the CA explained:

Measured by this yardstick, defendants-appellants [herein respondents] are purchasers in good faith
and for value. Amado Burgos bought the subject property (for his children Anita, Angelina, Angelito
and Amy) free from any lien or encumbrance or any notice of adverse claim annotated thereto. He
was presented with a clean title already in the name of the seller. If a person purchases a piece of
land on the assurance that the seller’s title thereto is valid, he should not run the risk of being told
later that his acquisition was ineffectual after all. If we were to void a sale of property covered by a
clean and unencumbered torrens title, public confidence in the Torrens System would be eroded and
transactions would have to be attended by complicated and inconclusive investigations and uncertain
proof of ownership. The consequences would be that land conflicts could proliferate and become
more abrasive, if not violent. (Words in bracket ours).10

Their motion for reconsideration having been denied by the CA in its equally challenged resolution of
June 9, 2000, petitioners are now with us via the present recourse, faulting the CA as follows:

A. THE HONORABLE COURT OF APPEALS DECIDED THIS CASE IN A WAY NOT IN ACCORD
WITH THE APPLICABLE DECISIONS OF THE HONORABLE SUPREME COURT.
B. THERE ARE SPECIAL AND IMPORTANT REASONS THAT REQUIRE A REVIEW OF THE CA
DECISION.

C. THE HONORABLE CA ACTED WITH GRAVE ABUSE OF DISCRETION AMOUNTING TO LACK


OF JURISDICTION WHEN IT COUNTERMANDED THE FINDINGS OF THE REGIONAL TRIAL
COURT EVEN ON POINTS AND QUESTIONS OF CREDIBILITY.

D. THE CA JUDGMENT THAT REVERSED THE RTC DECISION IS NOT SUPPORTED BY THE
EVIDENCE ON RECORD AND IS CONTRARY TO ESTABLISHED PRECEDENTS LAID DOWN BY
THE HONORABLE SUPREME COURT.

E. THE CA ERRED IN LAW IN PRACTICALLY HOLDING THAT A DEAD MAN ANGEL RUFLOE
(ANGEL NEVER SIGNED) VALIDLY DISPOSED OF HIS PROPERTY (A HOUSE AND LOT
COVERED BY A TCT THROUGH A FALSIFIED DEED OF SALE) AFTER HIS DEATH FOUR (4)
YEARS BEFORE THE EXECUTION OF THE DEED.

F. THE CA ERRED IN LAW IN HOLDING ANITA, ANGELINA, AMY AND ANGELITO BURGOS AND
THEIR SUCCESOR-IN-INTEREST (THEIR AUNT) LEONARDA BURGOS ARE BUYERS IN GOOD
FAITH.

G. THE CA IGNORED THE PLAIN PROVISIONS OF THE CIVIL CODE THAT "IN ALL
CONTRACTUAL, PROPERTY OR OTHER RELATIONS, WHEN ONE OF THE PARTIES IS AT A
DISADVANTAGE ON ACCOUNT OF HIS MORAL DEPENDENCE, IGNORANCE, INDIGENCE,
MENTAL WEAKNESS, TENDER AGE OR OTHER HANDICAP, THE COURT MUST BE VIGILANT
FOR HIS PROTECTION."11

In a gist, the issues to be resolved are (1) whether the sale of the subject property by Delos Reyes to
the Burgos siblings and the subsequent sale by the siblings to Leonarda were valid and binding; and
(2) whether respondents were innocent purchasers in good faith and for value despite the forged
deed of sale of their transferor Delos Reyes.

The issues necessitate an inquiry into the facts. While, as a rule, factual issues are not within the
province of this Court, nonetheless, in light of the conflicting factual findings of the two (2) courts
below, an examination of the facts obtaining in this case is in order.

The Rufloes aver that inasmuch as the Deed of Sale purportedly executed by them in favor of Delos
Reyes was a forgery, she could not pass any valid right or title to the Burgos siblings and Leonarda.
The Rufloes also contend that since the Burgos siblings and Leonarda acquired the subject property
with notice that another person has a right to or interest in such property, they cannot be considered
innocent purchasers in good faith and for value.

For their part, the Burgos siblings and Leonarda insist that their title is valid and binding. They
maintain that under the Torrens System, a person dealing with registered land may safely rely on the
correctness on the certificate of title without the need of further inquiry. For this reason, the Court
cannot disregard the right of an innocent third person who relies on the correctness of the certificate
of title even if the sale is void.

We find merit in the petition.

The issue concerning the validity of the deed of sale between the Rufloes and Delos Reyes had
already been resolved with finality in Civil Case No. M-7690 by the RTC of Pasay City which declared
that the signatures of the alleged vendors, Angel and Adoracion Rufloe, had been forged. 12 It is
undisputed that the forged deed of sale was null and void and conveyed no title. It is a well-settled
principle that no one can give what one does not have, nemo dat quod non habet. One can sell only
what one owns or is authorized to sell, and the buyer can acquire no more right than what the seller
can transfer legally.13 Due to the forged deed of sale, Delos Reyes acquired no right over the subject
property which she could convey to the Burgos siblings. All the transactions subsequent to the
falsified sale between the spouses Rufloe and Delos Reyes are likewise void, including the sale made
by the Burgos siblings to their aunt, Leonarda.

We now determine whether respondents Burgos siblings and Leonarda Burgos were purchasers in
good faith. It has been consistently ruled that a forged deed can legally be the root of a valid title
when an innocent purchaser for value intervenes.14

An innocent purchaser for value is one who buys the property of another without notice that some
other person has a right to or interest in it, and who pays a full and fair price at the time of the
purchase or before receiving any notice of another person’s claim.15 The burden of proving the status
of a purchaser in good faith and for value lies upon one who asserts that status. This onus
probandi cannot be discharged by mere invocation of the ordinary presumption of good faith. 16

As a general rule, every person dealing with registered land, as in this case, may safely rely on the
correctness of the certificate of title issued therefor and will in no way oblige him to go beyond the
certificate to determine the condition of the property. However, this rule admits of an unchallenged
exception:

… a person dealing with registered land has a right to rely on the Torrens certificate of title and to
dispense with the need of inquiring further except when the party has actual knowledge of facts and
circumstances that would impel a reasonably cautious man to make such inquiry or when the
purchaser has knowledge of a defect or the lack of title in his vendor or of sufficient facts to induce a
reasonably prudent man to inquire into the status of the title of the property in litigation. The presence
of anything which excites or arouses suspicion should then prompt the vendee to look beyond the
certificate and investigate the title of the vendor appearing on the face of said certificate. One who
falls within the exception can neither be denominated an innocent purchaser for value nor a
purchaser in good faith and, hence, does not merit the protection of the law. 17

The circumstances surrounding this case point to the absolute lack of good faith on the part of
respondents. The evidence shows that the Rufloes caused a notice of adverse claim to be annotated
on the title of Delos Reyes as early as November 5, 1979.18 The annotation of an adverse claim is a
measure designed to protect the interest of a person over a piece of real property, and serves as a
notice and warning to third parties dealing with said property that someone is claiming an interest on
the same or may have a better right than the registered owner thereof. Despite the notice of adverse
claim, the Burgos siblings still purchased the property in question.

Too, at the time the Burgos siblings bought the subject property on December 4, 1984, Civil Case No.
M-7690,19an action for damages, and Criminal Case No. 10914-P,20 for estafa, filed by the Rufloes
against Delos Reyes, were both pending before the RTC of Pasay City. This circumstance should
have alerted the Burgos siblings as to the validity of Delos Reyes’ title and her authority and legal
right to sell the property.

Equally significant is the fact that Delos Reyes was not in possession of the subject property when
she sold the same to the Burgos siblings. It was Amado Burgos who bought the property for his
children, the Burgos siblings. Amado was not personally acquainted with Delos Reyes prior to the
sale because he bought the property through a real estate broker, a certain Jose Anias, and not from
Delos Reyes herself. There was no showing that Amado or any of the Burgos siblings exerted any
effort to personally verify with the Register of Deeds if Delos Reyes’ certificate of title was clean and
authentic. They merely relied on the title as shown to them by the real estate broker. An ordinarily
prudent man would have inquired into the authenticity of the certificate of title, the property’s location
and its owners. Although it is a recognized principle that a person dealing with registered land need
not go beyond its certificate of title, it is also a firmly established rule that where circumstances exist
which would put a purchaser on guard and prompt him to investigate further, such as the presence of
occupants/tenants on the property offered for sale, it is expected that the purchaser would inquire first
into the nature of possession of the occupants, i.e., whether or not the occupants possess the land in
the concept of an owner. Settled is the rule that a buyer of real property that is in the possession of a
person other than the seller must be wary and should investigate the rights of those in possession.
Otherwise, without such inquiry, the buyer can hardly be regarded as a buyer in good faith. 21

In the same vein, Leonarda cannot be categorized as a purchaser in good faith. Since it was the
Rufloes who continued to have actual possession of the property, Leonarda should have investigated
the nature of their possession.

We cannot ascribe good faith to those who have not shown any diligence in protecting their rights.
Respondents had knowledge of facts that should have led them to inquire and investigate in order to
acquaint themselves with possible defects in the title of the seller of the
property.1avvphi1.zw+ However, they failed to do so. Thus, Leonarda, as well as the Burgos siblings,
cannot take cover under the protection the law accords to purchasers in good faith and for value.
They cannot claim valid title to the property.

Moreover, the defense of indefeasibility of a Torrens title does not extend to a transferee who takes it
with notice of a flaw in the title of his transferor. To be effective, the inscription in the registry must
have been made in good faith. A holder in bad faith of a certificate of title is not entitled to the
protection of the law, for the law cannot be used as a shield for fraud. 22

We quote with approval the following findings of the trial court showing that the sale between the
Burgos siblings and Leonarda is simulated:

1. The sale was not registered, a circumstance which is inconceivable in a legitimate transfer.
A true vendee would not brook any delay in registering the sale in his favor. Not only because
registration is the operative act that effects property covered by the Torrens System, but also
because registration and issuance of new title to the transferee, enable this transferee to
assume domiciliary and possessory rights over the property. These benefits of ownership shall
be denied him if the titles of the property shall remain in the name of vendor. Therefore, it is
inconceivable as contrary to behavioral pattern of a true buyer and the empirical knowledge of
man to assume that a buyer who invested on the property he bought would be uninvolved and
not endeavor to register the property he bought. The nonchalance of Leonarda amply
demonstrates the pretended sale to her, and the evident scheme of her brother Amado who
invested on the property he bought.

2. Despite the sale of property to Leonarda, the sellers continued paying taxes on the property
from the time they acquired it from Elvira in 1984 up to the present or a period of ten years.
The tax payment receipts remained in the name of Anita and her siblings, (Exhibits "16" to "16-
H"). On the other hand, Leonarda does not even pretend to have paid any tax on the land she
allegedly bought in 1985. Even the Tax Declaration issued in 1988, three years after the sale
to her (Leonarda) is still in the name of her nieces and nephew. These circumstances can only
account for the fact that her nieces and nephew remained the owners of the land and
continued paying taxes thereon.
3. Leonarda never exercised the attributes of ownership. Far from it, she vested the exercise of
domiciliary and possessory rights in her brother Amado the father of Anita, Angelina, Angelito
and Amy, by constituting him with full power including the ejectment of plaintiffs, to defend and
to enter a compromise of any case he may file. She allowed the children of Amado to remain
as the registered owners of the property without pressing for its transfer to her.

4. And, this simulated sale is the handiwork of Amado who apparently acted advisedly to make
it appear that his sister Leonarda as the second transferee of the property is an innocent
purchaser for value. Since he or his children could not plausibly assume the stance of a buyer
in good faith from the forger Elvira Delos Reyes, knowing of Elvira’s defective title, Amado
hoped that the entry of his sister Leonarda, might conjure the image and who might pass off as
an innocent purchaser, specially considering that the notice of adverse claim of the Plaintiffs
which was annotated in Elvira’s title was not, strangely enough, NOT carried over in the title of
his children, who were made to appear as the sellers to their Aunt Leonarda. It was a neat
chicanery of Amado to bring the property out of the reach of Plaintiffs thru a series of transfers
involving a third party, to make her appear as an innocent purchaser for value. His sister could
be manipulated to evict or oust the real owners from their own property thru a documentary
manipulation. Unfortunately, his scheme has not passed unnoticed by a discerning and
impartial evaluator, like this court. The Municipal Court of Muntinlupa in Civil Case No. 17446
has even established that Amado’s children Anita and others are buyers in bad faith who knew
of the defective title of their transferor Elvira Delos Reyes, the forger, as aforestated.

These circumstances taken altogether would show that the sale, which occurred between Leonarda
and the Burgos siblings, was simply a scheme designed to cleanse the title passed on to them by the
forger Delos Reyes. Respondents had to resort to this strategy because they were fully aware that
their title, having originated from the forged deed of sale of Delos Reyes, was not a clean and valid
title. The trial court explained, thus:

And, this simulated sale is the handiwork of Amado who apparently acted advisedly to make it appear
that his sister Leonarda as the second transferee of the property is an innocent purchaser for value.
Since he or his children could not plausibly assume the stamp of a buyer in good faith from the forger
Elvira Delos Reyes, knowing Elvira’s defective title, Amado had hoped that the entry of his sister
Leonarda, might conjure the image and might pass off as an innocent purchaser. xxx. It was a neat
chicanery of Amado to bring the property out of the reach of plaintiffs [herein petitioners] thru a series
of transfers involving a third party, to make her appear as an innocent purchaser for value.
Unfortunately, his scheme has not passed unnoticed by a discerning and impartial evaluator, like this
Court.23 (Words in bracket ours)

Patently, the Burgos siblings were not innocent purchasers for value and the simulated sale to
Leonarda did not remove the defect in their title.

Accordingly, we sustain the trial court’s award of ₱20,000.00 as moral damages, ₱50,000.00 as
exemplary damages, and P50,000.00 as attorney’s fees. 24

However, the actual damages in the amount of ₱134,200.00 should be deleted. In view of this Court’s
ruling that the property rightfully belongs to petitioners and must be restored to them, there is no more
basis for the award of said actual damages to the Rufloes.

WHEREFORE, the petition for review is hereby GRANTED. The assailed decision and resolution of
the Court of Appeals in CA-G.R. CV. No. 49939 are REVERSED and SET ASIDE. Accordingly, the
decision of the trial court is hereby REVIVED, except the award of actual damages which must be
deleted.
SO ORDERED.

G.R. No. L-2746 December 6, 1906

MATEO CARIÑO, petitioner-appellant,


vs.
THE INSULAR GOVERNMENT, respondent-appellee.

Coudert Brothers for appellant.

Attorney-General Wilfley for appellee.

WILLARD, J.:

The appellant, on the 23d of June, 1903, by his attorney in fact, Metcalf A. Clarke, filed a petition in
the Court of Land Registration asking that he be inscribed as the owner of a tract of land in the
municipality of Baguio, in the province of Benguet, containing 146 hectares. The Government of the
Philippine Islands, appeared in the Court of Land Registration and opposed the petition. The
Government of the United States that the land was part of the military reservation of Baguio.
Judgment was entered in the Court of Land Registration in favor of the petitioner, from which
judgment the respondents appealed in accordance with the law then in force to the Court of First
Instance of the province of Benguet. The case was therein tried de novo, and judgment was entered
dismissing the petition. The petitioner has brought the case here by bill of exceptions.

The petitioner presented no documentary evidence of title, except a possessory information obtained
in 1901. By the provisions of the Mortgage Law, under which this possessory information was
obtained (art. 394), it produced only those effects which the laws give to mere possession.

The petition not having shown any title from the Government, and the land being agricultural, the
case is governed by the decisions of this court in the cases of Valenton et al. vs. Murciano 1 (2 Off.
Gaz., 434); Cansino et al. vs. Valdez et al. 2 (4 Off. Gaz., 488); and Tiglao vs. The Insular
Government 3 (4 Off. Gaz., 747). In these cases it was held that the mere possession of land
such as that in controversy in this case would give the possessor and title thereto as against
the Government; in other words, that the statute of limitations did not run against the State in
reference to its agricultural lands.lawphil.net

The petitioner, however, insists that although the statute of limitations as such did not run against the
Government of Spain in the Philippine Islands, yet a grant is to be conclusively presumed from
immemorial use and occupation. To say that the presumption of a grant is presumption of law is, in
our opinion, simply to say that it amounts to a statute of limitations; and for a court to hold that the
statute of limitations does not run against the Government as to its public agricultural lands, and at
the same time to hold that if a person has been in possession of such lands for thirty years it is
conclusively presumed that the Government has given him a deed therefor, would be to make two
rulings directly inconsistent with each other.

Considered as a presumption of fact, the contention could not be sustained in this particular case.
Here the surrounding circumstances are incompatible with the existence of a grant, It is known that
for nearly three hundred years all attempts to convert the Igorots of the Province of Benguet to the
Christian religion completely failed, and that during that time they remained practically in the same
condition as they were when the Islands were first occupied by the Spaniards. To presume as a
matter of fact that during that time, and down to at least 1880, the provisions of the laws relating to
the grant, adjustment, and sale of public were taken advantage of by these deeds from the
Government for these lands would be to presume something which did not exist. The appellant says
in his brief (p.10):

The Igorot, no less than the American Indian, is an aborigine, and is equally ignorant of the
forms of law and procedure necessary to protect his interests.

There is, moreover, in the case evidence that in 1894 the petitioner sought to obtain title from the
Government in accordance with the laws then in force. In 1901 he made a contract with Metalcalf A.
Clarke, by the terms of which he agreed to sell the land to Clarke for 6,000 pesos when he obtained
title thereto from the Government, and this contract he does not say that he is the owner, but simply
that he is in possession thereof. The court below found that the land is now worth upwards of
P50,000.

The possession of the land has not been of such a character as to require the presumption of a grant.
No one has lived upon it for many years. It was never used for anything but pasturage of animals,
except insignificant portions thereof, and since the insurrection against Spain it has apparently not
been used by the petitioner for any purpose.

The petitioner relies upon the case of the United States vs. Chaves (159 U.S., 452) and the case of
The United States vs. Chaves (175 U.S., 509). In the case of Hays vs. The United States (175 U.S.
248) the court said at page 261;

But this presumption is subject to the limitation that where title is claimed from a deed which is
shown to be void, it will not be presumed that there was an independent grant
(Smith vs. Highbee, 12 Vermont,. 113), or where surrounding circumstances are inconsistent
with the theory of a grant. (Townsend vs. Downer, 32 Vermont, 183).

The substance of this doctrine is that lapse of time any be treated as helping out the
presumption of a grant, but where a void grant is shown, it affords no presumption that another
valid grant was made. Nor does such presumption arise if the surrounding circumstances are
incompatible with the existence of a grant. In this case under consideration we can not find any
evidence which justifies us in believing that a legal grant can have been made, and under
those circumstances we can not consider possession since the date of the treaty as
dispensing with the requirement that the title, if not perfect at that time, was one which the
claimant would have a lawful right to make perfect had the territory not been acquired by the
United States.

In the case of Chaves vs. The United States (175 U.S., 552) the court made the following statement
at page 562:

Finally, it distinctly appears that the possession of the parties is insufficient in length of time to
prove a valid title. In United States vs. Chaves (159 U.S., 452) the possession was under the
claim of a grant made by the governor of New Mexico to the alleged grantees. The grant had
been lost, but it had been seen and read by witnesses, and its existence had been proved by
evidence sufficient, as we stated in the opinion (p. 460), to warrant 'the finding of the court
below that the complainant's title was derived from the Republic of Mexico, and was complete
and perfect at the date when the United States acquired sovereignty in the territory of New
Mexico, within which the land was situated. We do not question the correctness to the remarks
made by Mr. Justice Shiras in regard to evidence of possession and the presumptions which
may under certain circumstances drawn as to the existence of a grant.

We do not deny the right of the duty of a court to presume its existence in a proper case, in
order to quiet a title and to give to long continued possession the quality of a rightful
possession under a legal right. We recognized and enforced such a rule in the case of United
States vs. Chaves decided at this term. in which the question is involved. We simply say in this
case that the possession was not a duration long enough to justify any such inference.

There is no proof of any valid grant, but on the contrary the evidence offered by the plaintiff
himself and upon which the bases the title that he asks the court to confirm, shows the
existence of a grant from a body which had no legal power to make it, and which, therefore,
conveyed no title whatever to its grantee, and the evidence is, as given by the plaintiff himself,
that it was under this grant alone that possession of the lands was taken. We can not presume
(within the time involved in this case) that any other and valid grant was ever made. The
possession of the plaintiff and of his grantors up to the time of the treaty of Guadalupe Hidalgo,
in 1848, had not been long enough to presume a grant. (Crispin vs. United States, 168 U.S.,
208; Hayes vs. United States, 170 U.S., 637, 649, 653; Hays vs. The United States, ante 248.)
The possession subsequently existing, we can not notice. Same authorities.

As we understand it, it is well settled in the United States that prescription does not run against the
Government as to its public lands — in other words, that if a person desires to obtain title to the public
lands of the United States situated within the boundaries of the States, he must do so in the way
pointed out by the law. We do not understand that a person in possession of unsurveyed public lands
in the State of Minnesota, for example, whose ancestors had occupied that the land for fortyh years,
could maintain in court a claim that he was the legal owner of the lands by granted the land to his
ancestors, a presumption founded not upon any proceedings taken in the General Land Office to
acquire a patent thereto, but upon the mere possession for that length of time.

The same is true of the public lands of Spain in the Philippine Islands. In the case of Valenton et al.
vs. Marciano it was said:

While the State has always recognized the right of the occupant to a deed if he proves a
possession for a sufficient length of time, yet it has always insisted that he must make that
proof before the proper administrative officers, and obtain from them his deed, and until he did
the State remained the absolute owner.

But in any event, and whatever the law may be elsewhere, it seems clear that this doctrine of
presumptive grant can not apply to the Philippines in view of the Spanish legislation for the Indies.
From time to time there were promulgated laws which required the person in possession of public
lands to exhibit their titles or grants thereto. If these titles or grants were found to be good, they were
confirmed, but if they were not, or if the persons had no grants or titles at all, they were evicted from
the land.

For example, in Law 14, title 12, 4, Recompilation of the Laws of the Indies, it is stated:

We therefore order and command that all viceroys and presidents of pretrial courts designate,
at such times as shall to them most expedient, a suitable period within which all possessors of
tracts, farms, plantations, and estates shall exhibit to them and to the court officers appointed
by them for this purpose their title deeds thereto. And those who are in possession by virtue of
proper deeds and receipts or by virtue of just prescriptive rights shall be protected, and all the
rest shall be restored to us to be disposed of at our will.

In the Royal Cedula of October 15, 1754, it was provided —

that any and all persons who, since the year 1700, and up to the date of promulgation and
publication of said order, shall have occupied royal lands, whether or not the same shall be
cultivated or tenanted, may, either in person or through their attorneys or representatives,
appear and exhibit to said subdelegates the titles and patents by virtue of which said lands are
occupied. Said subdelegates will designate as the period within which documents must be
presented a term sufficient in length and proportionate to the distance the interested party may
have to travel for the purpose of making the presentation. Said subdelegates will at the same
time warn the parties interested that in term designated, without a just and valid reason
therefor, they will be deprived of and evicted from their lands, and they will be granted to
others.

In the regulations of June 25, 1880, it was provided as follows:

ART. 8. If the interested parties shall not ask an adjustment of the lands whose possession
they are unlawfully enjoining within the time of one year, or, the adjustment having been
granted by the authorities, they shall fail to fulfill their obligation in connection with the
compromise, by paying the proper sum into the treasury, the latter will, by virtue of the
authority vested in it, reassert the ownership of the Stated over the lands, and will, after fixing
the whole thereof, proceed to sell at public auction that part of the same which, either because
it may have been reduced to cultivation or is not located within the forest zone, is not deemed
advisable to preserve as State forest reservations. 4

In the royal decree of the 13th of February, 1894, published in the Official Gazzette of Manila of the
17th of April, 01894, it is provided in article 4 as follows:

ART. 4. The title to all agricultural lands which were capable of adjustment (composicion)
under the royal decree of the 25th of June, 1880, but the adjustments of which decree in the
Gaceta de Manila, will revert to the State. Any claim to such lands by those who might have
applied for the adjustment of the same, but who have not done so as the above mentioned
date, will not avail them in any way or at any time.

In view of these provisions of the law, it seems to us impossible to say that as to the public
agricultural lands in the Philippines there existed a conclusive presumption after a lapse of thirty or
any other number of years that the Government of Spain had granted to the possessor thereof a legal
title thereto.

The plaintiff is not entitled to the benefits of paragraph 6 of section 54 of Act No. 926, the Public Land
Act, for the reason that act is not applicable to the Province of Benguet. The judgment of the court
below is affirmed, with the costs of this instance against the appellant.

After the expiration of twenty days let judgment be entered accordingly and ten days thereafter the
case be returned to the court below for execution. So ordered.

G.R. No. 135385 December 6, 2000


ISAGANI CRUZ and CESAR EUROPA, petitioners,
vs.
SECRETARY OF ENVIRONMENT AND NATURAL RESOURCES, SECRETARY OF BUDGET
AND MANAGEMENT and CHAIRMAN and COMMISSIONERS OF THE NATIONAL COMMISSION
ON INDIGENOUS PEOPLES, respondents.
HON. JUAN M .FLAVIER, HON. PONCIANO BENNAGEN, BAYANI ASCARRAGA, EDTAMI
MANSAYANGAN, BASILIO WANDAG, EVELYN DUNUAN, YAOM TUGAS, ALFREMO
CARPIANO, LIBERATO A. GABIN, MATERNIDAD M. COLAS, NARCISA M. DALUPINES, BAI
KIRAM-CONNIE SATURNO, BAE MLOMO-BEATRIZ T. ABASALA, DATU BALITUNGTUNG-
ANTONIO D. LUMANDONG, DATU MANTUMUKAW TEOFISTO SABASALES, DATU
EDUAARDO BANDA, DATU JOEL UNAD, DATU RAMON BAYAAN, TIMUAY JOSE ANOY,
TIMUAY MACARIO D. SALACAO, TIMUAY EDWIN B. ENDING, DATU SAHAMPONG MALANAW
VI, DATU BEN PENDAO CABIGON, BAI NANAPNAY-LIZA SAWAY, BAY INAY DAYA-MELINDA
S. REYMUNDO, BAI TINANGHAGA HELINITA T. PANGAN, DATU MAKAPUKAW ADOLINO L.
SAWAY, DATU MAUDAYAW-CRISPEN SAWAY, VICKY MAKAY, LOURDES D. AMOS, GILBERT
P. HOGGANG, TERESA GASPAR, MANUEL S. ONALAN, MIA GRACE L. GIRON, ROSEMARIE
G. PE, BENITO CARINO, JOSEPH JUDE CARANTES, LYNETTE CARANTES-VIVAL, LANGLEY
SEGUNDO, SATUR S. BUGNAY, CARLING DOMULOT, ANDRES MENDIOGRIN, LEOPOLDO
ABUGAN, VIRGILIO CAYETANO, CONCHITA G. DESCAGA, LEVY ESTEVES, ODETTE G.
ESTEVEZ, RODOLFO C. AGUILAR, MAURO VALONES, PEPE H. ATONG, OFELIA T. DAVI,
PERFECTO B. GUINOSAO, WALTER N. TIMOL, MANUEL T. SELEN, OSCAR DALUNHAY, RICO
O. SULATAN, RAFFY MALINDA, ALFREDO ABILLANOS, JESSIE ANDILAB, MIRLANDO H.
MANGKULINTAS, SAMIE SATURNO, ROMEO A. LINDAHAY, ROEL S. MANSANG-CAGAN,
PAQUITO S. LIESES, FILIPE G. SAWAY, HERMINIA S. SAWAY, JULIUS S. SAWAY, LEONARDA
SAWAY, JIMMY UGYUB, SALVADOR TIONGSON, VENANCIO APANG, MADION MALID, SUKIM
MALID, NENENG MALID, MANGKATADONG AUGUSTO DIANO, JOSEPHINE M. ALBESO,
MORENO MALID, MARIO MANGCAL, FELAY DIAMILING, SALOME P. SARZA, FELIPE P.
BAGON, SAMMY SALNUNGAN, ANTONIO D. EMBA, NORMA MAPANSAGONOS, ROMEO
SALIGA, SR., JERSON P. GERADA, RENATO T. BAGON, JR., SARING MASALONG, SOLEDAD
M. GERARDA, ELIZABETH L. MENDI, MORANTE S. TIWAN, DANILO M. MALUDAO, MINORS
MARICEL MALID, represented by her father CORNELIO MALID, MARCELINO M. LADRA,
represented by her father MONICO D. LADRA, JENNYLYN MALID, represented by her father
TONY MALID, ARIEL M. EVANGELISTA, represented by her mother LINAY BALBUENA,
EDWARD M. EMUY, SR., SUSAN BOLANIO, OND, PULA BATO B'LAAN TRIBAL FARMER'S
ASSOCIATION, INTER-PEOPLE'S EXCHANGE, INC. and GREEN FORUM-WESTERN
VISAYAS, intervenors.
COMMISSION ON HUMAN RIGHTS, intervenor.
IKALAHAN INDIGENOUS PEOPLE and HARIBON FOUNDATION FOR THE CONSERVATION OF
NATURAL RESOURCES, INC., intervenor.

RESOLUTION

PER CURIAM:

Petitioners Isagani Cruz and Cesar Europa brought this suit for prohibition and mandamus as citizens
and taxpayers, assailing the constitutionality of certain provisions of Republic Act No. 8371 (R.A.
8371), otherwise known as the Indigenous Peoples Rights Act of 1997 (IPRA), and its Implementing
Rules and Regulations (Implementing Rules).

In its resolution of September 29, 1998, the Court required respondents to comment.1 In compliance,
respondents Chairperson and Commissioners of the National Commission on Indigenous Peoples
(NCIP), the government agency created under the IPRA to implement its provisions, filed on October
13, 1998 their Comment to the Petition, in which they defend the constitutionality of the IPRA and
pray that the petition be dismissed for lack of merit.

On October 19, 1998, respondents Secretary of the Department of Environment and Natural
Resources (DENR) and Secretary of the Department of Budget and Management (DBM) filed through
the Solicitor General a consolidated Comment. The Solicitor General is of the view that the IPRA is
partly unconstitutional on the ground that it grants ownership over natural resources to indigenous
peoples and prays that the petition be granted in part.

On November 10, 1998, a group of intervenors, composed of Sen. Juan Flavier, one of the authors of
the IPRA, Mr. Ponciano Bennagen, a member of the 1986 Constitutional Commission, and the
leaders and members of 112 groups of indigenous peoples (Flavier, et. al), filed their Motion for
Leave to Intervene. They join the NCIP in defending the constitutionality of IPRA and praying for the
dismissal of the petition.

On March 22, 1999, the Commission on Human Rights (CHR) likewise filed a Motion to Intervene
and/or to Appear as Amicus Curiae. The CHR asserts that IPRA is an expression of the principle of
parens patriae and that the State has the responsibility to protect and guarantee the rights of those
who are at a serious disadvantage like indigenous peoples. For this reason it prays that the petition
be dismissed.

On March 23, 1999, another group, composed of the Ikalahan Indigenous People and the Haribon
Foundation for the Conservation of Natural Resources, Inc. (Haribon, et al.), filed a motion to
Intervene with attached Comment-in-Intervention. They agree with the NCIP and Flavier, et al. that
IPRA is consistent with the Constitution and pray that the petition for prohibition and mandamus be
dismissed.

The motions for intervention of the aforesaid groups and organizations were granted.

Oral arguments were heard on April 13, 1999. Thereafter, the parties and intervenors filed their
respective memoranda in which they reiterate the arguments adduced in their earlier pleadings and
during the hearing.

Petitioners assail the constitutionality of the following provisions of the IPRA and its Implementing
Rules on the ground that they amount to an unlawful deprivation of the State’s ownership over lands
of the public domain as well as minerals and other natural resources therein, in violation of the
regalian doctrine embodied in Section 2, Article XII of the Constitution:

"(1) Section 3(a) which defines the extent and coverage of ancestral domains, and Section 3(b)
which, in turn, defines ancestral lands;

"(2) Section 5, in relation to section 3(a), which provides that ancestral domains including inalienable
public lands, bodies of water, mineral and other resources found within ancestral domains are private
but community property of the indigenous peoples;

"(3) Section 6 in relation to section 3(a) and 3(b) which defines the composition of ancestral domains
and ancestral lands;

"(4) Section 7 which recognizes and enumerates the rights of the indigenous peoples over the
ancestral domains;
(5) Section 8 which recognizes and enumerates the rights of the indigenous peoples over the
ancestral lands;

"(6) Section 57 which provides for priority rights of the indigenous peoples in the harvesting,
extraction, development or exploration of minerals and other natural resources within the areas
claimed to be their ancestral domains, and the right to enter into agreements with nonindigenous
peoples for the development and utilization of natural resources therein for a period not exceeding 25
years, renewable for not more than 25 years; and

"(7) Section 58 which gives the indigenous peoples the responsibility to maintain, develop, protect
and conserve the ancestral domains and portions thereof which are found to be necessary for critical
watersheds, mangroves, wildlife sanctuaries, wilderness, protected areas, forest cover or
reforestation."2

Petitioners also content that, by providing for an all-encompassing definition of "ancestral domains"
and "ancestral lands" which might even include private lands found within said areas, Sections 3(a)
and 3(b) violate the rights of private landowners.3

In addition, petitioners question the provisions of the IPRA defining the powers and jurisdiction of the
NCIP and making customary law applicable to the settlement of disputes involving ancestral domains
and ancestral lands on the ground that these provisions violate the due process clause of the
Constitution.4

These provisions are:

"(1) sections 51 to 53 and 59 which detail the process of delineation and recognition of
ancestral domains and which vest on the NCIP the sole authority to delineate ancestral
domains and ancestral lands;

"(2) Section 52[i] which provides that upon certification by the NCIP that a particular area is an
ancestral domain and upon notification to the following officials, namely, the Secretary of
Environment and Natural Resources, Secretary of Interior and Local Governments, Secretary
of Justice and Commissioner of the National Development Corporation, the jurisdiction of said
officials over said area terminates;

"(3) Section 63 which provides the customary law, traditions and practices of indigenous
peoples shall be applied first with respect to property rights, claims of ownership, hereditary
succession and settlement of land disputes, and that any doubt or ambiguity in the
interpretation thereof shall be resolved in favor of the indigenous peoples;

"(4) Section 65 which states that customary laws and practices shall be used to resolve
disputes involving indigenous peoples; and

"(5) Section 66 which vests on the NCIP the jurisdiction over all claims and disputes involving
rights of the indigenous peoples."5

Finally, petitioners assail the validity of Rule VII, Part II, Section 1 of the NCIP Administrative Order
No. 1, series of 1998, which provides that "the administrative relationship of the NCIP to the Office of
the President is characterized as a lateral but autonomous relationship for purposes of policy and
program coordination." They contend that said Rule infringes upon the President’s power of control
over executive departments under Section 17, Article VII of the Constitution.6
Petitioners pray for the following:

"(1) A declaration that Sections 3, 5, 6, 7, 8, 52[I], 57, 58, 59, 63, 65 and 66 and other related
provisions of R.A. 8371 are unconstitutional and invalid;

"(2) The issuance of a writ of prohibition directing the Chairperson and Commissioners of the
NCIP to cease and desist from implementing the assailed provisions of R.A. 8371 and its
Implementing Rules;

"(3) The issuance of a writ of prohibition directing the Secretary of the Department of
Environment and Natural Resources to cease and desist from implementing Department of
Environment and Natural Resources Circular No. 2, series of 1998;

"(4) The issuance of a writ of prohibition directing the Secretary of Budget and Management to
cease and desist from disbursing public funds for the implementation of the assailed provisions
of R.A. 8371; and

"(5) The issuance of a writ of mandamus commanding the Secretary of Environment and
Natural Resources to comply with his duty of carrying out the State’s constitutional mandate to
control and supervise the exploration, development, utilization and conservation of Philippine
natural resources."7

After due deliberation on the petition, the members of the Court voted as follows:

Seven (7) voted to dismiss the petition. Justice Kapunan filed an opinion, which the Chief Justice and
Justices Bellosillo, Quisumbing, and Santiago join, sustaining the validity of the challenged provisions
of R.A. 8371. Justice Puno also filed a separate opinion sustaining all challenged provisions of the
law with the exception of Section 1, Part II, Rule III of NCIP Administrative Order No. 1, series of
1998, the Rules and Regulations Implementing the IPRA, and Section 57 of the IPRA which he
contends should be interpreted as dealing with the large-scale exploitation of natural resources and
should be read in conjunction with Section 2, Article XII of the 1987 Constitution. On the other hand,
Justice Mendoza voted to dismiss the petition solely on the ground that it does not raise a justiciable
controversy and petitioners do not have standing to question the constitutionality of R.A. 8371.

Seven (7) other members of the Court voted to grant the petition. Justice Panganiban filed a separate
opinion expressing the view that Sections 3 (a)(b), 5, 6, 7 (a)(b), 8, and related provisions of R.A.
8371 are unconstitutional. He reserves judgment on the constitutionality of Sections 58, 59, 65, and
66 of the law, which he believes must await the filing of specific cases by those whose rights may
have been violated by the IPRA. Justice Vitug also filed a separate opinion expressing the view that
Sections 3(a), 7, and 57 of R.A. 8371 are unconstitutional. Justices Melo, Pardo, Buena, Gonzaga-
Reyes, and De Leon join in the separate opinions of Justices Panganiban and Vitug.

As the votes were equally divided (7 to 7) and the necessary majority was not obtained, the case was
redeliberated upon. However, after redeliberation, the voting remained the same. Accordingly,
pursuant to Rule 56, Section 7 of the Rules of Civil Procedure, the petition is DISMISSED.

Attached hereto and made integral parts thereof are the separate opinions of Justices Puno, Vitug,
Kapunan, Mendoza, and Panganiban.

SO ORDERED.
Davide, Jr., C.J., Bellosillo, Melo, Quisumbing, Pardo, Buena, Gonzaga-Reyes, Ynares-Santiago, and
De Leon, Jr., JJ., concur.
Puno, Vitug, Kapunan, Mendoza and Panganiban JJ., see separate opinion

Footnotes
1 Rollo, p. 114.
2 Petition, Rollo, pp. 16-23.
3 Id. at 23-25.

4 Section1, Article III of the Constitution states: "No person shall be deprived of life, liberty or
property without due process of law, nor shall any person be denied the equal protection of the
laws."
5 Rollo, pp. 25-27.
6 Id. at 27-28.

7 Transcript of Stenographic Notes of the hearing held on April 13, 1999, pp. 5-6.

The Lawphil Project - Arellano Law Foundation

SEPARATE OPINION

PUNO, J.:

PRECIS

A classic essay on the utility of history was written in 1874 by Friedrich Nietzsche entitled "On the
Uses and Disadvantages of History for Life." Expounding on Nietzsche's essay, Judge Richard
Posner1 wrote:2

"Law is the most historically oriented, or if you like the most backward-looking, the most 'past-
dependent,' of the professions. It venerates tradition, precedent, pedigree, ritual, custom, ancient
practices, ancient texts, archaic terminology, maturity, wisdom, seniority, gerontocracy, and
interpretation conceived of as a method of recovering history. It is suspicious of innovation,
discontinuities, 'paradigm shifts,' and the energy and brashness of youth. These ingrained attitudes
are obstacles to anyone who wants to re-orient law in a more pragmatic direction. But, by the same
token, pragmatic jurisprudence must come to terms with history."

When Congress enacted the Indigenous Peoples Rights Act (IPRA), it introduced radical concepts
into the Philippine legal system which appear to collide with settled constitutional and jural precepts
on state ownership of land and other natural resources. The sense and subtleties of this law cannot
be appreciated without considering its distinct sociology and the labyrinths of its history. This Opinion
attempts to interpret IPRA by discovering its soul shrouded by the mist of our history. After all, the
IPRA was enacted by Congress not only to fulfill the constitutional mandate of protecting the
indigenous cultural communities' right to their ancestral land but more importantly, to correct a grave
historical injustice to our indigenous people.

This Opinion discusses the following:

I. The Development of the Regalian Doctrine in the Philippine Legal System.

A. The Laws of the Indies

B. Valenton v. Murciano

C. The Public Land Acts and the Torrens System

D. The Philippine Constitutions

II. The Indigenous Peoples Rights Act (IPRA).

A. Indigenous Peoples

1. Indigenous Peoples: Their History

2. Their Concept of Land

III. The IPRA is a Novel Piece of Legislation.

A. Legislative History

IV. The Provisions of the IPRA Do Not Contravene the Constitution.

A. Ancestral domains and ancestral lands are the private property of indigenous peoples and
do not constitute part of the land of the public domain.

1. The right to ancestral domains and ancestral lands: how acquired

2. The concept of native title

(a) Cariño v. Insular Government

(b) Indian Title to land

(c) Why the Cariño doctrine is unique

3. The option of securing a torrens title to the ancestral land

B. The right of ownership and possession by the ICCs/IPs to their ancestral domains is a
limited form of ownership and does not include the right to alienate the same.
1. The indigenous concept of ownership and customary law

C. Sections 7 (a), 7 (b) and 57 of the IPRA do not violate the Regalian Doctrine enshrined in
Section 2, Article XII of the 1987 Constitution.

1. The rights of ICCs/IPs over their ancestral domains and lands

2. The right of ICCs/IPs to develop lands and natural resources within the ancestral
domains does not deprive the State of ownership over the natural resources, control
and supervision in their development and exploitation.

(a) Section 1, Part II, Rule III of the Implementing Rules goes beyond the
parameters of Section 7(a) of the law on ownership of ancestral domains and
is ultra vires.

(b) The small-scale utilization of natural resources in Section 7 (b) of the IPRA is
allowed under Paragraph 3, Section 2, Article XII of the 1987 Consitution.

(c) The large-scale utilization of natural resources in Section 57 of the IPRA may
be harmonized with Paragraphs 1 and 4, Section 2, Article XII of the 1987
Constitution.

V. The IPRA is a Recognition of Our Active Participation in the International Indigenous Movement.

DISCUSSION

I. THE DEVELOPMENT OF THE REGALIAN DOCTRINE IN THE PHILIPPINE LEGAL SYSTEM.

A. The Laws of the Indies

The capacity of the State to own or acquire property is the state's power of dominium.3 This was the
foundation for the early Spanish decrees embracing the feudal theory of jura regalia. The "Regalian
Doctrine" or jura regalia is a Western legal concept that was first introduced by the Spaniards into
the country through the Laws of the Indies and the Royal Cedulas. The Laws of the Indies, i.e.,
more specifically, Law 14, Title 12, Book 4 of the Novisima Recopilacion de Leyes de las Indias, set
the policy of the Spanish Crown with respect to the Philippine Islands in the following manner:

"We, having acquired full sovereignty over the Indies, and all lands, territories, and possessions not
heretofore ceded away by our royal predecessors, or by us, or in our name, still pertaining to the royal
crown and patrimony, it is our will that all lands which are held without proper and true deeds of grant
be restored to us as they belong to us, in order that after reserving before all what to us or to our
viceroys, audiencias, and governors may seem necessary for public squares, ways, pastures, and
commons in those places which are peopled, taking into consideration not only their present
condition, but also their future and their probable increase, and after distributing to the natives what
may be necessary for tillage and pasturage, confirming them in what they now have and giving them
more if necessary, all the rest of said lands may remain free and unencumbered for us to dispose of
as we may wish.

We therefore order and command that all viceroys and presidents of pretorial courts designate at
such time as shall to them seem most expedient, a suitable period within which all possessors of
tracts, farms, plantations, and estates shall exhibit to them and to the court officers appointed by them
for this purpose, their title deeds thereto. And those who are in possession by virtue of proper deeds
and receipts, or by virtue of just prescriptive right shall be protected, and all the rest shall be restored
to us to be disposed of at our will."4

The Philippines passed to Spain by virtue of "discovery" and conquest. Consequently, all lands
became the exclusive patrimony and dominion of the Spanish Crown. The Spanish Government took
charge of distributing the lands by issuing royal grants and concessions to Spaniards, both military
and civilian.5 Private land titles could only be acquired from the government either by purchase or by
the various modes of land grant from the Crown.6

The Laws of the Indies were followed by the Ley Hipotecaria, or the Mortgage Law of 1893.7 The
Spanish Mortgage Law provided for the systematic registration of titles and deeds as well as
possessory claims. The law sought to register and tax lands pursuant to the Royal Decree of 1880.
The Royal Decree of 1894, or the "Maura Law," was partly an amendment of the Mortgage Law as
well as the Laws of the Indies, as already amended by previous orders and decrees.8 This was the
last Spanish land law promulgated in the Philippines. It required the "adjustment" or registration of all
agricultural lands, otherwise the lands shall revert to the state.

Four years later, by the Treaty of Paris of December 10, 1898, Spain ceded to the government of
the United States all rights, interests and claims over the national territory of the Philippine Islands. In
1903, the United States colonial government, through the Philippine Commission, passed Act No.
926, the first Public Land Act.

B. Valenton v. Murciano

In 1904, under the American regime, this Court decided the case of Valenton v. Murciano.9

Valenton resolved the question of which is the better basis for ownership of land: long-time
occupation or paper title. Plaintiffs had entered into peaceful occupation of the subject land in 1860.
Defendant's predecessor-in-interest, on the other hand, purchased the land from the provincial
treasurer of Tarlac in 1892. The lower court ruled against the plaintiffs on the ground that they had
lost all rights to the land by not objecting to the administrative sale. Plaintiffs appealed the judgment,
asserting that their 30-year adverse possession, as an extraordinary period of prescription in
the Partidas and the Civil Code, had given them title to the land as against everyone, including the
State; and that the State, not owning the land, could not validly transmit it.

The Court, speaking through Justice Willard, decided the case on the basis of "those special laws
which from earliest time have regulated the disposition of the public lands in the colonies."10 The
question posed by the Court was: "Did these special laws recognize any right of prescription as
against the State as to these lands; and if so, to what extent was it recognized?"

Prior to 1880, the Court said, there were no laws specifically providing for the disposition of land in
the Philippines. However, it was understood that in the absence of any special law to govern a
specific colony, the Laws of the Indies would be followed. Indeed, in the Royal Order of July 5, 1862,
it was decreed that until regulations on the subject could be prepared, the authorities of the Philippine
Islands should follow strictly the Laws of the Indies, the Ordenanza of the Intendentes of 1786, and
the Royal Cedula of 1754.11

Quoting the preamble of Law 14, Title 12, Book 4 of the Recopilacion de Leyes de las Indias, the
court interpreted it as follows:
"In the preamble of this law there is, as is seen, a distinct statement that all those lands belong to the
Crown which have not been granted by Philip, or in his name, or by the kings who preceded
him. This statement excludes the idea that there might be lands not so granted, that did not
belong to the king. It excludes the idea that the king was not still the owner of all ungranted
lands, because some private person had been in the adverse occupation of them. By the mandatory
part of the law all the occupants of the public lands are required to produce before the authorities
named, and within a time to be fixed by them, their title papers. And those who had good title or
showed prescription were to be protected in their holdings. It is apparent that it was not the intention
of the law that mere possession for a length of time should make the possessors the owners of the
land possessed by them without any action on the part of the authorities."12

The preamble stated that all those lands which had not been granted by Philip, or in his name, or by
the kings who preceded him, belonged to the Crown.13 For those lands granted by the king, the
decree provided for a system of assignment of such lands. It also ordered that all possessors of
agricultural land should exhibit their title deed, otherwise, the land would be restored to the Crown. 14

The Royal Cedula of October 15, 1754 reinforced the Recopilacion when it ordered the Crown's
principal subdelegate to issue a general order directing the publication of the Crown's instructions:

"x x x to the end that any and all persons who, since the year 1700, and up to the date of the
promulgation and publication of said order, shall have occupied royal lands, whether or not x x x
cultivated or tenanted, may x x x appear and exhibit to said subdelegates the titles and patents by
virtue of which said lands are occupied. x x x. Said subdelegates will at the same time warn the
parties interested that in case of their failure to present their title deeds within the term designated,
without a just and valid reason therefor, they will be deprived of and evicted from their lands, and they
will be granted to others."15

On June 25, 1880, the Crown adopted regulations for the adjustment of lands "wrongfully occupied"
by private individuals in the Philippine Islands. Valenton construed these regulations together with
contemporaneous legislative and executive interpretations of the law, and concluded that plaintiffs'
case fared no better under the 1880 decree and other laws which followed it, than it did under the
earlier ones. Thus as a general doctrine, the Court stated:

"While the State has always recognized the right of the occupant to a deed if he proves a possession
for a sufficient length of time, yet it has always insisted that he must make that proof before the
proper administrative officers, and obtain from them his deed, and until he did that the State
remained the absolute owner."16

In conclusion, the Court ruled: "We hold that from 1860 to 1892 there was no law in force in these
Islands by which the plaintiffs could obtain the ownership of these lands by prescription, without any
action by the State."17 Valenton had no rights other than those which accrued to mere possession.
Murciano, on the other hand, was deemed to be the owner of the land by virtue of the grant by the
provincial secretary. In effect, Valenton upheld the Spanish concept of state ownership of public land.

As a fitting observation, the Court added that "[t]he policy pursued by the Spanish Government
from earliest times, requiring settlers on the public lands to obtain title deeds therefor from
the State, has been continued by the American Government in Act No. 926."18

C. The Public Land Acts and the Torrens System

Act No. 926, the first Public Land Act, was passed in pursuance of the provisions of the the Philippine
Bill of 1902. The law governed the disposition of lands of the public domain. It prescribed rules and
regulations for the homesteading, selling, and leasing of portions of the public domain of the
Philippine Islands, and prescribed the terms and conditions to enable persons to perfect their titles to
public lands in the Islands. It also provided for the "issuance of patents to certain native settlers upon
public lands," for the establishment of town sites and sale of lots therein, for the completion of
imperfect titles, and for the cancellation or confirmation of Spanish concessions and grants in the
Islands." In short, the Public Land Act operated on the assumption that title to public lands in the
Philippine Islands remained in the government;19 and that the government's title to public land sprung
from the Treaty of Paris and other subsequent treaties between Spain and the United States.20 The
term "public land" referred to all lands of the public domain whose title still remained in the
government and are thrown open to private appropriation and settlement,21 and excluded the
patrimonial property of the government and the friar lands. 22

Act No. 926 was superseded in 1919 by Act 2874, the second Public Land Act. This new law
was passed under the Jones Law. It was more comprehensive in scope but limited the exploitation of
agricultural lands to Filipinos and Americans and citizens of other countries which gave Filipinos the
same privileges.23 After the passage of the 1935 Constitution, Act 2874 was amended in 1936
by Commonwealth Act No. 141. Commonwealth Act No. 141 remains the present Public Land Law
and it is essentially the same as Act 2874. The main difference between the two relates to the
transitory provisions on the rights of American citizens and corporations during the Commonwealth
period at par with Filipino citizens and corporations.24

Grants of public land were brought under the operation of the Torrens system under Act 496,
or the Land Registration Law of 1903. Enacted by the Philippine Commission, Act 496 placed all
public and private lands in the Philippines under the Torrens system. The law is said to be almost a
verbatim copy of the Massachussetts Land Registration Act of 1898,25 which, in turn, followed the
principles and procedure of the Torrens system of registration formulated by Sir Robert Torrens who
patterned it after the Merchant Shipping Acts in South Australia. The Torrens system requires that the
government issue an official certificate of title attesting to the fact that the person named is the owner
of the property described therein, subject to such liens and encumbrances as thereon noted or the
law warrants or reserves.26 The certificate of title is indefeasible and imprescriptible and all claims to
the parcel of land are quieted upon issuance of said certificate. This system highly facilitates land
conveyance and negotiation.27

D. The Philippine Constitutions

The Regalian doctrine was enshrined in the 1935 Constitution. One of the fixed and dominating
objectives of the 1935 Constitutional Convention was the nationalization and conservation of the
natural resources of the country.28There was an overwhelming sentiment in the Convention in
favor of the principle of state ownership of natural resources and the adoption of the Regalian
doctrine.29 State ownership of natural resources was seen as a necessary starting point to secure
recognition of the state's power to control their disposition, exploitation, development, or
utilization.30 The delegates to the Constitutional Convention very well knew that the concept of State
ownership of land and natural resources was introduced by the Spaniards, however, they were not
certain whether it was continued and applied by the Americans. To remove all doubts, the Convention
approved the provision in the Constitution affirming the Regalian doctrine. 31

Thus, the 1935 Constitution, in Section 1 of Article XIII on "Conservation and Utilization of Natural
Resources," reads as follows:

"Sec. 1. All agricultural, timber, and mineral lands of the public domain, waters, minerals, coal,
petroleum, and other mineral oils, all forces of potential energy, and other natural resources of
the Philippines belong to the State, and their disposition, exploitation, development, or
utilization shall be limited to citizens of the Philippines, or to corporations or associations at
least sixty per centum of the capital of which is owned by such citizens, subject to any
existing right, grant, lease, or concession at the time of the inauguration of the Government
established under this Constitution. Natural resources, with the exception of public
agricultural land, shall not be alienated, and no license, concession, or lease for the exploitation,
development, or utilization of any of the natural resources shall be granted for a period exceeding
twenty-five years, except as to water rights for irrigation, water supply, fisheries, or industrial uses
other than the development of water power, in which cases beneficial use may be the measure and
the limit of the grant."

The 1973 Constitution reiterated the Regalian doctrine in Section 8, Article XIV on the "National
Economy and the Patrimony of the Nation," to wit:

"Sec. 8. All lands of the public domain, waters, minerals, coal, petroleum and other mineral
oils, all forces of potential energy, fisheries, wildlife, and other natural resources of the
Philippines belong to the State. With the exception of agricultural, industrial or commercial,
residential, and resettlement lands of the public domain, natural resources shall not be
alienated, and no license, concession, or lease for the exploration, development, exploitation,
or utilization of any of the natural resources shall be granted for a period exceeding twenty-
five years, renewable for not more than twenty-five years, except as to water rights for irrigation,
water supply, fisheries, or industrial uses other than the development of water power, in which cases
beneficial use may be the measure and the limit of the grant."

The 1987 Constitution reaffirmed the Regalian doctrine in Section 2 of Article XII on "National
Economy and Patrimony," to wit:

"Sec. 2. All lands of the public domain, waters, minerals, coal, petroleum, and other mineral
oils, all forces of potential energy, fisheries, forests or timber, wildlife, flora and fauna, and
other natural resources are owned by the State. With the exception of agricultural lands, all
other natural resources shall not be alienated. The exploration, development and utilization of
natural resources shall be under the full control and supervision of the State. The State may
directly undertake such activities or it may enter into co-production, joint venture, or
production-sharing agreements with Filipino citizens, or corporations or associations at least
sixty per centum of whose capital is owned by such citizens. Such agreements may be for a
period not exceeding twenty-five years, renewable for not more than twenty-five years, and under
such terms and conditions as may be provided by law. In cases of water rights for irrigation, water
supply, fisheries, or industrial uses other than the development of water power, beneficial use may be
the measure and limit of the grant.

x x x."

Simply stated, all lands of the public domain as well as all natural resources enumerated therein,
whether on public or private land, belong to the State. It is this concept of State ownership that
petitioners claim is being violated by the IPRA.

II. THE INDIGENOUS PEOPLES RIGHTS ACT.

Republic Act No. 8371 is entitled "An Act to Recognize, Protect and Promote the Rights of
Indigenous Cultural Communities/ Indigenous Peoples, Creating a National Commission on
Indigenous Peoples, Establishing Implementing Mechanisms, Appropriating Funds Therefor, and for
Other Purposes." It is simply known as "The Indigenous Peoples Rights Act of 1997" or the
IPRA.
The IPRA recognizes the existence of the indigenous cultural communities or indigenous
peoples (ICCs/IPs) as a distinct sector in Philippine society. It grants these people the ownership
and possession of their ancestral domains and ancestral lands, and defines the extent of
these lands and domains. The ownership given is the indigenous concept of ownership under
customary law which traces its origin to native title.

Other rights are also granted the ICCs/IPs, and these are:

- the right to develop lands and natural resources;

- the right to stay in the territories;

- the right in case of displacement;

- the right to safe and clean air and water;

- the right to claim parts of reservations;

- the right to resolve conflict;32

- the right to ancestral lands which include

a. the right to transfer land/property to/among members of the same ICCs/IPs, subject
to customary laws and traditions of the community concerned;

b. the right to redemption for a period not exceeding 15 years from date of transfer, if
the transfer is to a non-member of the ICC/IP and is tainted by vitiated consent of the
ICC/IP, or if the transfer is for an unconscionable consideration.33

Within their ancestral domains and ancestral lands, the ICCs/IPs are given the right to self-
governance and empowerment,34 social justice and human rights,35 the right to preserve and protect
their culture, traditions, institutions and community intellectual rights, and the right to develop their
own sciences and technologies.36

To carry out the policies of the Act, the law created the National Commission on Indigenous Peoples
(NCIP). The NCIP is an independent agency under the Office of the President and is composed of
seven (7) Commissioners belonging to ICCs/IPs from each of the ethnographic areas- Region I and
the Cordilleras; Region II; the rest of Luzon; Island groups including Mindoro, Palawan, Romblon,
Panay and the rest of the Visayas; Northern and Western Mindanao; Southern and Eastern
Mindanao; and Central Mindanao.37 The NCIP took over the functions of the Office for Northern
Cultural Communities and the Office for Southern Cultural Communities created by former President
Corazon Aquino which were merged under a revitalized structure.38

Disputes involving ICCs/IPs are to be resolved under customary laws and practices. When still
unresolved, the matter may be brought to the NCIP, which is granted quasi-judicial powers.39 The
NCIP's decisions may be appealed to the Court of Appeals by a petition for review.

Any person who violates any of the provisions of the Act such as, but not limited to, unauthorized
and/or unlawful intrusion upon ancestral lands and domains shall be punished in accordance with
customary laws or imprisoned from 9 months to 12 years and/or fined from ₱100,000.00 to
₱500,000.00 and obliged to pay damages.40
A. Indigenous Peoples

The IPRA is a law dealing with a specific group of people, i.e., the Indigenous Cultural Communities
(ICCs) or the Indigenous Peoples (IPs). The term "ICCs" is used in the 1987 Constitution while that of
"IPs" is the contemporary international language in the International Labor Organization (ILO)
Convention 16941 and the United Nations (UN) Draft Declaration on the Rights of Indigenous
Peoples.42

ICCs/IPs are defined by the IPRA as:

"Sec. 3 [h]. Indigenous Cultural Communities/ Indigenous Peoples- refer to a group of people or
homogeneous societies identified by self-ascription and ascription by others, who have continuously
lived as organized community on communally bounded and defined territory, and who have, under
claims of ownership since time immemorial, occupied, possessed and utilized such territories, sharing
common bonds of language, customs, traditions and other distinctive cultural traits, or who have,
through resistance to political, social and cultural inroads of colonization, non-indigenous religions
and cultures, became historically differentiated from the majority of Filipinos. ICCs/IPs shall likewise
include peoples who are regarded as indigenous on account of their descent from the populations
which inhabited the country, at the time of conquest or colonization, or at the time of inroads of non-
indigenous religions and cultures, or the establishment of present state boundaries, who retain some
or all of their own social, economic, cultural and political institutions, but who may have been
displaced from their traditional domains or who may have resettled outside their ancestral domains."

Indigenous Cultural Communities or Indigenous Peoples refer to a group of people or


homogeneous societies who have continuously lived as an organized community on
communally bounded and defined territory. These groups of people have actually occupied,
possessed and utilized their territories under claim of ownership since time immemorial. They share
common bonds of language, customs, traditions and other distinctive cultural traits, or, they, by their
resistance to political, social and cultural inroads of colonization, non-indigenous religions and
cultures, became historically differentiated from the Filipino majority. ICCs/IPs also include
descendants of ICCs/IPs who inhabited the country at the time of conquest or colonization, who retain
some or all of their own social, economic, cultural and political institutions but who may have been
displaced from their traditional territories or who may have resettled outside their ancestral domains.

1. Indigenous Peoples: Their History

Presently, Philippine indigenous peoples inhabit the interiors and mountains of Luzon, Mindanao,
Mindoro, Negros, Samar, Leyte, and the Palawan and Sulu group of islands. They are composed of
110 tribes and are as follows:

1. In the Cordillera Autonomous Region- Kankaney, Ibaloi, Bontoc, Tinggian or Itneg, Ifugao,
Kalinga, Yapayao, Aeta or Agta or Pugot, and Bago of Ilocos Norte and Pangasinan; Ibanag of
Isabela, Cagayan; Ilongot of Quirino and Nueva Vizcaya; Gaddang of Quirino, Nueva Vizcaya,
Itawis of Cagayan; Ivatan of Batanes, Aeta of Cagayan, Quirino and Isabela.

2. In Region III- Aetas.

3. In Region IV- Dumagats of Aurora, Rizal; Remontado of Aurora, Rizal, Quezon; Alangan or
Mangyan, Batangan, Buid or Buhid, Hanunuo and Iraya of Oriental and Occidental Mindoro;
Tadyawan of Occidental Mindoro; Cuyonon, Palawanon, Tagbanua and Tao't bato of Palawan.
4. In Region V- Aeta of Camarines Norte and Camarines Sur; Aeta-Abiyan, Isarog, and
Kabihug of Camarines Norte; Agta, and Mayon of Camarines Sur; Itom of Albay, Cimaron of
Sorsogon; and the Pullon of Masbate and Camarines Sur.

5. In Region VI- Ati of Negros Occidental, Iloilo and Antique, Capiz; the Magahat of Negros
Occidental; the Corolano and Sulod.

6. In Region VII- Magahat of Negros Oriental and Eskaya of Bohol.

7. In Region IX- the Badjao numbering about 192,000 in Tawi-Tawi, Zamboanga del Sur; the
Kalibugan of Basilan, the Samal, Subanon and Yakat.

8. Region X- Numbering 1.6 million in Region X alone, the IPs are: the Banwaon, Bukidnon,
Matigsalog, Talaanding of Bukidnon; the Camiguin of Camiguin Island; the Higa-unon of
Agusan del Norte, Agusan del Sur, Bukidnon and Misamis Occidental; the Tigwahanon of
Agusan del Sur, Misamis Oriental and and Misamis Occidental, the Manobo of the Agusan
provinces, and the Umayamnon of Agusan and Bukidnon.

9. In Region XI- There are about 1,774,065 IPs in Region XI. They are tribes of the Dibabaon,
Mansaka of Davao del Norte; B'laan, Kalagan, Langilad, T'boli and Talaingod of Davao del
Sur; Mamamanua of Surigao del Sur; Mandaya of the Surigao provinces and Davao Oriental;
Manobo Blit of South Cotabato; the Mangguangon of Davao and South Cotabato; Matigsalog
of Davao del Norte and Del Sur; Tagakaolo, Tasaday and Ubo of South Cotabato; and Bagobo
of Davao del sur and South Cotabato.

10. In Region XII- Ilianen, Tiruray, Maguindanao, Maranao, Tausug, Yakan/Samal, and
Iranon.43

How these indigenous peoples came to live in the Philippines goes back to as early as 25,000
to 30,000 B.C.

Before the time of Western contact, the Philippine archipelago was peopled largely by the Negritos,
Indonesians and Malays.44 The strains from these groups eventually gave rise to common cultural
features which became the dominant influence in ethnic reformulation in the archipelago. Influences
from the Chinese and Indian civilizations in the third or fourth millenium B.C. augmented these ethnic
strains. Chinese economic and socio-cultural influences came by way of Chinese porcelain, silk and
traders. Indian influence found their way into the religious-cultural aspect of pre-colonial society.45

The ancient Filipinos settled beside bodies of water. Hunting and food gathering became
supplementary activities as reliance on them was reduced by fishing and the cultivation of the
soil.46 From the hinterland, coastal, and riverine communities, our ancestors evolved an essentially
homogeneous culture, a basically common way of life where nature was a primary
factor. Community life throughout the archipelago was influenced by, and responded to, common
ecology. The generally benign tropical climate and the largely uniform flora and fauna favored
similarities, not differences.47 Life was essentially subsistence but not harsh.48

The early Filipinos had a culture that was basically Malayan in structure and form. They had
languages that traced their origin to the Austronesian parent-stock and used them not only as media
of daily communication but also as vehicles for the expression of their literary moods. 49 They
fashioned concepts and beliefs about the world that they could not see, but which they sensed to be
part of their lives.50 They had their own religion and religious beliefs. They believed in the immortality
of the soul and life after death. Their rituals were based on beliefs in a ranking deity whom they called
Bathalang Maykapal, and a host of other deities, in the environmental spirits and in soul spirits. The
early Filipinos adored the sun, the moon, the animals and birds, for they seemed to consider the
objects of Nature as something to be respected. They venerated almost any object that was close to
their daily life, indicating the importance of the relationship between man and the object of nature. 51

The unit of government was the "barangay," a term that derived its meaning from the Malay word
"balangay," meaning, a boat, which transported them to these shores. 52 The barangay was basically
a family-based community and consisted of thirty to one hundred families. Each barangay was
different and ruled by a chieftain called a "dato." It was the chieftain's duty to rule and govern his
subjects and promote their welfare and interests. A chieftain had wide powers for he exercised all the
functions of government. He was the executive, legislator and judge and was the supreme
commander in time of war.53

Laws were either customary or written. Customary laws were handed down orally from
generation to generation and constituted the bulk of the laws of the barangay. They were
preserved in songs and chants and in the memory of the elder persons in the community.54 The
written laws were those that the chieftain and his elders promulgated from time to time as the
necessity arose.55 The oldest known written body of laws was the Maragtas Code by Datu Sumakwel
at about 1250 A.D. Other old codes are the Muslim Code of Luwaran and the Principal Code of
Sulu.56 Whether customary or written, the laws dealt with various subjects, such as inheritance,
divorce, usury, loans, partnership, crime and punishment, property rights, family relations and
adoption. Whenever disputes arose, these were decided peacefully through a court composed by the
chieftain as "judge" and the barangay elders as "jury." Conflicts arising between subjects of different
barangays were resolved by arbitration in which a board composed of elders from neutral barangays
acted as arbiters.57

Baranganic society had a distinguishing feature: the absence of private property in land. The
chiefs merely administered the lands in the name of the barangay. The social order was an extension
of the family with chiefs embodying the higher unity of the community. Each individual, therefore,
participated in the community ownership of the soil and the instruments of production as a member of
the barangay.58 This ancient communalism was practiced in accordance with the concept of mutual
sharing of resources so that no individual, regardless of status, was without sustenance. Ownership
of land was non-existent or unimportant and the right of usufruct was what regulated the
development of lands.59 Marine resources and fishing grounds were likewise free to all. Coastal
communities depended for their economic welfare on the kind of fishing sharing concept similar to
those in land communities.60 Recognized leaders, such as the chieftains and elders, by virtue of their
positions of importance, enjoyed some economic privileges and benefits. But their rights, related to
either land and sea, were subject to their responsibility to protect the communities from danger and to
provide them with the leadership and means of survival.61

Sometime in the 13th century, Islam was introduced to the archipelago in Maguindanao. The
Sultanate of Sulu was established and claimed jurisdiction over territorial areas represented today by
Tawi-tawi, Sulu, Palawan, Basilan and Zamboanga. Four ethnic groups were within this jurisdiction:
Sama, Tausug, Yakan and Subanon.62The Sultanate of Maguindanao spread out from Cotabato
toward Maranao territory, now Lanao del Norte and Lanao del Sur.63

The Muslim societies evolved an Asiatic form of feudalism where land was still held in
common but was private in use. This is clearly indicated in the Muslim Code of Luwaran. The Code
contains a provision on the lease of cultivated lands. It, however, has no provision for the acquisition,
transfer, cession or sale of land.64
The societies encountered by Magellan and Legaspi therefore were primitive economies where
most production was geared to the use of the producers and to the fulfillment of kinship obligations.
They were not economies geared to exchange and profit. 65 Moreover, the family basis of barangay
membership as well as of leadership and governance worked to splinter the population of the islands
into numerous small and separate communities.66

When the Spaniards settled permanently in the Philippines in 1565, they found the Filipinos
living in barangay settlements scattered along water routes and river banks. One of the first
tasks imposed on the missionaries and the encomenderos was to collect all scattered Filipinos
together in a reduccion.67 As early as 1551, the Spanish government assumed an unvarying
solicitous attitude towards the natives.68 The Spaniards regarded it a sacred "duty to conscience and
humanity to civilize these less fortunate people living in the obscurity of ignorance" and to accord
them the "moral and material advantages" of community life and the "protection and vigilance
afforded them by the same laws."69

The Spanish missionaries were ordered to establish pueblos where the church and convent would be
constructed. All the new Christian converts were required to construct their houses around the church
and the unbaptized were invited to do the same.70 With the reduccion, the Spaniards attempted to
"tame" the reluctant Filipinos through Christian indoctrination using the convento/casa
real/plaza complex as focal point. The reduccion, to the Spaniards, was a "civilizing" device to make
the Filipinos law-abiding citizens of the Spanish Crown, and in the long run, to make them ultimately
adopt Hispanic culture and civilization.71

All lands lost by the old barangays in the process of pueblo organization as well as all lands
not assigned to them and the pueblos, were now declared to be crown lands or realengas,
belonging to the Spanish king. It was from the realengas that land grants were made to non-
Filipinos.72

The abrogation of the Filipinos' ancestral rights in land and the introduction of the concept of
public domain were the most immediate fundamental results of Spanish colonial theory and
law.73 The concept that the Spanish king was the owner of everything of value in the Indies or
colonies was imposed on the natives, and the natives were stripped of their ancestral rights to
land.74

Increasing their foothold in the Philippines, the Spanish colonialists, civil and religious, classified the
Filipinos according to their religious practices and beliefs, and divided them into three types . First
were the Indios, the Christianized Filipinos, who generally came from the lowland populations.
Second, were the Moros or the Muslim communities, and third, were the infieles or the indigenous
communities.75

The Indio was a product of the advent of Spanish culture. This class was favored by the Spaniards
and was allowed certain status although below the Spaniards. The Moros and infieles were
regarded as the lowest classes.76

The Moros and infieles resisted Spanish rule and Christianity. The Moros were driven from
Manila and the Visayas to Mindanao; while the infieles, to the hinterlands. The Spaniards did not
pursue them into the deep interior. The upland societies were naturally outside the immediate
concern of Spanish interest, and the cliffs and forests of the hinterlands were difficult and
inaccessible, allowing the infieles, in effect, relative security.77 Thus, the infieles, which were
peripheral to colonial administration, were not only able to preserve their own culture but also
thwarted the Christianization process, separating themselves from the newly evolved Christian
community.78 Their own political, economic and social systems were kept constantly alive and vibrant.
The pro-Christian or pro-Indio attitude of colonialism brought about a generally mutual feeling of
suspicion, fear, and hostility between the Christians on the one hand and the non-Christians on the
other. Colonialism tended to divide and rule an otherwise culturally and historically related populace
through a colonial system that exploited both the virtues and vices of the Filipinos.79

President McKinley, in his instructions to the Philippine Commission of April 7, 1900,


addressed the existence of the infieles:

"In dealing with the uncivilized tribes of the Islands, the Commission should adopt the same
course followed by Congress in permitting the tribes of our North American Indians to
maintain their tribal organization and government, and under which many of those tribes are now
living in peace and contentment, surrounded by civilization to which they are unable or unwilling to
conform. Such tribal government should, however, be subjected to wise and firm regulation; and,
without undue or petty interference, constant and active effort should be exercised to prevent
barbarous practices and introduce civilized customs."80

Placed in an alternative of either letting the natives alone or guiding them in the path of civilization,
the American government chose "to adopt the latter measure as one more in accord with humanity
and with the national conscience."81

The Americans classified the Filipinos into two: the Christian Filipinos and the non-Christian
Filipinos. The term "non-Christian" referred not to religious belief, but to a geographical area, and
more directly, "to natives of the Philippine Islands of a low grade of civilization, usually living in tribal
relationship apart from settled communities."82

Like the Spaniards, the Americans pursued a policy of assimilation. In 1903, they passed Act
No. 253 creating the Bureau of Non-Christian Tribes (BNCT). Under the Department of the
Interior, the BNCT's primary task was to conduct ethnographic research among unhispanized
Filipinos, including those in Muslim Mindanao, with a "special view to determining the most
practicable means for bringing about their advancement in civilization and prosperity." The BNCT
was modeled after the bureau dealing with American Indians. The agency took a keen
anthropological interest in Philippine cultural minorities and produced a wealth of valuable materials
about them.83

The 1935 Constitution did not carry any policy on the non-Christian Filipinos. The raging issue
then was the conservation of the national patrimony for the Filipinos.

In 1957, the Philippine Congress passed R.A. No. 1888, an "Act to effectuate in a more rapid and
complete manner the economic, social, moral and political advancement of the non-Christian Filipinos
or national cultural minorities and to render real, complete, and permanent the integration of all said
national cultural minorities into the body politic, creating the Commission on National
Integration charged with said functions." The law called for a policy of integration of indigenous
peoples into the Philippine mainstream and for this purpose created the Commission on National
Integration (CNI).84 The CNI was given, more or less, the same task as the BNCT during the
American regime. The post-independence policy of integration was like the colonial policy of
assimilation understood in the context of a guardian-ward relationship.85

The policy of assimilation and integration did not yield the desired result. Like the Spaniards and
Americans, government attempts at integration met with fierce resistance. Since World War II, a
tidal wave of Christian settlers from the lowlands of Luzon and the Visayas swamped the highlands
and wide open spaces in Mindanao.86 Knowledge by the settlers of the Public Land Acts and the
Torrens system resulted in the titling of several ancestral lands in the settlers' names. With
government initiative and participation, this titling displaced several indigenous peoples from
their lands. Worse, these peoples were also displaced by projects undertaken by the national
government in the name of national development.87

It was in the 1973 Constitution that the State adopted the following provision:

"The State shall consider the customs, traditions, beliefs, and interests of national cultural
communities in the formulation and implementation of State policies."88

For the first time in Philippine history, the "non-Christian tribes" or the "cultural minorities"
were addressed by the highest law of the Republic, and they were referred to as "cultural
communities." More importantly this time, their "uncivilized" culture was given some recognition and
their "customs, traditions, beliefs and interests" were to be considered by the State in the formulation
and implementation of State policies. President Marcos abolished the CNI and transferred its
functions to the Presidential Adviser on National Minorities (PANAMIN). The PANAMIN was
tasked to integrate the ethnic groups that sought full integration into the larger community, and at the
same time "protect the rights of those who wish to preserve their original lifeways beside the larger
community."89 In short, while still adopting the integration policy, the decree recognized the
right of tribal Filipinos to preserve their way of life.90

In 1974, President Marcos promulgated P.D. No. 410, otherwise known as the Ancestral Lands
Decree. The decree provided for the issuance of land occupancy certificates to members of the
national cultural communities who were given up to 1984 to register their claims. 91 In 1979,
the Commission on the Settlement of Land Problems was created under E.O. No. 561 which
provided a mechanism for the expeditious resolution of land problems involving small settlers,
landowners, and tribal Filipinos.92

Despite the promulgation of these laws, from 1974 to the early 1980's, some 100,000 Kalingas and
Bontoks of the Cordillera region were displaced by the Chico River dam project of the National Power
Corporation (NPC). The Manobos of Bukidnon saw their land bulldozed by the Bukidnon Sugar
Industries Company (BUSCO). In Agusan del Sur, the National Development Company was
authorized by law in 1979 to take approximately 40,550 hectares of land that later became the NDC-
Guthrie plantation in Agusan del Sur. Most of the land was possessed by the Agusan
natives.93 Timber concessions, water projects, plantations, mining, and cattle ranching and other
projects of the national government led not only to the eviction of the indigenous peoples from their
land but also to the reduction and destruction of their natural environment. 94

The Aquino government signified a total shift from the policy of integration to one of
preservation. Invoking her powers under the Freedom Constitution, President Aquino created
the Office of Muslim Affairs, Office for Northern Cultural Communities and the Office for
Southern Cultural Communities all under the Office of the President.95

The 1987 Constitution carries at least six (6) provisions which insure the right of tribal
Filipinos to preserve their way of life.96 This Constitution goes further than the 1973
Constitution by expressly guaranteeing the rights of tribal Filipinos to their ancestral domains
and ancestral lands. By recognizing their right to their ancestral lands and domains, the State
has effectively upheld their right to live in a culture distinctly their own.

2. Their Concept of Land

Indigenous peoples share distinctive traits that set them apart from the Filipino mainstream. They
are non-Christians. They live in less accessible, marginal, mostly upland areas. They have a system
of self-government not dependent upon the laws of the central administration of the Republic of the
Philippines. They follow ways of life and customs that are perceived as different from those of the rest
of the population.97 The kind of response the indigenous peoples chose to deal with colonial threat
worked well to their advantage by making it difficult for Western concepts and religion to erode their
customs and traditions. The "infieles societies" which had become peripheral to colonial
administration, represented, from a cultural perspective, a much older base of archipelagic culture.
The political systems were still structured on the patriarchal and kinship oriented arrangement of
power and authority. The economic activities were governed by the concepts of an ancient
communalism and mutual help. The social structure which emphasized division of labor and
distinction of functions, not status, was maintained. The cultural styles and forms of life portraying the
varieties of social courtesies and ecological adjustments were kept constantly vibrant. 98

Land is the central element of the indigenous peoples' existence. There is no traditional concept
of permanent, individual, land ownership. Among the Igorots, ownership of land more accurately
applies to the tribal right to use the land or to territorial control. The people are the secondary owners
or stewards of the land and that if a member of the tribe ceases to work, he loses his claim of
ownership, and the land reverts to the beings of the spirit world who are its true and primary owners.
Under the concept of "trusteeship," the right to possess the land does not only belong to the present
generation but the future ones as well.99

Customary law on land rests on the traditional belief that no one owns the land except the gods and
spirits, and that those who work the land are its mere stewards. 100 Customary law has a strong
preference for communal ownership, which could either be ownership by a group of individuals or
families who are related by blood or by marriage,101 or ownership by residents of the same locality
who may not be related by blood or marriage. The system of communal ownership under customary
laws draws its meaning from the subsistence and highly collectivized mode of economic production.
The Kalingas, for instance, who are engaged in team occupation like hunting, foraging for forest
products, and swidden farming found it natural that forest areas, swidden farms, orchards, pasture
and burial grounds should be communally-owned.102 For the Kalingas, everybody has a common right
to a common economic base. Thus, as a rule, rights and obligations to the land are shared in
common.

Although highly bent on communal ownership, customary law on land also sanctions
individual ownership. The residential lots and terrace rice farms are governed by a limited system
of individual ownership. It is limited because while the individual owner has the right to use and
dispose of the property, he does not possess all the rights of an exclusive and full owner as defined
under our Civil Code.103 Under Kalinga customary law, the alienation of individually-owned land is
strongly discouraged except in marriage and succession and except to meet sudden financial needs
due to sickness, death in the family, or loss of crops.104 Moreover, and to be alienated should first be
offered to a clan-member before any village-member can purchase it, and in no case may land be
sold to a non-member of the ili.105

Land titles do not exist in the indigenous peoples' economic and social system. The concept
of individual land ownership under the civil law is alien to them. Inherently colonial in origin,
our national land laws and governmental policies frown upon indigenous claims to ancestral
lands. Communal ownership is looked upon as inferior, if not inexistent.106

III. THE IPRA IS A NOVEL PIECE OF LEGISLATION.

A. The Legislative History of the IPRA


It was to address the centuries-old neglect of the Philippine indigenous peoples that the Tenth
Congress of the Philippines, by their joint efforts, passed and approved R.A. No. 8371, the
Indigenous Peoples Rights Act (IPRA) of 1997. The law was a consolidation of two Bills- Senate
Bill No. 1728 and House Bill No. 9125.

Principally sponsored by Senator Juan M. Flavier,107 Senate Bill No. 1728 was a consolidation of
four proposed measures referred to the Committees on Cultural Communities, Environment and
Natural Resources, Ways and Means, as well as Finance. It adopted almost en toto the
comprehensive version of Senate Bill Nos. 1476 and 1486 which was a result of six regional
consultations and one national consultation with indigenous peoples nationwide.108 At the
Second Regular Session of the Tenth Congress, Senator Flavier, in his sponsorship speech, gave a
background on the situation of indigenous peoples in the Philippines, to wit:

"The Indigenous Cultural Communities, including the Bangsa Moro, have long suffered from the
dominance and neglect of government controlled by the majority. Massive migration of their Christian
brothers to their homeland shrunk their territory and many of the tribal Filipinos were pushed to the
hinterlands. Resisting the intrusion, dispossessed of their ancestral land and with the massive
exploitation of their natural resources by the elite among the migrant population, they became
marginalized. And the government has been an indispensable party to this insidious conspiracy
against the Indigenous Cultural Communities (ICCs). It organized and supported the resettlement of
people to their ancestral land, which was massive during the Commonwealth and early years of the
Philippine Republic. Pursuant to the Regalian Doctrine first introduced to our system by Spain
through the Royal Decree of 13 February 1894 or the Maura Law, the government passed laws to
legitimize the wholesale landgrabbing and provide for easy titling or grant of lands to migrant
homesteaders within the traditional areas of the ICCs." 109

Senator Flavier further declared:

"The IPs are the offsprings and heirs of the peoples who have first inhabited and cared for the land
long before any central government was established. Their ancestors had territories over which they
ruled themselves and related with other tribes. These territories- the land- include people, their
dwelling, the mountains, the water, the air, plants, forest and the animals. This is their environment in
its totality. Their existence as indigenous peoples is manifested in their own lives through political,
economic, socio-cultural and spiritual practices. The IPs culture is the living and irrefutable proof to
this.

Their survival depends on securing or acquiring land rights; asserting their rights to it; and depending
on it. Otherwise, IPs shall cease to exist as distinct peoples."110

To recognize the rights of the indigenous peoples effectively, Senator Flavier proposed a bill based
on two postulates: (1) the concept of native title; and (2) the principle of parens patriae.

According to Senator Flavier, "[w]hile our legal tradition subscribes to the Regalian Doctrine
reinstated in Section 2, Article XII of the 1987 Constitution," our "decisional laws" and jurisprudence
passed by the State have "made exception to the doctrine." This exception was first laid down in the
case of Cariño v. Insular Government where:

"x x x the court has recognized long occupancy of land by an indigenous member of the cultural
communities as one of private ownership, which, in legal concept, is termed "native title." This ruling
has not been overturned. In fact, it was affirmed in subsequent cases." 111
Following Cariño, the State passed Act No. 926, Act No. 2874, C.A. No. 141, P.D. 705, P.D. 410,
P.D. 1529, R.A. 6734 (the Organic Act for the Autonomous Region of Muslim Mindanao). These laws,
explicitly or implicitly, and liberally or restrictively, recognized "native title" or "private right" and the
existence of ancestral lands and domains. Despite the passage of these laws, however, Senator
Flavier continued:

"x x x the executive department of government since the American occupation has not implemented
the policy. In fact, it was more honored in its breach than in its observance, its wanton disregard
shown during the period unto the Commonwealth and the early years of the Philippine Republic when
government organized and supported massive resettlement of the people to the land of the ICCs."

Senate Bill No. 1728 seeks to genuinely recognize the IPs right to own and possess their ancestral
land. The bill was prepared also under the principle of parens patriae inherent in the supreme power
of the State and deeply embedded in Philippine legal tradition. This principle mandates that persons
suffering from serious disadvantage or handicap, which places them in a position of actual inequality
in their relation or transaction with others, are entitled to the protection of the State.

Senate Bill No. 1728 was passed on Third Reading by twenty-one (21) Senators voting in favor
and none against, with no abstention.112

House Bill No. 9125 was sponsored by Rep. Zapata, Chairman of the Committee on Cultural
Communities. It was originally authored and subsequently presented and defended on the floor
by Rep. Gregorio Andolana of North Cotabato.113

Rep. Andolana's sponsorhip speech reads as follows:

"This Representation, as early as in the 8th Congress, filed a bill of similar implications that would
promote, recognize the rights of indigenous cultural communities within the framework of national
unity and development.

Apart from this, Mr. Speaker, is our obligation, the government's obligation to assure and ascertain
that these rights shall be well-preserved and the cultural traditions as well as the indigenous laws that
remained long before this Republic was established shall be preserved and promoted. There is a
need, Mr. Speaker, to look into these matters seriously and early approval of the substitute bill shall
bring into reality the aspirations, the hope and the dreams of more than 12 million Filipinos that they
be considered in the mainstream of the Philippine society as we fashion for the year 2000." 114

Rep. Andolana stressed that H.B. No. 9125 is based on the policy of preservation as mandated in the
Constitution. He also emphasized that the rights of IPs to their land was enunciated in Cariño v.
Insular Government which recognized the fact that they had vested rights prior to the establishment
of the Spanish and American regimes.115

After exhaustive interpellation, House Bill No. 9125, and its corresponding amendments, was
approved on Second Reading with no objections.

IV. THE PROVISIONS OF THE IPRA DO NOT CONTRAVENE THE CONSTITUTION.

A. Ancestral Domains and Ancestral Lands are the Private Property of Indigenous Peoples
and Do Not Constitute Part of the Land of the Public Domain.
The IPRA grants to ICCs/IPs a distinct kind of ownership over ancestral domains and
ancestral lands. Ancestral lands are not the same as ancestral domains. These are defined in
Section 3 [a] and [b] of the Indigenous Peoples Right Act, viz:

"Sec. 3 a) Ancestral Domains. - Subject to Section 56 hereof, refer to all areas generally belonging
to ICCs/IPs comprising lands, inland waters, coastal areas, and natural resources therein, held under
a claim of ownership, occupied or possessed by ICCs/IPs by themselves or through their ancestors,
communally or individually since time immemorial, continuously to the present except when
interrupted by war, force majeure or displacement by force, deceit, stealth or as a consequence of
government projects or any other voluntary dealings entered into by government and private
individuals/corporations, and which are necessary to ensure their economic, social and cultural
welfare. It shall include ancestral lands, forests, pasture, residential, agricultural, and other lands
individually owned whether alienable and disposable or otherwise, hunting grounds, burial grounds,
worship areas, bodies of water, mineral and other natural resources, and lands which may no longer
be exclusively occupied by ICCs/IPs but from which they traditionally had access to for their
subsistence and traditional activities, particularly the home ranges of ICCs/IPs who are still nomadic
and/or shifting cultivators;

b) Ancestral Lands.- Subject to Section 56 hereof, refers to land occupied, possessed and utilized
by individuals, families and clans who are members of the ICCs/IPs since time immemorial, by
themselves or through their predecessors-in-interest, under claims of individual or traditional group
ownership, continuously, to the present except when interrupted by war, force majeure or
displacement by force, deceit, stealth, or as a consequence of government projects and other
voluntary dealings entered into by government and private individuals/corporations, including, but not
limited to, residential lots, rice terraces or paddies, private forests, swidden farms and tree lots."

Ancestral domains are all areas belonging to ICCs/IPs held under a claim of ownership, occupied or
possessed by ICCs/IPs by themselves or through their ancestors, communally or individually since
time immemorial, continuously until the present, except when interrupted by war, force majeure or
displacement by force, deceit, stealth or as a consequence of government projects or any other
voluntary dealings with government and/or private individuals or corporations. Ancestral domains
comprise lands, inland waters, coastal areas, and natural resources therein and includes
ancestral lands, forests, pasture, residential, agricultural, and other lands individually owned
whether alienable or not, hunting grounds, burial grounds, worship areas, bodies of water,
mineral and other natural resources. They also include lands which may no longer be exclusively
occupied by ICCs/IPs but from which they traditionally had access to for their subsistence and
traditional activities, particularly the home ranges of ICCs/IPs who are still nomadic and/or shifting
cultivators.116

Ancestral lands are lands held by the ICCs/IPs under the same conditions as ancestral domains
except that these are limited to lands and that these lands are not merely occupied and possessed
but are also utilized by the ICCs/IPs under claims of individual or traditional group ownership. These
lands include but are not limited to residential lots, rice terraces or paddies, private forests, swidden
farms and tree lots.117

The procedures for claiming ancestral domains and lands are similar to the procedures embodied in
Department Administrative Order (DAO) No. 2, series of 1993, signed by then Secretary of the
Department of Environment and Natural Resources (DENR) Angel Alcala. 118 DAO No. 2 allowed the
delineation of ancestral domains by special task forces and ensured the issuance of Certificates of
Ancestral Land Claims (CALC's) and Certificates of Ancestral Domain Claims (CADC's) to IPs.
The identification and delineation of these ancestral domains and lands is a power conferred by the
IPRA on the National Commission on Indigenous Peoples (NCIP).119 The guiding principle in
identification and delineation is self-delineation.120 This means that the ICCs/IPs have a decisive role
in determining the boundaries of their domains and in all the activities pertinent thereto. 121

The procedure for the delineation and recognition of ancestral domains is set forth in Sections 51
and 52 of the IPRA. The identification, delineation and certification of ancestral lands is in Section 53
of said law.

Upon due application and compliance with the procedure provided under the law and upon finding by
the NCIP that the application is meritorious, the NCIP shall issue a Certificate of Ancestral Domain
Title (CADT) in the name of the community concerned.122 The allocation of lands within the
ancestral domain to any individual or indigenous corporate (family or clan) claimants is left to the
ICCs/IPs concerned to decide in accordance with customs and traditions.123 With respect to
ancestral lands outside the ancestral domain, the NCIP issues a Certificate of Ancestral Land Title
(CALT).124

CADT's and CALT's issued under the IPRA shall be registered by the NCIP before the Register of
Deeds in the place where the property is situated.125

(1) Right to Ancestral Domains and Ancestral Lands: How Acquired

The rights of the ICCs/IPs to their ancestral domains and ancestral lands may be acquired in two
modes: (1) by native title over both ancestral lands and domains; or (2) by torrens title under
the Public Land Act and the Land Registration Act with respect to ancestral lands only.

(2) The Concept of Native Title

Native title is defined as:

"Sec. 3 [l]. Native Title- refers to pre-conquest rights to lands and domains which, as far back as
memory reaches, have been held under a claim of private ownership by ICCs/IPs, have never been
public lands and are thus indisputably presumed to have been held that way since before the
Spanish Conquest."126

Native title refers to ICCs/IPs' preconquest rights to lands and domains held under a claim of private
ownership as far back as memory reaches. These lands are deemed never to have been public lands
and are indisputably presumed to have been held that way since before the Spanish Conquest. The
rights of ICCs/IPs to their ancestral domains (which also include ancestral lands) by virtue of native
title shall be recognized and respected.127 Formal recognition, when solicited by ICCs/IPs concerned,
shall be embodied in a Certificate of Ancestral Domain Title (CADT), which shall recognize the title of
the concerned ICCs/IPs over the territories identified and delineated. 128

Like a torrens title, a CADT is evidence of private ownership of land by native title. Native title,
however, is a right of private ownership peculiarly granted to ICCs/IPs over their ancestral lands and
domains. The IPRA categorically declares ancestral lands and domains held by native title as never
to have been public land. Domains and lands held under native title are, therefore, indisputably
presumed to have never been public lands and are private.

(a) Cariño v. Insular Government129


The concept of native title in the IPRA was taken from the 1909 case of Cariño v. Insular
Government.130 Cariño firmly established a concept of private land title that existed irrespective of
any royal grant from the State.

In 1903, Don Mateo Cariño, an Ibaloi, sought to register with the land registration court 146 hectares
of land in Baguio Municipality, Benguet Province. He claimed that this land had been possessed and
occupied by his ancestors since time immemorial; that his grandfather built fences around the
property for the holding of cattle and that his father cultivated some parts of the land. Cariño inherited
the land in accordance with Igorot custom. He tried to have the land adjusted under the Spanish land
laws, but no document issued from the Spanish Crown.131In 1901, Cariño obtained a possessory title
to the land under the Spanish Mortgage Law.132 The North American colonial government, however,
ignored his possessory title and built a public road on the land prompting him to seek a Torrens title to
his property in the land registration court. While his petition was pending, a U.S. military
reservation133 was proclaimed over his land and, shortly thereafter, a military detachment was
detailed on the property with orders to keep cattle and trespassers, including Cariño, off the land.134

In 1904, the land registration court granted Cariño's application for absolute ownership to the land.
Both the Government of the Philippine Islands and the U.S. Government appealed to the C.F.I. of
Benguet which reversed the land registration court and dismissed Cariño's application. The Philippine
Supreme Court135 affirmed the C.F.I. by applying the Valenton ruling. Cariño took the case to the
U.S. Supreme Court.136 On one hand, the Philippine government invoked the Regalian doctrine and
contended that Cariño failed to comply with the provisions of the Royal Decree of June 25, 1880,
which required registration of land claims within a limited period of time. Cariño, on the other,
asserted that he was the absolute owner of the land jure gentium, and that the land never formed part
of the public domain.

In a unanimous decision written by Justice Oliver Wendell Holmes, the U.S. Supreme Court held:

"It is true that Spain, in its earlier decrees, embodied the universal feudal theory that all lands were
held from the Crown, and perhaps the general attitude of conquering nations toward people not
recognized as entitled to the treatment accorded to those in the same zone of civilization with
themselves. It is true, also, that in legal theory, sovereignty is absolute, and that, as against foreign
nations, the United States may assert, as Spain asserted, absolute power. But it does not follow that,
as against the inhabitants of the Philippines, the United States asserts that Spain had such power.
When theory is left on one side, sovereignty is a question of strength, and may vary in degree. How
far a new sovereign shall insist upon the theoretical relation of the subjects to the head in the past,
and how far it shall recognize actual facts, are matters for it to decide." 137

The U.S. Supreme Court noted that it need not accept Spanish doctrines. The choice was with the
new colonizer. Ultimately, the matter had to be decided under U.S. law.

The Cariño decision largely rested on the North American constitutionalist's concept of "due process"
as well as the pronounced policy "to do justice to the natives." 138 It was based on the strong mandate
extended to the Islands via the Philippine Bill of 1902 that "No law shall be enacted in said islands
which shall deprive any person of life, liberty, or property without due process of law, or deny to any
person therein the equal protection of the laws." The court declared:

"The acquisition of the Philippines was not like the settlement of the white race in the United States.
Whatever consideration may have been shown to the North American Indians, the dominant purpose
of the whites in America was to occupy land. It is obvious that, however stated, the reason for our
taking over the Philippines was different. No one, we suppose, would deny that, so far as consistent
with paramount necessities, our first object in the internal administration of the islands is to do justice
to the natives, not to exploit their country for private gain. By the Organic Act of July 1, 1902, chapter
1369, section 12 (32 Statutes at Large, 691), all the property and rights acquired there by the United
States are to be administered 'for the benefit of the inhabitants thereof.' It is reasonable to suppose
that the attitude thus assumed by the United States with regard to what was unquestionably its own is
also its attitude in deciding what it will claim for its own. The same statute made a bill of rights,
embodying the safeguards of the Constitution, and, like the Constitution, extends those safeguards to
all. It provides that 'no law shall be enacted in said islands which shall deprive any person of life,
liberty, or property without due process of law, or deny to any person therein the equal protection of
the laws.' In the light of the declaration that we have quoted from section 12, it is hard to believe that
the United States was ready to declare in the next breath that "any person" did not embrace the
inhabitants of Benguet, or that it meant by "property" only that which had become such by
ceremonies of which presumably a large part of the inhabitants never had heard, and that it proposed
to treat as public land what they, by native custom and by long association,- of the profoundest
factors in human thought,- regarded as their own."139

The Court went further:

"Every presumption is and ought to be against the government in a case like the present. It might,
perhaps, be proper and sufficient to say that when, as far back as testimony or memory goes,
the land has been held by individuals under a claim of private ownership, it will be presumed
to have been held in the same way from before the Spanish conquest, and never to have been
public land. Certainly in a case like this, if there is doubt or ambiguity in the Spanish law, we ought to
give the applicant the benefit of the doubt."140

The court thus laid down the presumption of a certain title held (1) as far back as testimony or
memory went, and (2) under a claim of private ownership. Land held by this title is presumed to
"never have been public land."

Against this presumption, the U.S. Supreme Court analyzed the Spanish decrees upheld in the 1904
decision ofValenton v. Murciano. The U.S. Supreme Court found no proof that the Spanish decrees
did not honor native title. On the contrary, the decrees discussed in Valenton appeared to recognize
that the natives owned some land, irrespective of any royal grant. The Regalian doctrine declared in
the preamble of the Recopilacion was all "theory and discourse" and it was observed that titles were
admitted to exist beyond the powers of the Crown, viz:

"If the applicant's case is to be tried by the law of Spain, we do not discover such clear proof
that it was bad by that law as to satisfy us that he does not own the land. To begin with, the
older decrees and laws cited by the counsel for the plaintiff in error seem to indicate pretty
clearly that the natives were recognized as owning some lands, irrespective of any royal
grant. In other words, Spain did not assume to convert all the native inhabitants of the Philippines
into trespassers or even into tenants at will. For instance, Book 4, title 12, Law 14 of the
the Recopilacion de Leyes de las Indias, cited for a contrary conclusion in Valenton v. Murciano, 3
Philippine 537, while it commands viceroys and others, when it seems proper, to call for the exhibition
of grants, directs them to confirm those who hold by good grants or justa prescripcion. It is true that
it begins by the characteristic assertion of feudal overlordship and the origin of all titles in the
King or his predecessors. That was theory and discourse. The fact was that titles were
admitted to exist that owed nothing to the powers of Spain beyond this recognition in their
books." (Emphasis supplied).141

The court further stated that the Spanish "adjustment" proceedings never held sway over
unconquered territories. The wording of the Spanish laws were not framed in a manner as to convey
to the natives that failure to register what to them has always been their own would mean loss of such
land. The registration requirement was "not to confer title, but simply to establish it;" it was "not
calculated to convey to the mind of an Igorot chief the notion that ancient family possessions were in
danger, if he had read every word of it."

By recognizing this kind of title, the court clearly repudiated the doctrine of Valenton. It was frank
enough, however, to admit the possibility that the applicant might have been deprived of his land
under Spanish law because of the inherent ambiguity of the decrees and concomitantly, the various
interpretations which may be given them. But precisely because of the ambiguity and of the
strong "due process mandate" of the Constitution, the court validated this kind of title.142 This
title was sufficient, even without government administrative action, and entitled the holder to a
Torrens certificate. Justice Holmes explained:

"It will be perceived that the rights of the applicant under the Spanish law present a problem not
without difficulties for courts of a legal tradition. We have deemed it proper on that account to notice
the possible effect of the change of sovereignty and the act of Congress establishing the fundamental
principles now to be observed. Upon a consideration of the whole case we are of the opinion that law
and justice require that the applicant should be granted what he seeks, and should not be deprived of
what, by the practice and belief of those among whom he lived, was his property, through a refined
interpretation of an almost forgotten law of Spain."143

Thus, the court ruled in favor of Cariño and ordered the registration of the 148 hectares in
Baguio Municipality in his name.144

Examining Cariño closer, the U.S. Supreme Court did not categorically refer to the title it upheld as
"native title." It simply said:

"The Province of Benguet was inhabited by a tribe that the Solicitor-General, in his argument,
characterized as a savage tribe that never was brought under the civil or military government
of the Spanish Crown. It seems probable, if not certain, that the Spanish officials would not
have granted to anyone in that province the registration to which formerly the plaintiff was
entitled by the Spanish Laws, and which would have made his title beyond question
good. Whatever may have been the technical position of Spain it does not follow that, in the view of
the United States, he had lost all rights and was a mere trespasser when the present government
seized his land. The argument to that effect seems to amount to a denial of native titles through an
important part of the Island of Luzon, at least, for the want of ceremonies which the Spaniards would
not have permitted and had not the power to enforce." 145

This is the only instance when Justice Holmes used the term "native title" in the entire length of
the Cariño decision. It is observed that the widespread use of the term "native title" may be traced to
Professor Owen James Lynch, Jr., a Visiting Professor at the University of the Philippines College of
Law from the Yale University Law School. In 1982, Prof. Lynch published an article in the Philippine
Law Journal entitled Native Title, Private Right and Tribal Land Law.146 This article was made
after Professor Lynch visited over thirty tribal communities throughout the country and studied the
origin and development of Philippine land laws.147 He discussed Cariño extensively and used the
term "native title" to refer to Cariño's title as discussed and upheld by the U.S. Supreme Court in said
case.

(b) Indian Title

In a footnote in the same article, Professor Lynch stated that the concept of "native title" as defined by
Justice Holmes in Cariño "is conceptually similar to "aboriginal title" of the American Indians.148 This
is not surprising, according to Prof. Lynch, considering that during the American regime, government
policy towards ICCs/IPs was consistently made in reference to native Americans. 149 This was clearly
demonstrated in the case of Rubi v. Provincial Board of Mindoro.150

In Rubi, the Provincial Board of Mindoro adopted a Resolution authorizing the provincial governor to
remove the Mangyans from their domains and place them in a permanent reservation in Sitio Tigbao,
Lake Naujan. Any Mangyan who refused to comply was to be imprisoned. Rubi and some Mangyans,
including one who was imprisoned for trying to escape from the reservation, filed for habeas corpus
claiming deprivation of liberty under the Board Resolution. This Court denied the petition on the
ground of police power. It upheld government policy promoting the idea that a permanent settlement
was the only successful method for educating the Mangyans, introducing civilized customs, improving
their health and morals, and protecting the public forests in which they roamed. 151 Speaking through
Justice Malcolm, the court said:

"Reference was made in the President's instructions to the Commission to the policy adopted by the
United States for the Indian Tribes. The methods followed by the Government of the Philippine
Islands in its dealings with the so-called non-Christian people is said, on argument, to be practically
identical with that followed by the United States Government in its dealings with the Indian tribes.
Valuable lessons, it is insisted, can be derived by an investigation of the American-Indian policy.

From the beginning of the United States, and even before, the Indians have been treated as "in a
state of pupilage." The recognized relation between the Government of the United States and the
Indians may be described as that of guardian and ward. It is for the Congress to determine when and
how the guardianship shall be terminated. The Indians are always subject to the plenary authority of
the United States.152

x x x.

As to the second point, the facts in the Standing Bear case and the Rubi case are not exactly
identical. But even admitting similarity of facts, yet it is known to all that Indian reservations do exist in
the United States, that Indians have been taken from different parts of the country and placed on
these reservations, without any previous consultation as to their own wishes, and that, when once so
located, they have been made to remain on the reservation for their own good and for the general
good of the country. If any lesson can be drawn from the Indian policy of the United States, it is that
the determination of this policy is for the legislative and executive branches of the government and
that when once so decided upon, the courts should not interfere to upset a carefully planned
governmental system. Perhaps, just as many forceful reasons exist for the segregation of the
Manguianes in Mindoro as existed for the segregation of the different Indian tribes in the United
States."153

Rubi applied the concept of Indian land grants or reservations in the Philippines. An Indian
reservation is a part of the public domain set apart by proper authority for the use and occupation of a
tribe or tribes of Indians.154 It may be set apart by an act of Congress, by treaty, or by executive order,
but it cannot be established by custom and prescription.155

Indian title to land, however, is not limited to land grants or reservations. It also covers the
"aboriginal right of possession or occupancy."156 The aboriginal right of possession depends on
the actual occupancy of the lands in question by the tribe or nation as their ancestral home, in the
sense that such lands constitute definable territory occupied exclusively by the particular tribe or
nation.157 It is a right which exists apart from any treaty, statute, or other governmental action,
although in numerous instances treaties have been negotiated with Indian tribes, recognizing their
aboriginal possession and delimiting their occupancy rights or settling and adjusting their
boundaries.158
American jurisprudence recognizes the Indians' or native Americans' rights to land they have
held and occupied before the "discovery" of the Americas by the Europeans. The earliest
definitive statement by the U.S. Supreme Court on the nature of aboriginal title was made in
1823 in Johnson & Graham's Lessee v. M'Intosh.159

In Johnson, the plaintiffs claimed the land in question under two (2) grants made by the chiefs of two
(2) Indian tribes. The U.S. Supreme Court refused to recognize this conveyance, the plaintiffs being
private persons. The only conveyance that was recognized was that made by the Indians to the
government of the European discoverer. Speaking for the court, Chief Justice Marshall pointed out
that the potentates of the old world believed that they had made ample compensation to the
inhabitants of the new world by bestowing civilization and Christianity upon them; but in addition, said
the court, they found it necessary, in order to avoid conflicting settlements and consequent war, to
establish the principle that discovery gives title to the government by whose subjects, or by
whose authority, the discovery was made, against all other European governments, which title
might be consummated by possession.160 The exclusion of all other Europeans gave to the nation
making the discovery the sole right of acquiring the soil from the natives and establishing settlements
upon it. As regards the natives, the court further stated that:

"Those relations which were to exist between the discoverer and the natives were to be regulated by
themselves. The rights thus acquired being exclusive, no other power could interpose between them.

In the establishment of these relations, the rights of the original inhabitants were, in no instance,
entirely disregarded; but were necessarily, to a considerable extent, impaired. They were admitted
to be the rightful occupants of the soil, with a legal as well as just claim to retain possession
of it, and to use it according to their own discretion; but their rights to complete sovereignty, as
independent nations, were necessarily diminished, and their power to dispose of the soil at their own
will, to whomsoever they pleased, was denied by the fundamental principle that discovery gave
exclusive title to those who made it.

While the different nations of Europe respected the right of the natives as occupants, they
asserted the ultimate dominion to be in themselves; and claimed and exercised, as a
consequence of this ultimate dominion, a power to grant the soil, while yet in possession of
the natives. These grants have been understood by all to convey a title to the grantees,
subject only to the Indian right of occupancy."161

Thus, the discoverer of new territory was deemed to have obtained the exclusive right to acquire
Indian land and extinguish Indian titles. Only to the discoverer- whether to England, France, Spain or
Holland- did this right belong and not to any other nation or private person. The mere acquisition of
the right nonetheless did not extinguish Indian claims to land. Rather, until the discoverer, by
purchase or conquest, exercised its right, the concerned Indians were recognized as the "rightful
occupants of the soil, with a legal as well as just claim to retain possession of it." Grants made by the
discoverer to her subjects of lands occupied by the Indians were held to convey a title to the
grantees, subject only to the Indian right of occupancy. Once the discoverer purchased the land from
the Indians or conquered them, it was only then that the discoverer gained an absolute title
unrestricted by Indian rights.

The court concluded, in essence, that a grant of Indian lands by Indians could not convey a title
paramount to the title of the United States itself to other parties, saying:

"It has never been contended that the Indian title amounted to nothing. Their right of possession
has never been questioned. The claim of government extends to the complete ultimate title,
charged with this right of possession, and to the exclusive power of acquiring that right."162
It has been said that the history of America, from its discovery to the present day, proves the
universal recognition of this principle.163

The Johnson doctrine was a compromise. It protected Indian rights and their native lands without
having to invalidate conveyances made by the government to many U.S. citizens. 164

Johnson was reiterated in the case of Worcester v. Georgia.165 In this case, the State of Georgia
enacted a law requiring all white persons residing within the Cherokee nation to obtain a license or
permit from the Governor of Georgia; and any violation of the law was deemed a high misdemeanor.
The plaintiffs, who were white missionaries, did not obtain said license and were thus charged with a
violation of the Act.

The U.S. Supreme Court declared the Act as unconstitutional for interfering with the treaties
established between the United States and the Cherokee nation as well as the Acts of Congress
regulating intercourse with them. It characterized the relationship between the United States
government and the Indians as:

"The Indian nations were, from their situation, necessarily dependent on some foreign potentate for
the supply of their essential wants, and for their protection from lawless and injurious intrusions into
their country. That power was naturally termed their protector. They had been arranged under the
protection of Great Britain; but the extinguishment of the British power in their neighborhood, and the
establishment of that of the United States in its place, led naturally to the declaration, on the part of
the Cherokees, that they were under the protection of the United States, and of no other power. They
assumed the relation with the United States which had before subsisted with Great Britain.

This relation was that of a nation claiming and receiving the protection of one more powerful, not that
of individuals abandoning their national character, and submitting as subjects to the laws of a
master."166

It was the policy of the U.S. government to treat the Indians as nations with distinct territorial
boundaries and recognize their right of occupancy over all the lands within their domains. Thus:

"From the commencement of our government Congress has passed acts to regulate trade and
intercourse with the Indians; which treat them as nations, respect their rights, and manifest a firm
purpose to afford that protection which treaties stipulate. All these acts, and especially that of 1802,
which is still in force, manifestly consider the several Indian nations as distinct political
communities, having territorial boundaries, within which their authority is exclusive, and
having a right to all the lands within those boundaries, which is not only acknowledged, but
guaranteed by the United States.

x x x.

"The Indian nations had always been considered as distinct, independent political
communities, retaining their original natural rights, as the undisputed possessors of the soil
from time immemorial, with the single exception of that imposed by irresistible power, which
excluded them from intercourse with any other European potentate than the first discoverer of the
coast of the particular region claimed: and this was a restriction which those European potentates
imposed on themselves, as well as on the Indians. The very term "nation," so generally applied to
them, means "a people distinct from others." x x x.167

The Cherokee nation, then, is a distinct community, occupying its own territory, with boundaries
accurately described, in which the laws of Georgia can have no force, and which the citizens of
Georgia have no right to enter but with the assent of the Cherokees themselves or in conformity with
treaties and with the acts of Congress. The whole intercourse between the United States and this
nation is, by our Constitution and laws, vested in the government of the United States." 168

The discovery of the American continent gave title to the government of the discoverer as against all
other European governments. Designated as the naked fee, 169 this title was to be consummated by
possession and was subject to the Indian title of occupancy. The discoverer acknowledged the
Indians' legal and just claim to retain possession of the land, the Indians being the original inhabitants
of the land. The discoverer nonetheless asserted the exclusive right to acquire the Indians' land-
either by purchase, "defensive" conquest, or cession- and in so doing, extinguish the Indian title. Only
the discoverer could extinguish Indian title because it alone asserted ultimate dominion in itself. Thus,
while the different nations of Europe respected the rights of the natives as occupants, they all
asserted the ultimate dominion and title to be in themselves.170

As early as the 19th century, it became accepted doctrine that although fee title to the lands
occupied by the Indians when the colonists arrived became vested in the sovereign- first the
discovering European nation and later the original 13 States and the United States- a right of
occupancy in the Indian tribes was nevertheless recognized. The Federal Government continued
the policy of respecting the Indian right of occupancy, sometimes called Indian title, which it accorded
the protection of complete ownership.171 But this aboriginal Indian interest simply constitutes
"permission" from the whites to occupy the land, and means mere possession not specifically
recognized as ownership by Congress.172 It is clear that this right of occupancy based upon aboriginal
possession is not a property right.173 It is vulnerable to affirmative action by the federal government
who, as sovereign, possessed exclusive power to extinguish the right of occupancy at will. 174 Thus,
aboriginal title is not the same as legal title. Aboriginal title rests on actual, exclusive and
continuous use and occupancy for a long time.175 It entails that land owned by Indian title must be
used within the tribe, subject to its laws and customs, and cannot be sold to another sovereign
government nor to any citizen.176 Such title as Indians have to possess and occupy land is in the
tribe, and not in the individual Indian; the right of individual Indians to share in the tribal property
usually depends upon tribal membership, the property of the tribe generally being held in communal
ownership.177

As a rule, Indian lands are not included in the term "public lands," which is ordinarily used to
designate such lands as are subject to sale or other disposal under general laws. 178 Indian land which
has been abandoned is deemed to fall into the public domain. 179 On the other hand, an Indian
reservation is a part of the public domain set apart for the use and occupation of a tribe of
Indians.180 Once set apart by proper authority, the reservation ceases to be public land, and until the
Indian title is extinguished, no one but Congress can initiate any preferential right on, or restrict the
nation's power to dispose of, them.181

The American judiciary struggled for more than 200 years with the ancestral land claims of
indigenous Americans.182 And two things are clear. First, aboriginal title is recognized. Second,
indigenous property systems are also recognized. From a legal point of view, certain benefits can be
drawn from a comparison of Philippine IPs to native Americans.183 Despite the similarities between
native title and aboriginal title, however, there are at present some misgivings on whether
jurisprudence on American Indians may be cited authoritatively in the Philippines. The U.S.
recognizes the possessory rights of the Indians over their land; title to the land, however, is deemed
to have passed to the U.S. as successor of the discoverer. The aboriginal title of ownership is not
specifically recognized as ownership by action authorized by Congress. 184 The protection of
aboriginal title merely guards against encroachment by persons other than the Federal
Government.185 Although there are criticisms against the refusal to recognize the native Americans'
ownership of these lands,186 the power of the State to extinguish these titles has remained firmly
entrenched.187

Under the IPRA, the Philippine State is not barred form asserting sovereignty over the ancestral
domains and ancestral lands.188 The IPRA, however, is still in its infancy and any similarities between
its application in the Philippines vis-à-vis American Jurisprudence on aboriginal title will depend on
the peculiar facts of each case.

(c) Why the Cariño doctrine is unique

In the Philippines, the concept of native title first upheld in Cariño and enshrined in the IPRA grants
ownership, albeit in limited form, of the land to the ICCs/IPs. Native title presumes that the land is
private and was never public. Cariño is the only case that specifically and categorically
recognizes native title. The long line of cases citing Cariño did not touch on native title and
the private character of ancestral domains and lands. Cariño was cited by the succeeding
cases to support the concept of acquisitive prescription under the Public Land Act which is a
different matter altogether. Under the Public Land Act, land sought to be registered must be public
agricultural land. When the conditions specified in Section 48 [b] of the Public Land Act are
complied with, the possessor of the land is deemed to have acquired, by operation of law, a right to a
grant of the land.189 The land ceases to be part of the public domain,190 ipso jure,191 and is converted
to private property by the mere lapse or completion of the prescribed statutory period.

It was only in the case of Oh Cho v. Director of Lands192 that the court declared that the rule that all
lands that were not acquired from the government, either by purchase or grant, belong to the public
domain has an exception. This exception would be any land that should have been in the possession
of an occupant and of his predecessors-in-interest since time immemorial. It is this kind of possession
that would justify the presumption that the land had never been part of the public domain or that it had
been private property even before the Spanish conquest. 193 Oh Cho, however, was decided under
the provisions of the Public Land Act and Cariño was cited to support the applicant's claim of
acquisitive prescription under the said Act.

All these years, Cariño had been quoted out of context simply to justify long, continuous, open and
adverse possession in the concept of owner of public agricultural land. It is this long, continuous,
open and adverse possession in the concept of owner of thirty years both for ordinary citizens194 and
members of the national cultural minorities195 that converts the land from public into private and
entitles the registrant to a torrens certificate of title.

(3) The Option of Securing a Torrens Title to the Ancestral Land Indicates that the Land is
Private.

The private character of ancestral lands and domains as laid down in the IPRA is
further strengthened by the option given to individual ICCs/IPs over their individually-owned
ancestral lands. For purposes of registration under the Public Land Act and the Land
Registration Act, the IPRA expressly converts ancestral land into public agricultural land
which may be disposed of by the State. The necessary implication is that ancestral land is
private. It, however, has to be first converted to public agricultural land simply for registration
purposes. To wit:

"Sec. 12. Option to Secure Certificate of Title Under Commonwealth Act 141, as amended, or the
Land Registration Act 496- Individual members of cultural communities, with respect to their
individually-owned ancestral lands who, by themselves or through their predecessors-in-interest,
have been in continuous possession and occupation of the same in the concept of owner since time
immemorial or for a period of not less than thirty (30) years immediately preceding the approval of this
Act and uncontested by the members of the same ICCs/IPs shall have the option to secure title to
their ancestral lands under the provisions of Commonwealth Act 141, as amended, or the Land
Registration Act 496.

For this purpose, said individually-owned ancestral lands, which are agricultural in character and
actually used for agricultural, residential, pasture, and tree farming purposes, including those with a
slope of eighteen percent (18%) or more, are hereby classified as alienable and disposable
agricultural lands.

The option granted under this section shall be exercised within twenty (20) years from the approval of
this Act."196

ICCs/IPs are given the option to secure a torrens certificate of title over their individually-owned
ancestral lands. This option is limited to ancestral lands only, not domains, and such lands must be
individually, not communally, owned.

Ancestral lands that are owned by individual members of ICCs/IPs who, by themselves or through
their predecessors-in-interest, have been in continuous possession and occupation of the same in the
concept of owner since time immemorial197 or for a period of not less than 30 years, which claims are
uncontested by the members of the same ICCs/IPs, may be registered under C.A. 141, otherwise
known as the Public Land Act, or Act 496, the Land Registration Act. For purposes of registration, the
individually-owned ancestral lands are classified as alienable and disposable agricultural lands of the
public domain, provided, they are agricultural in character and are actually used for agricultural,
residential, pasture and tree farming purposes. These lands shall be classified as public agricultural
lands regardless of whether they have a slope of 18% or more.

The classification of ancestral land as public agricultural land is in compliance with the requirements
of the Public Land Act and the Land Registration Act. C.A. 141, the Public Land Act, deals specifically
with lands of the public domain.198 Its provisions apply to those lands "declared open to disposition or
concession" x x x "which have not been reserved for public or quasi-public purposes, nor
appropriated by the Government, nor in any manner become private property, nor those on which a
private right authorized and recognized by this Act or any other valid law x x x or which having been
reserved or appropriated, have ceased to be so." 199 Act 496, the Land Registration Act, allows
registration only of private lands and public agricultural lands. Since ancestral domains and lands
are private, if the ICC/IP wants to avail of the benefits of C.A. 141 and Act 496, the IPRA itself
converts his ancestral land, regardless of whether the land has a slope of eighteen per cent
(18%) or over,200 from private to public agricultural land for proper disposition.

The option to register land under the Public Land Act and the Land Registration Act has nonetheless
a limited period. This option must be exercised within twenty (20) years from October 29, 1997, the
date of approval of the IPRA.

Thus, ancestral lands and ancestral domains are not part of the lands of the public domain.
They are private and belong to the ICCs/IPs. Section 3 of Article XII on National Economy and
Patrimony of the 1987 Constitution classifies lands of the public domain into four categories: (a)
agricultural, (b) forest or timber, (c) mineral lands, and (d) national parks. Section 5 of the same
Article XII mentions ancestral lands and ancestral domains but it does not classify them under any of
the said four categories. To classify them as public lands under any one of the four classes will
render the entire IPRA law a nullity. The spirit of the IPRA lies in the distinct concept of ancestral
domains and ancestral lands. The IPRA addresses the major problem of the ICCs/IPs which is loss of
land. Land and space are of vital concern in terms of sheer survival of the ICCs/IPs. 201
The 1987 Constitution mandates the State to "protect the rights of indigenous cultural
communities to their ancestral lands" and that "Congress provide for the applicability of
customary laws x x x in determining the ownership and extent of ancestral domain."202 It is the
recognition of the ICCs/IPs distinct rights of ownership over their ancestral domains and
lands that breathes life into this constitutional mandate.

B. The right of ownership and possession by the ICCs/IPs of their ancestral domains is a
limited form of ownership and does not include the right to alienate the same.

Registration under the Public Land Act and Land Registration Act recognizes the concept of
ownership under the civil law. This ownership is based on adverse possession for a specified period,
and harkens to Section 44 of the Public Land Act on administrative legalization (free patent) of
imperfect or incomplete titles and Section 48 (b) and (c) of the same Act on the judicial confirmation
of imperfect or incomplete titles. Thus:

"Sec. 44. Any natural-born citizen of the Philippines who is not the owner of more than twenty-four
hectares and who since July fourth, 1926 or prior thereto, has continuously occupied and cultivated,
either by himself or through his predecessors-in-interest, a tract or tracts of agricultural public lands
subject to disposition, or who shall have paid the real estate tax thereon while the same has not been
occupied by any person shall be entitled, under the provisions of this chapter, to have a free patent
issued to him for such tract or tracts of such land not to exceed twenty-four hectares.

A member of the national cultural minorities who has continuously occupied and cultivated,
either by himself or through his predecessors-in-interest, a tract or tracts of land, whether
disposable or not since July 4, 1955, shall be entitled to the right granted in the preceding
paragraph of this section: Provided, That at the time he files his free patent application he is
not the owner of any real property secured or disposable under the provision of the Public
Land Law.203

x x x.

"Sec. 48. The following described citizens of the Philippines, occupying lands of the public domain or
claiming to own any such lands or an interest therein, but whose titles have not been perfected or
completed, may apply to the Court of First Instance of the province where the land is located for
confirmation of their claims and the issuance of a certificate of title therefor, under the Land
Registration Act, to wit:

(a) [perfection of Spanish titles] xxx.

(b) Those who by themselves or through their predecessors-in-interest have been in open,
continuous, exclusive, and notorious possession and occupation of agricultural lands of the
public domain, under a bona fide claim of acquisition or ownership, for at least thirty years
immediately preceding the filing of the application for confirmation of title except when
prevented by war or force majeure. These shall be conclusively presumed to have performed
all the conditions essential to a Government grant and shall be entitled to a certificate of title
under the provisions of this Chapter.

(c) Members of the national cultural minorities who by themselves or through their
predecessors-in-interest have been in open, continuous, exclusive and notorious
possession and occupation of lands of the public domain suitable to agriculture,
whether disposable or not, under a bona fide claim of ownership for at least 30 years
shall be entitled to the rights granted in sub-section (b) hereof."204
Registration under the foregoing provisions presumes that the land was originally public agricultural
land but because of adverse possession since July 4, 1955 (free patent) or at least thirty years
(judicial confirmation), the land has become private. Open, adverse, public and continuous
possession is sufficient, provided, the possessor makes proper application therefor. The possession
has to be confirmed judicially or administratively after which a torrens title is issued.

A torrens title recognizes the owner whose name appears in the certificate as entitled to all the rights
of ownership under the civil law. The Civil Code of the Philippines defines ownership in Articles 427,
428 and 429. This concept is based on Roman Law which the Spaniards introduced to the Philippines
through the Civil Code of 1889. Ownership, under Roman Law, may be exercised over things or
rights. It primarily includes the right of the owner to enjoy and dispose of the thing owned. And the
right to enjoy and dispose of the thing includes the right to receive from the thing what it
produces,205 the right to consume the thing by its use,206 the right to alienate, encumber, transform or
even destroy the thing owned,207 and the right to exclude from the possession of the thing owned by
any other person to whom the owner has not transmitted such thing. 208

1. The Indigenous Concept of Ownership and Customary Law.

Ownership of ancestral domains by native title does not entitle the ICC/IP to a torrens title but to a
Certificate of Ancestral Domain Title (CADT). The CADT formally recognizes the indigenous concept
of ownership of the ICCs/IPs over their ancestral domain. Thus:

"Sec. 5. Indigenous concept of ownership.- Indigenous concept of ownership sustains the view that
ancestral domains and all resources found therein shall serve as the material bases of their cultural
integrity. The indigenous concept of ownership generally holds that ancestral domains are the
ICCs/IPs private but community property which belongs to all generations and therefore cannot be
sold, disposed or destroyed. It likewise covers sustainable traditional resource rights."

The right of ownership and possession of the ICCs/IPs to their ancestral domains is held
under the indigenous concept of ownership. This concept maintains the view that ancestral
domains are the ICCs/IPs private but community property. It is private simply because it is not
part of the public domain. But its private character ends there. The ancestral domain is owned
in common by the ICCs/IPs and not by one particular person. The IPRA itself provides that areas
within the ancestral domains, whether delineated or not, are presumed to be communally
held.209 These communal rights, however, are not exactly the same as co-ownership rights
under the Civil Code.210 Co-ownership gives any co-owner the right to demand partition of the
property held in common. The Civil Code expressly provides that "no co-owner shall be obliged to
remain in the co-ownership." Each co-owner may demand at any time the partition of the thing in
common, insofar as his share is concerned.211To allow such a right over ancestral domains may be
destructive not only of customary law of the community but of the very community itself. 212

Communal rights over land are not the same as corporate rights over real property, much less
corporate condominium rights. A corporation can exist only for a maximum of fifty (50) years
subject to an extension of another fifty years in any single instance. 213 Every stockholder has the right
to disassociate himself from the corporation.214 Moreover, the corporation itself may be dissolved
voluntarily or involuntarily.215

Communal rights to the land are held not only by the present possessors of the land but
extends to all generations of the ICCs/IPs, past, present and future, to the domain. This is the
reason why the ancestral domain must be kept within the ICCs/IPs themselves. The domain cannot
be transferred, sold or conveyed to other persons. It belongs to the ICCs/IPs as a community.
Ancestral lands are also held under the indigenous concept of ownership. The lands are
communal. These lands, however, may be transferred subject to the following limitations: (a) only to
the members of the same ICCs/IPs; (b) in accord with customary laws and traditions; and (c) subject
to the right of redemption of the ICCs/IPs for a period of 15 years if the land was transferred to a non-
member of the ICCs/IPs.

Following the constitutional mandate that "customary law govern property rights or relations in
determining the ownership and extent of ancestral domains," 216 the IPRA, by legislative fiat,
introduces a new concept of ownership. This is a concept that has long existed under
customary law.217

Custom, from which customary law is derived, is also recognized under the Civil Code as a
source of law.218Some articles of the Civil Code expressly provide that custom should be applied in
cases where no codal provision is applicable.219 In other words, in the absence of any applicable
provision in the Civil Code, custom, when duly proven, can define rights and liabilities. 220

Customary law is a primary, not secondary, source of rights under the IPRA and uniquely applies to
ICCs/IPs. Its recognition does not depend on the absence of a specific provision in the civil
law. The indigenous concept of ownership under customary law is specifically acknowledged and
recognized, and coexists with the civil law concept and the laws on land titling and land
registration.221

To be sure, the indigenous concept of ownership exists even without a paper title. The CADT is
merely a "formal recognition" of native title. This is clear from Section 11 of the IPRA, to wit:

"Sec. 11. Recognition of Ancestral Domain Rights.- The rights of ICCs/IPs to their ancestral domains
by virtue of Native Title shall be recognized and respected. Formal recognition, when solicited by
ICCs/IPs concerned shall be embodied in a Certificate of Ancestral Domain Title, which shall
recognize the title of the concerned ICCs/IPs over the territories identified and delineated."

The moral import of ancestral domain, native land or being native is "belongingness" to the land,
being people of the land- by sheer force of having sprung from the land since time beyond recall, and
the faithful nurture of the land by the sweat of one's brow. This is fidelity of usufructuary relation to the
land- the possession of stewardship through perduring, intimate tillage, and the mutuality of blessings
between man and land; from man, care for land; from the land, sustenance for man. 222

C. Sections 7 (a), 7 (b) and 57 of the IPRA Do Not Violate the Regalian Doctrine Enshrined in
Section 2, Article XII of the 1987 Constitution.

1. The Rights of ICCs/IPs Over Their Ancestral Domains and Lands

The IPRA grants the ICCs/IPs several rights over their ancestral domains and ancestral lands.
Section 7 provides for the rights over ancestral domains:

"Sec. 7. Rights to Ancestral Domains.- The rights of ownership and possession of ICCs/IPs to their
ancestral domains shall be recognized and protected. Such rights include:

a) Right of Ownership.- The right to claim ownership over lands, bodies of water
traditionally and actually occupied by ICCs/IPs, sacred places, traditional hunting and
fishing grounds, and all improvements made by them at any time within the domains;
b) Right to Develop Lands and Natural Resources.- Subject to Section 56 hereof, the right
to develop, control and use lands and territories traditionally occupied, owned, or used;
to manage and conserve natural resources within the territories and uphold the
responsibilities for future generations; to benefit and share the profits from allocation
and utilization of the natural resources found therein; the right to negotiate the terms
and conditions for the exploration of natural resources in the areas for the purpose of
ensuring ecological, environmental protection and the conservation measures,
pursuant to national and customary laws; the right to an informed and intelligent
participation in the formulation and implementation of any project, government or private, that
will affect or impact upon the ancestral domains and to receive just and fair compensation for
any damages which they may sustain as a result of the project; and the right to effective
measures by the government to prevent any interference with, alienation and encroachment
upon these rights;"

c) Right to Stay in the Territories.- The right to stay in the territory and not to be removed
therefrom. No ICCs/IPs will be relocated without their free and prior informed consent, nor
through any means other than eminent domain. x x x;

d) Right in Case of Displacement.- In case displacement occurs as a result of natural


catastrophes, the State shall endeavor to resettle the displaced ICCs/IPs in suitable areas
where they can have temporary life support systems: x x x;

e) Right to Regulate the Entry of Migrants.- Right to regulate the entry of migrant settlers and
organizations into their domains;

f) Right to Safe and Clean Air and Water.-For this purpose, the ICCs/IPs shall have access to
integrated systems for the management of their inland waters and air space;

g) Right to Claim Parts of Reservations.- The right to claim parts of the ancestral domains
which have been reserved for various purposes, except those reserved and intended for
common and public welfare and service;

h) Right to Resolve Conflict.- Right to resolve land conflicts in accordance with customary laws
of the area where the land is located, and only in default thereof shall the complaints be
submitted to amicable settlement and to the Courts of Justice whenever necessary."

Section 8 provides for the rights over ancestral lands:

"Sec. 8. Rights to Ancestral Lands.- The right of ownership and possession of the ICCs/IPs to their
ancestral lands shall be recognized and protected.

a) Right to transfer land/property.- Such right shall include the right to transfer land or property
rights to/among members of the same ICCs/IPs, subject to customary laws and traditions of
the community concerned.

b) Right to Redemption.- In cases where it is shown that the transfer of land/property rights by
virtue of any agreement or devise, to a non-member of the concerned ICCs/IPs is tainted by
the vitiated consent of the ICCs/IPs, or is transferred for an unconscionable consideration or
price, the transferor ICC/IP shall have the right to redeem the same within a period not
exceeding fifteen (15) years from the date of transfer."
Section 7 (a) defines the ICCs/IPs the right of ownership over their ancestral domains which covers
(a) lands, (b) bodies of water traditionally and actually occupied by the ICCs/IPs, (c) sacred places,
(d) traditional hunting and fishing grounds, and (e) all improvements made by them at any time within
the domains. The right of ownership includes the following rights: (1) the right to develop lands and
natural resources; (b) the right to stay in the territories; (c) the right to resettlement in case of
displacement; (d) the right to regulate the entry of migrants; (e) the right to safe and clean air and
water; (f) the right to claim parts of the ancestral domains as reservations; and (g) the right to resolve
conflict in accordance with customary laws.

Section 8 governs their rights to ancestral lands. Unlike ownership over the ancestral domains,
Section 8 gives the ICCs/IPs also the right to transfer the land or property rights to members of the
same ICCs/IPs or non-members thereof. This is in keeping with the option given to ICCs/IPs to
secure a torrens title over the ancestral lands, but not to domains.

2. The Right of ICCs/IPs to Develop Lands and Natural Resources Within the Ancestral Domains
Does Not Deprive the State of Ownership Over the Natural Resources and Control and Supervision in
their Development and Exploitation.

The Regalian doctrine on the ownership, management and utilization of natural resources is declared
in Section 2, Article XII of the 1987 Constitution, viz:

"Sec. 2. All lands of the public domain, waters, minerals, coal, petroleum, and other mineral
oils, all forces of potential energy, fisheries, forests or timber, wildlife, flora and fauna, and
other natural resources are owned by the State. With the exception of agricultural lands, all other
natural resources shall not be alienated. The exploration, development, and utilization of natural
resources shall be under the full control and supervision of the State. The State may directly
undertake such activities, or, it may enter into co-production, joint venture, or production-
sharing agreements with Filipino citizens, or corporations or associations at least sixty per
centum of whose capital is owned by such citizens. Such agreements may be for a period not
exceeding twenty-five years, renewable for not more than twenty-five years, and under such terms
and conditions as may be provided by law. In cases of water rights for irrigation, water supply,
fisheries, water supply, fisheries, or industrial uses other than the development of water power,
beneficial use may be the measure and limit of the grant.

The State shall protect the nation's marine wealth in its archipelagic waters, territorial sea, and
exclusive economic zone, and reserve its use and enjoyment exclusively to Filipino citizens.

The Congress may, by law, allow small-scale utilization of natural resources by Filipino
citizens, as well as cooperative fish farming, with priority to subsistence fishermen and fishworkers in
rivers, lakes, bays, and lagoons.

The President may enter into agreements with foreign-owned corporations involving either technical
or financial assistance for large-scale exploration, development, and utilization of minerals,
petroleum, and other mineral oils according to the general terms and conditions provided by law,
based on real contributions to the economic growth and general welfare of the country. In such
agreements, the state shall promote the development and use of local scientific and technical
resources.

The President shall notify the Congress of every contract entered into in accordance with this
provision, within thirty days from its execution."223
All lands of the public domain and all natural resources- waters, minerals, coal, petroleum, and
other mineral oils, all forces of potential energy, fisheries, forests or timber, wildlife, flora and fauna,
and other natural resources- are owned by the State. The Constitution provides that in the
exploration, development and utilization of these natural resources, the State exercises full control
and supervision, and may undertake the same in four (4) modes:

1. The State may directly undertake such activities; or

2. The State may enter into co-production, joint venture or production-sharing agreements with
Filipino citizens or qualified corporations;

3. Congress may, by law, allow small-scale utilization of natural resources by Filipino citizens;

4. For the large-scale exploration, development and utilization of minerals, petroleum and other
mineral oils, the President may enter into agreements with foreign-owned
corporations involving technical or financial assistance.

As owner of the natural resources, the State is accorded primary power and responsibility in
the exploration, development and utilization of these natural resources. The State may directly
undertake the exploitation and development by itself, or, it may allow participation by the private
sector through co-production,224 joint venture,225or production-sharing agreements.226 These
agreements may be for a period of 25 years, renewable for another 25 years. The State, through
Congress, may allow the small-scale utilization of natural resources by Filipino citizens. For the large-
scale exploration of these resources, specifically minerals, petroleum and other mineral oils, the
State, through the President, may enter into technical and financial assistance agreements with
foreign-owned corporations.

Under the Philippine Mining Act of 1995, (R.A. 7942) and the People's Small-Scale Mining Act of
1991 (R.A. 7076) the three types of agreements, i.e., co-production, joint venture or production-
sharing, may apply to both large-scale227 and small-scale mining.228 "Small-scale mining" refers to
"mining activities which rely heavily on manual labor using simple implements and methods and do
not use explosives or heavy mining equipment."229

Examining the IPRA, there is nothing in the law that grants to the ICCs/IPs ownership over the
natural resources within their ancestral domains. The right of ICCs/IPs in their ancestral domains
includes ownership, but this "ownership" is expressly defined and limited in Section 7 (a) as:

"Sec. 7. a) Right of ownership- The right to claim ownership over lands, bodies of water traditionally
and actually occupied by ICCs/IPs, sacred places, traditional hunting and fishing grounds, and all
improvements made by them at any time within the domains;"

The ICCs/IPs are given the right to claim ownership over "lands, bodies of water traditionally and
actually occupied by ICCs/IPs, sacred places, traditional hunting and fishing grounds, and all
improvements made by them at any time within the domains." It will be noted that this enumeration
does not mention bodies of water not occupied by the
ICCs/IPs, minerals, coal, wildlife, flora and fauna in the traditional hunting grounds, fish in the
traditional fishing grounds, forests or timber in the sacred places, etc. and all other natural resources
found within the ancestral domains. Indeed, the right of ownership under Section 7 (a) does not
cover "waters, minerals, coal, petroleum and other mineral oils, all forces of potential
energy, fisheries, forests or timber, wildlife, flora and fauna and all other natural resources"
enumerated in Section 2, Article XII of the 1987 Constitution as belonging to the State.
The non-inclusion of ownership by the ICCs/IPs over the natural resources in Section 7(a) complies
with the Regalian doctrine.

(a) Section 1, Part II, Rule III of the Implementing Rules Goes Beyond the Parameters of Sec. 7
(a) of the IPRA And is Unconstitutional.

The Rules Implementing the IPRA230 in Section 1, Part II, Rule III reads:

"Section 1. Rights of Ownership. ICCs/IPs have rights of ownership over lands, waters, and natural
resources and all improvements made by them at any time within the ancestral domains/ lands.
These rights shall include, but not limited to, the right over the fruits, the right to possess, the right to
use, right to consume, right to exclude and right to recover ownership, and the rights or interests over
land and natural resources. The right to recover shall be particularly applied to lands lost through
fraud or any form or vitiated consent or transferred for an unconscionable price."

Section 1 of the Implementing Rules gives the ICCs/IPs rights of ownership over "lands, waters and
natural resources." The term "natural resources" is not one of those expressly mentioned in Section 7
(a) of the law. Our Constitution and jurisprudence clearly declare that the right to claim ownership
over land does not necessarily include the right to claim ownership over the natural resources found
on or under the land.231 The IPRA itself makes a distinction between land and natural resources.
Section 7 (a) speaks of the right of ownership only over the land within the ancestral domain.
It is Sections 7 (b) and 57 of the law that speak of natural resources, and these provisions, as
shall be discussed later, do not give the ICCs/IPs the right of ownership over these resources.

The constitutionality of Section 1, Part II, Rule III of the Implementing Rules was not specifically and
categorically challenged by petitioners. Petitioners actually assail the constitutionality of the
Implementing Rules in general.232Nevertheless, to avoid any confusion in the implementation of the
law, it is necessary to declare that the inclusion of "natural resources" in Section 1, Part II, Rule III of
the Implementing Rules goes beyond the parameters of Section 7 (b) of the law and is contrary to
Section 2, Article XII of the 1987 Constitution.

(b) The Small-Scale Utilization of Natural Resources In Sec. 7 (b) of the IPRA Is Allowed Under
Paragraph 3, Section 2 of Article XII of the Constitution.

Ownership over natural resources remain with the State and the IPRA in Section 7 (b) merely grants
the ICCs/IPs the right to manage them, viz:

"Sec. 7 (b) Right to Develop Lands and Natural Resources.- Subject to Section 56 hereof, right to
develop, control and use lands and territories traditionally occupied, owned, or used; to manage and
conserve natural resources within the territories and uphold the responsibilities for future
generations; to benefit and share the profits from allocation and utilization of the natural resources
found therein; the right to negotiate the terms and conditions for the exploration of natural resources
in the areas for the purpose of ensuring ecological, environmental protection and the conservation
measures, pursuant to national and customary laws; the right to an informed and intelligent
participation in the formulation and implementation of any project, government or private, that will
affect or impact upon the ancestral domains and to receive just and fair compensation for any
damages which they may sustain as a result of the project; and the right to effective measures by the
government to prevent any interference with, alienation and encroachment upon these rights;"

The right to develop lands and natural resources under Section 7 (b) of the IPRA enumerates the
following rights:
a) the right to develop, control and use lands and territories traditionally occupied;

b) the right to manage and conserve natural resources within the territories and uphold the
responsibilities for future generations;

c) the right to benefit and share the profits from the allocation and utilization of the natural
resources found therein;

d) the right to negotiate the terms and conditions for the exploration of natural resources for the
purpose of ensuring ecological, environmental protection and the conservation measures,
pursuant to national and customary laws;

e) the right to an informed and intelligent participation in the formulation and implementation of
any project, government or private, that will affect or impact upon the ancestral domains and to
receive just and fair compensation for any damages which they may sustain as a result of the
project;

f) the right to effective measures by the government to prevent any interference with, alienation
and encroachment upon these rights.233

Ownership over the natural resources in the ancestral domains remains with the State and the
ICCs/IPs are merely granted the right to "manage and conserve" them for future generations,
"benefit and share" the profits from their allocation and utilization, and "negotiate the terms
and conditions for their exploration" for the purpose of "ensuring ecological and
environmental protection and conservation measures." It must be noted that the right to negotiate
the terms and conditions over the natural resources covers only their exploration which must be for
the purpose of ensuring ecological and environmental protection of, and conservation measures in
the ancestral domain. It does not extend to the exploitation and development of natural resources.

Simply stated, the ICCs/IPs' rights over the natural resources take the form of management or
stewardship. For the ICCs/IPs may use these resources and share in the profits of their utilization or
negotiate the terms for their exploration. At the same time, however, the ICCs/IPs must ensure that
the natural resources within their ancestral domains are conserved for future generations and that the
"utilization" of these resources must not harm the ecology and environment pursuant to national and
customary laws.234

The limited rights of "management and use" in Section 7 (b) must be taken to contemplate
small-scale utilization of natural resources as distinguished from large-scale. Small-scale
utilization of natural resources is expressly allowed in the third paragraph of Section 2, Article
XII of the Constitution "in recognition of the plight of forest dwellers, gold panners, marginal
fishermen and others similarly situated who exploit our natural resources for their daily sustenance
and survival."235 Section 7 (b) also expressly mandates the ICCs/IPs to manage and conserve these
resources and ensure environmental and ecological protection within the domains, which duties, by
their very nature, necessarily reject utilization in a large-scale.

(c) The Large-Scale Utilization of Natural Resources In Section 57 of the IPRA Is Allowed
Under Paragraphs 1 and 4, Section 2, Article XII of the 1987 Constitution.

Section 57 of the IPRA provides:

"Sec. 57. Natural Resources within Ancestral Domains.- The ICCs/IPs shall have priority rights in
the harvesting, extraction, development or exploitation of any natural resources within the
ancestral domains. A non-member of the ICCs/IPs concerned may be allowed to take part in the
development and utilization of the natural resources for a period of not exceeding twenty-five (25)
years renewable for not more than twenty-five (25) years: Provided, That a formal and written
agreement is entered into with the ICCs/IPs concerned or that the community, pursuant to its own
decision-making process, has agreed to allow such operation: Provided finally, That the NCIP may
exercise visitorial powers and take appropriate action to safeguard the rights of the ICCs/IPs under
the same contract."

Section 57 speaks of the "harvesting, extraction, development or exploitation of natural


resources within ancestral domains" and "gives the ICCs/IPs 'priority rights' therein." The
terms "harvesting, extraction, development or exploitation" of any natural resources within the
ancestral domains obviously refer to large-scale utilization. It is utilization not merely for
subsistence but for commercial or other extensive use that require technology other than manual
labor.236 The law recognizes the probability of requiring a non-member of the ICCs/IPs to participate
in the development and utilization of the natural resources and thereby allows such participation for a
period of not more than 25 years, renewable for another 25 years. This may be done on condition that
a formal written agreement be entered into by the non-member and members of the ICCs/IPs.

Section 57 of the IPRA does not give the ICCs/IPs the right to "manage and conserve" the natural
resources. Instead, the law only grants the ICCs/IPs "priority rights" in the development or exploitation
thereof. Priority means giving preference. Having priority rights over the natural resources does not
necessarily mean ownership rights. The grant of priority rights implies that there is a superior
entity that owns these resources and this entity has the power to grant preferential rights over the
resources to whosoever itself chooses.

Section 57 is not a repudiation of the Regalian doctrine. Rather, it is an affirmation of the said doctrine
that all natural resources found within the ancestral domains belong to the State. It incorporates by
implication the Regalian doctrine, hence, requires that the provision be read in the light of Section 2,
Article XII of the 1987 Constitution. Interpreting Section 2, Article XII of the 1987
Constitution237 in relation to Section 57 of IPRA, the State, as owner of these natural
resources, may directly undertake the development and exploitation of the natural resources
by itself, or in the alternative, it may recognize the priority rights of the ICCs/IPs as owners of
the land on which the natural resources are found by entering into a co-production, joint
venture, or production-sharing agreement with them. The State may likewise enter into any of
said agreements with a non-member of the ICCs/IPs, whether natural or juridical, or enter into
agreements with foreign-owned corporations involving either technical or financial assistance
for the large-scale exploration, development and utilization of minerals, petroleum, and other
mineral oils, or allow such non-member to participate in its agreement with the ICCs/IPs. If the
State decides to enter into an agreement with a non-ICC/IP member, the National Commission on
Indigenous Peoples (NCIP) shall ensure that the rights of the ICCs/IPs under the agreement shall be
protected. The agreement shall be for a period of 25 years, renewable for another 25 years.

To reiterate, in the large-scale utilization of natural resources within the ancestral domains, the State,
as owner of these resources, has four (4) options: (1) it may, of and by itself, directly undertake the
development and exploitation of the natural resources; or (2) it may recognize the priority rights of the
ICCs/IPs by entering into an agreement with them for such development and exploitation; or (3) it
may enter into an agreement with a non-member of the ICCs/IPs, whether natural or juridical, local or
foreign; or (4) it may allow such non-member to participate in the agreement with the ICCs/IPs.

The rights granted by the IPRA to the ICCs/IPs over the natural resources in their ancestral
domains merely gives the ICCs/IPs, as owners and occupants of the land on which the
resources are found, the right to the small-scale utilization of these resources, and at the
same time, a priority in their large-scale development and exploitation. Section 57 does not
mandate the State to automatically give priority to the ICCs/IPs. The State has several options
and it is within its discretion to choose which option to pursue. Moreover, there is nothing in the
law that gives the ICCs/IPs the right to solely undertake the large-scale development of the natural
resources within their domains. The ICCs/IPs must undertake such endeavour always under State
supervision or control. This indicates that the State does not lose control and ownership over the
resources even in their exploitation. Sections 7 (b) and 57 of the law simply give due respect to the
ICCs/IPs who, as actual occupants of the land where the natural resources lie, have traditionally
utilized these resources for their subsistence and survival.

Neither is the State stripped of ownership and control of the natural resources by the following
provision:

"Section 59. Certification Precondition.- All departments and other governmental agencies shall
henceforth be strictly enjoined from issuing, renewing or granting any concession, license or lease, or
entering into any production-sharing agreement. without prior certification from the NCIP that the area
affected does not overlap with any ancestral domain. Such certification shall only be issued after a
field-based investigation is conducted by the Ancestral Domains Office of the area
concerned: Provided, That no certification shall be issued by the NCIP without the free and prior
informed and written consent of the ICCs/IPs concerned: Provided, further, That no department,
government agency or government-owned or -controlled corporation may issue new concession,
license, lease, or production sharing agreement while there is a pending application for a
CADT: Provided, finally, That the ICCs/IPs shall have the right to stop or suspend, in accordance with
this Act, any project that has not satisfied the requirement of this consultation process."

Concessions, licenses, lease or production-sharing agreements for the exploitation of natural


resources shall not be issued, renewed or granted by all departments and government agencies
without prior certification from the NCIP that the area subject of the agreement does not overlap with
any ancestral domain. The NCIP certification shall be issued only after a field-based investigation
shall have been conducted and the free and prior informed written consent of the ICCs/IPs obtained.
Non-compliance with the consultation requirement gives the ICCs/IPs the right to stop or suspend any
project granted by any department or government agency.

As its subtitle suggests, this provision requires as a precondition for the issuance of any concession,
license or agreement over natural resources, that a certification be issued by the NCIP that the area
subject of the agreement does not lie within any ancestral domain. The provision does not vest the
NCIP with power over the other agencies of the State as to determine whether to grant or deny any
concession or license or agreement. It merely gives the NCIP the authority to ensure that the
ICCs/IPs have been informed of the agreement and that their consent thereto has been obtained.
Note that the certification applies to agreements over natural resources that do not necessarily lie
within the ancestral domains. For those that are found within the said domains, Sections 7(b) and 57
of the IPRA apply.

V. THE IPRA IS A RECOGNITION OF OUR ACTIVE PARTICIPATION IN THE INDIGENOUS


INTERNATIONAL MOVEMENT.

The indigenous movement can be seen as the heir to a history of anti-imperialism stretching back to
prehistoric times. The movement received a massive impetus during the 1960's from two sources.
First, the decolonization of Asia and Africa brought into the limelight the possibility of peoples
controlling their own destinies. Second, the right of self-determination was enshrined in the UN
Declaration on Human Rights.238 The rise of the civil rights movement and anti-racism brought to the
attention of North American Indians, Aborigines in Australia, and Maori in New Zealand the possibility
of fighting for fundamental rights and freedoms.

In 1974 and 1975, international indigenous organizations were founded, 239 and during the 1980's,
indigenous affairs were on the international agenda. The people of the Philippine Cordillera were the
first Asians to take part in the international indigenous movement. It was the Cordillera People's
Alliance that carried out successful campaigns against the building of the Chico River Dam in 1981-
82 and they have since become one of the best-organized indigenous bodies in the world.240

Presently, there is a growing concern for indigenous rights in the international scene. This came as a
result of the increased publicity focused on the continuing disrespect for indigenous human rights and
the destruction of the indigenous peoples' environment, together with the national governments'
inability to deal with the situation.241Indigenous rights came as a result of both human rights and
environmental protection, and have become a part of today's priorities for the international agenda.242

International institutions and bodies have realized the necessity of applying policies, programs and
specific rules concerning IPs in some nations. The World Bank, for example, first adopted a policy on
IPs as a result of the dismal experience of projects in Latin America. 243 The World Bank now seeks to
apply its current policy on IPs to some of its projects in Asia. This policy has provided an influential
model for the projects of the Asian Development Bank.244

The 1987 Philippine Constitution formally recognizes the existence of ICCs/IPs and declares as a
State policy the promotion of their rights within the framework of national unity and
development.245 The IPRA amalgamates the Philippine category of ICCs with the international
category of IPs,246 and is heavily influenced by both the International Labor Organization (ILO)
Convention 169 and the United Nations (UN) Draft Declaration on the Rights of Indigenous
Peoples.247

ILO Convention No. 169 is entitled the "Convention Concerning Indigenous and Tribal Peoples in
Independent Countries"248 and was adopted on June 27, 1989. It is based on the Universal
Declaration of Human Rights, the International Covenant on Economic, Social and Cultural Rights,
the International Covenant on Civil and Political Rights, and many other international instruments on
the prevention of discrimination.249 ILO Convention No. 169 revised the "Convention Concerning the
Protection and Integration of Indigenous and Other Tribal and Semi-Tribal Populations in Independent
Countries" (ILO No. 107) passed on June 26, 1957. Developments in international law made it
appropriate to adopt new international standards on indigenous peoples "with a view to removing the
assimilationist orientation of the earlier standards," and recognizing the aspirations of these peoples
to exercise control over their own institutions, ways of life and economic development." 250

CONCLUSION

The struggle of the Filipinos throughout colonial history had been plagued by ethnic and religious
differences. These differences were carried over and magnified by the Philippine government through
the imposition of a national legal order that is mostly foreign in origin or derivation. 251 Largely
unpopulist, the present legal system has resulted in the alienation of a large sector of society,
specifically, the indigenous peoples. The histories and cultures of the indigenes are relevant to the
evolution of Philippine culture and are vital to the understanding of contemporary problems.252 It is
through the IPRA that an attempt was made by our legislators to understand Filipino society not in
terms of myths and biases but through common experiences in the course of history. The Philippines
became a democracy a centennial ago and the decolonization process still continues. If the evolution
of the Filipino people into a democratic society is to truly proceed democratically, i.e., if the Filipinos
as a whole are to participate fully in the task of continuing democratization, 253 it is this Court's duty to
acknowledge the presence of indigenous and customary laws in the country and affirm their co-
existence with the land laws in our national legal system.

With the foregoing disquisitions, I vote to uphold the constitutionality of the Indigenous Peoples Rights
Act of 1997.

Footnotes
1Chief Judge, US Court of Appeals for the Seventh Circuit; Senior Lecturer, University of
Chicago Law School.
2 The University of Chicago Law Review, Vol. 67, Summer 2000, No. 3, p. 573.
3 Dominium is distinguished from imperium which is the government authority possessed by
the state expressed in the concept of sovereignty- Lee Hong Hok v. David, 48 SCRA 372, 377
[1972].
4Valenton v. Murciano, 3 Phil. 537, 543 [1904]; See also Florencio D.R. Ponce, The Philippine
Torrens System, p. 13 [1964].
5 Antonio H. Noblejas, Land Titles and Deeds, p. 5 [1986]; these grants were better known as
repartimientos and encomiendas. Repartimientos were handouts to the military as fitting
reward for their services to the Spanish crown. The encomiendas were given to Spaniards to
administer and develop with the right to receive and enjoy for themselves the tributes of the
natives assigned to them.- Ponce, supra, p. 12, citing Benitez, History of the Philippines, pp.
125-126.
6 Narciso Pena, Registration of Land Titles and Deeds, p. 2 [1994].
7The Mortgage Law is a misnomer because it is primarily a law on registration of property and
secondarily a mortgage law- Ponce, supra, at 16.
8 Ponce, supra, at 15.

9 3 Phil. 537 [1904].


10 Id. at 540.
11 Id. at 548.
12 Id. at 543-544.
13 Id. at 543.
14Id. at 542-543. These comments by the court are clear expressions of the concept that
Crown holdings embraced both imperium and dominium—Ma. Lourdes Aranal-Sereno and
Roan Libarios, The Interface Between National Land Law and Kalinga Land Law, 58 P.L.J.
420, 423 [1983].
15 Id. at 545-546.
16 Id. at 543.
17 Id. at 557.
18 Id. at 553-554; Valenton was applied in Cansino v. Valdez, 6 Phil. 320 [1906]; Tiglao v.
Insular Government, 7 Phil. 80 [1906]; and Cariño v. Insular Government, 7 Phil. 132 [1906];
all decided by the Philippine Supreme Court.
19 Please see Section 70, Act 926.
20 Ponce, supra, at 33.
21 Montano v. Insular Government, 12 Phil. 572 [1909]; also cited in Ponce, supra, at 32.
22Archbishop of Manila v. Director of Lands, 27 Phil. 245 [1914]; also cited in Ponce, supra, at
32.
23 Antonio H. Noblejas, Land Titles and Deeds, p. 250 [1961].
24 Ponce, supra, at 32.
25 Peña, Registration of Land Titles and Deeds, p. 26 [1982]; Noblejas, supra, at 32.
26 Noblejas, supra, at 32.
27 Ponce, supra, at 123-124; Noblejas, supra, at 33.

28 2 Aruego, The Framing of the Philippine Constitution, p. 592 [1937].


29 Id. at 600.
30 Id. at 600-601.
31 Ibid.

32 Section 7.
33 Section 8.
34 Sections 13 to 20.
35 Sections 21 to 28.
36 Sections 29 to 37.
37 Sections 38 and 40.
38 Sections 74 to 77.
39 Section 69.
40 Section 73.
41Convention Conerning Indigenous and Tribal Peoples in Independent Countries, June 27,
1989.
42Guide to R.A. 8371, published by the Coalition for Ips Rights and ancestral Domains in
cooperation with the ILO and Bilance-Asia Department, p. 4 [1999]—hereinafter referred to
as Guide to R.A. 8371.

43 Taken from the list of IPs sbmitted by Rep. Andolana to the house of Representatives during
the deliberations on H.B. No. 9125—Interpellations of Aug. 20, 1997, pp. 00086-00095. "lost
tribes" such as the Lutangan and Tatang have not been included.
44 How these people came to the Philippines may be explained by two theories. One view,
generally linked to Professor Otley H. Beyer, suggests the "wave theory"—a series of arrivals
in the archipelago bringing in different types and levels of culture. The Negritos, dark-skinned
pygmies, came between 25,000 to 30,000 B.C. Their cultural remains are preserved by the
Negrito-type Filipinos found in Luzon, Visayas and Mindanao. Their relatively inferior culture
did not enable them to overcome the pressures from the second wave of people, the
Indonesians A and B who came in 5,000 and 3,500 B.C. They are represented today by the
Kalinga, Gaddang, Isneg, Mangyan, Tagbanua, Manobo, Mandaya, Subanon, and Sama. The
first group was pushed inland as the second occupied the coastal and downriver settlements.
The last wave involved Malay migrations between 500 B.C. and 1,500 A.D. they had a more
advanced culture based on metal age technology. They are represented by the Christianized
and Islamized Filipinos who pushed the Indonesian groups inland and occupied much of the
coastal, lowland and downstream areas.

A second view is postulated by Robert Fox, F. Landa Jocana, Alfredo Evangelista, and
Jesus Peralta. Jocano maintains that the Negritos, Indonesians and Malays stand co-
equal as ethnic groups without any one being dominant, racially or culturally. The
geographic distribution of the ethno-linguistic groups, which shows overlapping of
otherwise similar racial strains in both upland and lowland cultures or coastal and inland
communities, suggests a random and unstructured advent of different kinds of groups in
the archipelago—Samuel K. Tan, A History of the Philippines, published by the Manila
Studies Association, Inc. and the Philippine National Historical society, Inc., pp. 33-34
[1997]; Teodoro A. Agoncillo, History of the Filipino People, p. 21 [1990].
45 Tan, supra, at 35-36.
46Onofre D. Corpuz, The Roots of the Filipino Nation, Philippine Centennial (1898-1998)
Edition, vol. 1, p. 13, Aklahi foundation, Inc. [1989]. It was in 800-1,000 A.D. that the Ifugaos of
Northern Luzon built the rice terraces—Id. at 37.
47 Id. at 5-6.
48 Id. at 13.
49 Teodoro A. Agoncillo, History of the Filipino People, p. 54 [1990].
50 Corpuz, supra, at 5.
51 Id. at 44-45.
52 Agoncillo, supra, at 40.
53 Id. at 40-41.
54Rafael Iriarte, History of the Judicial System, the Philippine Indigenous Era Prior to 1565,
unpublished work submitted as entry to the Centennial Essay-Writing Contest sponsored by
the National Centennial Commission and the Supreme Court in 1997, p. 103, citing Perfecto V.
Fernandez, Customs Laws in Pre-Conquest Philippines, UP Law Center, p. 10 [1976].
55 Agoncillo, supra, at 41.
56Amelia Alonzo, The History of the Judicial System in the Philippines, Indigenous Era Prior to
1565, unpublished work submitted as entry to the Centennial Essay-Writing Contest sponsored
by the National Centennial Commission and the Supreme Court in 1997.
57 Agoncillo, supra, at 42.
58 Renato Constantino, A Past Revisited , p. 38 [1975].
59Samuel K. Tan, A History of the Philippines, published by the Manila Studies Ass’n., Inc. and
the Phil. National Historical Society, Inc., p. 43 [1997].
60 Id.
61 Id. at 43-44.
62 Tan, supra, at 47-48.
63 Id. at 48-49.
64 Cacho v. Government of the P.I., 28 Phil. 616, 625-627 [1914]; see also Ponce, The
Philippine Torrens System, pp. 11-12 [1964]. In Philippine pre-colonial history, there was only
one recorded transaction on the purchase of land. The Maragtas Code tells us of the purchase
of Panay Island by ten Bornean datus led by Datu Puti from the Atis under Marikudo in the
13th century. The purchase price for the island was a gold salakot and a long gold necklace –
Agoncillo, supra, at 25.
65 Constantino, supra, at 38.
66 Corpuz, supra, at 39.
67 Resettlement- "bajo el son de la campana" (under the sound of the bell) or "bajo el toque de
la campana" (Under the peal of the bell).
68 People v. Cayat, 68 Phil. 12, 17 [1939].
69 Id. at 17, citing the Decree of the Governor-General of the Philippines, Jan. 14, 1887.
70 Agoncillo, supra, at 80.
71 Id. at 80.
72 Corpuz, supra, at 277-278.
73 Id. at 277.
74Id., N.B. But see discussion in Cariño v. Insular Government, infra, where the United States
Supreme Court found that the Spanish decrees in the Philippines appeared to recognize that
the natives owned some land. Whether in the implementation of these decrees the
natives’ ancestral rights to land were actually respected was not discussed by the U.S.
Supreme Court; see also Note 131, infra.
75 Tan, supra, at 49-50.
76 Id. at 67.
77 Id. at 52-53.
78 Id. at 53.
79 Id. at 55.
80 People v. Cayat, 68 Phil. 12, 17 [1939].
81Memorandum of the Secretary of the Interior, quoted in Rubi v. Provincial Board of Mindoro,
39 Phil. 660, 714 [1919]; also cited in People v. Cayat, supra, at 17-18.
82 Rubi v. Provincial Board of Mindoro, supra, at 693.
83 Charles Macdonald, Indigenous Peoples of the Philippines: Between Segregation and
Integration, Indigenous Peoples of Asia, p. 348, ed. by R.H. Barnes, A. Gray and B.
Kingsburry, pub. by Association for Asian Studies [1995]. The BNCT made a Bontok and
subanon ethnography, a history of Sulu genealogy, and a compilation on unhispanized
peoples in northern Luzon.—Owen J. Lynch, Jr., The Philippine Colonial Dichotomy: Attraction
and Disenfranchisement, 63 P. L. J. 139-140 [1988].
84 R.A. No. 1888 of 1957.
85See People v. Cayat, supra, at 21; See also Rubi v. Provincial Board of Mindoro, 39 Phil.
660, 694 [1919]
86 MacDonald, Indigenous Peoples of the Philippines, supra, at 351.
87The construction of the Ambuklao and Binga dams in the 1950’s resulted in the eviction of
hundreds of Ibaloi families – Cerilo Rico S. Abelardo, Ancestral Domain Rights: Issues,
Responses, and Recommendations, Ateneo Law Journal, vol. 38, No. 1, p. 92 [1993].
88 Section 11, Art. XV, 1973 Constitution.
89 Presidential Decrees Nos. 1017 and 1414.
90The PANAMIN, however, concentrated funds and resources on image-building, publicity,
and impact projects. In Mindanao, the agency resorted to a policy of forced resettlement on
reservations, militarization and intimidation- MacDonald, Indigenous Peoples of the
Philippines, supra, at 349-350.

91No occupancy certificates were issued, however, because the government failed to release
the decree’s implementing rules and regulations- Abelardo, supra, at 120-121.
92 Id., Note 177.
93 Id., at 93-94.
94 MacDonald, Indigenous People of the Philippines, supra, at 351.
95 E.O. Nos. 122-A, 122-B and 122-C. The preamble of E.O. No. 122-B states:

"Believing that the new government is committed to formulate more vigorous policies,
plans, programs, and projects for tribal Filipinos, otherwise known as Indigenous
Cultural Communities, taking into consideration their communal aspirations, customs,
traditions, beliefs, and interests, in order to promote and preserve their rich cultural
heritage and insure their participation in the country’s development for national unity;
xxx"
96Article II, sec. 22; Article VI, sec. 5, par. 2; Article XII, sec. 5; Article XIII, sec. 6; Article XIV,
sec. 17; and Article XVI, sec. 12.
97 MacDonald, Indigenous Peoples of the Philippines, supra, at 345.

98 Samuel K. Tan, A History of the Philippines, p. 54 [1997].


99Cordillera Studies Program, Land Use and Ownership and Public Policy in the Cordillera,
29-30 [n.d.]; also cited in Dante B. Gatmaytan, Ancestral Domain Recognition in the
Philippines: Trends in Jurisprudence and Legislation, 5 Phil. Nat. Res. L.J. No. 1, pp. 47-48
[1992].
100Abelardo, Ancestral Domain Rights, supra, at 98-99, citing Ponciano L. Bennagen,
Indigenous Attitudes Toward Land and Natural Resources of Tribal Filipinos, 31 National
Council of Churches in the Philippines Newsletter, Oct.-Dec. 1991, at 4-9.
101 Id. at 99, citing June Prill-Brett, Bontok Land Tenure (UP Law library, mimeographed).
102Ma. Lourdes Aranal-Sereno and Roan Libarios, The Interface of National Land Law and
Kalinga Law, 58 P.L.J. 420, 440-441 [1983].
103 Ibid.
104 Ibid.
105 Ibid.
106 Ma. Lourdes Aranal-Sereno and Roan Libarios, The Interface, supra, at 420.
107
Senate Bill No. 1728 was co-sponsored by Senator Macapagal-Arroyo and co-authored by
Senators Alvarez, Magsaysay, Revilla, Mercado, Enrile, Honasan, Tatad, Maceda, Shahani,
Osmena and Romulo.

The Eighth Congress, through Senators Rasul, Estrada and Romulo filed a bill to
operationalize the mandate of the 1987 Constitution on indigenous peoples. The bill
was reported out, sponsored an interpellated but never enacted into law. In the Ninth
Congress, the bill filed by Senators Rasul and Macapagal-Arroyo was never sponsored
and deliberated upon in the floor.
108
Sponsorship Speech of Senator Flavier, Legislative History of SBN 1728, Tenth Congress,
Second Regular Session, Senate, Oct. 16, 1996, pp. 15-16.
109 Id. at 12.
110 Id. at 17-18.
111 Id. at 13.

112Journal of the Tenth Congress of the Philippines, Senate, Session No. 5, Aug. 5-6, 1997,
pp. 86-87.
113
Co-authors of the bill were Reps. Ermita, Teves, Plaza, Calalay, Recto, Fua, Luciano,
Abad, Cosalan, Aumentado, de la Cruz, Bautista, Singson, Damasing, Romualdo, Montilla,
Germino, Verceles—Proceedings of Sept. 4, 1997, pp. 00107-00108.
114 Sponsorship speech of Rep. Andolana of House Bill No. 9125, March 20, 1997.
115 Interpellation of Aug. 20, 1997, 6:16 p.m., p. 00061.
116 Section 3 [a], IPRA.
117 Section 3 [b], IPRA.
118 Guide to R.A. 8371, p. 14.
119 Section 44 [e], IPRA.
120 Section 51, IPRA.
121 Guide to R.A. 8371, p. 15.
122A CADT refers to a title formally recognizing the right of possession and ownership of
ICCs/IPs over their ancestral domains identified and delineated in acordance with the IPRA—
Rule II [c], Rules & Regulations Implementing the IPRA, NCIP Admin. Order No. 1.
123 Section 53 [a], IPRA.
124A CALT refers to a title formally recognizing the rights of the ICCs/IPs over their ancestral
lands- Rule II [d], Implementing Rules, NCIP A.O. No. 1.
125 Section 52 [k], IPRA.
126 Section 3 [1], IPRA.
127 Section 11, IPRA.
128 Ibid.

129 41 Phil. 935 (1909), 212 U.S. 449, 53 L.Ed. 594.


130
Sponsorship Speech of Senator Juan Flavier, Leg. History of SBN 1728, Tenth Congress,
Second Regular Session, Oct. 16, 1996, p. 13.
131 It was the practice of the Spanish colonial government not to issue titles to Igorots—Owen
J. Lynch, Jr., Invisible Peoples and a Hidden Agenda: The Origins of Contemporary Philippine
Land Laws (1900-1913), 63 P.L.J. 249, 288 [1988], citing the testimony of Benguet Provincial
Governnor William F. Pack, Records at 47, Cariño.
132 Maura Law or the Royal Decree of Feb. 13, 1894.
133 Later named Camp John Hay.
134 Lynch, Invisible Peoples, supra, at 288-289.

135 7 Phil. 132 [1906].


136 In 1901, Cariño had entered into a promissory agreement with a U.S. merchant in Manila.
The note obliged Cariño to sell the land at issue "as soon as he obtains from the Government
of the United States, or its representatives in the Philippines, real and definitive title." See
Lynch, Invisible Peoples, supra, at 290, citing Government’s Exhibit G, Records, at 137-138,
Cariño.
137 Cariño v. Insular Government, supra, at 939.
138Ibid.

139Id. at 940.
140Id. at 941.
141Id. at 941-942.
142Aranal-Sereno and Libarios, The Interface Between Kalinga Land Law, supra at 428-This
artcile was one of those circulated among the Constitutional Commissioners in the formulation
of Sec. 5, Article XII of the 1987 Constitution (4 Record of the Constitutional Commission 33).
143Id. at 944.
144Certificate
of Title No. 2 covering the 148 hectares of Baguio Municipality was issued not in
the name of Cariño who died on June 6, 1908, but to his lawyers John Hausserman and
Charles Cohn and his attorney-in-fact Metcalf Clarke. Hausserman, Cohn and Clarke sold the
land to the U.S. Government in a Deed of Quitclaim-Richel B. Langit, Igorot Descendants
Claim Rights to Camp John Hay, Manila Times, p. 1, Jan. 12, 1998.
145Id. at 939.
14657 P.L.J. 268, 293-296 [1982].
147From 1987 to 1988, Prof. Lynch allowed the P.L.J. to publish parts of his doctoral
dissertation at the Yale Law School entitled "Invisible Peoples: A History of Philippine Land
Law." Please see the Legal Bases of Philippine Colonial Sovereignty: An Inquiry, 62 P.L.J. 279
[1987]; Land Rights, Land Laws and Land Usurpation: The Spanish Era (1568-1898), 63 P.L.J.
82 [1988]; The Colonial Dichotomy: Attraction and Disenfranchisement, 63 P.L.J. 112; Invisible
Peoples and a Hidden Agenda: The Origins of Contemporary Philippine Land Laws (1900-
1913), 63 P.L.J. 249.
148"Native title" is a common law recognition of pre-existing aboriginal land interests in
Autsralia- Maureen Tehan, Customary Title, Heritage Protection, and Property Rights in
Australia: Emerging Patterns of Land Use in the Post-Mabo Era, 7 Pacific Rim Law & Policy
Journal, No. 3, p. 765 [June 1998].
149Lynch, Native Titles, supra, Note 164, p. 293.

15039 Phil. 660 [1919].


151Id. at 712-713.
152Id. at 694.
153Id. at 700.
15442 C.J.S., Indians, Sec. 29 [1944 ed.].
155There are 3 kinds of Indian reservations: (a) those created by treaties prior to 1871; (b)
those created by acts of Congress since 1871; and (c) those made by Executive Orders where
the President has set apart public lands for the use of the Indians in order to keep them within
a certain territory- 42 C.J.S., Indians, Sec. 29 citing Sioux Tribe of Indians v. U.S. 94 Ct. Cl.
150, 170, certiorari granted 62 S. Ct. 631, 315 U.S. 790, 86 L. Ed. 1194, affirmed 62 S. Ct.
1095, 316 U.S. 317, 86 L. Ed. 1501. It is observed that the first two kinds may include
lands possessed by aboriginal title. The last kind covers Indian reservations proper.

Until 1871, Indian tribes were recognized by the United States as possessing the
attributes of nations to the extent that treaties were made with them. In that year,
however, Congress, by statute, declared its intention thereafter to make the Indian
tribes amenable directly to the power and authority of the United States by the
immediate exercise of its legislative power over them, instead of by treaty. Since then,
Indian affairs have been regulated by acts if Congress and by contracts with the Indian
tribes practically amounting to treaties- 41 Am Jur 2d, Indians, Sec. 55 [1995 ed].
15642 C.J.S. Indians, Sec. 28 [1944 ed.].
157Ibid.;
see also U.S. v. Santa Fe Pac. R. Co., Ariz., 62 S. Ct. 248, 314 U.S. 339, 86 L. Ed.
260 [1941].
158Ibid.

1598 Wheat 543, 5 L. Ed. 681 [1823].


160Id. at 680.
161Id. at 689.
162Id. at 696; see also 41 ALR Fed 425, Annotation: Proof and Extinguishment of Aboriginal
Title to Indian Lands, Sec. 2[a] [1979].
163Buttz v. Northern Pac.R. Co., Dak., 7 S. Ct. 100, 119 U.S. 55, 30 L. Ed. 330, 335 [1886].
164Lynch, Native Title, supra, at 293-294; Cohen, Original Indian Title, 32 Minn. L.R. 48-49
[1947].
1656 Pet 515, 8 L.Ed. 483 [1832].
166Id. at 499.
167Id. at 500.
168Id. at 501.

169The title of the government to Indian lands, the naked fee, is a sovereign title, the
government having no landlord from whom it holds the fee- Shoshone Tribe of Indians of Wind
River Reservation in Wyoming v. U.S., 85 Ct. Cl. 331, certiorari granted U.S. v. Shoshone
Tribe of Indians, 58 S. Ct. 609, 303 U.S. 629, 82 L. Ed. 1090, affirmed 58 S. Ct. 794, 304 U.S.
111, 82 L. Ed. 1213, 1218-1219 [1938].
170Buttzv. Northern Pac. R. Co., Dak., at 30 L. Ed. 330, 335; Beecher v. Wetherby, Wis., 95
U.S. 517, 24 L. Ed. 440, 441 [1877]; see also 42 C.J.S., Indians, Sec. 28 [1944 ed.].
171Annotation, Proof and Extinguishment of Aboriginal title to Indian Lands, 41 ALR Fed 425,
Sec. 2 [b] [1979]- hereinafter cited as Aboriginal Title to Indian Lands.
172Ibid.;see also Tee Hit Ton Indians v. U.S., 348 U.S. 272, 99 L. Ed. 314, 320, 75 S. Ct. 313
[1955], reh den 348 U.S. 965, 99 L. Ed. 753, 75 S. Ct. 521.
173Ibid.; Tee Hit Ton Indians v. U.S., at 99 L. Ed. 320.
174Oneida Indian Nation v. County of Oneida, 414 U.S. 661, 39 L. Ed. 2d 73, 94 S Ct. 772
[1974]; U.S. v. Alcea Bank of Tillamooks, 329 U.S. 40, 91 L. Ed. 29. 67 S. Ct. 167 [1946].
175For compensation under the Indian Claims Commission Act, the proof of aboriginal title rests
on actual, exclusive and continuous use and occupancy for a long time prior to the loss of the
property. (The Indian Claims Commission Act awards compensation to Indians whose
aboriginal titles were extinguished by the government through military conquest, creation of a
reservation, forced confinement of Indians and removal of Indians from certain portions of the
land an the designation of Indian land into forest preserve, grazing district, etc.) - Aboriginal
Title to Indian Lands, supra, at Secs. 2[a], 3[a], pp. 431, 433, 437.
176Aboriginal Title to Indian Lands, supra, at Sec. 2[b], p. 435.
17741 Am Jr 2d, Indians, Sec. 59 [1995 ed.].

178An allotment of Indian land contains restrictions on alienation of the land. These restrictions
extend to a devise of the land by will- Missouri, K. & T.R. Co. v. U.S., 235 U.S. 37, 59 L. Ed.
116,. 35 S. Ct. 6 [1914]; A railroad land grant that falls within Indian land is null and void-
Northern P. R. Co. v. U.S., 227 U.S. 355, 57 L.Ed. 544,33 S. Ct. 368 [1913]; Portions of Indian
land necessary for a railroad right of way were, by the terms of the treaty, declared "public
land," implying that land beyond the right of way was private- Kindred v. Union P.R. Co., 225
U.S. 582, 56 L. Ed. 1216, 32 S. Ct. 780 [1912]; see also 41 Am Jur 2d, Indians, Sec. 58 [1995
ed].
179Aboriginal Title to Indian Lands, supra, at Sec. 2[a], p. 433.
18042 C.J.S. Indians, Sec. 29 [1944 ed.]
181Ibid.

182North American Indians have made much progress in establishing a relationship with the
national government and developing their own laws. Some have their own government-
recognized constitutions. Usually the recognition of Indian tribes depends on whether the tribe
has a reservation. North American tribes have reached such an advanced stage that the main
issues today evolve around complex jurisdictional and litigation matters. Tribes have acquired
the status of sovereign nations within another nation, possessing the right to change and grow-
Jose Paulo Kastrup, The Internationalization of Indigenous Rights from the Environmental and
Human Rights Perspective, Texas International Law Journal, vol. 32: 97, 104 [1997].
183Lynch, Native Title, supra, at 293.
184Dante Gatmaytan, Ancestral Domain Recognition in the Philippines: Trends in
Jurisprudence and Legislation, 5 Phil. Nat. Res. L.J. No. 1, pp. 43, 40 [Aug. 1992]; see also
Tee Hit Ton Indians v. U.S., supra, at 320.
185Ibid.

186D. Gatmaytan, supra, citing Churchill, The Earth is Our Mother: Struggles for American
Indian Land and Liberation in the Contemporary United States, The State of Native America:
Genocide, Colonization and Resistance 139 (M. Jaimes 1992); and Indian Law Resource
Center, United States Denial of Indian Property Rights: A Study in Lawless Power and Racial
Discrimination, Rethinking Indian Law 15 (National Lawyers Guild, Committee on Native
American Struggles 1982).
187Id.,
Note 28, stating that some earlier decisions of the U.S. Supreme Court have held that
Congress is subject to the strictures of the Constitution in dealing with Indians. When an Indian
property is taken for non-Indian use, the U.S. government is liable for payment of
compensation, and an uncompensated taking may be enjoined. F. Cohen, Handbook of
Federal Indian Law 217 [1982], citing Shoshone Tribe v. U.S. 299 U.S. 476 [1937]; Choate v.
Trapp, 224 U.S. 665 [1912]; and Lane v. Pueblo of Santa Rosa, 249 U.S. 110 [1919].
188See Discussion, infra, Part IV (c) (2).
189Susi v. Razon, 48 Phil. 424 [1925]; Herico v. Dar, 95 SCRA 437 [1980].
190Ibid.

191Directorof Lands v. Intermediate Appellate Court, 146 SCRA 509 [1986]; Director of
Lands v. Buyco, 216 SCRA 78 [1992]; Republic v. Court of Appeals and Lapina, 235 SCRA
567 [1994].
19275 Phil. 890 [1946].
193Id. at 892.

194Sec. 48 [b], C.A. 141.


195Sec. 48 [c], C.A. 141, as amended. This provision was added in 1964 by R.A. 3872.
196Section 12, IPRA.
197"Time immemorial" refers "to a period of time when as far back as memory can go, certain
ICCs/Ips are known to have occupied, possessed in the concept of owner, and utilized a
defined territory devolved to them, by operation of customary law or inherited from their
ancestors, in accordance with their customs and traditions." (Sec. 3 [p], IPRA).
198Section 2, C.A. 141.
199Section 8, C.A. 141.
200The classification of ancestral lands 18% in slope or over as alienable in the IPRA is an
exception to Section 15, P.D. 705, the Revised Forestry Code.
201Charles MacDonald, Indigenous Peoples of the Philippines: Between Segregation and
Integration, Indigenous Peoples of Asia, supra, at pp. 345, 350.

202 Section 5, Article XII, 1987 Constitution.


203 Words in bold were amendments introduced by R.A. 3872 in 1964.
204Words in bold were amendments introduced by R.A. 3872 on June 18, 1964. On January
25, 1977, however, Sec. 48 [b] and 48 [c] were further amended by P.D. 1073 stating that
these provisions on cultural minorities apply only to alienable and disposable lands of the
public domain- Please see Republic v. CA and Paran, 201 SCRA 1, 10-11 [1991].
205 Jus utendi, jus fruendi.
206 Jus abutendi.
207 Jus disponendi.
208Jus vindicandi. Please see Tolentino, Civil Code, vol. II, pp. 45-46 [1992]; see also
Tolentino, vol. I, pp. 12-14.
209 Sec. 55, IPRA provides:

"Sec. 55. Communal rights.- Subject to Section 56 hereof, areas within the ancestral
domains, whether delineated or not, shall be presumed to be communally held:
provided, That communal rights under this Act shall not be construed as co-ownership
as provided in Republic Act No. 386, otherwise known as the New Civil Code."
210 Ibid.
211 Article 494, Civil Code.
212
Antonio M. La Vina, Arguments for Communal Title, Part II, 2 Phil. Nat. Res. L. J. 23 [Dec.
1989].
213 Section 11, Corporation Code.
214 Sections 60-72, Corporation Code.
215
Section 117, Corporation Code. Please see also La Vina, Arguments for Communal Title,
Part II, supra, at 23.

216 Section 5, par. 2, Article XII, 1987 Constitution.


217
Customary law is recognized by the Local Government Code of 1991 in solving disputes
among members of the indigenous communities, viz:

"Sec. 412 (c) Conciliation among members of indigenous cultural communities.- The
customs and traditions of indigenous cultural communities shall be applied in settling
disputes between members of the cultural communities."
218 Law writes custom into contract-Hongkong & Shanghai Bank v. Peters, 16 Phil. 284 [1910].

The Civil Code provides:

"Art. 11. Customs which are contrary to law, public order or public policy shall not be
countenanced."

"Art. 12. A custom must be proved as a fact, according to the rules of evidence."

219Article 78 on marriages between Mohammedans or pagans who live in the non-Christian


provinces- this is now Art. 33 of the Family Code; Art. 118, now Art. 74 of the Family Code on
property relations between spouses; Art. 577 on the usufructuary of woodland; Art. 657 on
easement of right of way for passage of livestock; Arts. 678, 1315, 1376, 1522, 1564 and
1577. Please see Aquino, Civil Code, vol. 1, p. 25.
220
Castle Bros. v. Gutierrez Hermanos, 11 Phil. 629 [1908]; In Re: Firm Name of Ozaeta
Romulo, 92 SCRA 1 [1979]; Yao Kee v. Sy-Gonzales, 167 SCRA 736 [1988]; Please see
Aquino, Civil Code, vol. 1, p. 26 for a list of other cases.
221This situation is analogous to the Muslim code or the Code of Muslim Personal Laws (P.D.
1083) which took effect on February 4, 1977 despite the effectivity of the Civil Code and the
Family Code. P.D. 1083 governs persons, family relations and succession among Muslims, the
adjudication and settlement of disputes, the organization of the Shari’a courts, etc.
222Mariflor P. Pagusara, The Kalinga Ili: Cultural-Ecological Reflections on Indigenous Theora
and Praxis of Man-Nature Relationship, Dakami Ya Nan Dagami, p. 36, Papers and
Proceedings of the 1st Cordillera Muti-Sectoral Land Congress, 11-14 March 1983, Cordillera
Consultative Committee [1984].
223 Section 2, Article XII.
224A "co-production agreement" is defined as one wherein the government provides input to
the mining operation other than the mineral resource- Section 26 (b), R.A. 7942, the Philippine
Mining Act of 1995.
225A "joint venture agreement" is one where a joint-venture company is organized by the
government and the contractor with both parties having equity shares, and the government
entitled to a share in the gross output- Section 26 (c), R.A. 7942.
226A mineral "production-sharing agreement" is one where the government grants to the
contractor the exclusive right to conduct mining operations within a contract area and shares in
the gross output. The contractor provides the financing, technology, management and
personnel necessary for the implementation of the agreement- Section 26 (a), R.A. 7942.
227 Section 26, R.A. 7942.
228 Section 3 [d], People's Small-Scale Mining Act of 1991 (R.A. 7076) provides:

"Sec. 3 [d] 'Small-scale mining contract' refers to co-production, joint venture or mineral
production sharing agreement between the State and a small-scale mining contractor
for the small-scale utilization of a plot of mineral land."
229 Section 3 [b], R.A. 7076.
230 NCIP Administrative Order No. 1, Series of 1998.
231In Republic v. Court of Appeals, 160 SCRA 228, 239 [1988], Cruz, J., ponente, it was
declared that if a person is the owner of a piece of agricultural land on which minerals are
discovered, his ownership of such land does not give him the right to extract or utilize the said
minerals without the permission of the State to which such minerals belong- also cited in H. de
Leon, Phil. Constitutional Law, Principles and Cases, vol. 2, pp. 800-801 [1999].
232 See Ground I, Grounds to Issue Writ of Prohibition, Petition, p. 14.
233 Section 7 (b) is subject to Section 56 of the same law which provides:
"Sec. 56. Existing Property Rights Regimes.- Property rights within the ancestral
domains already existing and/or vested upon effectivity of this Act, shall be recognized
and respected."

The law took effect 15 days upon publication in the O.G. or in any 2 newspapers of
general circulation (Sec. 84, IPRA). The IPRA was published in the Chronicle and
Malaya on Nov. 7, 1997.
234
Section 9 of the IPRA also gives the ICCs/IPs the ff. responsibilities over their ancestral
domains:

(a) Maintain Ecological Balance- To preserve, restore, and maintain a balanced ecology
in the ancestral domain by protecting the flora and fauna, watershed areas, and other
reserves;

(b) Restore Denuded Areas.- To actively initiate, undertake and participate in the
reforestation of denuded areas and other development programs and projects subject to
just and reasonable renumeration;

(c) Observe Laws.- To observe and comply with the provisions of this Act and the rules
and regulations for its effective implementation."

Section 58 of the same law also mandates that ancestral domains or portions thereof,
which are found to be necessary for critical watersheds, mangroves, wildlife
sanctuaries, wilderness, protected areas, forest cover, or reforestation as determined by
appropriate agencies with the full participation of the ICCs/IPs concerned shall be
maintained, managed and developed for such purposes. The ICCs/IPs concerned shall
be given the responsibility to maintain, develop, protect and conserve such areas with
the full and effective assistance of government agencies.
235Hector S. de Leon, Textbook on the New Philippine Constitution pp. 473-474 [1987] citing
the 1986 UP Law Constitution Project, The National Economy and Patrimony, p. 11.
236Under the Small-Scale Mining Act of 1991, "small-scale mining" refers to "mining activities
which rely heavily on manual labor using simple implements and methods and do not use
explosives or heavy mining equipment"- Section 3 [b], R.A. 7076.
237 See infra., pp. 77-79?.
238
Andrew Gray, The Indigenous Movement in Asia, Indigenous Peoples of Asia, ed. By
Barnes, Gray and Kingsbury, pub. By Ass'n. for Asian Studies, at 35, 42 [1995].
239 E.g. International Indian Treaty Council, World Council of IPs.
240 Gray, The Indigenous Movement in Asia, supra, at 44, citing the International Work Group
for Indigenous Affairs, 1988.
241
Jose Paulo Kastrup, The Internationalization of Indigenous Rights from the Environmental
and Human Rights Perspective, 32 Texas International Law Journal 97, 102 [1997].
242 Benedict Kingsbury, "Indigenous Peoples" in International Law: A Constructivist Approach
to the Asian Controversy, The American Journal of International Law, vol. 92: 414, 429 [1998].
243The World Bank supported the Chico Dam project. Due to the Kalingas' opposition, the WB
pulled out of the project but the conflict between the Philippine government and the natives
endured long after- Marcus Colchester, Indigenous Peoples' Rights and Sustainable Resource
Use in South and Southeast Asia, Indigenous Peoples of Asia, supra, pp. 59, 71-72.
244 Kingsbury, supra, at 417.

245 Section 22, Article II, 1987 Constitution.


246
Interpellation of Senator Flavier on S.B. No. 1728, Deliberation on Second Reading,
November 20, 1996, p. 20.
247Guide to R.A. 8371, Coalition for IPs Rights and Ancestral Domains, the International Labor
Organization, and the ILO-Bilance- Asia Dep't, p. 3 [1999].
248 Also referred to as the "Indigenous and Tribal Peoples Convention, 1989."
249 See Introduction to ILO Convention No. 169, par. 4.
250 Id., pars. 5 and 6.

251
Perfecto V. Fernandez, Towards a Definition of National Policy on Recognition of Ethnic
Law within the Philippine Legal Order, 55 P.L.J. 383, 385 [1980].
252
Samuel K. Tan, A History of the Philippines, Manila Studies Association, Inc. and the Phil.
National Historical Society, Inc., p. 6 [1997].
253 Fernandez, supra, at 385, 391.

The Lawphil Project - Arellano Law Foundation

SEPARATE OPINION

VITUG, J.:

An issue of grave national interest indeed deserves a proper place in any forum and, when it
shows itself in a given judicial controversy, the rules of procedure, like locus standi, the
propriety of the specific remedy invoked, or the principle of hierarchy of courts, that may
ordinarily be raised by party-litigants, should not be so perceived as good and inevitable
justifications for advocating timidity, let alone isolationism, by the Court.

A cardinal requirement, to which I agree, is that one who invokes the Court’s adjudication must have
a personal and substantial interest in the dispute;1 indeed, the developing trend would require
a logical nexus between the status asserted and the claim sought to be adjudicated in order to ensure
that one is the proper and appropriate party to invoke judicial power. 2 The rule requires a party to
aptly show a personal stake in the outcome of the case or an injury to himself that can be redressed
by a favorable decision so as to warrant his invocation of the Court’s jurisdiction and to render legally
feasible the exercise of the Court’s remedial powers in his behalf. If it were otherwise, the exercise of
that power can easily become too unwieldy by its sheer magnitude and scope to a point that may, in
no small measure, adversely affect its intended essentiality, stability and consequentiality.

Nevertheless, where a most compelling reason exits, such as when the matter is of transcendental
importance and paramount interest to the nation,3 the Court must take the liberal approach that
recognizes the legal standing of nontraditional plaintiffs, such as citizens and taxpayers, to raise
constitutional issues that affect them.4 This Court thus did so in a case5 that involves the conservation
of our forests for ecological needs. Until and exact balance is struck, the Court must accept an
eclectic notion that can free itself from the bondage of legal nicety and hold trenchant
technicalities subordinate to what may be considered to be of overriding concern.

The petition seeks a declaration by the Court of unconstitutionality of certain provisions of Republic
Act No. 8371, a law that obviously is yet incapable of exact equation in its significance to the nation
and its people now and in the generations yet to come. Republic Act No. 8371, otherwise also known
as the Indigenous Peoples Rights Act of 1997 ("IPRA"), enacted into law in 1997 and made effective
on 22 November 1997, is apparently intended to be a legislative response to the 1987 Constitution
which recognizes the rights of indigenous cultural communities "within the framework of national unity
and development"6 and commands the State, "subject to the provisions of this Constitution and
national development policies and programs," to protect the rights of indigenous cultural
communities to their ancestral lands in order to ensure their economic, social, and cultural well-
being.7

Among the assailed provisions in IPRA is its Section 3(a) which defines "ancestral domains" to
embrace "all areas generally belonging to ICCs/IPs comprising lands, inland waters, coastal
areas, and natural resources" including "ancestral lands, forest, pasture, residential,
agricultural, and other lands individually owned whether alienable and disposable or
otherwise," over which indigenous cultural communities/indigenous
peoples ("ICCs/IPs") could exercise virtual ownership and control.

IPRA effectively withdraws from the public domain the so-called ancestral domains covering
literally millions of hectares. The notion of community property would comprehend not only
matters of proprietary interest but also some forms of self-governance over the curved-out
territory. This concept is elaborated in Section 7 of the law which states that the "rights of ownership
and possession of ICCs/IPs to their ancestral domains shall be recognized and protected," subsumed
under which would encompass the right of ownership (paragraph a); the right to develop, control
and use lands and natural resources, including "the right to negotiate the terms and conditions
for the exploration of natural resources in the areas for the purpose of ensuring ecological,
environmental protection and the conservation measures, pursuant to national and customary laws;"
(par. b); the right to stay in the territories (par. c); the right to return to their abandoned lands in
case of displacement (par. d); the right to regulate entry of migrants (par. e); the right to claim
parts of ancestral domains previously reserved (par. g); and the right to resolve land conflicts
in accordance primarily with customary law (par. h). Concurrently, Section 57 states that
ICCs/IPs shall be given "priority rights in the harvesting, extraction, development or exploitation of any
natural resources within the ancestral domains." These provisions of IPRA, in their totality, are, in
my view, beyond the context of the fundamental law and virtually amount to an undue
delegation, if not an unacceptable abdication, of State authority over a significant area of the
country and its patrimony.
Article XII of the 1987 Constitution expresses that all "lands of the public domain, waters,
minerals, coal, petroleum, and other mineral oils, all forces of potential energy, fisheries,
forest or timber, wildlife, flora and fauna, and other natural resources are owned by the State,"
and, with the exception of agricultural lands, "shall not be alienated." It ordains that the
"exploration, development, and utilization of natural resources shall be under the full control
and supervision of the State."8

These provisions had roots in the 1935 Constitution which, along with some other specific mandates
in the 1935 Constitution, forming Article XII under the title "Conservation and Utilization of Natural
Resources", were derived largely from the report of the Committee on Nationalization and
Preservation of Lands and other Natural Resources.9 According to the Committee report, among the
principles upon which these provisions were based, was "that the land, minerals, forest and other
natural resources constitute the exclusive heritage of the Filipino Nation," and should thereby "be
preserved for those under the sovereign authority of the Nation and for their posterity." 10 The
delegates to the 1934 Constitutional Convention were of the unanimous view that the "policy on
natural resources, being fundamental to the nation’s survival should not be left to the changing mood
of the lawmaking body."11

The 1987 Constitution, like the precursor provisions in the 1935 and 1973 Constitutions, thus
expresses this regalian doctrine of the old, and the domainial doctrine of the new, that all lands and
natural resources belong to the state other than those which it recognizes to be of private
ownership. Except for agricultural lands of the public domain which alone may be alienated,
forest or timber, and mineral lands, as well as all other natural resources, of the country must
remain with the state, the exploration, development and utilization of which shall be subject to
its full control and supervision albeit allowing it to enter into co-production, joint venture or
production-sharing agreements, or into agreements with foreign-owned corporations involving
technical or financial assistance for large-scale exploration, development and utilization.12

The decision of the United States Supreme Court in Cariño vs. Insular Government,13 holding that a
parcel of land held since time immemorial by individuals under a claim of private ownership is
presumed never to have been public land and cited to downgrade the application of the regalian
doctrine, cannot override the collective will of the people expressed in the Constitution. It is in them
that sovereignty resides and from them that all government authority emanates. 14 It is not then for a
court ruling or any piece of legislation to be conformed to by the fundamental law, but it is for the
former to adapt to the latter, and it is the sovereign act that must, between them, stand inviolate.

The second paragraph of Section 5 of Article XII of the Constitution allows Congress to provide "for
the applicability of customary laws governing property rights or relations in determining the ownership
and extent of ancestral domains." I do not see this statement as saying that Congress may enact a
law that would simply express that "customary laws shall govern" and end it there. Had it been so, the
Constitution could have itself easily provided without having to still commission Congress to do it. Mr.
Chief Justice Davide has explained this authority of Congress, during the deliberations of the 1986
Constitutional Convention, thus:

"Mr. Davide. x x x Insofar as the application of the customary laws governing property rights or
relations in determining the ownership and extent of the ancestral domain is concerned, it is
respectfully submitted that the particular matter must be submitted to Congress. I understand that the
idea of Comm. Bennagen is for the possibility of the codification of these customary laws. So before
these are codified, we cannot now mandate that the same must immediately be applicable. We leave
it to Congress to determine the extent of the ancestral domain and the ownership thereof in relation to
whatever may have been codified earlier. So, in short, let us not put the cart ahead of the horse." 15
The constitutional aim, it seems to me, is to get Congress to look closely into the customary
laws and, with specificity and by proper recitals, to hew them to, and make them part of, the
stream of laws. The "due process clause," as I so understand it in Tanada vs. Tuvera16 would
require an apt publication of a legislative enactment before it is permitted to take force and effect. So,
also, customary laws, when specifically enacted to become part of statutory law, must first undergo
that publication to render them correspondingly binding and effective as such.

Undoubtedly, IPRA has several good points, and I would respectfully urge Congress to re-
examine the law. Indeed, the State is exhorted to protect the rights of indigenous cultural
communities to their ancestral lands, a task that would entail a balancing of interest between
their specific needs and the imperatives of national interest.

WHEREFORE, I vote to grant the petition.

Footnotes
1People vs. Vera, 65 Phil. 56, 89; Macasiano vs. National Housing Authority, 224 SCRA 236,
244.
2 Am Jur § 189, p. 591, S. vD., 410 US 641, 35 L Ed 2d 536, 93 S Ct 1146.
3Legaspi vs. Civil Service Commission, 150 SCRA 530, 540; Tañada vs. Tuvera, 136 SCRA
27, 36, 37.
4Defensor Santiago, Miriam, Constitutional Law, First Edition, 1994, p. 11; see also Rev. Fr.
Joaquin Bernas, S.J., on the 1987 Constitution of the Republic of the Philippines, 1996 Ed., pp.
336-337.
5 Oposa vs. Factoran, Jr., 224 SCRA 792.
6 Art. 11, Sec. 22.
7 Art. XII, Sec. 5.
8 Sec. 2.
9 II Aruego, The Framing of the Philippine Constitution, p. 594.
10 Ibid., p. 595.
11 Ibid., p. 600.
12
CONST., Art. XII, Sec. 2; Miners Association of the Philippines, Inc., vs. Factoran, Jr., 240
SCRA 100.
13 41 Phil. 935.
14 CONST., Art. II, Sec. 1.
15 4 Record of the Constitutional Commission 32.
16 146 SCRA 446.

The Lawphil Project - Arellano Law Foundation

SEPARATE OPINION

KAPUNAN, J.:

You ask if we own the land. . . How can you own that which will outlive you? Only the race own the
land because only the race lives forever. To claim a piece of land is a birthright of every man. The
lowly animals claim their place; how much more man? Man is born to live. Apu Kabunian, lord of us
all, gave us life and placed us in the world to live human lives. And where shall we obtain life? From
the land. To work (the land) is an obligation, not merely a right. In tilling the land, you possess it. And
so land is a grace that must be nurtured. To enrich it and make it fructify is the eternal exhortation of
Apu Kabunian to all his children. Land is sacred. Land is beloved. From its womb springs …life.

- Macli-ing Dulag, Chieftain of the Kalinga Tribe (quoted in Ponciano L. Bennagen, "Tribal Filipinos" in
Indigenous View of Land and the Environment, ed. Shelton H. Davis, the World Bank Discussion
Papers, No. 188, pp. 71-72.)

It is established doctrine that a statute should be construed whenever possible in harmony with,
rather than in violation of, the Constitution.1 The presumption is that the legislature intended to enact
a valid, sensible and just law and one which operates no further than may be necessary to effectuate
the specific purpose of the law.2

The challenged provisions of the Indigenous Peoples Rights Act (IPRA) must be construed in view of
such presumption of constitutionality. Further, the interpretation of these provisions should take into
account the purpose of the law, which is to give life to the constitutional mandate that the rights of the
indigenous peoples be recognized and protected.

The struggle of our indigenous peoples to reclaim their ancestral lands and domains and therefore,
their heritage, is not unique. It is one that they share with the red-skinned "Indians" of the United
States, with the aborigines of Australia, the Maori of New Zealand and the Sazmi of Sweden, to name
a few. Happily, the nations in which these indigenous peoples live all have enacted measures in an
attempt to heal an oppressive past by the promise of a progressive future. Thus has the international
community realized the injustices that have been perpetrated upon the indigenous peoples. This
sentiment among the family of nations is expressed in a number of documents, the most recent and
most comprehensive of which is the Draft United Nations Declaration on the Rights of Indigenous
Peoples which was adopted by the UN Sub-Commission on Prevention of Discrimination and
Protection of Minorities by its resolution on August 26, 1994. Among the rights recognized by the UN
Draft is the restitution of lands, territories and even the resources which the indigenous peoples have
traditionally owned or otherwise occupied or used, and which have been confiscated, occupied, used
or damaged without the free and informed consent of the indigenous peoples.

A Historical Backdrop on the Indigenous Peoples


The term "indigenous" traces its origin to the Old Latin word indu, meaning "within." In the sense the
term has come to be used, it is nearer in meaning to the Latin word indigenus, which means
"native."3 "Indigenous" refers to that which originated or has been produced naturally in a particular
land, and has not been introduced from the outside.4 In international law, the definition of what
constitutes "indigenous peoples" attains some degree of controversy. No definition of the term
"indigenous peoples" has been adopted by the United Nations (UN), although UN practice has been
guided by a working definition in the 1986 Report of UN Special Rapporteur Martinez Cobo: 5

Indigenous communities, peoples and nations are those which, having a historical continuity with pre-
invasion and pre-colonial societies that developed on their territories, consider themselves distinct
from other sections of the societies now prevailing in those territories, or parts of them. They form at
present non-dominant sections of society and are determined to preserve, develop and transmit to
future generations their ancestral territories, and their ethnic identity, as the basis of their continued
existence as peoples, in accordance with their own cultural patterns, social institutions and legal
systems.

This historical continuity may consist of the continuation, for an extended period reaching into the
present, of one or more of the following factors:

(a) Occupation of ancestral lands, or at least of part of them;

(b) Common ancestry with the original occupants of these lands;

(c) Culture in general, or in specific manifestations (such as religion, living under a tribal
system, membership of an indigenous community, dress, means of livelihood, life-style, etc.);

(d) Language (whether used as the only language, as mother-tongue, as the habitual means of
communication at home or in the family, or as the main, preferred, habitual, general or normal
language);

(e) Residence in certain parts of the country; or in certain regions of the world;

(f) Other relevant facts.6

In Philippine constitutional law, the term "indigenous peoples" pertains to those groups of Filipinos
who have retained a high degree of continuity from pre-Conquest culture.7 Philippine legal history,
however, has not been kind to the indigenous peoples, characterized them as
"uncivilized,"8 "backward people,"9 with "barbarous practices"10 and "a low order of intelligence."11

Drawing inspiration from both our fundamental law and international law, IPRA now employs the
politically-correct conjunctive term "indigenous peoples/indigenous cultural communities" as follows:

Sec. 3. Definition of Terms.- For purposes of this Act, the following terms shall mean:

xxx

(h) Indigenous peoples/Indigenous cultural communities. - refer to a group of people or homogenous


societies identified by self-ascription and ascription by others, who have continuously lived as
organized community on communally bounded and defined territory, and who have, under claims of
ownership since time immemorial, occupied, possessed and utilized such territories, sharing common
bonds of language, customs, traditions, and other distinctive cultural traits, or who have, through
resistance to political, social and cultural inroads of colonization, non-indigenous religions and
cultures, became historically differentiated from the majority of Filipinos. Indigenous peoples shall
likewise include peoples who are regarded as indigenous on account of their descent from the
populations which inhabited the country at the time of conquest or colonization, or at the time of
inroads of non-indigenous religions and cultures, or the establishment of present State boundaries,
who retain some or all of their own social, economic, cultural and political institutions, but who may
have been displaced from their traditional domains or who may have resettled outside their ancestral
domains x x x.

Long before the Spaniards set foot in these islands, the indigenous peoples were already plowing our
soil and hunting in our forests. The Filipinos of Aeta and Malay stock, who were the original
inhabitants of our archipelago, were, at that time, practicing a native culture. From the time the
Spaniards arrived up to the early part of the American regime, 12 these native inhabitants resisted
foreign invasion, relentlessly fighting for their lands. Today, from the remote uplands of Northern
Luzon, to Palawan, Mindoro and Mindanao, the indigenous peoples continue to live on and cultivate
their ancestral lands, the lands of their forefathers.

Though Filipinos today are essentially of the same stock as the indigenous peoples, our national
culture exhibits only the last vestiges of this native culture. Centuries of colonial rule and neocolonial
domination have created a discernible distinction between the cultural majority and the group of
cultural minorities.13 The extant Philippine national culture is the culture of the majority; its indigenous
roots were replaced by foreign cultural elements that are decidedly pronounced, if not
dominant.14 While the culture of the majority reoriented itself to Western influence, the culture of the
minorities has retained its essentially native character.

One of every six Filipinos is a member of an indigenous cultural community. Around twelve million
Filipinos are members of the one hundred and ten or so indigenous cultural
communities,15 accounting for more than seventeen per centum of the estimated seventy million
Filipinos16 in our country. Sadly, the indigenous peoples are one of the poorest sectors of Philippine
society. The incidence of poverty and malnutrition among them is significantly higher than the national
average. The indigenous peoples are also among the most powerless. Perhaps because of their
inability to speak the language of law and power, they have been relegated to the fringes of society.
They have little, if any, voice in national politics and enjoy the least protection from economic
exploitation.

The Constitutional Policies on Indigenous Peoples

The framers of the 1987 Constitution, looking back to the long destitution of our less fortunate
brothers, fittingly saw the historic opportunity to actualize the ideals of people empowerment and
social justice, and to reach out particularly to the marginalized sectors of society, including the
indigenous peoples. They incorporated in the fundamental law several provisions recognizing and
protecting the rights and interests of the indigenous peoples, to wit:

Sec. 22. The State recognizes and promotes the rights of indigenous peoples within the framework of
national unity and development.17

Sec. 5. The State, subject to the provisions of this Constitution and national development policies and
programs, shall protect the rights of indigenous cultural communities to their ancestral lands to ensure
their economic, social, and cultural well-being.

The Congress may provide for the applicability of customary laws governing property rights and
relations in determining the ownership and extent of ancestral domains.18
Sec. 1. The Congress shall give the highest priority to the enactment of measures that protect and
enhance the right of all the people to human dignity, reduce social, economic and political
inequalities, and remove cultural inequities by equitably diffusing wealth and political power for the
common good.

To this end, the State shall regulate the acquisition, ownership, use and disposition of property and its
increments.19

Sec. 6. The State shall apply the principles of agrarian reform or stewardship, whenever applicable in
accordance with law, in the disposition and utilization of other natural resources, including lands of
the public domain under lease or concession, subject to prior rights, homestead rights of small
settlers, and the rights of indigenous communities to their ancestral lands.20

Sec. 17. The State shall recognize, respect, and protect the rights of indigenous cultural communities
to preserve and develop their cultures, traditions, and institutions. It shall consider these rights in the
formulation of national plans and policies.21

Sec. 12. The Congress may create a consultative body to advise the President on policies affecting
indigenous cultural communities, the majority of the members of which shall come from such
communities.22

IPRA was enacted precisely to implement the foregoing constitutional provisions. It provides, among
others, that the State shall recognize and promote the rights of indigenous peoples within the
framework of national unity and development, protect their rights over the ancestral lands and
ancestral domains and recognize the applicability of customary laws governing property rights or
relations in determining the ownership and extent of the ancestral domains. 23 Moreover, IPRA
enumerates the civil and political rights of the indigenous peoples; 24 spells out their social and cultural
rights;25 acknowledges a general concept of indigenous property right and recognizes title
thereto;26 and creates the NCIP as an independent agency under the Office of the President. 27

Preliminary Issues

A. The petition presents an actual controversy.

The time-tested standards for the exercise of judicial review are: (1) the existence of an appropriate
case; (2) an interest personal and substantial by the party raising the constitutional question; (3) the
plea that the function be exercised at the earliest opportunity; and (4) the necessity that the
constitutional question be passed upon in order to decide the case. 28

Courts can only decide actual controversies, not hypothetical questions or cases. 29 The threshold
issue, therefore, is whether an "appropriate case" exists for the exercise of judicial review in the
present case.

An "actual case or controversy" means an existing case or controversy which is both ripe for
resolution and susceptible of judicial determination, and that which is not conjectural or
anticipatory,30 or that which seeks to resolve hypothetical or feigned constitutional problems. 31 A
petition raising a constitutional question does not present an "actual controversy," unless it alleges a
legal right or power. Moreover, it must show that a conflict of rights exists, for inherent in the term
"controversy" is the presence of opposing views or contentions.32 Otherwise, the Court will be forced
to resolve issues which remain unfocused because they lack such concreteness provided when a
question emerges precisely framed from a clash of adversary arguments exploring every aspect of a
multi-faceted situation embracing conflicting and demanding interests. 33 The controversy must also
be justiciable; that is, it must be susceptible of judicial determination.34

In the case at bar, there exists a live controversy involving a clash of legal rights. A law has been
enacted, and the Implementing Rules and Regulations approved. Money has been appropriated and
the government agencies concerned have been directed to implement the statute. It cannot be
successfully maintained that we should await the adverse consequences of the law in order to
consider the controversy actual and ripe for judicial resolution. It is precisely the contention of the
petitioners that the law, on its face, constitutes an unconstitutional abdication of State ownership over
lands of the public domain and other natural resources. Moreover, when the State machinery is set
into motion to implement an alleged unconstitutional statute, this Court possesses sufficient authority
to resolve and prevent imminent injury and violation of the constitutional process.

B. Petitioners, as citizens and taxpayers, have the requisite standing to raise the constitutional
questions herein.

In addition to the existence of an actual case or controversy, a person who assails the validity of a
statute must have a personal and substantial interest in the case, such that, he has sustained, or will
sustain, a direct injury as a result of its enforcement.35 Evidently, the rights asserted by petitioners as
citizens and taxpayers are held in common by all the citizens, the violation of which may result only in
a "generalized grievance".36 Yet, in a sense, all citizen’s and taxpayer’s suits are efforts to air
generalized grievances about the conduct of government and the allocation of power. 37

In several cases, the Court has adopted a liberal attitude with regard to standing. 38 The proper party
requirement is considered as merely procedural,39 and the Court has ample discretion with regard
thereto.40 As early as 1910, the Court in the case of Severino vs. Governor General 41 held:

x x x When the relief is sought merely for the protection of private rights, the relator must show some
personal or special interest in the subject matter, since he is regarded as the real party in interest and
his right must clearly appear. Upon the other hand, when the question is one of public right and
the object of the mandamus is to procure the enforcement of a public duty, the people are regarded
as the real party in interest, and the relator at whose instigation the proceedings are instituted
need not show that he has any legal or special interest in the result, it being sufficient to show
that he is a citizen and as such interested in the execution of the laws.42

This Court has recognized that a "public right," or that which belongs to the people at large, may also
be the subject of an actual case or controversy. In Severino, we ruled that a private citizen may
enforce a "public right" in behalf of other citizens. We opined therein that:

… The right which [petitioner] seeks to enforce is not greater or different from that of any other
qualified elector in the municipality of Silay. It is also true that the injury which he would suffer in case
he fails to obtain the relief sought would not be greater or different from that of the other electors;
but he is seeking to enforce a public right as distinguished from a private right. The real party in
interest is the public, or the qualified electors of the town of Silay. Each elector has the same right
and would suffer the same injury. Each elector stands on the same basis with reference to
maintaining a petition whether or not the relief sought by the relator should be granted. 43

In Tañada v. Tuvera,44 the Court enforced the "public right" to due process and to be informed of
matters of public concern.

In Garcia vs. Board of Investments,45 the Court upheld the "public right" to be heard or consulted on
matters of national concern.
In Oposa v. Factoran,46 the Court recognized the "public right" of citizens to "a balanced and healthful
ecology which, for the first time in our nation’s constitutional history, is solemnly incorporated in the
fundamental law."47 Mr. Justice (now Chief Justice) Hilario G. Davide, Jr., delivering the opinion of the
Court, stated that:

Such a right belongs to a different category of rights altogether for it concerns nothing less than self-
preservation and self-perpetuation-aptly and fittingly stressed by petitioners-the advancement of
which may even be said to predate all governments and constitutions. As a matter of fact, these
basic rights need not even be written in the Constitution for they are assumed to exist from
the inception of humankind.48

Petitioners, as citizens, possess the "public right" to ensure that the national patrimony is not
alienated and diminished in violation of the Constitution. Since the government, as the guardian of the
national patrimony, holds it for the benefit of all Filipinos without distinction as to ethnicity, it follows
that a citizen has sufficient interest to maintain a suit to ensure that any grant of concessions covering
the national economy and patrimony strictly complies with constitutional requirements. Thus, the
preservation of the integrity and inviolability of the national patrimony is a proper subject of a citizen’s
suit.

In addition, petitioners, as taxpayers, possess the right to restrain officials from wasting public funds
through the enforcement of an unconstitutional statute. It is well-settled that a taxpayer has the right
to enjoin public officials from wasting public funds through the implementation of an unconstitutional
statute,49 and by necessity, he may assail the validity of a statute appropriating public funds. 50 The
taxpayer has paid his taxes and contributed to the public coffers and, thus, may inquire into the
manner by which the proceeds of his taxes are spent. The expenditure by an official of the State for
the purpose of administering an invalid law constitutes a misapplication of such funds. 51

The IPRA appropriates funds as indicated in its title: "An Act to Recognize, Protect and Promote the
Rights of Indigenous Cultural Communities/Indigenous Peoples, Creating the National Commission
on Indigenous Peoples, Establishing Implementing Mechanisms, Appropriating Funds Therefor,
and for Other Purposes." In the same manner, Section 79 authorizes for the expenditure of public
funds by providing that "the amount necessary to finance [its] initial implementation shall be charged
against the current year's appropriation for the Office for Northern Cultural Communities (the
"ONCC") and the Office for Southern Cultural Communities (the "OSCC")," 52which were merged as
organic offices of the NCIP.53 Thus, the IPRA is a valid subject of a taxpayer’s suit.

C. The petition for prohibition and mandamus is not an improper remedy.

Prohibition is an extraordinary writ directed against any tribunal, corporation, board, officer or person,
whether exercising judicial, quasi-judicial or ministerial functions, ordering said entity or person to
desist from further proceedings when said proceedings are without or in excess of said entity’s or
person’s jurisdiction, or are accompanied with grave abuse of discretion, and there is no appeal or
any other plain, speedy and adequate remedy in the ordinary course of law. 54 Mandamus, on the
other hand, is an extraordinary writ commanding a tribunal, corporation, board, officer or person,
immediately or at some other specified time, to do the act required to be done, when said entity or
person unlawfully neglects the performance of an act which the law specifically enjoins as a duty
resulting from an office, trust or station, or when said entity or person unlawfully excludes another
from the use and enjoyment of a right or office to which such other is entitled, and there is no other
plain, speedy and adequate remedy in the ordinary course of law. 55

In this case, the petitioners pray that respondents be restrained from implementing the challenged
provisions of the IPRA and its Implementing Rules and the assailed DENR Circular No. 2, series of
1998, and that the same officials be enjoined from disbursing public funds for the implementation of
the said law and rules. They further ask that the Secretary of the DENR be compelled to perform his
duty to control and supervise the activities pertaining to natural resources.

Prohibition will lie to restrain the public officials concerned from implementing the questioned
provisions of the IPRA and from disbursing funds in connection therewith if the law is found to be
unconstitutional. Likewise, mandamus will lie to compel the Secretary of the DENR to perform his
duty to control and supervise the exploration, development, utilization and conservation of the
country’s natural resources. Consequently, the petition for prohibition and mandamus is not an
improper remedy for the relief sought.

D. Notwithstanding the failure of petitioners to observe the hierarchy of courts, the Court assumes
jurisdiction over the petition in view of the importance of the issues raised therein.

Between two courts of concurrent original jurisdiction, it is the lower court that should initially pass
upon the issues of a case. That way, as a particular case goes through the hierarchy of courts, it is
shorn of all but the important legal issues or those of first impression, which are the proper subject of
attention of the appellate court. This is a procedural rule borne of experience and adopted to improve
the administration of justice.

This Court has consistently enjoined litigants to respect the hierarchy of courts. Although this Court
has concurrent jurisdiction with the Regional Trial Courts and the Court of Appeals to issue writs
of certiorari, prohibition, mandamus, quo warranto, habeas corpus and injunction,56 such concurrence
does not give a party unrestricted freedom of choice of court forum. The resort to this Court’s primary
jurisdiction to issue said writs shall be allowed only where the redress desired cannot be obtained in
the appropriate courts or where exceptional and compelling circumstances justify such
invocation.57 We held in People v. Cuaresma58 that:

A becoming regard for judicial hierarchy most certainly indicates that petitions for the issuance of
extraordinary writs against first level ("inferior") courts should be filed with the Regional Trial Court,
and those against the latter, with the Court of Appeals. A direct invocation of the Supreme Court’s
original jurisdiction to issue these writs should be allowed only where there are special and
important reasons therefor, clearly and specifically set out in the petition. This is established
policy. It is a policy necessary to prevent inordinate demands upon the Court’s time and attention
which are better devoted to those matters within its exclusive jurisdiction, and to prevent further over-
crowding of the Court’s docket x x x.59 (Emphasis supplied.)

IPRA aims to rectify the historical injustice inflicted upon indigenous peoples. Its impact upon the lives
not only of the indigenous peoples but also upon the lives of all Filipinos cannot be denied. The
resolution of this case by the Court at the earliest opportunity is necessary if the aims of the law are to
be achieved. This reason is compelling enough to allow petitioners’ invocation of this Court’s
jurisdiction in the first instance.

Substantive Issues

Primary Issue

The issue of prime concern raised by petitioners and the Solicitor General revolves around the
constitutionality of certain provisions of IPRA, specifically Sections 3(a), 3(b), 5, 6, 7, 8, 57, 58 and
59. These provisions allegedly violate Section 2, Article XII of the Constitution, which states:
Sec. 2. All lands of the public domain, waters, minerals, coal, petroleum, and other mineral oils, all
forces of potential energy, fisheries, forests or timber, wildlife, flora and fauna, and other natural
resources are owned by the State. With the exception of agricultural lands, all other natural resources
shall not be alienated. The exploration, development, and utilization of natural resources shall be
under the full control and supervision of the State. The State may directly undertake such activities, or
it may enter into co-production, joint venture, or production-sharing agreements with Filipino citizens,
or corporations or associations at least sixty per centum of whose capital is owned by such citizens.
Such agreements may be for a period not exceeding twenty-five years, renewable for not more than
twenty-five years, and under such terms and conditions as may be provided by law. In cases of water
rights for irrigation, water supply, fisheries, or industrial uses other than the development of water
power, beneficial use may be the measure and limit of the grant.

The State shall protect the nation’s marine wealth in its archipelagic waters, territorial sea, and
exclusive economic zone, and reserve its use and enjoyment exclusively to Filipino citizens.

The Congress, may, by law, allow small-scale utilization of natural resources by Filipino citizens, as
well as cooperative fish farming, with priority to subsistence fishermen and fishworkers in rivers,
lakes, bays and lagoons.

The President may enter into agreements with foreign-owned corporations involving either technical
or financial assistance for large-scale exploration, development and utilization of minerals, petroleum,
and other mineral oils according to the general terms and conditions provided by law, based on real
contributions to the economic growth and general welfare of the country. In such agreements, the
State shall promote the development and use of local scientific and technical resources.

The President shall notify the Congress of every contract entered into in accordance with this
provision, within thirty days from its execution.

Under IPRA, indigenous peoples may obtain the recognition of their right of ownership 60 over
ancestral lands and ancestral domains by virtue of native title.61 The term "ancestral lands" under
the statute refers to lands occupied by individuals, families and clans who are members of
indigenous cultural communities, including residential lots, rice terraces or paddies, private forests,
swidden farms and tree lots. These lands are required to have been "occupied, possessed and
utilized" by them or through their ancestors "since time immemorial, continuously to the
present".62 On the other hand, "ancestral domains" is defined as areas generally belonging to
indigenous cultural communities, including ancestral lands, forests, pasture, residential and
agricultural lands, hunting grounds, worship areas, and lands no longer occupied exclusively by
indigenous cultural communities but to which they had traditional access, particularly the home
ranges of indigenous cultural communities who are still nomadic or shifting cultivators. Ancestral
domains also include inland waters, coastal areas and natural resources therein. 63Again, the same
are required to have been "held under a claim of ownership, occupied or possessed by ICCs/IPs, by
themselves or through their ancestors, communally or individually since time immemorial,
continuously to the present".64 Under Section 56, property rights within the ancestral domains already
existing and/or vested upon effectivity of said law "shall be recognized and respected."

Ownership is the crux of the issue of whether the provisions of IPRA pertaining to ancestral lands,
ancestral domains, and natural resources are unconstitutional. The fundamental question is, who,
between the State and the indigenous peoples, are the rightful owners of these properties?

It bears stressing that a statute should be construed in harmony with, and not in violation, of the
fundamental law.65 The reason is that the legislature, in enacting a statute, is assumed to have acted
within its authority and adhered to the constitutional limitations. Accordingly, courts should presume
that it was the intention of the legislature to enact a valid, sensible, and just law and one which
operates no further than may be necessary to effectuate the specific purpose of the law. 66

A. The provisions of IPRA recognizing the ownership of indigenous peoples over the ancestral lands
and ancestral domains are not unconstitutional.

In support of their theory that ancestral lands and ancestral domains are part of the public domain
and, thus, owned by the State, pursuant to Section 2, Article XII of the Constitution, petitioners and
the Solicitor General advance the following arguments:

First, according to petitioners, the King of Spain under international law acquired exclusive dominion
over the Philippines by virtue of discovery and conquest. They contend that the Spanish King under
the theory of jura regalia, which was introduced into Philippine law upon Spanish conquest in 1521,
acquired title to all the lands in the archipelago.

Second, petitioners and the Solicitor General submit that ancestral lands and ancestral domains are
owned by the State. They invoke the theory of jura regalia which imputes to the State the ownership
of all lands and makes the State the original source of all private titles. They argue that the Philippine
State, as successor to Spain and the United States, is the source of any asserted right of ownership
in land.

Third, petitioners and the Solicitor General concede that the Cariño doctrine exists. However,
petitioners maintain that the doctrine merely states that title to lands of the public domain may be
acquired by prescription. The Solicitor General, for his part, argues that the doctrine applies only to
alienable lands of the public domain and, thus, cannot be extended to other lands of the public
domain such as forest or timber, mineral lands, and national parks.

Fourth, the Solicitor General asserts that even assuming that native title over ancestral lands and
ancestral domains existed by virtue of the Cariño doctrine, such native title was extinguished upon
the ratification of the 1935 Constitution.

Fifth, petitioners admit that Congress is mandated under Section 5, Article XII of the Constitution to
protect that rights of indigenous peoples to their ancestral lands and ancestral domains. However,
they contend that the mandate is subject to Section 2, Article XII and the theory of jura
regalia embodied therein. According to petitioners, the recognition and protection under R.A. 8371 of
the right of ownership over ancestral lands and ancestral domains is far in excess of the legislative
power and constitutional mandate of Congress.

Finally, on the premise that ancestral lands and ancestral domains are owned by the State,
petitioners posit that R.A. 8371 violates Section 2, Article XII of the Constitution which prohibits the
alienation of non-agricultural lands of the public domain and other natural resources.

I am not persuaded by these contentions.

Undue reliance by petitioners and the Solicitor General on the theory of jura regalia is
understandable. Not only is the theory well recognized in our legal system; it has been regarded,
almost with reverence, as the immutable postulate of Philippine land law. It has been incorporated
into our fundamental law and has been recognized by the Court. 67

Generally, under the concept of jura regalia, private title to land must be traced to some grant,
express or implied, from the Spanish Crown or its successors, the American Colonial government,
and thereafter, the Philippine Republic. The belief that the Spanish Crown is the origin of all land titles
in the Philippines has persisted because title to land must emanate from some source for it cannot
issue forth from nowhere.68

In its broad sense, the term "jura regalia" refers to royal rights,69 or those rights which the King has by
virtue of his prerogatives.70 In Spanish law, it refers to a right which the sovereign has over anything
in which a subject has a right of property or propriedad.71 These were rights enjoyed during feudal
times by the king as the sovereign.

The theory of the feudal system was that title to all lands was originally held by the King, and while
the use of lands was granted out to others who were permitted to hold them under certain conditions,
the King theoretically retained the title.72 By fiction of law, the King was regarded as the original
proprietor of all lands, and the true and only source of title, and from him all lands were held.73 The
theory of jura regalia was therefore nothing more than a natural fruit of conquest.74

The Regalian theory, however, does not negate native title to lands held in private ownership since
time immemorial. In the landmark case of Cariño vs. Insular Government75 the United States
Supreme Court, reversing the decision76of the pre-war Philippine Supreme Court, made the following
pronouncement:

x x x Every presumption is and ought to be taken against the Government in a case like the present.
It might, perhaps, be proper and sufficient to say that when, as far back as testimony or memory
goes, the land has been held by individuals under a claim of private ownership, it will be
presumed to have been held in the same way from before the Spanish conquest, and never to
have been public land. x x x.77 (Emphasis supplied.)

The above ruling institutionalized the recognition of the existence of native title to land, or ownership
of land by Filipinos by virtue of possession under a claim of ownership since time immemorial and
independent of any grant from the Spanish Crown, as an exception to the theory of jura regalia.

In Cariño, an Igorot by the name of Mateo Cariño applied for registration in his name of an ancestral
land located in Benguet. The applicant established that he and his ancestors had lived on the land,
had cultivated it, and had used it as far they could remember. He also proved that they had all been
recognized as owners, the land having been passed on by inheritance according to native custom.
However, neither he nor his ancestors had any document of title from the Spanish Crown. The
government opposed the application for registration, invoking the theory of jura regalia. On appeal,
the United States Supreme Court held that the applicant was entitled to the registration of his native
title to their ancestral land.

Cariño was decided by the U.S. Supreme Court in 1909, at a time when decisions of the U.S. Court
were binding as precedent in our jurisdiction.78 We applied the Cariño doctrine in the 1946 case of Oh
Cho vs. Director of Lands,79 where we stated that "[a]ll lands that were not acquired from the
Government either by purchase or by grant, belong to the public domain, but [a]n exception to the
rule would be any land that should have been in the possession of an occupant and of his
predecessors in interest since time immemorial, for such possession would justify the presumption
that the land had never been part of the public domain or that it had been private property even
before the Spanish conquest."80

Petitioners however aver that the U.S. Supreme Court’s ruling in Cariño was premised on the fact that
the applicant had complied with the requisites of acquisitive prescription, having established that he
and his predecessors-in-interest had been in possession of the property since time immemorial. In
effect, petitioners suggest that title to the ancestral land applied for by Cariño was transferred from
the State, as original owner, to Cariño by virtue of prescription. They conclude that the doctrine
cannot be the basis for decreeing "by mere legislative fiat…that ownership of vast tracts of land
belongs to [indigenous peoples] without judicial confirmation." 81

The Solicitor General, for his part, claims that the Cariño doctrine applies only to alienable lands of
the public domain and, as such, cannot be extended to other lands of the public domain such as
forest or timber, mineral lands, and national parks.

There is no merit in these contentions.

A proper reading of Cariño would show that the doctrine enunciated therein applies only to lands
which have always been considered as private, and not to lands of the public domain, whether
alienable or otherwise. A distinction must be made between ownership of land under native title and
ownership by acquisitive prescription against the State. Ownership by virtue of native title
presupposes that the land has been held by its possessor and his predecessors-in-interest in the
concept of an owner since time immemorial. The land is not acquired from the State, that is, Spain or
its successors-in-interest, the United States and the Philippine Government. There has been no
transfer of title from the State as the land has been regarded as private in character as far back as
memory goes. In contrast, ownership of land by acquisitive prescription against the State involves a
conversion of the character of the property from alienable public land to private land, which
presupposes a transfer of title from the State to a private person. Since native title assumes that the
property covered by it is private land and is deemed never to have been part of the public domain, the
Solicitor General’s thesis that native title under Cariño applies only to lands of the public domain is
erroneous. Consequently, the classification of lands of the public domain into agricultural, forest or
timber, mineral lands, and national parks under the Constitution82 is irrelevant to the application of
the Cariño doctrine because the Regalian doctrine which vests in the State ownership of lands of the
public domain does not cover ancestral lands and ancestral domains.

Legal history supports the Cariño doctrine.

When Spain acquired sovereignty over the Philippines by virtue of its discovery and occupation
thereof in the 16th century and the Treaty of Tordesillas of 1494 which it entered into with
Portugal,83 the continents of Asia, the Americas and Africa were considered as terra nullius although
already populated by other peoples.84 The discovery and occupation by the European States, who
were then considered as the only members of the international community of civilized nations, of
lands in the said continents were deemed sufficient to create title under international law. 85

Although Spain was deemed to have acquired sovereignty over the Philippines, this did not mean that
it acquired title to all lands in the archipelago. By virtue of the colonial laws of Spain, the Spanish
Crown was considered to have acquired dominion only over the unoccupied and unclaimed portions
of our islands.86

In sending the first expedition to the Philippines, Spain did not intend to deprive the natives of their
property. Miguel Lopez de Legazpi was under instruction of the Spanish King to do no harm to the
natives and to their property. In this regard, an authority on the early Spanish colonial period in the
Philippines wrote:

The government of [the King of Spain] Philip II regarded the Philippines as a challenging opportunity
to avoid a repetition of the sanguinary conquests of Mexico and Peru. In his written instructions for
the Adelantado Legazpi, who commanded the expedition, Philip II envisaged a bloodless pacification
of the archipelago. This extraordinary document could have been lifted almost verbatim from the
lectures of the Dominican theologian, Francisco de Vitoria, delivered in the University of Salamanca.
The King instructed Legazpi to inform the natives that the Spaniards had come to do no harm to their
persons or to their property. The Spaniards intended to live among them in peace and in friendship
and "to explain to them the law of Jesus Christ by which they will be saved." Although the Spanish
expedition could defend themselves if attacked, the royal instructions admonished the commander to
commit no aggressive act which might arouse native hostility.87

Spanish colonial laws recognized and respected Filipino landholdings including native land
occupancy.88 Thus, the Recopilación de Leyes de las Indias expressly conferred ownership of lands
already held by the natives.89The royal decrees of 1880 and 1894 did not extinguish native title to
land in the Philippines. The earlier royal decree, dated June 25, 1880, provided that all those in
"unlawful possession of royal lands" must legalize their possession by means of adjustment
proceedings,90 and within the period specified. The later royal decree, dated February 13, 1894,
otherwise known as the Maura Law, declared that titles that were capable of adjustment under the
royal decree of 1880, but for which adjustment was not sought, were forfeited. Despite the harsh
wording of the Maura Law, it was held in the case of Cariño that the royal decree of 1894 should not
be construed as confiscation of title, but merely as the withdrawal of the privilege of registering such
title.91

Neither was native title disturbed by the Spanish cession of the Philippines to the United States,
contrary to petitioners’ assertion that the US merely succeeded to the rights of Spain, including the
latter’s rights over lands of the public domain.92 Under the Treaty of Paris of December 10, 1898, the
cession of the Philippines did not impair any right to property existing at the time.93 During the
American colonial regime, native title to land was respected, even protected. The Philippine Bill of
1902 provided that property and rights acquired by the US through cession from Spain were to be
administered for the benefit of the Filipinos.94 In obvious adherence to libertarian principles,
McKinley’s Instructions, as well as the Philippine Bill of 1902, contained a bill of rights embodying the
safeguards of the US Constitution. One of these rights, which served as an inviolable rule upon every
division and branch of the American colonial government in the Philippines,95 was that "no person
shall be deprived of life, liberty, or property without due process of law." 96 These vested rights
safeguarded by the Philippine Bill of 1902 were in turn expressly protected by the due process clause
of the 1935 Constitution. Resultantly, property rights of the indigenous peoples over their ancestral
lands and ancestral domains were firmly established in law.

Nonetheless, the Solicitor General takes the view that the vested rights of indigenous peoples to their
ancestral lands and domains were "abated by the direct act by the sovereign Filipino people of
ratifying the 1935 Constitution."97 He advances the following arguments:

The Sovereign, which is the source of all rights including ownership, has the power to restructure the
consolidation of rights inherent in ownership in the State. Through the mandate of the Constitutions
that have been adopted, the State has wrested control of those portions of the natural resources it
deems absolutely necessary for social welfare and existence. It has been held that the State may
impair vested rights through a legitimate exercise of police power.

Vested rights do not prohibit the Sovereign from performing acts not only essential to but
determinative of social welfare and existence. To allow otherwise is to invite havoc in the established
social system. x x x

Time-immemorial possession does not create private ownership in cases of natural resources that
have been found from generation to generation to be critical to the survival of the Sovereign and its
agent, the State.98

Stated simply, the Solicitor General’s argument is that the State, as the source of all titles to land, had
the power to re-vest in itself, through the 1935 Constitution, title to all lands, including ancestral lands
and ancestral domains. While the Solicitor General admits that such a theory would necessarily
impair vested rights, he reasons out that even vested rights of ownership over ancestral lands and
ancestral domains are not absolute and may be impaired by the legitimate exercise of police power.

I cannot agree. The text of the provision of the 1935 Constitution invoked by the Solicitor General,
while embodying the theory of jura regalia, is too clear for any misunderstanding. It simply declares
that "all agricultural, timber, and mineral lands of the public domain, waters, minerals, coal, petroleum,
and other mineral oils, all forces of potential energy, and other natural resources of the Philippines
belong to the State."99 Nowhere does it state that certain lands which are "absolutely necessary for
social welfare and existence," including those which are not part of the public domain, shall thereafter
be owned by the State. If there is any room for constitutional construction, the provision should be
interpreted in favor of the preservation, rather than impairment or extinguishment, of vested rights.
Stated otherwise, Section 1, Article XII of the 1935 Constitution cannot be construed to mean that
vested right which had existed then were extinguished and that the landowners were divested of their
lands, all in the guise of "wrest[ing] control of those portions of the natural resources [which the State]
deems absolutely necessary for social welfare and existence." On the contrary, said Section restated
the fundamental rule against the diminution of existing rights by expressly providing that the
ownership of lands of the public domain and other natural resources by the State is "subject to any
existing right, grant, lease, or concessions." The "existing rights" that were intended to be protected
must, perforce, include the right of ownership by indigenous peoples over their ancestral lands and
domains. The words of the law should be given their ordinary or usual meaning, 100 and the term
"existing rights" cannot be assigned an unduly restrictive definition.

Petitioners concede that Congress is mandated under Section 5, Article XII of the 1987
Constitution101to protect the rights of indigenous peoples to their ancestral lands and ancestral
domains. Nonetheless, they contend that the recognition and protection under IPRA of the right of
ownership of indigenous peoples over ancestral lands and ancestral domains are far in excess of the
legislative power and constitutional mandate of the Congress,102since such recognition and protection
amount to the alienation of lands of the public domain, which is proscribed under Section 2, Article XII
of the Constitution.

Section 5, Article XII of the Constitution expresses the sovereign intent to "protect the rights of
indigenous peoples to their ancestral lands." In its general and ordinary sense, the term "right" refers
to any legally enforceable claim.103 It is a power, privilege, faculty or demand inherent in one person
and incident upon another.104 When used in relation to property, "right" includes any interest in or title
to an object, or any just and legal claim to hold, use and enjoy it. 105 Said provision in the Constitution
cannot, by any reasonable construction, be interpreted to exclude the protection of the right of
ownership over such ancestral lands. For this reason, Congress cannot be said to have exceeded its
constitutional mandate and power in enacting the provisions of IPRA, specifically Sections 7(a) and 8,
which recognize the right of ownership of the indigenous peoples over ancestral lands.

The second paragraph of Section 5, Article XII also grants Congress the power to "provide for the
applicability of customary laws governing property rights or relations in determining the ownership
and extent of ancestral domains." In light of this provision, does Congress have the power to decide
whether ancestral domains shall be private property or part of the public domain? Also, does
Congress have the power to determine whether the "extent" of ancestral domains shall include the
natural resources found therein?

It is readily apparent from the constitutional records that the framers of the Constitution did not intend
Congress to decide whether ancestral domains shall be public or private property. Rather, they
acknowledged that ancestral domains shall be treated as private property, and that customary laws
shall merely determine whether such private ownership is by the entire indigenous cultural
community, or by individuals, families, or clans within the community. The discussion below between
Messrs. Regalado and Bennagen and Mr. Chief Justice Davide, then members of the 1986
Constitutional Commission, is instructive:

MR. REGALADO. Thank you, Madame President. May I seek some clarifications from either
Commissioner Bennagen or Commissioner Davide regarding this phrase "CONGRESS SHALL
PROVIDE FOR THE APPLICABILITY OF CUSTOMARY LAWS GOVERNING PROPERTY RIGHTS
OR RELATIONS in determining the ownership and extent of the ancestral domain," because
ordinarily it is the law on ownership and the extent thereof which determine the property rights or
relations arising therefrom. On the other hand, in this proposed amendment the phraseology is that it
is the property rights or relations which shall be used as the basis in determining the ownership and
extent of the ancestral domain. I assume there must be a certain difference in the customary laws
and our regular civil laws on property.

MR. DAVIDE. That is exactly the reason, Madam President, why we will leave it to Congress to make
the necessary exception to the general law on property relations.

MR. REGALADO. I was thinking if Commissioner Bennagen could give us an example of such a
customary law wherein it is the property rights and relations that determine the ownership and the
extent of that ownership, unlike the basic fundamental rule that it is the ownership and the extent of
ownership which determine the property rights and relations arising therefrom and consequent
thereto. Perhaps, these customary laws may have a different provision or thrust so that we could
make the corresponding suggestions also by way of an amendment.

MR. DAVIDE. That is exactly my own perception.

MR. BENNAGEN. Let me put it this way.

There is a range of customary laws governing certain types of ownership. There would be
ownership based on individuals, on clan or lineage, or on community. And the thinking
expressed in the consultation is that this should be codified and should be recognized in relation to
existing national laws. That is essentially the concept. 106 (Emphasis supplied.)

The intention to treat ancestral domains as private property is also apparent from the following
exchange between Messrs. Suarez and Bennagen:

MR. SUAREZ. When we speak of customary laws governing property rights or relations in
determining the ownership and extent of the ancestral domain, are we thinking in terms of the tribal
ownership or community ownership or of private ownership within the ancestral lands or ancestral
domain?

MR. BENNAGEN. The concept of customary laws is that it is considered as ownership by


private individuals, clans and even communities.

MR. SUAREZ. So, there will be two aspects to this situation. This means that the State will set aside
the ancestral domain and there is a separate law for that. Within the ancestral domain it could accept
more specific ownership in terms of individuals within the ancestral lands.

MR. BENNAGEN. Individuals and groups within the ancestral domain. 107 (Emphasis supplied.)

It cannot be correctly argued that, because the framers of the Constitution never expressly
mentioned Cariño in their deliberations, they did not intend to adopt the concept of native title to land,
or that they were unaware of native title as an exception to the theory of jura regalia.108 The framers
of the Constitution, as well as the people adopting it, were presumed to be aware of the prevailing
judicial doctrines concerning the subject of constitutional provisions, and courts should take these
doctrines into consideration in construing the Constitution.109

Having thus recognized that ancestral domains under the Constitution are considered as private
property of indigenous peoples, the IPRA, by affirming or acknowledging such ownership through its
various provisions, merely abides by the constitutional mandate and does not suffer any vice of
unconstitutionality.

Petitioners interpret the phrase "subject to the provisions of this Constitution and national
development policies and programs" in Section 5, Article XII of the Constitution to mean "as subject to
the provision of Section 2, Article XII of the Constitution," which vests in the State ownership of all
lands of the public domain, mineral lands and other natural resources. Following this interpretation,
petitioners maintain that ancestral lands and ancestral domains are the property of the State.

This proposition is untenable. Indeed, Section 2, Article XII reiterates the declarations made in the
1935 and 1973 Constitutions on the state policy of conservation and nationalization of lands of the
public domain and natural resources, and is of paramount importance to our national economy and
patrimony. A close perusal of the records of the 1986 Constitutional Commission reveals that the
framers of the Constitution inserted the phrase "subject to the provisions of this Constitution" mainly
to prevent the impairment of Torrens titles and other prior rights in the determination of what
constitutes ancestral lands and ancestral domains, to wit:

MR. NATIVIDAD. Just one question. I want to clear this section protecting ancestral lands. How does
this affect the Torrens title and other prior rights?

MR. BENNAGEN. I think that was also discussed in the committee hearings and we did say that in
cases where due process is clearly established in terms of prior rights, these two have to be
respected.

MR. NATIVIDAD. The other point is: How vast is this ancestral land? Is it true that parts of Baguio
City are considered as ancestral lands?

MR. BENNAGEN. They could be regarded as such. If the Commissioner still recalls, in one of the
publications that I provided the Commissioners, the parts could be considered as ancestral domain in
relation to the whole population of Cordillera but not in relation to certain individuals or certain groups.

MR. NATIVIDAD. The Commissioner means that the whole Baguio City is considered as ancestral
land?

MR. BENNAGEN. Yes, in the sense that it belongs to Cordillera or in the same manner that Filipinos
can speak of the Philippine archipelago as ancestral land, but not in terms of the right of a particular
person or particular group to exploit, utilize, or sell it.

MR. NATIVIDAD. But is clear that the prior rights will be respected.

MR. BENNAGEN. Definitely. 110

Thus, the phrase "subject to the provisions of this Constitution" was intended by the framers of the
Constitution as a reiteration of the constitutional guarantee that no person shall be deprived of
property without due process of law.
There is another reason why Section 5 of Article XII mandating the protection of rights of the
indigenous peoples to their ancestral lands cannot be construed as subject to Section 2 of the same
Article ascribing ownership of all public lands to the State. The Constitution must be construed as a
whole. It is a rule that when construction is proper, the whole Constitution is examined in order to
determine the meaning of any provision. That construction should be used which would give effect to
the entire instrument.111

Thus, the provisions of the Constitution on State ownership of public lands, mineral lands and other
natural resources should be read together with the other provisions thereof which firmly recognize the
rights of the indigenous peoples. These, as set forth hereinbefore, 112 include: Section 22, Article II,
providing that the State recognizes and promotes the rights of indigenous peoples within the
framework of national unity and development; Section 5, Article XII, calling for the protection of the
rights of indigenous cultural communities to their ancestral lands to ensure their economic, social, and
cultural well-being, and for the applicability of customary laws governing property rights and relations
in determining the ownership and extent of ancestral domains; Section 1, Article XIII, directing the
removal or reduction of social, economic, political and cultural inequities and inequalities by equitably
diffusing wealth and political power for the common good; Section 6, Article XIII, directing the
application of the principles of agrarian reform or stewardship in the disposition and utilization of other
natural resources, subject to prior rights, homestead rights of small settlers, and the rights of
indigenous communities to their ancestral lands; Section 17, Article XIV, decreeing that the State
shall recognize, respect, and protect the rights of indigenous cultural communities to preserve and
develop their cultures, traditions, and institutions; and Section 12, Article XVI, authorizing the
Congress to create a consultative body to advise the President on policies affecting indigenous
cultural communities.

Again, as articulated in the Constitution, the first goal of the national economy is the more equitable
distribution of opportunities, income, and wealth.113 Equity is given prominence as the first
objective of national economic development.114 The framers of the Constitution did not, by the phrase
"subject to the provisions of this Constitution and national development policies and programs,"
intend to establish a hierarchy of constitutional norms. As explained by then Commissioner (now
Chief Justice) Hilario G. Davide, Jr., it was not their objective to make certain interests primary or
paramount, or to create absolute limitations or outright prohibitions; rather, the idea is towards the
balancing of interests:

BISHOP BACANI. In Commissioner Davide’s formulation of the first sentence, he says: "The State,
SUBJECT TO THE provisions of this Constitution AND NATIONAL DEVELOPMENT POLICIES AND
PROGRAMS shall guarantee the rights of cultural or tribal communities to their ancestral lands to
insure their economic, social and cultural well-being." There are at least two concepts here which
receive different weights very often. They are the concepts of national development policies and
programs, and the rights of cultural or tribal communities to their ancestral lands, et cetera. I would
like to ask: When the Commissioner proposed this amendment, which was the controlling concept? I
ask this because sometimes the rights of cultural minorities are precisely transgressed in the interest
of national development policies and programs. Hence, I would like to know which is the controlling
concept here. Is it the rights of indigenous peoples to their ancestral lands or is it national
development policies and programs.

MR. DAVIDE. It is not really a question of which is primary or which is more paramount. The
concept introduced here is really the balancing of interests. That is what we seek to attain. We
have to balance the interests taking into account the specific needs and the specific interests also of
these cultural communities in like manner that we did so in the autonomous regions.115 (Emphasis
supplied.)
B. The provisions of R.A. 8371 do not infringe upon the State’s ownership over the natural resources
within the ancestral domains.

Petitioners posit that IPRA deprives the State of its ownership over mineral lands of the public domain
and other natural resources,116 as well as the State’s full control and supervision over the exploration,
development and utilization of natural resources.117 Specifically, petitioners and the Solicitor General
assail Sections 3 (a),118 5,119and 7120 of IPRA as violative of Section 2, Article XII of the Constitution
which states, in part, that "[a]ll lands of the public domain, waters, minerals, coal, petroleum, and
other mineral oils, all forces of potential energy, fisheries, forests or timber, wildlife, flora and fauna,
and other natural resources are owned by the State."121 They would have the Court declare as
unconstitutional Section 3(a) of IPRA because the inclusion of natural resources in the definition of
ancestral domains purportedly results in the abdication of State ownership over these resources.

I am not convinced.

Section 3(a) merely defines the coverage of ancestral domains, and describes the extent, limit and
composition of ancestral domains by setting forth the standards and guidelines in determining
whether a particular area is to be considered as part of and within the ancestral domains. In other
words, Section 3(a) serves only as a yardstick which points out what properties are within the
ancestral domains. It does not confer or recognize any right of ownership over the natural resources
to the indigenous peoples. Its purpose is definitional and not declarative of a right or title.

The specification of what areas belong to the ancestral domains is, to our mind, important to ensure
that no unnecessary encroachment on private properties outside the ancestral domains will result
during the delineation process. The mere fact that Section 3(a) defines ancestral domains to include
the natural resources found therein does not ipso facto convert the character of such natural
resources as private property of the indigenous peoples. Similarly, Section 5 in relation to Section
3(a) cannot be construed as a source of ownership rights of indigenous people over the natural
resources simply because it recognizes ancestral domains as their "private but community property."

The phrase "private but community property" is merely descriptive of the indigenous peoples’ concept
of ownership as distinguished from that provided in the Civil Code. In Civil Law, "ownership" is the
"independent and general power of a person over a thing for purposes recognized by law and within
the limits established thereby."122 The civil law concept of ownership has the following attributes: jus
utendi or the right to receive from the thing that which it produces, jus abutendi or the right to
consume the thing by its use, jus disponendi or the power to alienate, encumber, transform and even
destroy that which is owned and jus vidicandi or the right to exclude other persons from the
possession the thing owned.123 In contrast, the indigenous peoples’ concept of ownership
emphasizes the importance of communal or group ownership. By virtue of the communal character of
ownership, the property held in common "cannot be sold, disposed or destroyed" 124 because it was
meant to benefit the whole indigenous community and not merely the individual member. 125

That IPRA is not intended to bestow ownership over natural resources to the indigenous peoples is
also clear from the deliberations of the bicameral conference committee on Section 7 which recites
the rights of indigenous peoples over their ancestral domains, to wit:

CHAIRMAN FLAVIER. Accepted. Section 8126 rights to ancestral domain, this is where we transferred
the other provision but here itself -

HON. DOMINGUEZ. Mr. Chairman, if I maybe allowed to make a very short Statement. Earlier, Mr.
Chairman, we have decided to remove the provisions on natural resources because we all
agree that that belongs to the State. Now, the plight or the rights of those indigenous communities
living in forest and areas where it could be exploited by mining, by dams, so can we not also provide
a provision to give little protection or either rights for them to be consulted before any mining areas
should be done in their areas, any logging done in their areas or any dam construction because this
has been disturbing our people especially in the Cordilleras. So, if there could be, if our lawyers or the
secretariat could just propose a provision for incorporation here so that maybe the right to
consultation and the right to be compensated when there are damages within their ancestral lands.

CHAIRMAN FLAVIER. Yes, very well taken but to the best of my recollection both are already
considered in subsequent sections which we are now looking for.

HON. DOMINGUEZ. Thank you.

CHAIRMAN FLAVIER. First of all there is a line that gives priority use for the indigenous people
where they are. Number two, in terms of the mines there is a need for prior consultation of source
which is here already. So, anyway it is on the record that you want to make sure that the secretariat
takes note of those two issues and my assurance is that it is already there and I will make sure that
they cross check.

HON. ADAMAT. I second that, Mr. Chairman.

CHAIRMAN FLAVIER. Okay, thank you. So we now move to Section 8, there is a Senate version you
do not have and if you agree we will adopt that.127 (Emphasis supplied.)

Further, Section 7 makes no mention of any right of ownership of the indigenous peoples over the
natural resources. In fact, Section 7(a) merely recognizes the "right to claim ownership over lands,
bodies of water traditionally and actually occupied by indigenous peoples, sacred places, traditional
hunting and fishing grounds, and all improvements made by them at any time within the domains."
Neither does Section 7(b), which enumerates certain rights of the indigenous peoples over the natural
resources found within their ancestral domains, contain any recognition of ownership vis-a-vis the
natural resources.

What is evident is that the IPRA protects the indigenous peoples’ rights and welfare in relation to the
natural resources found within their ancestral domains,128 including the preservation of the ecological
balance therein and the need to ensure that the indigenous peoples will not be unduly displaced
when State-approved activities involving the natural resources located therein are undertaken.

Finally, the concept of native title to natural resources, unlike native title to land, has not been
recognized in the Philippines. NCIP and Flavier, et al. invoke the case of Reavies v. Fianza129 in
support of their thesis that native title to natural resources has been upheld in this jurisdiction. 130 They
insist that "it is possible for rights over natural resources to vest on a private (as opposed to a public)
holder if these were held prior to the 1935 Constitution." 131However, a judicious examination
of Reavies reveals that, contrary to the position of NCIP and Flavier, et al., the Court did not
recognize native title to natural resources. Rather, it merely upheld the right of the indigenous peoples
to claim ownership of minerals under the Philippine Bill of 1902.

While as previously discussed, native title to land or private ownership by Filipinos of land by virtue of
time immemorial possession in the concept of an owner was acknowledged and recognized as far
back during the Spanish colonization of the Philippines, there was no similar favorable treatment as
regards natural resources. The unique value of natural resources has been acknowledged by the
State and is the underlying reason for its consistent assertion of ownership and control over said
natural resources from the Spanish regime up to the present. 132 Natural resources, especially
minerals, were considered by Spain as an abundant source of revenue to finance its battles in wars
against other nations. Hence, Spain, by asserting its ownership over minerals wherever these may be
found, whether in public or private lands, recognized the separability of title over lands and that over
minerals which may be found therein. 133

On the other hand, the United States viewed natural resources as a source of wealth for its nationals.
As the owner of natural resources over the Philippines after the latter’s cession from Spain, the
United States saw it fit to allow both Filipino and American citizens to explore and exploit minerals in
public lands, and to grant patents to private mineral lands. A person who acquired ownership over a
parcel of private mineral land pursuant to the laws then prevailing could exclude other persons, even
the State, from exploiting minerals within his property.134 Although the United States made a
distinction between minerals found in public lands and those found in private lands, title in these
minerals was in all cases sourced from the State. The framers of the 1935 Constitution found it
necessary to maintain the State’s ownership over natural resources to insure their conservation for
future generations of Filipinos, to prevent foreign control of the country through economic domination;
and to avoid situations whereby the Philippines would become a source of international conflicts,
thereby posing danger to its internal security and independence. 135

The declaration of State ownership and control over minerals and other natural resources in the 1935
Constitution was reiterated in both the 1973136 and 1987 Constitutions.137

Having ruled that the natural resources which may be found within the ancestral domains belong to
the State, the Court deems it necessary to clarify that the jurisdiction of the NCIP with respect to
ancestral domains under Section 52 [i] of IPRA extends only to the lands and not to
the natural resources therein.

Section 52[i] provides:

Turnover of Areas Within Ancestral Domains Managed by Other Government Agencies. - The
Chairperson of the NCIP shall certify that the area covered is an ancestral domain. The secretaries of
the Department of Agrarian Reform, Department of Environment and Natural Resources, Department
of Interior and Local Government, and Department of Justice, the Commissioner of the National
Development Corporation, and any other government agency claiming jurisdiction over the area shall
be notified thereof. Such notification shall terminate any legal basis for the jurisdiction previously
claimed.

Undoubtedly, certain areas that are claimed as ancestral domains may still be under the
administration of other agencies of the Government, such as the Department of Agrarian Reform, with
respect to agricultural lands, and the Department of Environment and Natural Resources with respect
to timber, forest and mineral lands. Upon the certification of these areas as ancestral domain
following the procedure outlined in Sections 51 to 53 of the IPRA, jurisdiction of the government
agency or agencies concerned over lands forming part thereof ceases. Nevertheless, the jurisdiction
of government agencies over the natural resources within the ancestral domains does not terminate
by such certification because said agencies are mandated under existing laws to administer the
natural resources for the State, which is the owner thereof. To construe Section 52[i] as divesting the
State, through the government agencies concerned, of jurisdiction over the natural resources within
the ancestral domains would be inconsistent with the established doctrine that all natural resources
are owned by the State.

C. The provisions of IPRA pertaining to the utilization of natural resources are not unconstitutional.

The IPRA provides that indigenous peoples shall have the right to manage and conserve the natural
resources found on the ancestral domains, to benefit from and share in the profits from the allocation
and utilization of these resources, and to negotiate the terms and conditions for the exploration of
such natural resources.138 The statute also grants them priority rights in the harvesting, extraction,
development or exploitation of any natural resources within the ancestral domains. 139 Before the
NCIP can issue a certification for the renewal, or grant of any concession, license or lease, or for the
perfection of any production-sharing agreement the prior informed written consent of the indigenous
peoples concerned must be obtained.140 In return, the indigenous peoples are given the responsibility
to maintain, develop, protect and conserve the ancestral domains or portions thereof which are found
to be necessary for critical watersheds, mangroves, wildlife sanctuaries, wilderness, protected areas,
forest cover, or reforestation.141

The Solicitor General argues that these provisions deny the State an active and dominant role in the
utilization of our country’s natural resources. Petitioners, on the other hand, allege that under the
Constitution the exploration, development and utilization of natural resources may only be undertaken
by the State, either directly or indirectly through co-production, joint venture, or production-sharing
agreements.142 To petitioners, no other method is allowed by the Constitution. They likewise submit
that by vesting ownership of ancestral lands and ancestral domains in the indigenous peoples, IPRA
necessarily gives them control over the use and enjoyment of such natural resources, to the prejudice
of the State.143

Section 2, Article XII of the Constitution provides in paragraph 1 thereof that the exploration,
development and utilization of natural resources must be under the full control and supervision of the
State, which may directly undertake such activities or enter into co-production, joint venture, or
production-sharing agreements. This provision, however, should not be read in isolation to avoid a
mistaken interpretation that any and all forms of utilization of natural resources other than the
foregoing are prohibited. The Constitution must be regarded as consistent with itself
throughout.144 No constitutional provision is to be separated from all the others, or to be considered
alone, all provisions bearing upon a particular subject are to be brought into view and to be so
interpreted as to effectuate the great purposes of the fundamental law. 145

In addition to the means of exploration, development and utilization of the country’s natural resources
stated in paragraph 1, Section 2 of Article XII, the Constitution itself states in the third paragraph of
the same section that Congress may, by law, allow small-scale utilization of natural resources by
its citizens.146 Further, Section 6, Article XIII, directs the State, in the disposition and utilization of
natural resources, to apply the principles of agrarian reform or stewardship. 147 Similarly, Section 7,
Article XIII mandates the State to protect the rights of subsistence fishermen to the preferential
use of marine and fishing resources.148 Clearly, Section 2, Article XII, when interpreted in view of the
pro-Filipino, pro-poor philosophy of our fundamental law, and in harmony with the other provisions of
the Constitution rather as a sequestered pronouncement, 149 cannot be construed as a prohibition
against any and all forms of utilization of natural resources without the State’s direct participation.

Through the imposition of certain requirements and conditions for the exploration, development and
utilization of the natural resources under existing laws,150 the State retains full control over such
activities, whether done on small-scale basis151 or otherwise.

The rights given to the indigenous peoples regarding the exploitation of natural resources under
Sections 7(b) and 57 of IPRA amplify what has been granted to them under existing laws, such as the
Small-Scale Mining Act of 1991 (R.A. 7076) and the Philippine Mining Act of 1995 (R.A. 7942). R.A.
7076 expressly provides that should an ancestral land be declared as a people’s small-scale mining
area, the members of the indigenous peoples living within said area shall be given priority in the
awarding of small-scale mining contracts.152 R.A. 7942 declares that no ancestral land shall be
opened for mining operations without the prior consent of the indigenous cultural community
concerned153 and in the event that the members of such indigenous cultural community give their
consent to mining operations within their ancestral land, royalties shall be paid to them by the
parties to the mining to the contract.154

In any case, a careful reading of Section 7(b) would reveal that the rights given to the indigenous
peoples are duly circumscribed. These rights are limited only to the following: "to manage and
conserve natural resources within territories and uphold it for future generations; to benefit and
share the profits from allocation and utilization of the natural resources found therein; to negotiate
the terms and conditions for the exploration of natural resources in the areas for the purpose of
ensuring ecological, environmental protection and the conservation measures, pursuant to national
and customary laws; to an informed and intelligent participation in the formulation and
implementation of any project, government or private, that will affect or impact upon the ancestral
domains and to receive just and fair compensation for any damages which they may sustain as a
result of the project, and the right to effective measures by the government to prevent any
interference with, alienation and encroachment of these rights."

It must be noted that the right to negotiate terms and conditions granted under Section 7(b) pertains
only to the exploration of natural resources. The term "exploration" refers only to the search or
prospecting of mineral resources, or any other means for the purpose of determining the existence
and the feasibility of mining them for profit.155 The exploration, which is merely a preliminary activity,
cannot be equated with the entire process of "exploration, development and utilization" of natural
resources which under the Constitution belong to the State.

Section 57, on the other hand, grants the indigenous peoples "priority rights" in the utilization of
natural resources and not absolute ownership thereof. Priority rights does not mean exclusive rights.
What is granted is merely the right of preference or first consideration in the award of privileges
provided by existing laws and regulations, with due regard to the needs and welfare of indigenous
peoples living in the area.

There is nothing in the assailed law which implies an automatic or mechanical character in the grant
of concessions. Nor does the law negate the exercise of sound discretion by government entities.
Several factors still have to be considered. For example, the extent and nature of utilization and the
consequent impact on the environment and on the indigenous peoples’ way of life are important
considerations. Moreover, the indigenous peoples must show that they live in the area and that they
are in the best position to undertake the required utilization.

It must be emphasized that the grant of said priority rights to indigenous peoples is not a blanket
authority to disregard pertinent laws and regulations. The utilization of said natural resources is
always subject to compliance by the indigenous peoples with existing laws, such as R.A. 7076 and
R.A. 7942 since it is not they but the State, which owns these resources.

It also bears stressing that the grant of priority rights does not preclude the State from undertaking
activities, or entering into co-production, joint venture or production-sharing agreements with private
entities, to utilize the natural resources which may be located within the ancestral domains. There is
no intention, as between the State and the indigenous peoples, to create a hierarchy of values;
rather, the object is to balance the interests of the State for national development and those of the
indigenous peoples.

Neither does the grant of priority rights to the indigenous peoples exclude non-indigenous peoples
from undertaking the same activities within the ancestral domains upon authority granted by the
proper governmental agency. To do so would unduly limit the ownership rights of the State over the
natural resources.
To be sure, the act of the State of giving preferential right to a particular sector in the utilization of
natural resources is nothing new. As previously mentioned, Section 7, Article XIII of the Constitution
mandates the protection by the State of "the rights of subsistence fishermen, especially of local
communities, to the preferential use of communal marine and fishing resources, both inland and
offshore."

Section 57 further recognizes the possibility that the exploration and exploitation of natural resources
within the ancestral domains may disrupt the natural environment as well as the traditional activities
of the indigenous peoples therein. Hence, the need for the prior informed consent of the indigenous
peoples before any search for or utilization of the natural resources within their ancestral domains is
undertaken.

In a situation where the State intends to directly or indirectly undertake such activities, IPRA requires
that the prior informed consent of the indigenous peoples be obtained. The State must, as a matter of
policy and law, consult the indigenous peoples in accordance with the intent of the framers of the
Constitution that national development policies and programs should involve a systematic
consultation to balance local needs as well as national plans. As may be gathered from the
discussion of the framers of the Constitution on this point, the national plan presumably takes into
account the requirements of the region after thorough consultation. 156 To this end, IPRA grants to the
indigenous peoples the right to an informed and intelligent participation in the formulation and
implementation of any project, government or private, and the right not to be removed therefrom
without their free and prior informed consent.157 As to non-members, the prior informed consent takes
the form of a formal and written agreement between the indigenous peoples and non-members under
the proviso in Section 57 in case the State enters into a co-production, joint venture, or production-
sharing agreement with Filipino citizens, or corporations. This requirement is not peculiar to IPRA.
Existing laws and regulations such as the Philippine Environmental Policy, 158 the Environmental
Impact System,159 the Local Government Code160 and the Philippine Mining Act of 1995161already
require increased consultation and participation of stakeholders, such as indigenous peoples, in the
planning of activities with significant environment impact.

The requirement in Section 59 that prior written informed consent of the indigenous peoples must be
procured before the NCIP can issue a certification for the "issuance, renewal, or grant of any
concession, license or lease, or to the perfection of any production-sharing agreement," must be
interpreted, not as a grant of the power to control the exploration, development and utilization of
natural resources, but merely the imposition of an additional requirement for such concession or
agreement. The clear intent of the law is to protect the rights and interests of the indigenous peoples
which may be adversely affected by the operation of such entities or licensees.

Corollary Issues

A. IPRA does not violate the Due Process clause.

The first corollary issue raised by petitioners is whether IPRA violates Section 1, Article III of the
Constitution, which provides that "no person shall be deprived of life, liberty, or property without due
process of law, nor shall any person be deprived the equal protection of the laws."

Petitioners maintain that the broad definition of ancestral lands and ancestral domains under Section
3(a) and 3(b) of IPRA includes private lands. They argue that the inclusion of private lands in the
ancestral lands and ancestral domains violates the due process clause. 162 Petitioners’ contention is
erroneous.
Sections 3(a) and 3(b) expressly provide that the definition of ancestral lands and ancestral domains
are "subject to Section 56," which reads:

Sec. 56. Existing Property Rights Regimes. – Property rights within the ancestral domains already
existing and/or vested upon effectivity of this Act, shall be recognized and protected.

Petitioners, however, contend that Section 56 aims to protect only the vested rights of indigenous
peoples, but not those who are not members of such communities. Following their interpretation,
IPRA, under Section 56, recognizes the rights of indigenous peoples to their ancestral lands and
ancestral domains, subject to the vested rights of the same communities to such ancestral
lands and ancestral domains. Such interpretation is obviously incorrect.

The "property rights" referred to in Section 56 belong to those acquired by individuals, whether
indigenous or non-indigenous peoples. Said provision makes no distinction as to the ethnic origins of
the ownership of these "property rights." The IPRA thus recognizes and respects "vested rights"
regardless of whether they pertain to indigenous or non-indigenous peoples. Where the law does not
distinguish, the courts should not distinguish.163What IPRA only requires is that these "property rights"
already exist and/or vested upon its effectivity.

Further, by the enactment of IPRA, Congress did not purport to annul any and all Torrens titles within
areas claimed as ancestral lands or ancestral domains. The statute imposes strict procedural
requirements for the proper delineation of ancestral lands and ancestral domains as safeguards
against the fraudulent deprivation of any landowner of his land, whether or not he is member of an
indigenous cultural community. In all proceedings for delineation of ancestral lands and ancestral
domains, the Director of Lands shall appear to represent the interest of the Republic of the
Philippines.164 With regard to ancestral domains, the following procedure is mandatory: first, petition
by an indigenous cultural community, or motu proprio by the NCIP; second, investigation and census
by the Ancestral domains Office ("ADO") of the NCIP; third, preliminary report by the ADO; fourth,
posting and publication; and lastly, evaluation by the NCIP upon submission of the final report of the
ADO.165 With regard to ancestral lands, unless such lands are within an ancestral domain, the
statute imposes the following procedural requirements: first, application; second, posting and
publication; third, investigation and inspection by the ADO; fourth, delineation; lastly, evaluation by
the NCIP upon submission of a report by the ADO.166 Hence, we cannot sustain the arguments of the
petitioners that the law affords no protection to those who are not indigenous peoples.

Neither do the questioned sections of IPRA on the composition and powers and jurisdiction of the
NCIP167 and the application of customary law,168 violate the due process clause of the Constitution.

Petitioners point out that IPRA provides that the NCIP shall be composed exclusively of members of
indigenous peoples,169 and that the NCIP shall have jurisdiction over all claims and disputes involving
indigenous peoples,170including even disputes between a member of such communities and one who
is not a member, as well as over disputes in the delineation of ancestral domains. 171 Petitioners clarify
that they do not claim that the members of the NCIP are incapable of being fair and impartial judges.
They merely contend that the NCIP will not appear to be impartial, because a party who is not a
member of an indigenous cultural community "who must defend his case against [one who is] before
judges who are all members of [indigenous peoples] cannot but harbor a suspicion that they do not
have the cold neutrality of an impartial judge."172

In addition, petitioners claim that IPRA prescribes that customary laws shall be applied first in
disputes involving property, succession and land,173 and that such laws shall likewise be used in
disputes involving indigenous peoples.174 They assert that "[w]hen the dispute involves a member of
an [indigenous cultural community and another who is not], a resolution of such a dispute based on
customary laws. . . would clearly be a denial of due process. . . [because those who are not
indigenous peoples] do not know what these customary laws are."175

Petitioners’ concerns are unfounded. The fact that the NCIP is composed of members of the
indigenous peoples does not mean that it (the NCIP) is incapable, or will appear to be so incapable,
of delivering justice to the non-indigenous peoples. A person’s possession of the trait of impartiality
desirable of a judge has nothing to do with his or her ethnic roots. In this wise, the indigenous peoples
are as capable of rendering justice as the non-indigenous peoples for, certainly, the latter have no
monopoly of the concept of justice.

In any case, there are sufficient checks in the law against any abuse by the NCIP of its quasi-judicial
powers. Section 67 states that the decision of the NCIP shall be appealable to the Court of Appeals
by petition for review. The regular remedies under our rules of procedure are likewise available to any
party aggrieved by the decision of the NCIP.

Anent the use of customary laws in determining the ownership and extent of ancestral domains,
suffice it to say that such is allowed under paragraph 2, Section 5 of Article XII of the Constitution.
Said provision states, "The Congress may provide for the applicability of customary laws governing
property rights and relations in determining the ownership and extent of the ancestral domains."
Notably, the use of customary laws under IPRA is not absolute, for the law speaks merely of primacy
of use.176 The IPRA prescribes the application of such customary laws where these present a
workable solution acceptable to the parties, who are members of the same indigenous group. This
interpretation is supported by Section 1, Rule IX of the Implementing Rules which states:

RULE IX. JURISDICTION AND PROCEDURES FOR ENFORCEMENT OF RIGHTS

Section 1. Primacy of Customary Law. All conflicts related to ancestral domains and lands, involving
ICCs/IPs, such as but not limited to conflicting claims and boundary disputes, shall be resolved by the
concerned parties through the application of customary laws in the area where the disputed ancestral
domain or land is located.

All conflicts related to the ancestral domains or lands where one of the parties is a non-ICC/IP
or where the dispute could not be resolved through customary law shall be heard and
adjudicated in accordance with the Rules on Pleadings, Practice and Procedures Before the
NCIP to be adopted hereafter. (Emphasis supplied.)

The application of customary law is limited to disputes concerning property rights or relations in
determining the ownership and extent of the ancestral domains,177 where all the parties involved
are members of indigenous peoples,178 specifically, of the same indigenous group. It therefore follows
that when one of the parties to a dispute is a non-member of an indigenous group, or when the
indigenous peoples involved belong to different groups, the application of customary law is not
required.

Like any other law, the objective of IPRA in prescribing the primacy of customary law in disputes
concerning ancestral lands and domains where all parties involved are indigenous peoples is justice.
The utilization of customary laws is in line with the constitutional policy of recognizing the application
thereof through legislation passed by Congress.

Furthermore, the recognition and use of customary law is not a novel idea in this jurisdiction. Under
the Civil Code, use of customary law is sanctioned, as long as it is proved as a fact according to the
rules of evidence,179 and it is not contrary to law, public order or public policy.180 Moreover, the Local
Government Code of 1991 calls for the recognition and application of customary laws to the
resolution of issues involving members of indigenous peoples. This law admits the operation of
customary laws in the settling of disputes if such are ordinarily used in barangays where majority of
the inhabitants are members of indigenous peoples.181

B. Section 1, Part II, Rule VII of the Implementing Rules of IPRA does not infringe upon the
President’s power of control over the Executive Department.

The second corollary issue is whether the Implementing Rules of IPRA violate Section 17, Article VII
of the Constitution, which provides that:

The President shall have control of all the executive departments, bureaus, and offices. He shall
ensure that the laws be faithfully executed.

The assailed provision of the Implementing Rules provides:

Rule VII. The National Commission on Indigenous Peoples (NCIP)

xxx

Part II: NCIP as an Independent Agency Under the Office of the President

Section 1. The NCIP is the primary agency of government for the formulation and implementation of
policies, plans and programs to recognize, promote and protect the rights and well-being of
indigenous peoples. It shall be an independent agency under the Office of the President. As such,
the administrative relationship of the NCIP to the Office of the President is characterized as a
lateral but autonomous relationship for purposes of policy and program coordination. This
relationship shall be carried out through a system of periodic reporting. Matters of day-to-day
administration or all those pertaining to internal operations shall be left to the discretion of the
Chairperson of the Commission, as the Chief Executive Officer.

Petitioners asseverate that the aforecited rule infringes upon the power of control of the President
over the NCIP by characterizing the relationship of the NCIP to the Office of the President as "lateral
but autonomous...for purposes of policy and program coordination."

Although both Section 40 of the IPRA and Section 1, Part II, Rule VII of the Implementing Rules
characterize the NCIP as an independent agency under the Office of the President, such
characterization does not remove said body from the President’s control and supervision.

The NCIP has been designated under IPRA as the primary government agency responsible for the
formulation and implementation of policies, plans and programs to promote and protect the rights and
well being of the indigenous peoples and the recognition of their ancestral domain as well as their
rights thereto.182 It has been granted administrative,183 quasi-legislative184 and quasi-judicial
powers185 to carry out its mandate. The diverse nature of the NCIP’s functions renders it impossible to
place said agency entirely under the control of only one branch of government and this, apparently, is
the reason for its characterization by Congress as an independent agency. An "independent agency"
is defined as an administrative body independent of the executive branch or one not subject to a
superior head of department, as distinguished from a "subordinate agency" or an administrative body
whose action is subject to administrative review or revision.186

That Congress did not intend to place the NCIP under the control of the President in all instances is
evident in the IPRA itself, which provides that the decisions of the NCIP in the exercise of its quasi-
judicial functions shall be appealable to the Court of Appeals,187 like those of the National Labor
Relations Commission (NLRC) and the Securities and Exchange Commission (SEC). Nevertheless,
the NCIP, although independent to a certain degree, was placed by Congress "under the office of the
President" and, as such, is still subject to the President’s power of control and supervision granted
under Section 17, Article VII of the Constitution188 with respect to its performance of administrative
functions, such as the following: (1) the NCIP must secure the President’s approval in obtaining loans
to finance its projects;189 (2) it must obtain the President’s approval for any negotiation for funds and
for the acceptance of gifts and/or properties in whatever from and from whatever source;190 (3) the
NCIP shall submit annual reports of its operations and achievements to the President, and advise the
latter on all matters relating to the indigenous peoples;191 and (4) it shall exercise such other powers
as may be directed by the President.192The President is also given the power to appoint the
Commissioners of the NCIP193 as well as to remove them from office for cause motu proprio or upon
the recommendation of any indigenous community.194

To recapitulate:

(1) The provisions of the IPRA (specifically Sections 3, paragraphs (a) and (b), 5, 6, 7, and 8)
affirming the ownership by the indigenous peoples of their ancestral lands and domains by
virtue of native title do not diminish the State’s ownership of lands of the public domain,
because said ancestral lands and domains are considered as private land, and never to have
been part of the public domain, following the doctrine laid down in Cariño vs. Insular
Government;195

(2) The constitutional provision vesting ownership over minerals, mineral lands and other
natural resources in the State is not violated by Sections 3, 5, 7, 56, 57, 58 and 59 of the IPRA
which grant certain rights to the indigenous peoples over the natural resources found within the
ancestral domains, e.g., to benefit from and share in the profits from the allocation and
utilization of the same, as well as priority rights in the harvesting, extraction, development or
exploitation thereof. The State retains full control over the exploration, development and
utilization of natural resources even with the grant of said rights to the indigenous peoples,
through the imposition of requirements and conditions for the utilization of natural resources
under existing laws, such as the Small-Scale Mining Act of 1991196and the Philippine Mining
Act of 1995.197Moreover, the rights granted to indigenous peoples for the utilization of natural
resources within their ancestral domains merely amplify what has been earlier granted to them
under the aforesaid laws;

(3) While the IPRA recognizes the rights of indigenous peoples with regard to their ancestral
lands and domains, it also protects the vested rights of persons, whether indigenous or non-
indigenous peoples, who may have acquired rights of ownership lands or rights to explore and
exploit natural resources within the ancestral lands and domains; 198

(4) The Due Process Clause of the Constitution is not violated by the provisions (Sections 40,
51-54, 62, 63, 65 and 66) of the IPRA which, among others, establish the composition of the
NCIP, and prescribe the application of customary law in certain disputes involving indigenous
peoples. The fact the NCIP is composed wholly of indigenous peoples does not mean that it is
incapable of being impartial. Moreover, the use of customary laws is sanctioned by paragraph
2, Section 5 of Article XII of the Constitution; and

(5) The provision of the Implementing Rules characterizing the NCIP as an independent
agency under the Office of the President does not infringe upon the President’s power of
control under Section 17, Article VII of the Constitution, since said provision as well as Section
40 of the IPRA expressly places the NCIP under the Office of the President, and therefore
under the President’s control and supervision with respect to its administrative functions.
However, insofar as the decisions of the NCIP in the exercise of its quasi-judicial powers are
concerned, the same are reviewable by the Court of Appeals, like those of the NLRC and the
SEC.

In view of the foregoing, I vote to DISMISS the petition.

G.R. No. 179987 September 3, 2013

HEIRS OF MARIO MALABANAN, (Represented by Sally A. Malabanan), Petitioners,


vs.
REPUBLIC OF THE PHILIPPINES, Respondent.

RESOLUTION

BERSAMIN, J.:

For our consideration and resolution are the motions for reconsideration of the parties who both assail
the decision promulgated on April 29, 2009, whereby we upheld the ruling of the Court of Appeals
(CA) denying the application of the petitioners for the registration of a parcel of land situated in
Barangay Tibig, Silang, Cavite on the ground that they had not established by sufficient evidence
their right to the registration in accordance with either Section 14(1) or Section 14(2) of Presidential
Decree No. 1529 (Property Registration Decree).

Antecedents

The property subject of the application for registration is a parcel of land situated in Barangay Tibig,
Silang Cavite, more particularly identified as Lot 9864-A, Cad-452-D, with an area of 71,324-square
meters. On February 20, 1998, applicant Mario Malabanan, who had purchased the property from
Eduardo Velazco, filed an application for land registration covering the property in the Regional Trial
Court (RTC) in Tagaytay City, Cavite, claiming that the property formed part of the alienable and
disposable land of the public domain, and that he and his predecessors-in-interest had been in open,
continuous, uninterrupted, public and adverse possession and occupation of the land for more than
30 years, thereby entitling him to the judicial confirmation of his title. 1

To prove that the property was an alienable and disposable land of the public domain, Malabanan
presented during trial a certification dated June 11, 2001 issued by the Community Environment and
Natural Resources Office (CENRO) of the Department of Environment and Natural Resources
(DENR), which reads:

This is to certify that the parcel of land designated as Lot No. 9864 Cad 452-D, Silang Cadastre as
surveyed for Mr. Virgilio Velasco located at Barangay Tibig, Silang, Cavite containing an area of
249,734 sq. meters as shown and described on the Plan Ap-04-00952 is verified to be within the
Alienable or Disposable land per Land Classification Map No. 3013 established under Project No. 20-
A and approved as such under FAO 4-1656 on March 15, 1982.2

After trial, on December 3, 2002, the RTC rendered judgment granting Malabanan’s application for
land registration, disposing thusly:
WHEREFORE, this Court hereby approves this application for registration and thus places under the
operation of Act 141, Act 496 and/or P.D. 1529, otherwise known as Property Registration Law, the
lands described in Plan Csd-04-0173123-D, Lot 9864-A and containing an area of Seventy One
Thousand Three Hundred Twenty Four (71,324) Square Meters, as supported by its technical
description now forming part of the record of this case, in addition to other proofs adduced in the
name of MARIO MALABANAN, who is of legal age, Filipino, widower, and with residence at Munting
Ilog, Silang, Cavite.

Once this Decision becomes final and executory, the corresponding decree of registration shall
forthwith issue.

SO ORDERED.3

The Office of the Solicitor General (OSG) appealed the judgment to the CA, arguing that Malabanan
had failed to prove that the property belonged to the alienable and disposable land of the public
domain, and that the RTC erred in finding that he had been in possession of the property in the
manner and for the length of time required by law for confirmation of imperfect title.

On February 23, 2007, the CA promulgated its decision reversing the RTC and dismissing the
application for registration of Malabanan. Citing the ruling in Republic v. Herbieto (Herbieto),4 the CA
declared that under Section 14(1) of the Property Registration Decree, any period of possession prior
to the classification of the land as alienable and disposable was inconsequential and should be
excluded from the computation of the period of possession. Noting that the CENRO-DENR
certification stated that the property had been declared alienable and disposable only on March 15,
1982, Velazco’s possession prior to March 15, 1982 could not be tacked for purposes of computing
Malabanan’s period of possession.

Due to Malabanan’s intervening demise during the appeal in the CA, his heirs elevated the CA’s
decision of February 23, 2007 to this Court through a petition for review on certiorari.

The petitioners assert that the ruling in Republic v. Court of Appeals and Corazon Naguit 5 (Naguit)
remains the controlling doctrine especially if the property involved is agricultural land. In this regard,
Naguit ruled that any possession of agricultural land prior to its declaration as alienable and
disposable could be counted in the reckoning of the period of possession to perfect title under the
Public Land Act (Commonwealth Act No. 141) and the Property Registration Decree. They point out
that the ruling in Herbieto, to the effect that the declaration of the land subject of the application for
registration as alienable and disposable should also date back to June 12, 1945 or earlier, was a
mere obiter dictum considering that the land registration proceedings therein were in fact found and
declared void ab initio for lack of publication of the notice of initial hearing.

The petitioners also rely on the ruling in Republic v. T.A.N. Properties, Inc. 6 to support their argument
that the property had been ipso jure converted into private property by reason of the open,
continuous, exclusive and notorious possession by their predecessors-in-interest of an alienable land
of the public domain for more than 30 years. According to them, what was essential was that the
property had been "converted" into private property through prescription at the time of the application
without regard to whether the property sought to be registered was previously classified as
agricultural land of the public domain.

As earlier stated, we denied the petition for review on certiorari because Malabanan failed to establish
by sufficient evidence possession and occupation of the property on his part and on the part of his
predecessors-in interest since June 12, 1945, or earlier.
Petitioners’ Motion for Reconsideration

In their motion for reconsideration, the petitioners submit that the mere classification of the land as
alienable or disposable should be deemed sufficient to convert it into patrimonial property of the
State. Relying on the rulings in Spouses De Ocampo v. Arlos,7 Menguito v. Republic8 and Republic v.
T.A.N. Properties, Inc.,9 they argue that the reclassification of the land as alienable or disposable
opened it to acquisitive prescription under the Civil Code; that Malabanan had purchased the property
from Eduardo Velazco believing in good faith that Velazco and his predecessors-in-interest had been
the real owners of the land with the right to validly transmit title and ownership thereof; that
consequently, the ten-year period prescribed by Article 1134 of the Civil Code, in relation to Section
14(2) of the Property Registration Decree, applied in their favor; and that when Malabanan filed the
application for registration on February 20, 1998, he had already been in possession of the land for
almost 16 years reckoned from 1982, the time when the land was declared alienable and disposable
by the State.

The Republic’s Motion for Partial Reconsideration

The Republic seeks the partial reconsideration in order to obtain a clarification with reference to the
application of the rulings in Naguit and Herbieto.

Chiefly citing the dissents, the Republic contends that the decision has enlarged, by implication, the
interpretation of Section 14(1) of the Property Registration Decree through judicial legislation. It
reiterates its view that an applicant is entitled to registration only when the land subject of the
application had been declared alienable and disposable since June 12, 1945 or earlier.

Ruling

We deny the motions for reconsideration.

In reviewing the assailed decision, we consider to be imperative to discuss the different classifications
of land in relation to the existing applicable land registration laws of the Philippines.

Classifications of land according to ownership

Land, which is an immovable property,10 may be classified as either of public dominion or of private
ownership.11Land is considered of public dominion if it either: (a) is intended for public use; or (b)
belongs to the State, without being for public use, and is intended for some public service or for the
development of the national wealth.12 Land belonging to the State that is not of such character, or
although of such character but no longer intended for public use or for public service forms part of the
patrimonial property of the State.13 Land that is other than part of the patrimonial property of the
State, provinces, cities and municipalities is of private ownership if it belongs to a private individual.

Pursuant to the Regalian Doctrine (Jura Regalia), a legal concept first introduced into the country
from the West by Spain through the Laws of the Indies and the Royal Cedulas,14 all lands of the
public domain belong to the State.15 This means that the State is the source of any asserted right to
ownership of land, and is charged with the conservation of such patrimony. 16

All lands not appearing to be clearly under private ownership are presumed to belong to the State.
Also, public lands remain part of the inalienable land of the public domain unless the State is shown
to have reclassified or alienated them to private persons.17
Classifications of public lands
according to alienability

Whether or not land of the public domain is alienable and disposable primarily rests on the
classification of public lands made under the Constitution. Under the 1935 Constitution, 18 lands of the
public domain were classified into three, namely, agricultural, timber and mineral.19 Section 10, Article
XIV of the 1973 Constitution classified lands of the public domain into seven, specifically, agricultural,
industrial or commercial, residential, resettlement, mineral, timber or forest, and grazing land, with the
reservation that the law might provide other classifications. The 1987 Constitution adopted the
classification under the 1935 Constitution into agricultural, forest or timber, and mineral, but added
national parks.20 Agricultural lands may be further classified by law according to the uses to which
they may be devoted.21 The identification of lands according to their legal classification is done
exclusively by and through a positive act of the Executive Department.22

Based on the foregoing, the Constitution places a limit on the type of public land that may be
alienated. Under Section 2, Article XII of the 1987 Constitution, only agricultural lands of the public
domain may be alienated; all other natural resources may not be.

Alienable and disposable lands of the State fall into two categories, to wit: (a) patrimonial lands of the
State, or those classified as lands of private ownership under Article 425 of the Civil Code, 23 without
limitation; and (b) lands of the public domain, or the public lands as provided by the Constitution, but
with the limitation that the lands must only be agricultural. Consequently, lands classified as forest or
timber, mineral, or national parks are not susceptible of alienation or disposition unless they are
reclassified as agricultural.24 A positive act of the Government is necessary to enable such
reclassification,25 and the exclusive prerogative to classify public lands under existing laws is vested
in the Executive Department, not in the courts.26 If, however, public land will be classified as neither
agricultural, forest or timber, mineral or national park, or when public land is no longer intended for
public service or for the development of the national wealth, thereby effectively removing the land
from the ambit of public dominion, a declaration of such conversion must be made in the form of a law
duly enacted by Congress or by a Presidential proclamation in cases where the President is duly
authorized by law to that effect.27Thus, until the Executive Department exercises its prerogative to
classify or reclassify lands, or until Congress or the President declares that the State no longer
intends the land to be used for public service or for the development of national wealth, the Regalian
Doctrine is applicable.

Disposition of alienable public lands

Section 11 of the Public Land Act (CA No. 141) provides the manner by which alienable and
disposable lands of the public domain, i.e., agricultural lands, can be disposed of, to wit:

Section 11. Public lands suitable for agricultural purposes can be disposed of only as follows, and not
otherwise:

(1) For homestead settlement;

(2) By sale;

(3) By lease; and

(4) By confirmation of imperfect or incomplete titles;

(a) By judicial legalization; or


(b) By administrative legalization (free patent).

The core of the controversy herein lies in the proper interpretation of Section 11(4), in relation to
Section 48(b) of the Public Land Act, which expressly requires possession by a Filipino citizen of the
land since June 12, 1945, or earlier, viz:

Section 48. The following-described citizens of the Philippines, occupying lands of the public domain
or claiming to own any such lands or an interest therein, but whose titles have not been perfected or
completed, may apply to the Court of First Instance of the province where the land is located for
confirmation of their claims and the issuance of a certificate of title thereafter, under the Land
Registration Act, to wit:

xxxx

(b) Those who by themselves or through their predecessors-in-interest have been in open,
continuous, exclusive, and notorious possession and occupation of alienable and disposable lands of
the public domain, under a bona fide claim of acquisition of ownership, since June 12, 1945, or
earlier, immediately preceding the filing of the applications for confirmation of title, except when
prevented by war or force majeure. These shall be conclusively presumed to have performed all the
conditions essential to a Government grant and shall be entitled to a certificate of title under the
provisions of this chapter. (Bold emphasis supplied)

Note that Section 48(b) of the Public Land Act used the words "lands of the public domain" or
"alienable and disposable lands of the public domain" to clearly signify that lands otherwise classified,
i.e., mineral, forest or timber, or national parks, and lands of patrimonial or private ownership, are
outside the coverage of the Public Land Act. What the law does not include, it excludes. The use of
the descriptive phrase "alienable and disposable" further limits the coverage of Section 48(b) to only
the agricultural lands of the public domain as set forth in Article XII, Section 2 of the 1987
Constitution. Bearing in mind such limitations under the Public Land Act, the applicant must satisfy
the following requirements in order for his application to come under Section 14(1) of the Property
Registration Decree,28 to wit:

1. The applicant, by himself or through his predecessor-in-interest, has been in possession


and occupation of the property subject of the application;

2. The possession and occupation must be open, continuous, exclusive, and notorious;

3. The possession and occupation must be under a bona fide claim of acquisition of
ownership;

4. The possession and occupation must have taken place since June 12, 1945, or earlier; and

5. The property subject of the application must be an agricultural land of the public domain.

Taking into consideration that the Executive Department is vested with the authority to classify lands
of the public domain, Section 48(b) of the Public Land Act, in relation to Section 14(1) of the Property
Registration Decree, presupposes that the land subject of the application for registration must have
been already classified as agricultural land of the public domain in order for the provision to apply.
Thus, absent proof that the land is already classified as agricultural land of the public domain, the
Regalian Doctrine applies, and overcomes the presumption that the land is alienable and disposable
as laid down in Section 48(b) of the Public Land Act. However, emphasis is placed on the
requirement that the classification required by Section 48(b) of the Public Land Act is classification or
reclassification of a public land as agricultural.

The dissent stresses that the classification or reclassification of the land as alienable and disposable
agricultural land should likewise have been made on June 12, 1945 or earlier, because any
possession of the land prior to such classification or reclassification produced no legal effects. It
observes that the fixed date of June 12, 1945 could not be minimized or glossed over by mere judicial
interpretation or by judicial social policy concerns, and insisted that the full legislative intent be
respected.

We find, however, that the choice of June 12, 1945 as the reckoning point of the requisite possession
and occupation was the sole prerogative of Congress, the determination of which should best be left
to the wisdom of the lawmakers. Except that said date qualified the period of possession and
occupation, no other legislative intent appears to be associated with the fixing of the date of June 12,
1945. Accordingly, the Court should interpret only the plain and literal meaning of the law as written
by the legislators.

Moreover, an examination of Section 48(b) of the Public Land Act indicates that Congress prescribed
no requirement that the land subject of the registration should have been classified as agricultural
since June 12, 1945, or earlier. As such, the applicant’s imperfect or incomplete title is derived only
from possession and occupation since June 12, 1945, or earlier. This means that the character of the
property subject of the application as alienable and disposable agricultural land of the public domain
determines its eligibility for land registration, not the ownership or title over it.

Alienable public land held by a possessor, either personally or through his predecessors-in-interest,
openly, continuously and exclusively during the prescribed statutory period is converted to private
property by the mere lapse or completion of the period.29 In fact, by virtue of this doctrine,
corporations may now acquire lands of the public domain for as long as the lands were already
converted to private ownership, by operation of law, as a result of satisfying the requisite period of
possession prescribed by the Public Land Act.30 It is for this reason that the property subject of the
application of Malabanan need not be classified as alienable and disposable agricultural land of the
public domain for the entire duration of the requisite period of possession.

To be clear, then, the requirement that the land should have been classified as alienable and
disposable agricultural land at the time of the application for registration is necessary only to dispute
the presumption that the land is inalienable.

The declaration that land is alienable and disposable also serves to determine the point at which
prescription may run against the State. The imperfect or incomplete title being confirmed under
Section 48(b) of the Public Land Act is title that is acquired by reason of the applicant’s possession
and occupation of the alienable and disposable agricultural land of the public domain. Where all the
necessary requirements for a grant by the Government are complied with through actual physical,
open, continuous, exclusive and public possession of an alienable and disposable land of the public
domain, the possessor is deemed to have acquired by operation of law not only a right to a grant, but
a grant by the Government, because it is not necessary that a certificate of title be issued in order that
such a grant be sanctioned by the courts.31

If one follows the dissent, the clear objective of the Public Land Act to adjudicate and quiet titles to
unregistered lands in favor of qualified Filipino citizens by reason of their occupation and cultivation
thereof for the number of years prescribed by law32 will be defeated. Indeed, we should always bear
in mind that such objective still prevails, as a fairly recent legislative development bears out, when
Congress enacted legislation (Republic Act No. 10023)33 in order to liberalize stringent requirements
and procedures in the adjudication of alienable public land to qualified applicants, particularly
residential lands, subject to area limitations.34

On the other hand, if a public land is classified as no longer intended for public use or for the
development of national wealth by declaration of Congress or the President, thereby converting such
land into patrimonial or private land of the State, the applicable provision concerning disposition and
registration is no longer Section 48(b) of the Public Land Act but the Civil Code, in conjunction with
Section 14(2) of the Property Registration Decree.35As such, prescription can now run against the
State.

To sum up, we now observe the following rules relative to the disposition of public land or lands of the
public domain, namely:

(1) As a general rule and pursuant to the Regalian Doctrine, all lands of the public domain
belong to the State and are inalienable. Lands that are not clearly under private ownership are
also presumed to belong to the State and, therefore, may not be alienated or disposed;

(2) The following are excepted from the general rule, to wit:

(a) Agricultural lands of the public domain are rendered alienable and disposable
through any of the exclusive modes enumerated under Section 11 of the Public Land
Act. If the mode is judicial confirmation of imperfect title under Section 48(b) of the
Public Land Act, the agricultural land subject of the application needs only to be
classified as alienable and disposable as of the time of the application, provided the
applicant’s possession and occupation of the land dated back to June 12, 1945, or
earlier. Thereby, a conclusive presumption that the applicant has performed all the
conditions essential to a government grant arises,36 and the applicant becomes the
owner of the land by virtue of an imperfect or incomplete title. By legal fiction, the land
has already ceased to be part of the public domain and has become private property.37

(b) Lands of the public domain subsequently classified or declared as no longer


intended for public use or for the development of national wealth are removed from the
sphere of public dominion and are considered converted into patrimonial lands or lands
of private ownership that may be alienated or disposed through any of the modes of
acquiring ownership under the Civil Code. If the mode of acquisition is prescription,
whether ordinary or extraordinary, proof that the land has been already converted to
private ownership prior to the requisite acquisitive prescriptive period is a condition sine
qua non in observance of the law (Article 1113, Civil Code) that property of the State not
patrimonial in character shall not be the object of prescription.

To reiterate, then, the petitioners failed to present sufficient evidence to establish that they and their
predecessors-in-interest had been in possession of the land since June 12, 1945. Without satisfying
the requisite character and period of possession - possession and occupation that is open,
continuous, exclusive, and notorious since June 12, 1945, or earlier - the land cannot be considered
ipso jure converted to private property even upon the subsequent declaration of it as alienable and
disposable. Prescription never began to run against the State, such that the land has remained
ineligible for registration under Section 14(1) of the Property Registration Decree. Likewise, the land
continues to be ineligible for land registration under Section 14(2) of the Property Registration Decree
unless Congress enacts a law or the President issues a proclamation declaring the land as no longer
intended for public service or for the development of the national wealth.1âwphi1
WHEREFORE, the Court DENIES the petitioners' Motion for Reconsideration and the respondent's
Partial Motion for Reconsideration for their lack of merit.

SO ORDERED.

[G.R. No. L-27873. November 29, 1983.]

HEIRS OF JOSE AMUNATEGUI, Petitioners, v. DIRECTOR OF FORESTRY, Respondent.

[G.R. No. L-30035. November 29, 1983.]

ROQUE BORRE and ENCARNACION DELFIN, Petitioners, v. ANGEL ALPASAN, HEIRS OF


MELQUIADES BORRE, EMETERIO BEREBER and HEIRS OF JOSE AMUNATEGUI and THE
CAPIZ COURT OF FIRST INSTANCE, Respondents.

SYLLABUS

1. CIVIL LAW; PUBLIC LAND ACT; FOREST LAND; CLASSIFICATION NOT LOST EVEN IF IT HAS
BEEN STRIPPED OF FOREST COVER; UNLESS RELEASED IN AN OFFICIAL PROCLAMATION
AS DISPOSABLE LANDS, RULES ON CONFIRMATION OF IMPERFECT TITLE DO NOT APPLY.
— A forested area classified as forest land of the public domain does not lose such classification
simply because loggers or settlers may have stripped it of its forest cover. Parcels of land classified
as forest land may actually be covered with grass or planted to crops by kaingin cultivators or other
farmers. "Forest lands" do not have to be on mountains or in out of the way places. Swampy areas
covered by mangrove trees, nipa palms, and other tress growing in brackish or sea water may also
be classified as forest land. The classification is descriptive of its legal nature or status and does not
have to be descriptive of what the land actually looks like. Unless and until the land classified as
"forest" is released in an official proclamation to that effect so that it may form part of the disposable
agricultural lands of the public domain, the rules on confirmation of imperfect title do not apply.

2. ID.; ID.; FOREST LANDS; ACQUISITIVE OWNERSHIP NOT ACQUIRED. — This Court ruled in
the leading case of Director of Forestry v. Muñoz (23 SCRA 1184) that possession of forest lands, no
matter how long, cannot ripen into private ownership. And in Republic v. Animas (56 SCRA 499), we
granted the petition on the ground that the ares covered by the patent and title was not disposable
public land, it being a part of the forest zone and any patent and title to said area is void ab initio. It
bears emphasizing that a positive act of Government is needed to declassify land which is classified
as forest and to convert it into alienable or disposable land for agricultural or other purposes.

3. ID.; ID.; CONFIRMATION, OF IMPERFECT TITLE CASES; BURDEN OF PROVING THAT THE
REQUIREMENTS OF THE LAW HAVE BEEN MET, RESTS ON THE APPLICANT. — In
confirmation of imperfect title cases, the applicant shoulders the burden of proving that he meets the
requirements of Section 48, Commonwealth Act No. 141, as amended by Republic Act No. 1942. He
must overcome the presumption that the land he is applying for is part of the public domain but that
he has an interest therein sufficient to warrant registration in his name because of an imperfect title
such as those derived from old Spanish grants or that he has had continuous, open, and notorious
possession and occupation of agricultural lands of the public domain under a bona fide claim of
acquisition of ownership for at least thirty (30) years preceding the filing of his application.
DECISION

GUTIERREZ, JR., J.:

The two petitions for review on certiorari before us question the decision of the Court
of Appeals which declared the disputed property as forest land, not subject to titling in favor of private
persons.

These two petitions have their genesis in an application for confirmation of imperfect title and its
registration filed with the Court of First Instance of Capiz. The parcel of land sought to be registered is
known as Lot No. 885 of the Cadastral Survey of Pilar, Capiz, and has an area of 645,703 square
meters.cralawnad

Roque Borre, petitioner in G.R. No, L-30035, and Melquiades Borre, filed the application for
registration. In due time, the heirs of Jose Amunategui, petitioners in G.R. No. L-27873 filed an
opposition to the application of Roque and Melquiades Borre. At the same time, they prayed that the
title to a portion of Lot No. 885 of Pilar Cadastre containing 527,747 square meters be confirmed and
registered in the names of said Heirs of Jose Amunategui.

The Director of Forestry, through the Provincial Fiscal of Capiz, also filed an opposition to the
application for registration of title claiming that the land was mangrove swamp which was still
classified as forest land and part of the public domain.

Another oppositor, Emeterio Bereber filed his opposition insofar as a portion of Lot No. 885 containing
117,956 square meters was concerned and prayed that title to said portion be confirmed and
registered in his name.

During the progress of the trial, applicant-petitioner Roque Borre sold whatever rights and interests he
may have on Lot No. 885 to Angel Alpasan. The latter also filed an opposition, claiming that he is
entitled to have said lot registered in his name.

After trial, the Court of First Instance of Capiz adjudicated 117,956 square meters to Emeterio
Bereber and the rest of the land containing 527,747 square meters was adjudicated in the proportion
of 5/6 share to Angel Alpasan and 1/6 share to Melquiades Borre.

Only the Heirs of Jose Amunategui and the Director of Forestry filed their respective appeals with the
Court of Appeals, The case was docketed as CA-G.R. No. 34190-R.

In its decision, the Court of Appeals held:jgc:chanrobles.com.ph

". . . the conclusion so far must have to be that as to the private litigants that have been shown to
have a better right over Lot 885 are, as to the northeastern portion of a little less than 117,956 square
meters, it was Emeterio Bereber and as to the rest of 527,747 square meters, it was the heirs of Jose
Amunategui; but the last question that must have to be considered is whether after all, the title that
these two (2) private litigants have shown did not amount to a registerable one in view of the
opposition and evidence of the Director of Forestry; . . .

". . . turning back the clock thirty (30) years from 1955 when the application was filed which would
place it at 1925, the fact must have to be accepted that during that period, the land was a classified
forest land so much so that timber licenses had to be issued to certain licensee before 1926 and after
that; that even Jose Amunategui himself took the trouble to ask for a license to cut timber within the
area; and this can only mean that the Bureau of Forestry had stood and maintained its ground that it
was a forest land as indeed the testimonial evidence referred to above persuasively indicates, and
the only time when the property was converted into a fishpond was sometime after 1950; or a bare
five (5) years before the filing of the application; but only after there had been a previous warning by
the District Forester that that could not be done because it was classified as a public forest; so that
having these in mind and remembering that even under Republic Act 1942 which came into effect in
1957, two (2) years after this case had already been filed in the lower Court, in order for applicant to
be able to demonstrate a registerable title he must have shown.

"‘open, continuous, exclusive and notorious possession and occupation of agricultural lands of the
public domain under a bona fide claim of acquisition of ownership for at least thirty (30) years,
preceding the filing of the application;’

the foregoing details cannot but justify the conclusion that not one of the applicants or oppositors had
shown that during the required period of thirty (30) years prescribed by Republic Act 1942 in order for
him to have shown a registerable title for the entire period of thirty (30) years before filing of the
application, he had been in

"‘open, continuous, exclusive and notorious possession and occupation of agricultural lands of the
public domain’,

it is evident that the Bureau of Forestry had insisted on its claim all throughout that period of thirty
(30) years and even before and applicants and their predecessors had made implicit recognition of
that; the result must be to deny all these applications; this Court stating that it had felt impelled
notwithstanding, just the same to resolve the conflicting positions of the private litigants among
themselves as to who of them had demonstrated a better right to possess because this Court
foresees that this litigation will go all the way to the Supreme Court and it is always better that the
findings be as complete as possible to enable the Highest Court to pass final judgment;

"IN VIEW WHEREOF, the decision must have to be as it is hereby reversed; the application as well
as all the oppositions with the exception of that of the Director of Forestry which is hereby sustained
are dismissed; no more pronouncement as to costs."cralaw virtua1aw library

A petition for review on certiorari was filed by the Heirs of Jose Amunategui contending that the
disputed lot had been in the possession of private persons for over thirty years and therefore in
accordance with Republic Act No. 1942, said lot could still be the subject of registration and
confirmation of title in the name of a private person in accordance with Act No. 496 known as the
Land Registration Act. On the other hand, another petition for review on certiorari was filed by Roque
Borre and Encarnacion Delfin, contending that the trial court committed grave abuse of discretion in
dismissing their complaint against the Heirs of Jose Amunategui. The Borre complaint was for the
annulment of the deed of absolute sale of Lot No. 885 executed by them in favor of the Heirs of
Amunategui. The complaint was dismissed on the basis of the Court of Appeals’ decision that the
disputed lot is part of the public domain. The petitioners also question the jurisdiction of the Court
of Appeals in passing upon the relative rights of the parties over the disputed lot when its
final decision after all is to declare said lot a part of the public domain classified as forest
land.chanrobles law library : red

The need for resolving the questions raised by Roque Borre and Encarnacion Delfin in
their petition depends on the issue raised by the Heirs of Jose Amunategui, that is, whether or not Lot
No. 885 is public forest land, not capable of registration in the names of the private applicants.
The Heirs of Jose Amunategui maintain that Lot No. 885 cannot be classified as forest land because
it is not thickly forested but is a "mangrove swamp." Although conceding that a "mangrove swamp" is
included in the classification of forest land in accordance with Section 1820 of the Revised
Administrative Code, the petitioners argue that no big trees classified in Section 1821 of said Code as
first, second and third groups are found on the land in question. Furthermore, they contend that Lot
885, even if it is a mangrove swamp, is still subject to land registration proceedings because the
property had been in actual possession of private persons for many years, and therefore, said land
was already "private land" better adapted and more valuable for agricultural than for forest purposes
and not required by the public interests to be kept under forest classification.

The petition is without merit.

A forested area classified as forest land of the public domain does not lose such classification simply
because loggers or settlers may have stripped it of its forest cover. Parcels of land classified as forest
land may actually be covered with grass or planted to crops by kaingin cultivators or other farmers.
"Forest lands" do not have to be on mountains or in out of the way places. Swampy areas covered by
mangrove trees, nipa palms, and other trees growing in brackish or sea water may also be classified
as forest land. The classification is descriptive of its legal nature or status and does not have to be
descriptive of what the land actually looks like. Unless and until the land classified as "forest" is
released in an official proclamation to that effect so that it may form part of the disposable agricultural
lands of the public domain, the rules on confirmation of imperfect title do not apply.

This Court ruled in the leading case of Director of Forestry v. Muñoz (23 SCRA 1184) that possession
of forest lands, no matter how long, cannot ripen into private ownership. And in Republic v. Animas
(56 SCRA 499), we granted the petition on the ground that the area covered by the patent and title
was not disposable public land, it being a part of the forest zone and any patent and title to said area
is void ab initio. It bears emphasizing that a positive act of Government is needed to declassify land
which is classified as forest and to convert it into alienable or disposable land for agricultural or other
purposes.

The findings of the Court of Appeals are particularly well-grounded in the instant petition.

The fact that no trees enumerated in Section 1821 of the Revised Administrative Code are found in
Lot No. 885 does not divest such land of its being classified as forest land, much less as land of the
public domain. The appellate court found that in 1912, the land must have been a virgin forest as
stated by Emeterio Bereber’s witness Deogracias Gavacao, and that as late as 1926, it must have
been a thickly forested area as testified by Jaime Bertolde. The opposition of the Director of Forestry
was strengthened by the appellate court’s finding that timber licenses had to be issued to certain
licensees and even Jose Amunategui himself took the trouble to ask for a license to cut timber within
the area. It was only sometime in 1950 that the property was converted into fishpond but only after a
previous warning from the District Forester that the same could not be done because it was classified
as "public forest." chanrobles.com:cralaw:red

In confirmation of imperfect title cases, the applicant shoulders the burden of proving that he meets
the requirements of Section 48, Commonwealth Act No. 141, as amended by Republic Act No. 1942.
He must overcome the presumption that the land he is applying for is part of the public domain but
that he has an interest therein sufficient to warrant registration in his name because of an imperfect
title such as those derived from old Spanish grants or that he has had continuous, open, and
notorious possession and occupation of agricultural lands of the public domain under a bona fide
claim of acquisition of ownership for at least thirty (30) years preceding the filing of his application.

The decision of the appellate court is not based merely on the presumptions implicit in
Commonwealth Act No. 141 as amended. The records show that Lot No. 88S never ceased to be
classified as forest land of the public domain.

In Republic v. Gonong (118 SCRA 729) we ruled:jgc:chanrobles.com.ph

"As held in Oh Cho v. Director of Lands, 75 Phil. 890, all lands that were not acquired from the
Government, either by purchase or by grant, belong to the public domain. An exception to the rule
would be any land that should have been in the possession of an occupant and of his predecessors
in-interests since time immemorial, for such possession would justify the presumption that the land
had never been part of the public domain or that it had been a private property even before the
Spanish conquest."cralaw virtua1aw library

In the instant petitions, the exception in the Oh Cho case does not apply. The evidence is clear that
Lot No. 885 had always been public land classified as forest.

Similarly, in Republic v. Vera (120 SCRA 210), we ruled:jgc:chanrobles.com.ph

". . . The possession of public land however long the period thereof may have extended, never
confers title thereto upon the possessor because the statute of limitations with regard to public land
does not operate against the State, unless the occupant can prove possession and occupation of the
same under claim of ownership for the required number of years to constitute a grant from the State.
(Director of Lands v. Reyes, 68 SCRA 177, 195)."cralaw virtua1aw library

We, therefore, affirm the finding that the disputed property Lot No. 885 is part of the public domain,
classified as public forest land. There is no need for us to pass upon the other issues raised
by petitioners Roque Borre and Encarnacion Delfin, as such issues are rendered moot by this
finding.chanrobles virtual lawlibrary

WHEREFORE, the petitions in G. R. No. L-30035 and G. R. No. L-27873 are DISMISSED for lack of
merit. Costs against the petitioners.

SO ORDERED.

G.R. No. 75042 November 29, 1988

REPUBLIC OF THE PHILIPPINES, petitioner,


vs.
INTERMEDIATE APPELLATE COURT, ROMAN CATHOLIC BISHOP OF LUCENA, represented
by Msgr. Jose T. Sanchez, and REGIONAL TRIAL COURT, BRANCH LIII, LUCENA
CITY, respondents.

The Solicitor General for petitioner.

Gilbert D. Camaligan for private respondent.

BIDIN, J.:

This is an appeal from the 1) decision * of the FIRST CIVIL CASES DIVISION of the then
Intermediate Appellate Court dated May 13, 1986, in AC G.R. No. 01410 entitled the ROMAN
CATHOLIC BISHOP OF Lucena, represented by Msgr. Jose T. Sanchez, applicant-appellee vs.
Republic of the Philippines, et al., Oppositors-appellants, affirming the decision ** of the then Court of
FIRST INSTANCE of Quezon, 9th Judicial District, Branch 1, dated November 4, 1980 in Land
Registration Case No. N-1106 entitled the ROMAN CATHOLIC BISHOP of Lucena, represented
by Msgr. Jose T. Sanchez, applicant vs. the Director of Lands and the Director, Bureau of Forest
Development, oppositors, ordering the registration of title to the parcel of land designated, as lots 1, 2
and 3 of plan PSD-65686 and its technical descriptions, and the parcel of land described in plan PSU-
112592 and its technical description, together with whatever improvements existing thereon, in the
name of the ROMAN CATHOLIC BISHOP of Lucena and 2) its resolution Dated June 19,1986,
denying appellant's "Motion for Reconsideration for lack of merit."

The factual background of the case as found by the Intermediate Appellate Court are as follows:

On February 2, 1979, the ROMAN CATHOLIC BISHOP of Lucena, represented by


Msgr. Jose T. Sanchez, filed an application for confirmation of title to four (4) parcels of
land. Three of said parcels, denominated as Lots 1, 2 and 3, respectively, of plan PSU-
65686 are situated in Barrio Masin, Municipality of Candelaria, Quezon Province. The
fourth parcels under plan PSU-112592 is located in Barrio Bucal (Taguan), same
municipality and province. As basis for the application, the applicant claimed title to the
various properties through either purchase or donation dating as far back as 1928.

The legal requirements of publication and posting were duly complied with, as was the
service of copies of notice of initial hearing on the proper government officials.

In behalf of the Director of Lands and the Director of the Bureau of Forest Development,
the Solicitor General filed an Opposition on April 20, 1979, alleging therein among
others, that the applicant did not have an imperfect title or title in fee simple to the parcel
of land being applied for.

At the initial hearing held on November 13, 1979, only the Provincial Fiscal in
representation of the Solicitor General appeared to interpose personal objection to the
application. Hence, an Order of General Default against the whole world was issued by
the Court a quo except for the Director of Lands and the Director of the Bureau of
Forest Development.

The preliminaries dispensed with, the applicant then introduced its proofs in support of
the petition, summed up by the lower court as follows:

With respect to Lots 1, 2, and 3, plan PSU-65686.

Lots 1, 2 and 3 of plan PSU-65686 respectively containing an area of


18,977, 6,910 and 16,221 square meters, are adjoining lots & are situated
in the Barrio of Masin, Municipality of Candelaria, Province of Quezon
(formerly Tayabas) (Exhibits F, F-1, F-2 and F-3). Said lots were surveyed
for the Roman Catholic Church on November 3, 1928 (Exhibit P-5) and
the survey plan approved on October 20, 1929 (Exhibit F-6).

Lot 1 was acquired by the Roman Catholic Church thru Rev. Father
Raymundo Esquenet by purchase from the spouses Atanacio Yranso and
Maria Coronado on October 20, 1928 (Exhibits G, G-1), portion of Lot 2
also by purchase thru Rev. Father Raymundo Esquenet from the spouses
Benito Maramot and Venancia Descaller on May 22, 1969 (Exhibits M, N-
1), while the remaining portion of Lot 2 and Lot 3 were already owned and
possessed by the Roman Catholic Church even prior to the survey of the
said three lots in 1928.

Records of burial of the Roman Catholic Church of Candelaria, Quezon


showed that even as early as November 1918, Lot 3 has already been
utilized by the Roman Catholic Church as its cemetery in Candelaria,
Quezon (Exhibit N, N-1 to N-5).<äre||anº•1àw>

These three lots presently constituted the Roman Catholic Church


cemetery in Candelaria, Quezon.

Lots 1, 2 and 3 are declared for taxation purposes in the name of the
Roman Catholic Church under Tax Declaration Nos. 22-19-02-079, 22-19-
02-077 and 22-19-02-082 as 'cemetery site' (Exhibit S, V and T).

With respect to the parcel of land described in plan PSU-112592:

This parcel of land situated in the barrio of Bucal (Taguan), Municipality of


Candelaria, Province of Quezon (formerly Tayabas) and more particularly
described in plan PSU-1 12592 and its technical description with an area
of 3,221 square meters (Exhibit 1) was formerly owned and possessed by
the spouses Paulo G. Macasaet, and Gabriela V. de Macasaet. Said
spouses, on February 26, 1941, donated this lot to the Roman Catholic
Church represented by Reverend Father Raymundo Esquenet (Exhibit J,
J-1 to J-4). It was surveyed for the Roman Catholic Church on Aug. 16,
1940 as church site and the corresponding survey plan approved on Jan.
15, 1941 (Exhibits I-1, I-2, 1-3).

Previously erected on this Lot was an old chapel which was demolished
and new chapel now stands in its place on the same site.

For his part, the Fiscal in a Manifestation dated July 22, 1980, said 'the State will not
adduce evidence in support of its opposition and will submit the instant case for
decision.'

Evaluating the applicant's submitted proofs, the court a quo concluded, on the basis of
acquisitive prescription at the very least, that the former had adequately shown title to
the parcels of land being claimed.

Since the acquisition of these four (4) lots by the applicant, it has been in
continuous possession and enjoyment thereof, and such possession,
together with its predecessors-in interest, covering a period of more than
52 years (at least from the date of the survey in 1928) with respect to lots
1 and 2, about 62 years with respect to lot 3, all of plan PSU- 65686; and
more than 39 years with respect to the fourth parcel described in plan
PSU-112592 (at least from the date of the survey in 1940) have been
open, public, continuous, peaceful, adverse against the whole world, and
in the concept of owner.

Accordingly, the court ordered the registration of the four parcels together with the
improvements thereon "in the name of the ROMAN CATHOLIC BISHOP OF LUCENA,
INC., a religious corporation sole duly registered and existing under the laws of the
Republic of the Philippines."

Against this decision, the Solicitor General filed a Motion for reconsideration on the
following grounds:

1. Article XIV, Section 11 of the New Constitution(1973) disqualifies a private


corporation from acquiring alienable lands for the public domain.

2. In the case at bar the application was filed after the effectivity on the New
Constitution on January 17, 1973.

which was denied by the lower court for lack of merit.

Still insisting of the alleged unconstitutionality of the registration (a point which,


incidentally, the appellant never raised in the lower court prior to its Motion for
Reconsideration), the Republic elevated this appeal. (Rollo, pp. 25-28)

On May 13, 1986, the first Civil Cases Division of the Intermediate Appellate Court rendered its
Decision the dispositive part of which reads:

WHEREFORE, finding the judgment a quo to be supported by law and the evidence on
record, the same is hereby AFFIRMED. No pronouncement as to costs.

SO ORDERED. (Rollo p. 30)

A reconsideration of the aforequoted Decision was sought by Appellant Republic of the Philippines,
but for lack of merit, its motion for reconsideration was denied on June 19, 1986, by Resolution of the
First Civil Case Division, Intermediate Appellate Court which resolution reads in full:

Considering appellant Republic of the Philippines "Motion for reconsideration" filed on


June 4, 1986; the Court RESOLVED to DENY the Motion for Reconsideration for lack of
merit, grounds raised therein having all been considered in the decision. (Rollo, p. 31)

Hence, this petition.

The following are the assigned errors raised by the petitioner in its petition:

1. The decision and the resolution in question are contrary to law and decisions of this
honorable Court in Meralco vs. Castro-Bartolome and Republic, 114 SCRA 799 (prom.
June 29,1982); Republic vs. Judge Villanueva and Iglesia ni Cristo, 114 SCRA 875,
June 29, 1982); and Republic vs. Judge Gonong and Iglesia ni Cristo, 118 SCRA 729-
733 (November 25,1982); Director of Lands vs. Hermanos y Hermanas, Inc. 141 SCRA
21-25 (Jan. 7,1986).

2. The lands applied for registration were the subject of a previous registration case
where a decree of registration was already issued.

3. Respondent corporation failed to establish the indentity of the lands applied for.
(Rollo, pp. 14-15)
The issue raised in this case involves the question of whether the Roman Catholic Bishop of Lucena,
as a corporation sole is qualified to apply for confirmation of its title to the four (4) parcels of land
subject of this case.

Corollary thereto is the question of whether or not a corporation sole should be treated as an ordinary
private corporation, for purpose of the application of Art. XIV, Sec. 11 of the 1973 Constitution.

Article XIV, Sec. 11 of the 1973 Constitution, in part provides:

Sec. 11. .... No private corporation or association may hold alienable lands of the public
domain except by lease not to exceed one thousand hectares in area; nor may any
citizen hold such lands by lease in excess of five hundred hectares....

Sec. 48 of the Public Land Act, in part, provides:

Sec. 48. The following described citizens of the Philippines occupying lands of the
public domain or claiming to own any such lands or an interest therein, but whose titles
have not been perfected or completed, may apply to the Court of First Instance of the
province where the land is located for confirmation of their claims and the issuance of a
Certificate of title therefor, under the Land Registration Act, to wit:

(a) ...

(b) Those who by themselves or through their predecessor-in-interest


have been in open, continuous, exclusive, and notorious possession and
occupation of agricultural lands of the public domain under a bona fide
claim of acquisition of ownership for at least thirty years immediately
preceding the filing of the application for confirmation of title except when
prevented by war or force majeure. These shall be conclusively presumed
to have performed all the conditions essential to a Government grant and
shall be entitled to a certificate of title under the provisions of this chapter.

(c) ...

In its Motion for Reconsideration, petitioner contends that the Roman Catholic Bishop of Lucena
(private respondent herein) which is admittedly a corporation sole is disqualified to own and register
its title over the parcels of land involved herein. (Rollo, p. 41)

In its petition it likewise argued that being a juridical entity, private respondent cannot avail of the
benefits of Sec. 48(b) of the public land law which applies to FILIPINO citizens or NATURAL persons.
On the other hand, private respondent in its MEMORANDUM espoused the contrary view.

There is no merit in this petition.

The parties herein do not dispute that since the acquisition of the four (4) lots by the applicant, it has
been in continuous possession and enjoyment thereof, and such possession, together with its
predecessors-in-interest, covering a period of more than 52 years (at least from the date of survey in
1928) with respect to lots 1 and 2, about 62 years with respect to lot 3, all of plan PSU-65686; and
more than 39 years with respect to the fourth parcel described in plan PSU-11 2592 (at least from the
date of the survey in 1940) have been open, public, continuous, peaceful, adverse against the whole
world, and in the concept of owner.
Being disputed before this Court is the matter of the applicability of Art. XIV Sec. 11 of the 1973
Constitution to the case at bar.

Petitioner argues that considering such constitutional prohibition, private respondent is disqualified to
own and register its title to the lots in question. Further, it argues that since the application for
registration was filed only on February 2, 1979, long after the 1973 Constitution took effect on
January 17, 1973, the application for registration and confirmation of title is ineffectual because at the
time it was filed, private corporation had been declared ineligible to acquire alienable lands of the
public domain pursuant to Art. XIV, Sec. 11 of the said constitution. (Rollo, p. 41)

The questioned posed before this Court has been settled in the case of DIRECTOR OF LANDS vs.
Intermediate Appellate Court (146 SCRA 509 [1986]) which reversed the ruling first enunciated in the
1982 case of Manila Electric Co. vs. CASTRO BARTOLOME, (114 SCRA 789 [1982]) imposing the
constitutional ban on public land acquisition by private corporations which ruling was declared
emphatically as res judicata on January 7, 1986 in Director of Lands vs. Hermanos y Hermanas de
Sta. Cruz de Mayo, Inc., (141 SCRA 21 [1986]).<äre||anº•1àw> In said case, (Director of Lands v.
IAC, supra), this Court stated that a determination of the character of the lands at the time of
institution of the registration proceedings must be made. If they were then still part of the public
domain, it must be answered in the negative.

If, on the other hand, they were already private lands, the constitutional prohibition against their
acquisition by private corporation or association obviously does not apply. In affirming the Decision of
the Intermediate Appellate Court in said case, this Court adopted the vigorous dissent of the then
Justice, later Chief Justice Claudio Teehankee, tracing the line of cases beginning with CARINO, 1 in
1909, thru SUSI, 2 in 1925, down to HERICO, 3 in 1980, which developed, affirmed and reaffirmed the
doctrine that open, exclusive and undisputed possession of alienable public land for the period
prescribed by law creates the legal fiction whereby the land, upon completion of the requisite
period ipso jure and without the need of judicial or other sanction, ceases to be public land and
becomes' private property. (DIRECTOR OF LANDS vs. IAC, supra, p. 518).

Nothing can more clearly demonstrate the logical inevitability of considering possession of public land
which is of the character and duration prescribed by statute as the equivalent of an express grant
from the state than the dictim of the statute itself; 4 that the possessor "... shall be conclusively
presumed to have performed all the conditions essential to a government grant and shall be entitled
to a certificate of title ..." No proof being admissable to overcome a conclusive presumption,
confirmation proceedings would, in truth be little more than a formality, at the most limited to
ascertaining whether the possession claimed is of the required character and length of time, and
registration thereunder would not confer title, but simply recognize a title already vested. The
proceedings would not ORIGINALLY convert the land from public to private land, but only confirm
such a conversion already effected by operation of law from the moment the required period of
possession became complete. As was so well put in Carino, "... There are indications that registration
was expected from all, but none sufficient to show that, for want of it, ownership actually gained would
be lost. The effect of the proof, wherever made, was not to confer title, but simply to establish it, as
already conferred by the decree, if not by earlier law. (DIRECTOR OF LANDS vs. IAC, supra, p. 520).

The open, continuous and exclusive possession of the four lots by private respondent can clearly be
gleaned from the following facts on record: Lot 1 and portion of Lot 2 was acquired by purchase in
1928 and 1929, respectively. The remaining portion of lots 2 and 3 was already owned and
possessed by private respondent even prior to the survey of said lots in 1928. In fact, records of
burial of the Roman Catholic Church of Candelaria, Quezon showed that as early as 1919, Lot 3 has
already been utilized by the Roman Catholic Church as its cemetery. That at present, said three lots
are utilized as the Roman Catholic Church of Candelaria, Quezon. That said lots are declared for
taxation purposes in the name of the Roman Catholic Church. The fourth parcel of land was acquired
by donation in 1941 and same lot is utilized as church site.

It must be emphasized that the Court is not here saying that a corporation sole should be treated like
an ordinary private corporation.

In Roman Catholic Apostolic Administration of Davao, Inc. vs. Land Registration Commission, et al.
(L-8451, December 20,1957,102 Phil. 596). We articulated:

In solving the problem thus submitted to our consideration, We can say the following: A
corporation sole is a special form of corporation usually associated with the clergy.
Conceived and introduced into the common law by sheer necessity, this legal creation
which was referred to as "that unhappy freak of English Law" was designed to facilitate
the exercise of the functions of ownership carried on by the clerics for and on behalf of
the church which was regarded as the property owner (See 1 Bouvier's Law Dictionary,
p. 682-683).

A corporation sole consists of one person only, and his successors (who will always be
one at a time), in some particular station, who are incorporated by law in order to give
them some legal capacities and advantages, particulary that of perpetuity, which in their
natural persons they could not have had. In this sense, the King is a sole corporation;
so is a bishop, or deans distinct from their several chapters (Reid vs. Barry, 93 fla. 849,
112 So. 846).

Pertinent to this case is the provision of Sec. 113 Batas Pambansa Blg. 68 which reads as follows:

Sec. 113. Acquisition and alienation of property. — Any corporation sole may purchase
and hold real estate and personal property for its church, charitable, benevolent or
educational purposes, and may receive bequests or gifts for such purposes. Such
corporation may mortgage or sell real property held by it upon obtaining an order for
that purpose from the Court of First Instance of the province where the property is
situated; but before the order is issued, proof must be made to the satisfaction of the
Court that notice of the application for leave to mortgage or sell has been given by
publication or otherwise in such manner and for such time as said court may have
directed, and that it is to the interest of the corporation that leave to mortgage or sell
should be granted. The application for leave to mortgage or sell must be made by
petition, duly verified by the chief archbishop, bishop, priest, minister, rabbi or presiding
elder acting as corporation sole, and may be opposed by any member of the religious
denomination, sect or church represented by the corporation sole: Provided, That in
cases where the rules, regulations and discipline of the religious denomination, sect or
church religious society or order concerned represented by such corporation sole
regulate the method of acquiring, holding, selling and mortgaging real estate and
personal property, such rules, regulations and discipline shall control and the
intervention of the courts shall not be necessary.

There is no doubt that a corporation sole by the nature of its Incorporation is vested with the right to
purchase and hold real estate and personal property. It need not therefore be treated as an ordinary
private corporation because whether or not it be so treated as such, the Constitutional provision
involved will, nevertheless, be not applicable.

In the light of the facts obtaining in this case and the ruling of this Court in Director of Lands vs. IAC,
(supra, 513), the lands subject of this petition were already private property at the time the application
for confirmation of title was filed in 1979. There is therefore no cogent reason to disturb the findings of
the appellate court.

WHEREFORE, the petition is dismissed for lack of merit and the appealed decision and Resolution of
the Intermediate Appellate Court is hereby AFFIRMED.

SO ORDERED.

G.R. No. L-28021 December 15, 1977

JULIAN SANTULAN substituted by his children named PATROCINIO, ADORACION, ARTURO,


CONSTANCIA, and PEPITA, all surnamed SANTULAN and minor grandchildren, JOCELYN,
ROSAURO and ROBERTO, all surnamed SANTULAN assisted by their guardian ad
litem, PATROCINIO SANTULAN petitioners-appellants,
vs.
THE EXECUTIVE SECRETARY, THE SECRETARY OF AGRICULTURE AND NATURAL
RESOURCES, THE DIRECTOR OF LANDS, and ANTONIO LUSIN, substituted by his Heirs
named TEODOSIA BALANZA (widow) and Children LEOPOLDO, ARMANDO. ALFONSO,
EMILIANO, MAGDALENA, ERLINDA and ESTRELLA (ESTER), all surnamed LUSIN, and Heirs
of CAROLINA LUSIN-LUCERO named MANOLITO LUCERO and MARIO LUCERO, respondents-
appellees.

Isidoro Crisostomo for appellants Heirs of Julian Santulan.

Romulo C. Felizmeña for appellees Heirs of Antonio Lusin.

Solicitor General Arturo A. Alafriz Assistant Solicitor General Esmeraldo Umali and Solicitor Conrado
T. Limcaoco for The Executive Secretary, etc.

AQUINO, J.

This case is about the lease of a parcel of foreshore land of the public domain with an area of about
four and one-half hectares located at Barrio Kaingin, Kawit, Cavite abutting on Bacoor Bay and the
Ankaw Creek.

It is protracted controversy that has been pending for more than thirty years between the rival
claimants Julian Santulan plan and Antonio Lusin, who have been succeeded by their heirs.

Santulan claimed that foreshore land was an extension of his land, Lot No. 986 of the Kawit cadastre,
with an area of 17,301 square meters, registered in his name in 1937 under Original Certificate of
Title No. 6 which was issued by virtue of a free patent. The northern boundary of Lot No. 986 is
Bacoor (Manila) Bay (Exh. A). The said foreshore land was allegedly formed by soil deposits
accumulated by the alluvial action of the sea.

On December 5, 1942 Santulan caused the said land to be surveyed. The survey plan was approved
by the Director of Lands in 1944 (Exh. B). On December 29, 1942 Santulan, pursuant to Lands
Administrative Order No. 7-1, filed an application, F.L.A. No. V 562, to lease for five years for
agricultural purposes an area of 36,120 square meters of the said foreshore land (Exh. F).

On that same date, December 29, 1942, Santulan, pursuant to Act No. 3077 and Lands
Administrative Order No. 8-3, filed with the Bureau of Lands an application for a revocable permit to
occupy the said land. He indicated therein that he would use the land for 11 capiz beds and oyster
beds, the planting of bakawan and pagatpat and later to be developed into a fishpond" (Exh. G).

Seven years later, or on December 22, 1949, Santulan filed with the Bureau of Fisheries an
application for an ordinary fishpond permit or lease of the said foreshore land (Special Use Permit,
pp. A. No. 5114, Exh. H).

At the instance of the Director of Fisheries, the Director of Forestry investigated the condition of the
said foreshore land. The latter in his first indorsement dated June 19, 1950 found that it was swampy
"and not an improved fishpond as alleged by Antonio Lusin" and that it is within the disposable areas
for agricultural purposes under the jurisdiction of the Bureau of Lands (Exh. L-1).

The chief of the division of commercial fisheries sent a letter to Lusin dated April 28, 1950 apprising
him that he was reported to have illegally entered the area covered by Santulan's fishpond permit
application and directing him to refrain from introducing improvements, with the warning that court
proceedings would be taken against him (Exh. J).

On January 12, 1951 an attorney, acting for the Director of Lands wrote the following letter to Lusin
advising him to vacate the disputed land and maintain the status quo:

Mr. Antonio Lusin


Caiñgin, Kawit, Cavite

S i r:

We have been informed that the area which is presently controverted by and between
you and Julian Santulan, under the applications noted above, was recently entered
by you and some companion and that you are destroying the dikes and other
improvements previously constructed thereon by said Julian Santulan.

If this information is true, and inasmuch as you are aware that the controversy is still
pending final adjudgment in this Office, is desired that you take proper advice and leave
the area and its existing improvements in status quo in order to avoid possible
confusion of rights which ma delay the final disposition of the area in question.

You are advised further that the acts imputed to you may make you liable to prosecution
and punishment under the law; and that whatever improvements you may make for
yourself in the premises will not legally accrue to your benefit, nor will they serve as
basis for a claim to preferential rights. (Paragraphing supplied, Exh, J-1).

Santulan declared the said foreshore land in his name for tax purposes. Tax Declaration No. 2923,
which took effect in 1948 and which cancelled Tax Declaration No. 13816 also in Santulan's name,
shows that the land was assessed at P460. He paid the realty taxes due on the said land for the
years 1945-46, 1948-55 and 195760 (Exh. C, D and E, el seq.).

On the other hand, Antonio Lusin in 1942 and 1945 (he died in 1962) filed with the Bureau of Lands
applications for a revocable-permit and lease of a foreshore land, respectively, for the purpose of
producing salt on the said land. He claimed that he had been in the continuous and exclusive
possession of the land since 1920, when it was still under water, and that he had used it as a site of
his fish corrals.

He allegedly converted two hectares of the said land into a fishpond. The entire area was enclosed
with mud dikes and provided with a concrete sluice gate and another sluice gate made of wood On
the northern part of the land bordering the bay were bamboo stakes placed at close intervals to serve
as water breakers to protect the mud dikes from being washed away by the action of the sea. Lusin
introduced the alleged improvements from 1951 to 1953.

The 1942 foreshore lease applications of Santulan and Lusin gave rise to Bureau of Lands Conflict
No. 8 (N). The Director of Lands in his decision in that case dated February 1, 1951 found that the
disputed land is foreshore land covered and uncovered by the flow and ebb of the ordinary tides that
it is an extension of Santulan's Lot No. 986 and it was formerly a part of the sea; that Santulan was
the first to enter the land and to make dikes thereon, and that Lusin entered the land later and made
dikes also (Exh. K made a part hereof for reference as Annex A).

The Director ruled that the disputed foreshore land was subject "to reparian rights which may he
invoked by Santulan as owner of the upland in accordance with section 32 of Lands Administrative
Order No. 7-1" (Exh. K). Hence the Director rejected Lusin's application for a foreshore lease and for
a revocable permit and gave due course to Santulan's foreshore lease application.

Lusin filed a motion for reconsideration. The Director in his order of October 19, 1951 denied that
motion. lie found that Lusin was a possessor in bad faith: that it is not true that Lusin had improved
and possessed the said foreshore land for twenty years, that the disputed area is covered by water,
two to three feet deep during ordinary tides and is exposed land after the ebb of the tides, and that
Lusin's alleged possession and improvements could not nullify Santulan's preferential right to lease
the land by reason of his riparian rights. The Director ordered Lusin to vacate the land within sixty
days from notice (Exh. L made a part hereof for reference as Annex B).

Lusin appealed to the Acting Secretary of Agriculture and Natural Resources who in his decision of
October 13, 1952 dismissed the appeal and affirmed the Director's 1951 decision (Exh. M made a
part hereof for reference as Annex C). Lusin's motion for reconsideration was denied in the
Secretary's order of February 28, 1953 (Exh. N made a part hereof for reference as Annex D).

Lusin asked for a reinvestigation of the case. His request was granted. The Department ordered a
reinvestigation on May 12, 1953.

After receipt of the report of reinvestigation, the Undersecretary of Agriculture and Natural Resources,
by authority of the Secretary, in his order of December 14, 1954, reaffirmed the rejection of Lusin's
revocable permit and foreshore lease applications but ordered Santulan to reimburse to Lusin the
appraised value of his improvements (Exh. O made a part hereof for reference as Annex E).

Lusin appealed to the President of the Philippines after his motion for reconsideration was denied in
the Undersecretary's order of May 19, 1955 (Exh. OO made a part hereof for reference as Annex F).

Executive Secretary Juan C. Pajo, by authority of the President, held in his decision of April 10, 1958
that section 32 of Lands Administrative Order No. 7-1 (promulgated by the Secretary of Agriculture
and Natural Resources on April 30, 1936 pursuant to Acts Nos. 2874 and 3038) was "rendered
obsolete" by section 67 of the Public Land Law which took effect on December 1, 1936 (Exh. P made
a part hereof for reference as Annex G).
On the basis of the foregoing ruling and since the record is silent as to whether or not the land in
question has been declared by the President as not necessary for the public service and as open to
disposition (Sec. 61, Public Land Law), the Executive Secretary sustained Lusin's appeal and
reversed the orders of the Director of Lands and the Secretary of Agriculture and Natural Resources
in favor of Santulan. Secretary Pajo decided the case in the alternative as follows:

On the assumption that the land in question has been declared open for disposition and
is not necessary for the public service, this Office directs that an oral bidding for the
leasing thereof to interested parties pursuant to the provisions of Section 67 of
Commonwealth Act .No. 141 be conducted and the contract of lease awarded to the
highest bidder whoever shall be the highest bidder, if other than the appellant, shall be
required to pay to the appellant the appraised value of the improvements introduced by
him on the land to be determined by that Department.

If the land in question has not been so declared, this Office directs that a revocable
permit under Section 68 of Commonwealth Act No. 141 be Id to the appellant requiring
him to pay permit fees since the year 1951.

Accordingly, the orders and decisions of that Department and the Bureau of Lands are
hereby revoked.

Santulan's case was distinguished from that of Gonzalo Monzon whose Lot No. 987 adjoins
Santulan's Lot No. 986. Executive Secretary Fred Ruiz Castro (now Chief Justice) in his decision of
May 10, 1954 upheld the preferential right of Monzon to lease the foreshore land north of his lot,
which foreshore land is adjacent to the foreshore land now in dispute in this case (Exh. Q made a part
hereof for reference as Annex H).

Santulan's motion for reconsideration was denied in the letter of the Acting Executive Secretary dated
August 20, 1959 (Exh. W).

On October 22, 1959 Santulan filed in the Court of First Instance of Cavite a petition for certiorari
wherein he alleged that the Executive Secretary committed a grave abuse of discretion in
misinterpreting certain provisions of Act No. 2874, Commonwealth Act No. 141, and Lands
Administrative Order No. 7-1.

In the lower court the parties agreed that the case Involves only a question of law. On August 18.
1961 the lower court dismissed the petition and affirmed the Executive Secretary's decision. Santulan
appealed to the Court of Appeals which in its resolution of July 21, 1967 elevated the record to this
Court on the ground that Santulan in his brief raised only the legal questions of whether the Public
Land Law repealed section 32 of Lands Administrative Order No. 7 1 and whether the Executive
Secretary's decision is "legally sound and correct" (CA-G. R. No. 30708-R).

It should be emphasized that. as found by tile investigators of the Bureau of Lands, Santulan was the
prior possessor of the foreshore land in question. lie had it surveyed in 1942. The survey plan Psu-
115357) was approved by the Director of Lands in 1944. Santulan paid the realty taxes on that land .

It should further be underscored that the regulations pie him a preferential right to lease the land as a
riparian owner. Lands Administrative Order No. 7-1 dated April 30. 1936. which was issued by the
Secretary of Agriculture and Natural Resources upon the recommendation of the Director of Lands for
the disposition of alienable lands of the public domain, provides:
32. Preference of the Reparian Owner — The owner of the property adjoining foreshore
lands, marshy lands or lands covered with water bordering upon shores or banks of
navigable lakes or rivers, shall be given preference to apply for such lands adjoining his
property as may not be needed for the public service, subject to the laws and
regulations governing lands of this nature, provided that he applies therefor within sixty
(60) days from the date he receives a communication from the Director of Lands
advising him of his preferential right.

Paragraph 32 quoted above is a substantial copy of paragraph 4 of Lands Administrative Order No. 8-
3 dated April 20, 1936, which was promulgated by the Secretary of Agriculture and Natural
Resources upon the recommendation of the Director of Lands for issuance of temporary permits of
occupation and use of agricultural lands of the public domain.

The word "riparian" in paragraphs 32 and 4 of the departmental regulations is used in a broad sense
as referring to any property having a water frontage (Shepard's Point Land Co. vs. Atlantic Hotel, 44
S. E. 39, 45, 132 N. C. 517, 65 C. J. S. 143, note 84). Strictly speaking, "riparian" refers to rivers. A
riparian owner is a person who owns land situated on the bank of a river.

But in paragraphs 32 and 4, the term "riparian owner" embraces not only the owners of lands on the
banks of rivers but also the littoral owners, meaning the owners of lands bordering the shore of the
sea or lake or other tidal waters. The littoral is the coastal region including both the land along the
coast and the water near the coast or the shore zone between the high and low watermarks.

Therefore, on the basis of paragraphs 32 and 4 of the said administrative regulations, Santulan or his
heirs Should be allowed to leased or occupy the said foreshore land.

But the Executive Secretary ruled that paragraph 32 was rendered obsolete by Commonwealth Act
No. 141 or, as held by the trial court, Lands Administrative Order No. 7-1 was repealed by the Public
Land Law. Is that conclusion correct? We hold that it is wrong.

It is true that Lands Administrative Orders Nos. 7-1 and 8-3 were issued when the 1919 Public Land
Act was in force or before the present Public Land Law took effect on December 1, 1936. But that
circumstance would not necessarily mean that the said departmental regulations are not good under
the 1936 Public Land Law.

In rationalizing the alleged repeal of paragraph 32, the Executive Secretary cited the following
provisions of Act No. 2874, the 1919 Public Land Act (15 Public Land laws 24):

SEC. 64. The lease or sale shall be adjudicated to the highest bidder; and if there is no
bidder besides the applicant, it shall be adjudicated to him. The provisions of section
twenty-seven of this Act shall be applied wherever applicable. If all or part of the lots
remain unleased or unsold the Director of Lands Shall from time to time announce in
the Offcial Gazette or otherwise the lease or sale of those lots if necessary . (Section 27
refers to sealed bidding).

The Executive Secretary held that the above-quoted section 64 was by the for provisions of on wealth
Act No. 141 which took effect on December 1, 1936:

SEC. 67. The lease or sale shall be made through oral bidding-, and ajudication shall be
made to the highest bidder. However, where m applicant has made improvements on
the land by virtue of a permit issued to him by competent authority, the sale or lease
shall be made by sealed bidding as prescribed in section twenty-six of this Act, the
provisions of which shall be applied wherever applicable. If all or Dart of the lots remain
unleased or unsold. the Director of Lands shall from time to time announce in
the Official Gazzate, or in any other newspapers of general circulation, the lease or sale
of those lots, if necessary. (Section 26, like section 27 of Act No. 2874, refers to sealed
bidding).

The Executive Secretary noted that under section 64 of Act No. 2874 sealed bidding was the general
rule of procedure in an award of a lease of foreshore land and that the t is entitled to equal the bid of
the highest bidder. On the other hand, under 67, oral bidding is the general rule.

Hence, the Executive Secretary assumed that, while under section 64 of the 1919 old Public Land
Act, the fact that the applicant has a preferential right to lease foreshore land was a crucial factor it is
thus under section 67 of the 1936 Public Land Law because in oral bidding the appellant is not
entitled to equal the bid of the highest bidder.

The Executive Secretary concluded that, because the preferential right of the applicant to lease
foreshore land was immaterial under 67 of the present Public Land Law, paragraph 32 of Lands
Administrative Order No. 7-1, which gives such preference. had become "idle and useless".

That conclusion is wrong because it is based on the erroneous hypothesis that section 64 of the 1919
Public Land Act is different from section 67 of the 1936 Public Land Law. They are not different. The
truth is that section 64 was amended by Act No. 3517 which took effect on February 4, 1919 (24
Public Laws 416). Section 64, as thus amended, is substantially the same as section 67 of the 1936
Public Land Law.

That fact was overlooked by the Executive Secretary. Hence, his conclusion, that paragraph 32 of
Lands Administrative Order No. 71 was repealed or rendered obsolete by section 67 of the present
Public Land Law, is wrong because its premise is wrong.

In other words, paragraph 32 of Lands Administrative Order No. 7-1, issued on April 30, 1936, was
promulgated under section 64 of the old Public Land Law, as amended. And since the amended
section 64 was substantially reproduced in section 67 of the 1936 Public Land Law, it is glaringly
incorrect to say that section 67 rendered obsolete the said paragraph 32. Paragraph 32 is still in force
and is good under the existing Public Land Law.

The foregoing discussion reveals that the Executive Summary's rationalization of the alleged repeal of
paragraph 32 of Lands Administrative Order No. 7-1 (identical to paragraph 4 of Lands Administrative
Order No. 8-3) is not only deficient in clarity and cogency but is predicated on the false assumption
that section 64 of the 1919 Public Land Act is different from section 67 of the present Public Land
Law. Consequently, the aforementioned decision of Executive Secretary Juan C. Pajo under review
bas to be set aside.

This case is governed by the precedent established in the case of Gonzalo Monzon, which, as
already noted, is similar to this cm since the foreshore land involved in the Monzon case is adjacent
to the foreshore land involved in this case.

In the Monzon case, the Office of the President, applying the oft-cited paragraph 32 of Lands
Administrative Order No. 7-1 held that Monzon, the littoral owner of the registered land abutting upon
the foreshore land, has the preferential right to lease the foreshore land,

The location of the lots of Santulan and Monzon and the foreshore lands abutting thereon is shown in
the following sketch bawd on the plan, Psu-115357 (Exh. B):
Manila Bay or Bacoor Bay

Disputed Area

Psu-1 15357 Psu- 1 15358

Foreshore land Forshore land

claimed by leased to

Julian Santulan Gonzalo Monzon

and

Antonio Lusin

Lot No. 986 Lot no. 987

Belonging to Belonging to

Julian Santulan Gonzalo Monzon

Considering that the foreshore land abutting upon Santolan's lot is in the same situation as the
foreshore land abutting upon Monzon's lot, there is no reason why Santulan should not enjoy, with
respect to the disputed foreshore land, the rights given to Monzon over the foreshore land adjacent to
his lot.

Now, then, is there any justification for giving to the littoral owner the preferential right to lease the
foreshore land abutting on his land?

That rule in paragraph 32 is in consonance with article 4 of the Spanish Law of Waters of 1866 which
provides that, while lands added to the shores by accretions and alluvial deposits caused by the
action of the sea form part of the public domain, such lands, "when they are no longer washed by the
waters of the sea and are not necessary for purposes of public utility, or for the establishment of
special industries, or for the coast guard service", shall be declared by the Government "to be the
property of the owner of the estates adjacent thereto and as increment thereof" (cited in Ignacio vs.
Director of Lands, 108 Phil. 335, 338).

In other words, article 4 recognizes the preferential right of the littoral owner (riparian according to
paragraph 32) to the foreshore land formed by accretions or alluvial deposits due to the action of the
sea (Ker & Co. vs. Cauden 6 Phil. 732, 736, 223 U.S. 268, 56 L. Ed. 432, 435; Jover vs. Insular
Government, 10 Phil. 522, 40 Phil. 1094, 1100, 221 U.S. 623, 55 L. Ed. 884).

The reason for that preferential right is the same as the justification for giving accretions to the
riparian owner, which is that accretion compensates the riparian owner for the diminutions which his
land suffers by reason of the destructive force of the waters (Cortes vs. City of Manila, 10 Phil. 567).
So, in the case of littoral lands, he who loses by the encroachments of the sea should gain by its
recession (Banks vs. Ogden 2 Wall. 57, 67, 17 L. Ed. 818, 821).

That preferential right is recognized in American jurisprudence where the rule is that the owner of the
land adjacent to navigable waters has certain riparian or littoral rights of a proprietary nature not
possessed by the general public which rights are incident to the ownership of the banks or the
uplands: riparian as respects the waters of a river and littoral as to sea waters or the waters of a lake
(65 C.J. S. 143-145).

It may be mentioned that the Director of Lands stated in his manifestation of October 26, 1977 that
Lands Administrative Orders Nos. 7-1 and 8-3 are still in force and have not been superseded by any
later regulations and that the directive of the President of the Philippines to the Director of Lands
dated May 24, 1966, stopping the grant of foreshore leases all along Manila Bay, towards Cavite and
Bataan, has not rendered the instant case moot and academic "because the foreshore lease
application involved is pending award."

In view of the foregoing considerations, the trial court's decision and the decision of the Executive
Secretary dated April 10, 1958 are reversed and set aside and the order of the Undersecretary of
Agriculture and Natural Resources dated December 14, 1954 and the orders of the Director of Lands
dated February I and October 19, 1951 are affirmed.

The lease application of Julian Santulan mentioned in the order of February 1, 1951 should be
recorded in the names of his heirs and the obligation to make reimbursement mentioned in the
dispositive part of the Undersecretary's order should now devolve upon the heirs of Santolan. The
reimbursement should be made to the heirs of the late Antonio Lusin The obligation to vacate the
disputed land, as required in the Director's order of October 19, 1951 devolves upon the heirs of
Lusin Costs in both instances against respondent heirs of Lusin (As amended by Resolution of
February 17, 1977.

SO ORDERED.

G.R. No. 133250 July 9, 2002

FRANCISCO I. CHAVEZ, petitioner,


vs.
PUBLIC ESTATES AUTHORITY and AMARI COASTAL BAY DEVELOPMENT
CORPORATION, respondents.

CARPIO, J.:

This is an original Petition for Mandamus with prayer for a writ of preliminary injunction and a
temporary restraining order. The petition seeks to compel the Public Estates Authority ("PEA" for
brevity) to disclose all facts on PEA's then on-going renegotiations with Amari Coastal Bay and
Development Corporation ("AMARI" for brevity) to reclaim portions of Manila Bay. The petition further
seeks to enjoin PEA from signing a new agreement with AMARI involving such reclamation.

The Facts

On November 20, 1973, the government, through the Commissioner of Public Highways, signed a
contract with the Construction and Development Corporation of the Philippines ("CDCP" for brevity) to
reclaim certain foreshore and offshore areas of Manila Bay. The contract also included the
construction of Phases I and II of the Manila-Cavite Coastal Road. CDCP obligated itself to carry out
all the works in consideration of fifty percent of the total reclaimed land.

On February 4, 1977, then President Ferdinand E. Marcos issued Presidential Decree No. 1084
creating PEA. PD No. 1084 tasked PEA "to reclaim land, including foreshore and submerged areas,"
and "to develop, improve, acquire, x x x lease and sell any and all kinds of lands." 1 On the same date,
then President Marcos issued Presidential Decree No. 1085 transferring to PEA the "lands reclaimed
in the foreshore and offshore of the Manila Bay"2 under the Manila-Cavite Coastal Road and
Reclamation Project (MCCRRP).

On December 29, 1981, then President Marcos issued a memorandum directing PEA to amend its
contract with CDCP, so that "[A]ll future works in MCCRRP x x x shall be funded and owned by PEA."
Accordingly, PEA and CDCP executed a Memorandum of Agreement dated December 29, 1981,
which stated:

"(i) CDCP shall undertake all reclamation, construction, and such other works in the MCCRRP
as may be agreed upon by the parties, to be paid according to progress of works on a unit
price/lump sum basis for items of work to be agreed upon, subject to price escalation, retention
and other terms and conditions provided for in Presidential Decree No. 1594. All the financing
required for such works shall be provided by PEA.

xxx

(iii) x x x CDCP shall give up all its development rights and hereby agrees to cede and transfer
in favor of PEA, all of the rights, title, interest and participation of CDCP in and to all the areas
of land reclaimed by CDCP in the MCCRRP as of December 30, 1981 which have not yet
been sold, transferred or otherwise disposed of by CDCP as of said date, which areas consist
of approximately Ninety-Nine Thousand Four Hundred Seventy Three (99,473) square meters
in the Financial Center Area covered by land pledge No. 5 and approximately Three Million
Three Hundred Eighty Two Thousand Eight Hundred Eighty Eight (3,382,888) square meters
of reclaimed areas at varying elevations above Mean Low Water Level located outside the
Financial Center Area and the First Neighborhood Unit."3

On January 19, 1988, then President Corazon C. Aquino issued Special Patent No. 3517, granting
and transferring to PEA "the parcels of land so reclaimed under the Manila-Cavite Coastal Road and
Reclamation Project (MCCRRP) containing a total area of one million nine hundred fifteen thousand
eight hundred ninety four (1,915,894) square meters." Subsequently, on April 9, 1988, the Register of
Deeds of the Municipality of Parañaque issued Transfer Certificates of Title Nos. 7309, 7311, and
7312, in the name of PEA, covering the three reclaimed islands known as the "Freedom Islands"
located at the southern portion of the Manila-Cavite Coastal Road, Parañaque City. The Freedom
Islands have a total land area of One Million Five Hundred Seventy Eight Thousand Four Hundred
and Forty One (1,578,441) square meters or 157.841 hectares.

On April 25, 1995, PEA entered into a Joint Venture Agreement ("JVA" for brevity) with AMARI, a
private corporation, to develop the Freedom Islands. The JVA also required the reclamation of an
additional 250 hectares of submerged areas surrounding these islands to complete the configuration
in the Master Development Plan of the Southern Reclamation Project-MCCRRP. PEA and AMARI
entered into the JVA through negotiation without public bidding. 4 On April 28, 1995, the Board of
Directors of PEA, in its Resolution No. 1245, confirmed the JVA.5On June 8, 1995, then President
Fidel V. Ramos, through then Executive Secretary Ruben Torres, approved the JVA.6

On November 29, 1996, then Senate President Ernesto Maceda delivered a privilege speech in the
Senate and denounced the JVA as the "grandmother of all scams." As a result, the Senate
Committee on Government Corporations and Public Enterprises, and the Committee on
Accountability of Public Officers and Investigations, conducted a joint investigation. The Senate
Committees reported the results of their investigation in Senate Committee Report No. 560 dated
September 16, 1997.7 Among the conclusions of their report are: (1) the reclaimed lands PEA seeks
to transfer to AMARI under the JVA are lands of the public domain which the government has not
classified as alienable lands and therefore PEA cannot alienate these lands; (2) the certificates of title
covering the Freedom Islands are thus void, and (3) the JVA itself is illegal.

On December 5, 1997, then President Fidel V. Ramos issued Presidential Administrative Order No.
365 creating a Legal Task Force to conduct a study on the legality of the JVA in view of Senate
Committee Report No. 560. The members of the Legal Task Force were the Secretary of Justice, 8 the
Chief Presidential Legal Counsel,9 and the Government Corporate Counsel.10 The Legal Task Force
upheld the legality of the JVA, contrary to the conclusions reached by the Senate Committees. 11

On April 4 and 5, 1998, the Philippine Daily Inquirer and Today published reports that there were on-
going renegotiations between PEA and AMARI under an order issued by then President Fidel V.
Ramos. According to these reports, PEA Director Nestor Kalaw, PEA Chairman Arsenio Yulo and
retired Navy Officer Sergio Cruz composed the negotiating panel of PEA.

On April 13, 1998, Antonio M. Zulueta filed before the Court a Petition for Prohibition with Application
for the Issuance of a Temporary Restraining Order and Preliminary Injunction docketed as G.R. No.
132994 seeking to nullify the JVA. The Court dismissed the petition "for unwarranted disregard of
judicial hierarchy, without prejudice to the refiling of the case before the proper court." 12

On April 27, 1998, petitioner Frank I. Chavez ("Petitioner" for brevity) as a taxpayer, filed the
instant Petition for Mandamus with Prayer for the Issuance of a Writ of Preliminary Injunction and
Temporary Restraining Order. Petitioner contends the government stands to lose billions of pesos in
the sale by PEA of the reclaimed lands to AMARI. Petitioner prays that PEA publicly disclose the
terms of any renegotiation of the JVA, invoking Section 28, Article II, and Section 7, Article III, of the
1987 Constitution on the right of the people to information on matters of public concern. Petitioner
assails the sale to AMARI of lands of the public domain as a blatant violation of Section 3, Article XII
of the 1987 Constitution prohibiting the sale of alienable lands of the public domain to private
corporations. Finally, petitioner asserts that he seeks to enjoin the loss of billions of pesos in
properties of the State that are of public dominion.

After several motions for extension of time,13 PEA and AMARI filed their Comments on October 19,
1998 and June 25, 1998, respectively. Meanwhile, on December 28, 1998, petitioner filed an
Omnibus Motion: (a) to require PEA to submit the terms of the renegotiated PEA-AMARI contract; (b)
for issuance of a temporary restraining order; and (c) to set the case for hearing on oral argument.
Petitioner filed a Reiterative Motion for Issuance of a TRO dated May 26, 1999, which the Court
denied in a Resolution dated June 22, 1999.

In a Resolution dated March 23, 1999, the Court gave due course to the petition and required the
parties to file their respective memoranda.

On March 30, 1999, PEA and AMARI signed the Amended Joint Venture Agreement ("Amended
JVA," for brevity). On May 28, 1999, the Office of the President under the administration of then
President Joseph E. Estrada approved the Amended JVA.

Due to the approval of the Amended JVA by the Office of the President, petitioner now prays that on
"constitutional and statutory grounds the renegotiated contract be declared null and void." 14

The Issues

The issues raised by petitioner, PEA15 and AMARI16 are as follows:


I. WHETHER THE PRINCIPAL RELIEFS PRAYED FOR IN THE PETITION ARE MOOT AND
ACADEMIC BECAUSE OF SUBSEQUENT EVENTS;

II. WHETHER THE PETITION MERITS DISMISSAL FOR FAILING TO OBSERVE THE
PRINCIPLE GOVERNING THE HIERARCHY OF COURTS;

III. WHETHER THE PETITION MERITS DISMISSAL FOR NON-EXHAUSTION OF


ADMINISTRATIVE REMEDIES;

IV. WHETHER PETITIONER HAS LOCUS STANDI TO BRING THIS SUIT;

V. WHETHER THE CONSTITUTIONAL RIGHT TO INFORMATION INCLUDES OFFICIAL


INFORMATION ON ON-GOING NEGOTIATIONS BEFORE A FINAL AGREEMENT;

VI. WHETHER THE STIPULATIONS IN THE AMENDED JOINT VENTURE AGREEMENT


FOR THE TRANSFER TO AMARI OF CERTAIN LANDS, RECLAIMED AND STILL TO BE
RECLAIMED, VIOLATE THE 1987 CONSTITUTION; AND

VII. WHETHER THE COURT IS THE PROPER FORUM FOR RAISING THE ISSUE OF
WHETHER THE AMENDED JOINT VENTURE AGREEMENT IS GROSSLY
DISADVANTAGEOUS TO THE GOVERNMENT.

The Court's Ruling

First issue: whether the principal reliefs prayed for in the petition are moot and academic
because of subsequent events.

The petition prays that PEA publicly disclose the "terms and conditions of the on-going negotiations
for a new agreement." The petition also prays that the Court enjoin PEA from "privately entering into,
perfecting and/or executing any new agreement with AMARI."

PEA and AMARI claim the petition is now moot and academic because AMARI furnished petitioner on
June 21, 1999 a copy of the signed Amended JVA containing the terms and conditions agreed upon
in the renegotiations. Thus, PEA has satisfied petitioner's prayer for a public disclosure of the
renegotiations. Likewise, petitioner's prayer to enjoin the signing of the Amended JVA is now moot
because PEA and AMARI have already signed the Amended JVA on March 30, 1999. Moreover, the
Office of the President has approved the Amended JVA on May 28, 1999.

Petitioner counters that PEA and AMARI cannot avoid the constitutional issue by simply fast-tracking
the signing and approval of the Amended JVA before the Court could act on the issue. Presidential
approval does not resolve the constitutional issue or remove it from the ambit of judicial review.

We rule that the signing of the Amended JVA by PEA and AMARI and its approval by the President
cannot operate to moot the petition and divest the Court of its jurisdiction. PEA and AMARI have still
to implement the Amended JVA. The prayer to enjoin the signing of the Amended JVA on
constitutional grounds necessarily includes preventing its implementation if in the meantime PEA and
AMARI have signed one in violation of the Constitution. Petitioner's principal basis in assailing the
renegotiation of the JVA is its violation of Section 3, Article XII of the Constitution, which prohibits the
government from alienating lands of the public domain to private corporations. If the Amended JVA
indeed violates the Constitution, it is the duty of the Court to enjoin its implementation, and if already
implemented, to annul the effects of such unconstitutional contract.
The Amended JVA is not an ordinary commercial contract but one which seeks to transfer title and
ownership to 367.5 hectares of reclaimed lands and submerged areas of Manila Bay to a
single private corporation. It now becomes more compelling for the Court to resolve the issue to
insure the government itself does not violate a provision of the Constitution intended to safeguard the
national patrimony. Supervening events, whether intended or accidental, cannot prevent the Court
from rendering a decision if there is a grave violation of the Constitution. In the instant case, if the
Amended JVA runs counter to the Constitution, the Court can still prevent the transfer of title and
ownership of alienable lands of the public domain in the name of AMARI. Even in cases where
supervening events had made the cases moot, the Court did not hesitate to resolve the legal or
constitutional issues raised to formulate controlling principles to guide the bench, bar, and the
public.17

Also, the instant petition is a case of first impression. All previous decisions of the Court involving
Section 3, Article XII of the 1987 Constitution, or its counterpart provision in the 1973
Constitution,18 covered agricultural lands sold to private corporations which acquired the lands from
private parties. The transferors of the private corporations claimed or could claim the right to judicial
confirmation of their imperfect titles19 under Title II of Commonwealth Act. 141 ("CA No. 141" for
brevity). In the instant case, AMARI seeks to acquire from PEA, a public corporation, reclaimed lands
and submerged areas for non-agricultural purposes by purchase under PD No. 1084 (charter of
PEA) and Title III of CA No. 141. Certain undertakings by AMARI under the Amended JVA constitute
the consideration for the purchase. Neither AMARI nor PEA can claim judicial confirmation of their
titles because the lands covered by the Amended JVA are newly reclaimed or still to be reclaimed.
Judicial confirmation of imperfect title requires open, continuous, exclusive and notorious occupation
of agricultural lands of the public domain for at least thirty years since June 12, 1945 or earlier.
Besides, the deadline for filing applications for judicial confirmation of imperfect title expired on
December 31, 1987.20

Lastly, there is a need to resolve immediately the constitutional issue raised in this petition because of
the possible transfer at any time by PEA to AMARI of title and ownership to portions of the reclaimed
lands. Under the Amended JVA, PEA is obligated to transfer to AMARI the latter's seventy percent
proportionate share in the reclaimed areas as the reclamation progresses. The Amended JVA even
allows AMARI to mortgage at any time the entire reclaimed area to raise financing for the reclamation
project.21

Second issue: whether the petition merits dismissal for failing to observe the principle
governing the hierarchy of courts.

PEA and AMARI claim petitioner ignored the judicial hierarchy by seeking relief directly from the
Court. The principle of hierarchy of courts applies generally to cases involving factual questions. As it
is not a trier of facts, the Court cannot entertain cases involving factual issues. The instant case,
however, raises constitutional issues of transcendental importance to the public.22 The Court can
resolve this case without determining any factual issue related to the case. Also, the instant case is a
petition for mandamus which falls under the original jurisdiction of the Court under Section 5, Article
VIII of the Constitution. We resolve to exercise primary jurisdiction over the instant case.

Third issue: whether the petition merits dismissal for non-exhaustion of administrative
remedies.

PEA faults petitioner for seeking judicial intervention in compelling PEA to disclose publicly certain
information without first asking PEA the needed information. PEA claims petitioner's direct resort to
the Court violates the principle of exhaustion of administrative remedies. It also violates the rule that
mandamus may issue only if there is no other plain, speedy and adequate remedy in the ordinary
course of law.

PEA distinguishes the instant case from Tañada v. Tuvera23 where the Court granted the petition for
mandamus even if the petitioners there did not initially demand from the Office of the President the
publication of the presidential decrees. PEA points out that in Tañada, the Executive Department had
an affirmative statutory duty under Article 2 of the Civil Code24 and Section 1 of Commonwealth Act
No. 63825 to publish the presidential decrees. There was, therefore, no need for the petitioners in
Tañada to make an initial demand from the Office of the President. In the instant case, PEA claims it
has no affirmative statutory duty to disclose publicly information about its renegotiation of the JVA.
Thus, PEA asserts that the Court must apply the principle of exhaustion of administrative remedies to
the instant case in view of the failure of petitioner here to demand initially from PEA the needed
information.

The original JVA sought to dispose to AMARI public lands held by PEA, a government corporation.
Under Section 79 of the Government Auditing Code,26 the disposition of government lands to private
parties requires public bidding. PEA was under a positive legal duty to disclose to the public the
terms and conditions for the sale of its lands. The law obligated PEA to make this public
disclosure even without demand from petitioner or from anyone. PEA failed to make this public
disclosure because the original JVA, like the Amended JVA, was the result of a negotiated contract,
not of a public bidding. Considering that PEA had an affirmative statutory duty to make the public
disclosure, and was even in breach of this legal duty, petitioner had the right to seek direct judicial
intervention.

Moreover, and this alone is determinative of this issue, the principle of exhaustion of administrative
remedies does not apply when the issue involved is a purely legal or constitutional question. 27 The
principal issue in the instant case is the capacity of AMARI to acquire lands held by PEA in view of
the constitutional ban prohibiting the alienation of lands of the public domain to private corporations.
We rule that the principle of exhaustion of administrative remedies does not apply in the instant case.

Fourth issue: whether petitioner has locus standi to bring this suit

PEA argues that petitioner has no standing to institute mandamus proceedings to enforce his
constitutional right to information without a showing that PEA refused to perform an affirmative duty
imposed on PEA by the Constitution. PEA also claims that petitioner has not shown that he will suffer
any concrete injury because of the signing or implementation of the Amended JVA. Thus, there is no
actual controversy requiring the exercise of the power of judicial review.

The petitioner has standing to bring this taxpayer's suit because the petition seeks to compel PEA to
comply with its constitutional duties. There are two constitutional issues involved here. First is the
right of citizens to information on matters of public concern. Second is the application of a
constitutional provision intended to insure the equitable distribution of alienable lands of the public
domain among Filipino citizens. The thrust of the first issue is to compel PEA to disclose publicly
information on the sale of government lands worth billions of pesos, information which the
Constitution and statutory law mandate PEA to disclose. The thrust of the second issue is to prevent
PEA from alienating hundreds of hectares of alienable lands of the public domain in violation of the
Constitution, compelling PEA to comply with a constitutional duty to the nation.

Moreover, the petition raises matters of transcendental importance to the public. In Chavez v.
PCGG,28 the Court upheld the right of a citizen to bring a taxpayer's suit on matters of transcendental
importance to the public, thus -
"Besides, petitioner emphasizes, the matter of recovering the ill-gotten wealth of the Marcoses
is an issue of 'transcendental importance to the public.' He asserts that ordinary taxpayers
have a right to initiate and prosecute actions questioning the validity of acts or orders of
government agencies or instrumentalities, if the issues raised are of 'paramount public
interest,' and if they 'immediately affect the social, economic and moral well being of the
people.'

Moreover, the mere fact that he is a citizen satisfies the requirement of personal interest, when
the proceeding involves the assertion of a public right, such as in this case. He invokes several
decisions of this Court which have set aside the procedural matter of locus standi, when the
subject of the case involved public interest.

xxx

In Tañada v. Tuvera, the Court asserted that when the issue concerns a public right and the
object of mandamus is to obtain the enforcement of a public duty, the people are regarded as
the real parties in interest; and because it is sufficient that petitioner is a citizen and as such is
interested in the execution of the laws, he need not show that he has any legal or special
interest in the result of the action. In the aforesaid case, the petitioners sought to enforce their
right to be informed on matters of public concern, a right then recognized in Section 6, Article
IV of the 1973 Constitution, in connection with the rule that laws in order to be valid and
enforceable must be published in the Official Gazette or otherwise effectively promulgated. In
ruling for the petitioners' legal standing, the Court declared that the right they sought to be
enforced 'is a public right recognized by no less than the fundamental law of the land.'

Legaspi v. Civil Service Commission, while reiterating Tañada, further declared that 'when a
mandamus proceeding involves the assertion of a public right, the requirement of personal
interest is satisfied by the mere fact that petitioner is a citizen and, therefore, part of the
general 'public' which possesses the right.'

Further, in Albano v. Reyes, we said that while expenditure of public funds may not have been
involved under the questioned contract for the development, management and operation of the
Manila International Container Terminal, 'public interest [was] definitely involved considering
the important role [of the subject contract] . . . in the economic development of the country and
the magnitude of the financial consideration involved.' We concluded that, as a consequence,
the disclosure provision in the Constitution would constitute sufficient authority for upholding
the petitioner's standing.

Similarly, the instant petition is anchored on the right of the people to information and access
to official records, documents and papers — a right guaranteed under Section 7, Article III of
the 1987 Constitution. Petitioner, a former solicitor general, is a Filipino citizen. Because of the
satisfaction of the two basic requisites laid down by decisional law to sustain petitioner's legal
standing, i.e. (1) the enforcement of a public right (2) espoused by a Filipino citizen, we rule
that the petition at bar should be allowed."

We rule that since the instant petition, brought by a citizen, involves the enforcement of constitutional
rights - to information and to the equitable diffusion of natural resources - matters of transcendental
public importance, the petitioner has the requisite locus standi.

Fifth issue: whether the constitutional right to information includes official information on on-
going negotiations before a final agreement.
Section 7, Article III of the Constitution explains the people's right to information on matters of public
concern in this manner:

"Sec. 7. The right of the people to information on matters of public concern shall be
recognized. Access to official records, and to documents, and papers pertaining to
official acts, transactions, or decisions, as well as to government research data used as
basis for policy development, shall be afforded the citizen, subject to such limitations as may
be provided by law." (Emphasis supplied)

The State policy of full transparency in all transactions involving public interest reinforces the people's
right to information on matters of public concern. This State policy is expressed in Section 28, Article
II of the Constitution, thus:

"Sec. 28. Subject to reasonable conditions prescribed by law, the State adopts and implements
a policy of full public disclosure of all its transactions involving public interest."
(Emphasis supplied)

These twin provisions of the Constitution seek to promote transparency in policy-making and in the
operations of the government, as well as provide the people sufficient information to exercise
effectively other constitutional rights. These twin provisions are essential to the exercise of freedom of
expression. If the government does not disclose its official acts, transactions and decisions to
citizens, whatever citizens say, even if expressed without any restraint, will be speculative and
amount to nothing. These twin provisions are also essential to hold public officials "at all times x x x
accountable to the people,"29 for unless citizens have the proper information, they cannot hold public
officials accountable for anything. Armed with the right information, citizens can participate in public
discussions leading to the formulation of government policies and their effective implementation. An
informed citizenry is essential to the existence and proper functioning of any democracy. As
explained by the Court in Valmonte v. Belmonte, Jr.30 –

"An essential element of these freedoms is to keep open a continuing dialogue or process of
communication between the government and the people. It is in the interest of the State that
the channels for free political discussion be maintained to the end that the government may
perceive and be responsive to the people's will. Yet, this open dialogue can be effective only to
the extent that the citizenry is informed and thus able to formulate its will intelligently. Only
when the participants in the discussion are aware of the issues and have access to information
relating thereto can such bear fruit."

PEA asserts, citing Chavez v. PCGG,31 that in cases of on-going negotiations the right to information
is limited to "definite propositions of the government." PEA maintains the right does not include
access to "intra-agency or inter-agency recommendations or communications during the stage when
common assertions are still in the process of being formulated or are in the 'exploratory stage'."

Also, AMARI contends that petitioner cannot invoke the right at the pre-decisional stage or before the
closing of the transaction. To support its contention, AMARI cites the following discussion in the 1986
Constitutional Commission:

"Mr. Suarez. And when we say 'transactions' which should be distinguished from contracts,
agreements, or treaties or whatever, does the Gentleman refer to the steps leading to the
consummation of the contract, or does he refer to the contract itself?
Mr. Ople: The 'transactions' used here, I suppose is generic and therefore, it can cover
both steps leading to a contract and already a consummated contract, Mr. Presiding
Officer.

Mr. Suarez: This contemplates inclusion of negotiations leading to the consummation of


the transaction.

Mr. Ople: Yes, subject only to reasonable safeguards on the national interest.

Mr. Suarez: Thank you."32 (Emphasis supplied)

AMARI argues there must first be a consummated contract before petitioner can invoke the right.
Requiring government officials to reveal their deliberations at the pre-decisional stage will degrade
the quality of decision-making in government agencies. Government officials will hesitate to express
their real sentiments during deliberations if there is immediate public dissemination of their
discussions, putting them under all kinds of pressure before they decide.

We must first distinguish between information the law on public bidding requires PEA to disclose
publicly, and information the constitutional right to information requires PEA to release to the public.
Before the consummation of the contract, PEA must, on its own and without demand from anyone,
disclose to the public matters relating to the disposition of its property. These include the size,
location, technical description and nature of the property being disposed of, the terms and conditions
of the disposition, the parties qualified to bid, the minimum price and similar information. PEA must
prepare all these data and disclose them to the public at the start of the disposition process, long
before the consummation of the contract, because the Government Auditing Code requires public
bidding. If PEA fails to make this disclosure, any citizen can demand from PEA this information at
any time during the bidding process.

Information, however, on on-going evaluation or review of bids or proposals being undertaken by


the bidding or review committee is not immediately accessible under the right to information. While
the evaluation or review is still on-going, there are no "official acts, transactions, or decisions" on the
bids or proposals. However, once the committee makes its official recommendation, there arises
a "definite proposition" on the part of the government. From this moment, the public's right to
information attaches, and any citizen can access all the non-proprietary information leading to such
definite proposition. In Chavez v. PCGG,33 the Court ruled as follows:

"Considering the intent of the framers of the Constitution, we believe that it is incumbent upon
the PCGG and its officers, as well as other government representatives, to disclose sufficient
public information on any proposed settlement they have decided to take up with the
ostensible owners and holders of ill-gotten wealth. Such information, though, must pertain
to definite propositions of the government, not necessarily to intra-agency or inter-agency
recommendations or communications during the stage when common assertions are still in the
process of being formulated or are in the "exploratory" stage. There is need, of course, to
observe the same restrictions on disclosure of information in general, as discussed earlier –
such as on matters involving national security, diplomatic or foreign relations, intelligence and
other classified information." (Emphasis supplied)

Contrary to AMARI's contention, the commissioners of the 1986 Constitutional Commission


understood that the right to information "contemplates inclusion of negotiations leading to the
consummation of the transaction."Certainly, a consummated contract is not a requirement for the
exercise of the right to information. Otherwise, the people can never exercise the right if no contract is
consummated, and if one is consummated, it may be too late for the public to expose its
defects.1âwphi1.nêt

Requiring a consummated contract will keep the public in the dark until the contract, which may be
grossly disadvantageous to the government or even illegal, becomes a fait accompli. This negates
the State policy of full transparency on matters of public concern, a situation which the framers of the
Constitution could not have intended. Such a requirement will prevent the citizenry from participating
in the public discussion of any proposed contract, effectively truncating a basic right enshrined in the
Bill of Rights. We can allow neither an emasculation of a constitutional right, nor a retreat by the State
of its avowed "policy of full disclosure of all its transactions involving public interest."

The right covers three categories of information which are "matters of public concern," namely: (1)
official records; (2) documents and papers pertaining to official acts, transactions and decisions; and
(3) government research data used in formulating policies. The first category refers to any document
that is part of the public records in the custody of government agencies or officials. The second
category refers to documents and papers recording, evidencing, establishing, confirming, supporting,
justifying or explaining official acts, transactions or decisions of government agencies or officials. The
third category refers to research data, whether raw, collated or processed, owned by the government
and used in formulating government policies.

The information that petitioner may access on the renegotiation of the JVA includes evaluation
reports, recommendations, legal and expert opinions, minutes of meetings, terms of reference and
other documents attached to such reports or minutes, all relating to the JVA. However, the right to
information does not compel PEA to prepare lists, abstracts, summaries and the like relating to the
renegotiation of the JVA.34 The right only affords access to records, documents and papers, which
means the opportunity to inspect and copy them. One who exercises the right must copy the records,
documents and papers at his expense. The exercise of the right is also subject to reasonable
regulations to protect the integrity of the public records and to minimize disruption to government
operations, like rules specifying when and how to conduct the inspection and copying. 35

The right to information, however, does not extend to matters recognized as privileged information
under the separation of powers.36 The right does not also apply to information on military and
diplomatic secrets, information affecting national security, and information on investigations of crimes
by law enforcement agencies before the prosecution of the accused, which courts have long
recognized as confidential.37 The right may also be subject to other limitations that Congress may
impose by law.

There is no claim by PEA that the information demanded by petitioner is privileged information rooted
in the separation of powers. The information does not cover Presidential conversations,
correspondences, or discussions during closed-door Cabinet meetings which, like internal
deliberations of the Supreme Court and other collegiate courts, or executive sessions of either house
of Congress,38 are recognized as confidential. This kind of information cannot be pried open by a co-
equal branch of government. A frank exchange of exploratory ideas and assessments, free from the
glare of publicity and pressure by interested parties, is essential to protect the independence of
decision-making of those tasked to exercise Presidential, Legislative and Judicial power. 39 This is not
the situation in the instant case.

We rule, therefore, that the constitutional right to information includes official information on on-going
negotiations before a final contract. The information, however, must constitute definite propositions
by the government and should not cover recognized exceptions like privileged information, military
and diplomatic secrets and similar matters affecting national security and public order. 40 Congress
has also prescribed other limitations on the right to information in several legislations. 41
Sixth issue: whether stipulations in the Amended JVA for the transfer to AMARI of lands,
reclaimed or to be reclaimed, violate the Constitution.

The Regalian Doctrine

The ownership of lands reclaimed from foreshore and submerged areas is rooted in the Regalian
doctrine which holds that the State owns all lands and waters of the public domain. Upon the Spanish
conquest of the Philippines, ownership of all "lands, territories and possessions" in the Philippines
passed to the Spanish Crown.42 The King, as the sovereign ruler and representative of the people,
acquired and owned all lands and territories in the Philippines except those he disposed of by grant or
sale to private individuals.

The 1935, 1973 and 1987 Constitutions adopted the Regalian doctrine substituting, however, the
State, in lieu of the King, as the owner of all lands and waters of the public domain. The Regalian
doctrine is the foundation of the time-honored principle of land ownership that "all lands that were not
acquired from the Government, either by purchase or by grant, belong to the public domain."43 Article
339 of the Civil Code of 1889, which is now Article 420 of the Civil Code of 1950, incorporated the
Regalian doctrine.

Ownership and Disposition of Reclaimed Lands

The Spanish Law of Waters of 1866 was the first statutory law governing the ownership and
disposition of reclaimed lands in the Philippines. On May 18, 1907, the Philippine Commission
enacted Act No. 1654 which provided for the lease, but not the sale, of reclaimed lands of the
government to corporations and individuals. Later, on November 29, 1919, the Philippine
Legislature approved Act No. 2874, the Public Land Act, which authorized the lease, but not the
sale, of reclaimed lands of the government to corporations and individuals. On November 7,
1936, the National Assembly passed Commonwealth Act No. 141, also known as the Public Land
Act, which authorized the lease, but not the sale, of reclaimed lands of the government to
corporations and individuals. CA No. 141 continues to this day as the general law governing the
classification and disposition of lands of the public domain.

The Spanish Law of Waters of 1866 and the Civil Code of 1889

Under the Spanish Law of Waters of 1866, the shores, bays, coves, inlets and all waters within the
maritime zone of the Spanish territory belonged to the public domain for public use. 44 The Spanish
Law of Waters of 1866 allowed the reclamation of the sea under Article 5, which provided as follows:

"Article 5. Lands reclaimed from the sea in consequence of works constructed by the State, or
by the provinces, pueblos or private persons, with proper permission, shall become the
property of the party constructing such works, unless otherwise provided by the terms of the
grant of authority."

Under the Spanish Law of Waters, land reclaimed from the sea belonged to the party undertaking the
reclamation, provided the government issued the necessary permit and did not reserve ownership of
the reclaimed land to the State.

Article 339 of the Civil Code of 1889 defined property of public dominion as follows:

"Art. 339. Property of public dominion is –


1. That devoted to public use, such as roads, canals, rivers, torrents, ports and bridges
constructed by the State, riverbanks, shores, roadsteads, and that of a similar character;

2. That belonging exclusively to the State which, without being of general public use, is
employed in some public service, or in the development of the national wealth, such as walls,
fortresses, and other works for the defense of the territory, and mines, until granted to private
individuals."

Property devoted to public use referred to property open for use by the public. In contrast, property
devoted to public service referred to property used for some specific public service and open only to
those authorized to use the property.

Property of public dominion referred not only to property devoted to public use, but also to property
not so used but employed to develop the national wealth. This class of property constituted
property of public dominion although employed for some economic or commercial activity to increase
the national wealth.

Article 341 of the Civil Code of 1889 governed the re-classification of property of public dominion into
private property, to wit:

"Art. 341. Property of public dominion, when no longer devoted to public use or to the defense
of the territory, shall become a part of the private property of the State."

This provision, however, was not self-executing. The legislature, or the executive department
pursuant to law, must declare the property no longer needed for public use or territorial defense
before the government could lease or alienate the property to private parties. 45

Act No. 1654 of the Philippine Commission

On May 8, 1907, the Philippine Commission enacted Act No. 1654 which regulated the lease of
reclaimed and foreshore lands. The salient provisions of this law were as follows:

"Section 1. The control and disposition of the foreshore as defined in existing law, and
the title to all Government or public lands made or reclaimed by the Government by
dredging or filling or otherwise throughout the Philippine Islands, shall be retained by the
Government without prejudice to vested rights and without prejudice to rights conceded to the
City of Manila in the Luneta Extension.

Section 2. (a) The Secretary of the Interior shall cause all Government or public lands made or
reclaimed by the Government by dredging or filling or otherwise to be divided into lots or
blocks, with the necessary streets and alleyways located thereon, and shall cause plats and
plans of such surveys to be prepared and filed with the Bureau of Lands.

(b) Upon completion of such plats and plans the Governor-General shall give notice to the
public that such parts of the lands so made or reclaimed as are not needed for public
purposes will be leased for commercial and business purposes, x x x.

xxx
(e) The leases above provided for shall be disposed of to the highest and best
bidder therefore, subject to such regulations and safeguards as the Governor-General may by
executive order prescribe." (Emphasis supplied)

Act No. 1654 mandated that the government should retain title to all lands reclaimed by the
government. The Act also vested in the government control and disposition of foreshore lands.
Private parties could lease lands reclaimed by the government only if these lands were no longer
needed for public purpose. Act No. 1654 mandated public bidding in the lease of government
reclaimed lands. Act No. 1654 made government reclaimed lands sui generis in that unlike other
public lands which the government could sell to private parties, these reclaimed lands were available
only for lease to private parties.

Act No. 1654, however, did not repeal Section 5 of the Spanish Law of Waters of 1866. Act No. 1654
did not prohibit private parties from reclaiming parts of the sea under Section 5 of the Spanish Law of
Waters. Lands reclaimed from the sea by private parties with government permission remained
private lands.

Act No. 2874 of the Philippine Legislature

On November 29, 1919, the Philippine Legislature enacted Act No. 2874, the Public Land Act. 46 The
salient provisions of Act No. 2874, on reclaimed lands, were as follows:

"Sec. 6. The Governor-General, upon the recommendation of the Secretary of


Agriculture and Natural Resources, shall from time to time classify the lands of the
public domain into –

(a) Alienable or disposable,

(b) Timber, and

(c) Mineral lands, x x x.

Sec. 7. For the purposes of the government and disposition of alienable or disposable public
lands, the Governor-General, upon recommendation by the Secretary of Agriculture and
Natural Resources, shall from time to time declare what lands are open to disposition or
concession under this Act."

Sec. 8. Only those lands shall be declared open to disposition or concession which
have been officially delimited or classified x x x.

xxx

Sec. 55. Any tract of land of the public domain which, being neither timber nor mineral land,
shall be classified as suitable for residential purposes or for commercial, industrial, or
other productive purposes other than agricultural purposes, and shall be open to
disposition or concession, shall be disposed of under the provisions of this chapter, and not
otherwise.

Sec. 56. The lands disposable under this title shall be classified as follows:

(a) Lands reclaimed by the Government by dredging, filling, or other means;


(b) Foreshore;

(c) Marshy lands or lands covered with water bordering upon the shores or banks of
navigable lakes or rivers;

(d) Lands not included in any of the foregoing classes.

x x x.

Sec. 58. The lands comprised in classes (a), (b), and (c) of section fifty-six shall be
disposed of to private parties by lease only and not otherwise, as soon as the Governor-
General, upon recommendation by the Secretary of Agriculture and Natural Resources,
shall declare that the same are not necessary for the public service and are open to
disposition under this chapter. The lands included in class (d) may be disposed of by
sale or lease under the provisions of this Act." (Emphasis supplied)

Section 6 of Act No. 2874 authorized the Governor-General to "classify lands of the public domain
into x x x alienable or disposable"47 lands. Section 7 of the Act empowered the Governor-General to
"declare what lands are open to disposition or concession." Section 8 of the Act limited alienable or
disposable lands only to those lands which have been "officially delimited and classified."

Section 56 of Act No. 2874 stated that lands "disposable under this title 48 shall be classified" as
government reclaimed, foreshore and marshy lands, as well as other lands. All these lands, however,
must be suitable for residential, commercial, industrial or other productive non-agricultural purposes.
These provisions vested upon the Governor-General the power to classify inalienable lands of the
public domain into disposable lands of the public domain. These provisions also empowered the
Governor-General to classify further such disposable lands of the public domain into government
reclaimed, foreshore or marshy lands of the public domain, as well as other non-agricultural lands.

Section 58 of Act No. 2874 categorically mandated that disposable lands of the public domain
classified as government reclaimed, foreshore and marshy lands "shall be disposed of to private
parties by lease only and not otherwise." The Governor-General, before allowing the lease of
these lands to private parties, must formally declare that the lands were "not necessary for the public
service." Act No. 2874 reiterated the State policy to lease and not to sell government reclaimed,
foreshore and marshy lands of the public domain, a policy first enunciated in 1907 in Act No. 1654.
Government reclaimed, foreshore and marshy lands remained sui generis, as the only alienable or
disposable lands of the public domain that the government could not sell to private parties.

The rationale behind this State policy is obvious. Government reclaimed, foreshore and marshy public
lands for non-agricultural purposes retain their inherent potential as areas for public service. This is
the reason the government prohibited the sale, and only allowed the lease, of these lands to private
parties. The State always reserved these lands for some future public service.

Act No. 2874 did not authorize the reclassification of government reclaimed, foreshore and marshy
lands into other non-agricultural lands under Section 56 (d). Lands falling under Section 56 (d) were
the only lands for non-agricultural purposes the government could sell to private parties. Thus, under
Act No. 2874, the government could not sell government reclaimed, foreshore and marshy lands to
private parties, unless the legislature passed a law allowing their sale.49

Act No. 2874 did not prohibit private parties from reclaiming parts of the sea pursuant to Section 5 of
the Spanish Law of Waters of 1866. Lands reclaimed from the sea by private parties with government
permission remained private lands.
Dispositions under the 1935 Constitution

On May 14, 1935, the 1935 Constitution took effect upon its ratification by the Filipino people. The
1935 Constitution, in adopting the Regalian doctrine, declared in Section 1, Article XIII, that –

"Section 1. All agricultural, timber, and mineral lands of the public domain, waters, minerals,
coal, petroleum, and other mineral oils, all forces of potential energy and other natural
resources of the Philippines belong to the State, and their disposition, exploitation,
development, or utilization shall be limited to citizens of the Philippines or to corporations or
associations at least sixty per centum of the capital of which is owned by such citizens, subject
to any existing right, grant, lease, or concession at the time of the inauguration of the
Government established under this Constitution. Natural resources, with the exception of
public agricultural land, shall not be alienated, and no license, concession, or lease for the
exploitation, development, or utilization of any of the natural resources shall be granted for a
period exceeding twenty-five years, renewable for another twenty-five years, except as to
water rights for irrigation, water supply, fisheries, or industrial uses other than the development
of water power, in which cases beneficial use may be the measure and limit of the grant."
(Emphasis supplied)

The 1935 Constitution barred the alienation of all natural resources except public agricultural lands,
which were the only natural resources the State could alienate. Thus, foreshore lands, considered
part of the State's natural resources, became inalienable by constitutional fiat, available only for lease
for 25 years, renewable for another 25 years. The government could alienate foreshore lands only
after these lands were reclaimed and classified as alienable agricultural lands of the public domain.
Government reclaimed and marshy lands of the public domain, being neither timber nor mineral
lands, fell under the classification of public agricultural lands.50 However, government reclaimed and
marshy lands, although subject to classification as disposable public agricultural lands, could only be
leased and not sold to private parties because of Act No. 2874.

The prohibition on private parties from acquiring ownership of government reclaimed and marshy
lands of the public domain was only a statutory prohibition and the legislature could therefore remove
such prohibition. The 1935 Constitution did not prohibit individuals and corporations from acquiring
government reclaimed and marshy lands of the public domain that were classified as agricultural
lands under existing public land laws. Section 2, Article XIII of the 1935 Constitution provided as
follows:

"Section 2. No private corporation or association may acquire, lease, or hold public


agricultural lands in excess of one thousand and twenty four hectares, nor may any
individual acquire such lands by purchase in excess of one hundred and forty hectares,
or by lease in excess of one thousand and twenty-four hectares, or by homestead in
excess of twenty-four hectares. Lands adapted to grazing, not exceeding two thousand
hectares, may be leased to an individual, private corporation, or association." (Emphasis
supplied)

Still, after the effectivity of the 1935 Constitution, the legislature did not repeal Section 58 of Act No.
2874 to open for sale to private parties government reclaimed and marshy lands of the public domain.
On the contrary, the legislature continued the long established State policy of retaining for the
government title and ownership of government reclaimed and marshy lands of the public domain.

Commonwealth Act No. 141 of the Philippine National Assembly


On November 7, 1936, the National Assembly approved Commonwealth Act No. 141, also known as
the Public Land Act, which compiled the then existing laws on lands of the public domain. CA No.
141, as amended, remains to this day the existing general law governing the classification and
disposition of lands of the public domain other than timber and mineral lands. 51

Section 6 of CA No. 141 empowers the President to classify lands of the public domain into "alienable
or disposable"52 lands of the public domain, which prior to such classification are inalienable and
outside the commerce of man. Section 7 of CA No. 141 authorizes the President to "declare what
lands are open to disposition or concession." Section 8 of CA No. 141 states that the government can
declare open for disposition or concession only lands that are "officially delimited and classified."
Sections 6, 7 and 8 of CA No. 141 read as follows:

"Sec. 6. The President, upon the recommendation of the Secretary of Agriculture and
Commerce, shall from time to time classify the lands of the public domain into –

(a) Alienable or disposable,

(b) Timber, and

(c) Mineral lands,

and may at any time and in like manner transfer such lands from one class to another,53 for the
purpose of their administration and disposition.

Sec. 7. For the purposes of the administration and disposition of alienable or disposable public
lands, the President, upon recommendation by the Secretary of Agriculture and
Commerce, shall from time to time declare what lands are open to disposition or
concession under this Act.

Sec. 8. Only those lands shall be declared open to disposition or concession which
have been officially delimited and classified and, when practicable, surveyed, and which
have not been reserved for public or quasi-public uses, nor appropriated by the
Government, nor in any manner become private property, nor those on which a private right
authorized and recognized by this Act or any other valid law may be claimed, or which, having
been reserved or appropriated, have ceased to be so. x x x."

Thus, before the government could alienate or dispose of lands of the public domain, the President
must first officially classify these lands as alienable or disposable, and then declare them open to
disposition or concession. There must be no law reserving these lands for public or quasi-public uses.

The salient provisions of CA No. 141, on government reclaimed, foreshore and marshy lands of the
public domain, are as follows:

"Sec. 58. Any tract of land of the public domain which, being neither timber nor mineral
land, is intended to be used for residential purposes or for commercial, industrial, or
other productive purposes other than agricultural, and is open to disposition or
concession, shall be disposed of under the provisions of this chapter and not
otherwise.

Sec. 59. The lands disposable under this title shall be classified as follows:
(a) Lands reclaimed by the Government by dredging, filling, or other means;

(b) Foreshore;

(c) Marshy lands or lands covered with water bordering upon the shores or banks of
navigable lakes or rivers;

(d) Lands not included in any of the foregoing classes.

Sec. 60. Any tract of land comprised under this title may be leased or sold, as the case may
be, to any person, corporation, or association authorized to purchase or lease public lands for
agricultural purposes. x x x.

Sec. 61. The lands comprised in classes (a), (b), and (c) of section fifty-nine shall be
disposed of to private parties by lease only and not otherwise, as soon as the President,
upon recommendation by the Secretary of Agriculture, shall declare that the same are not
necessary for the public service and are open to disposition under this chapter. The lands
included in class (d) may be disposed of by sale or lease under the provisions of this
Act." (Emphasis supplied)

Section 61 of CA No. 141 readopted, after the effectivity of the 1935 Constitution, Section 58 of Act
No. 2874 prohibiting the sale of government reclaimed, foreshore and marshy disposable lands of the
public domain. All these lands are intended for residential, commercial, industrial or other non-
agricultural purposes. As before, Section 61 allowed only the lease of such lands to private parties.
The government could sell to private parties only lands falling under Section 59 (d) of CA No. 141, or
those lands for non-agricultural purposes not classified as government reclaimed, foreshore and
marshy disposable lands of the public domain. Foreshore lands, however, became inalienable under
the 1935 Constitution which only allowed the lease of these lands to qualified private parties.

Section 58 of CA No. 141 expressly states that disposable lands of the public domain intended for
residential, commercial, industrial or other productive purposes other than agricultural "shall be
disposed of under the provisions of this chapter and not otherwise." Under Section 10 of CA
No. 141, the term "disposition" includes lease of the land. Any disposition of government reclaimed,
foreshore and marshy disposable lands for non-agricultural purposes must comply with Chapter IX,
Title III of CA No. 141,54 unless a subsequent law amended or repealed these provisions.

In his concurring opinion in the landmark case of Republic Real Estate Corporation v. Court of
Appeals,55Justice Reynato S. Puno summarized succinctly the law on this matter, as follows:

"Foreshore lands are lands of public dominion intended for public use. So too are lands
reclaimed by the government by dredging, filling, or other means. Act 1654 mandated that the
control and disposition of the foreshore and lands under water remained in the national
government. Said law allowed only the 'leasing' of reclaimed land. The Public Land Acts of
1919 and 1936 also declared that the foreshore and lands reclaimed by the government were
to be "disposed of to private parties by lease only and not otherwise." Before leasing, however,
the Governor-General, upon recommendation of the Secretary of Agriculture and Natural
Resources, had first to determine that the land reclaimed was not necessary for the public
service. This requisite must have been met before the land could be disposed of. But even
then, the foreshore and lands under water were not to be alienated and sold to private
parties. The disposition of the reclaimed land was only by lease. The land remained
property of the State." (Emphasis supplied)
As observed by Justice Puno in his concurring opinion, "Commonwealth Act No. 141 has remained in
effect at present."

The State policy prohibiting the sale to private parties of government reclaimed, foreshore and
marshy alienable lands of the public domain, first implemented in 1907 was thus reaffirmed in CA No.
141 after the 1935 Constitution took effect. The prohibition on the sale of foreshore lands, however,
became a constitutional edict under the 1935 Constitution. Foreshore lands became inalienable as
natural resources of the State, unless reclaimed by the government and classified as agricultural
lands of the public domain, in which case they would fall under the classification of government
reclaimed lands.

After the effectivity of the 1935 Constitution, government reclaimed and marshy disposable lands of
the public domain continued to be only leased and not sold to private parties. 56 These lands
remained sui generis, as the only alienable or disposable lands of the public domain the government
could not sell to private parties.

Since then and until now, the only way the government can sell to private parties government
reclaimed and marshy disposable lands of the public domain is for the legislature to pass a law
authorizing such sale. CA No. 141 does not authorize the President to reclassify government
reclaimed and marshy lands into other non-agricultural lands under Section 59 (d). Lands classified
under Section 59 (d) are the only alienable or disposable lands for non-agricultural purposes that the
government could sell to private parties.

Moreover, Section 60 of CA No. 141 expressly requires congressional authority before lands under
Section 59 that the government previously transferred to government units or entities could be sold to
private parties. Section 60 of CA No. 141 declares that –

"Sec. 60. x x x The area so leased or sold shall be such as shall, in the judgment of the
Secretary of Agriculture and Natural Resources, be reasonably necessary for the purposes for
which such sale or lease is requested, and shall not exceed one hundred and forty-four
hectares: Provided, however, That this limitation shall not apply to grants, donations, or
transfers made to a province, municipality or branch or subdivision of the Government for the
purposes deemed by said entities conducive to the public interest; but the land so granted,
donated, or transferred to a province, municipality or branch or subdivision of the
Government shall not be alienated, encumbered, or otherwise disposed of in a manner
affecting its title, except when authorized by Congress: x x x." (Emphasis supplied)

The congressional authority required in Section 60 of CA No. 141 mirrors the legislative authority
required in Section 56 of Act No. 2874.

One reason for the congressional authority is that Section 60 of CA No. 141 exempted government
units and entities from the maximum area of public lands that could be acquired from the State.
These government units and entities should not just turn around and sell these lands to private
parties in violation of constitutional or statutory limitations. Otherwise, the transfer of lands for non-
agricultural purposes to government units and entities could be used to circumvent constitutional
limitations on ownership of alienable or disposable lands of the public domain. In the same manner,
such transfers could also be used to evade the statutory prohibition in CA No. 141 on the sale of
government reclaimed and marshy lands of the public domain to private parties. Section 60 of CA No.
141 constitutes by operation of law a lien on these lands. 57
In case of sale or lease of disposable lands of the public domain falling under Section 59 of CA No.
141, Sections 63 and 67 require a public bidding. Sections 63 and 67 of CA No. 141 provide as
follows:

"Sec. 63. Whenever it is decided that lands covered by this chapter are not needed for public
purposes, the Director of Lands shall ask the Secretary of Agriculture and Commerce (now the
Secretary of Natural Resources) for authority to dispose of the same. Upon receipt of such
authority, the Director of Lands shall give notice by public advertisement in the same manner
as in the case of leases or sales of agricultural public land, x x x.

Sec. 67. The lease or sale shall be made by oral bidding; and adjudication shall be made
to the highest bidder. x x x." (Emphasis supplied)

Thus, CA No. 141 mandates the Government to put to public auction all leases or sales of alienable
or disposable lands of the public domain.58

Like Act No. 1654 and Act No. 2874 before it, CA No. 141 did not repeal Section 5 of the Spanish
Law of Waters of 1866. Private parties could still reclaim portions of the sea with government
permission. However, the reclaimed land could become private land only if classified as
alienable agricultural land of the public domain open to disposition under CA No. 141. The 1935
Constitution prohibited the alienation of all natural resources except public agricultural lands.

The Civil Code of 1950

The Civil Code of 1950 readopted substantially the definition of property of public dominion found in
the Civil Code of 1889. Articles 420 and 422 of the Civil Code of 1950 state that –

"Art. 420. The following things are property of public dominion:

(1) Those intended for public use, such as roads, canals, rivers, torrents, ports and bridges
constructed by the State, banks, shores, roadsteads, and others of similar character;

(2) Those which belong to the State, without being for public use, and are intended for some
public service or for the development of the national wealth.

x x x.

Art. 422. Property of public dominion, when no longer intended for public use or for public
service, shall form part of the patrimonial property of the State."

Again, the government must formally declare that the property of public dominion is no longer needed
for public use or public service, before the same could be classified as patrimonial property of the
State.59 In the case of government reclaimed and marshy lands of the public domain, the declaration
of their being disposable, as well as the manner of their disposition, is governed by the applicable
provisions of CA No. 141.

Like the Civil Code of 1889, the Civil Code of 1950 included as property of public dominion those
properties of the State which, without being for public use, are intended for public service or the
"development of the national wealth." Thus, government reclaimed and marshy lands of the State,
even if not employed for public use or public service, if developed to enhance the national wealth, are
classified as property of public dominion.
Dispositions under the 1973 Constitution

The 1973 Constitution, which took effect on January 17, 1973, likewise adopted the Regalian
doctrine. Section 8, Article XIV of the 1973 Constitution stated that –

"Sec. 8. All lands of the public domain, waters, minerals, coal, petroleum and other mineral
oils, all forces of potential energy, fisheries, wildlife, and other natural resources of the
Philippines belong to the State. With the exception of agricultural, industrial or
commercial, residential, and resettlement lands of the public domain, natural resources
shall not be alienated, and no license, concession, or lease for the exploration, development,
exploitation, or utilization of any of the natural resources shall be granted for a period
exceeding twenty-five years, renewable for not more than twenty-five years, except as to water
rights for irrigation, water supply, fisheries, or industrial uses other than the development of
water power, in which cases, beneficial use may be the measure and the limit of the grant."
(Emphasis supplied)

The 1973 Constitution prohibited the alienation of all natural resources with the exception of
"agricultural, industrial or commercial, residential, and resettlement lands of the public domain." In
contrast, the 1935 Constitution barred the alienation of all natural resources except "public agricultural
lands." However, the term "public agricultural lands" in the 1935 Constitution encompassed industrial,
commercial, residential and resettlement lands of the public domain.60 If the land of public domain
were neither timber nor mineral land, it would fall under the classification of agricultural land of the
public domain. Both the 1935 and 1973 Constitutions, therefore, prohibited the alienation of all
natural resources except agricultural lands of the public domain.

The 1973 Constitution, however, limited the alienation of lands of the public domain to individuals who
were citizens of the Philippines. Private corporations, even if wholly owned by Philippine citizens,
were no longer allowed to acquire alienable lands of the public domain unlike in the 1935
Constitution. Section 11, Article XIV of the 1973 Constitution declared that –

"Sec. 11. The Batasang Pambansa, taking into account conservation, ecological, and
development requirements of the natural resources, shall determine by law the size of land of
the public domain which may be developed, held or acquired by, or leased to, any qualified
individual, corporation, or association, and the conditions therefor. No private corporation or
association may hold alienable lands of the public domain except by lease not to exceed
one thousand hectares in area nor may any citizen hold such lands by lease in excess of five
hundred hectares or acquire by purchase, homestead or grant, in excess of twenty-four
hectares. No private corporation or association may hold by lease, concession, license or
permit, timber or forest lands and other timber or forest resources in excess of one hundred
thousand hectares. However, such area may be increased by the Batasang Pambansa upon
recommendation of the National Economic and Development Authority." (Emphasis supplied)

Thus, under the 1973 Constitution, private corporations could hold alienable lands of the public
domain only through lease. Only individuals could now acquire alienable lands of the public domain,
and private corporations became absolutely barred from acquiring any kind of alienable land
of the public domain. The constitutional ban extended to all kinds of alienable lands of the public
domain, while the statutory ban under CA No. 141 applied only to government reclaimed, foreshore
and marshy alienable lands of the public domain.

PD No. 1084 Creating the Public Estates Authority


On February 4, 1977, then President Ferdinand Marcos issued Presidential Decree No. 1084 creating
PEA, a wholly government owned and controlled corporation with a special charter. Sections 4 and 8
of PD No. 1084, vests PEA with the following purposes and powers:

"Sec. 4. Purpose. The Authority is hereby created for the following purposes:

(a) To reclaim land, including foreshore and submerged areas, by dredging, filling or
other means, or to acquire reclaimed land;

(b) To develop, improve, acquire, administer, deal in, subdivide, dispose, lease and sell any
and all kinds of lands, buildings, estates and other forms of real property, owned, managed,
controlled and/or operated by the government;

(c) To provide for, operate or administer such service as may be necessary for the efficient,
economical and beneficial utilization of the above properties.

Sec. 5. Powers and functions of the Authority. The Authority shall, in carrying out the purposes
for which it is created, have the following powers and functions:

(a)To prescribe its by-laws.

xxx

(i) To hold lands of the public domain in excess of the area permitted to private corporations
by statute.

(j) To reclaim lands and to construct work across, or otherwise, any stream, watercourse,
canal, ditch, flume x x x.

xxx

(o) To perform such acts and exercise such functions as may be necessary for the attainment
of the purposes and objectives herein specified." (Emphasis supplied)

PD No. 1084 authorizes PEA to reclaim both foreshore and submerged areas of the public domain.
Foreshore areas are those covered and uncovered by the ebb and flow of the tide. 61 Submerged
areas are those permanently under water regardless of the ebb and flow of the tide.62 Foreshore and
submerged areas indisputably belong to the public domain 63 and are inalienable unless reclaimed,
classified as alienable lands open to disposition, and further declared no longer needed for public
service.

The ban in the 1973 Constitution on private corporations from acquiring alienable lands of the public
domain did not apply to PEA since it was then, and until today, a fully owned government corporation.
The constitutional ban applied then, as it still applies now, only to "private corporations and
associations." PD No. 1084 expressly empowers PEA "to hold lands of the public domain" even
"in excess of the area permitted to private corporations by statute." Thus, PEA can hold title to
private lands, as well as title to lands of the public domain.

In order for PEA to sell its reclaimed foreshore and submerged alienable lands of the public domain,
there must be legislative authority empowering PEA to sell these lands. This legislative authority is
necessary in view of Section 60 of CA No.141, which states –
"Sec. 60. x x x; but the land so granted, donated or transferred to a province, municipality, or
branch or subdivision of the Government shall not be alienated, encumbered or otherwise
disposed of in a manner affecting its title, except when authorized by Congress; x x x."
(Emphasis supplied)

Without such legislative authority, PEA could not sell but only lease its reclaimed foreshore and
submerged alienable lands of the public domain. Nevertheless, any legislative authority granted to
PEA to sell its reclaimed alienable lands of the public domain would be subject to the constitutional
ban on private corporations from acquiring alienable lands of the public domain. Hence, such
legislative authority could only benefit private individuals.

Dispositions under the 1987 Constitution

The 1987 Constitution, like the 1935 and 1973 Constitutions before it, has adopted the Regalian
doctrine. The 1987 Constitution declares that all natural resources are "owned by the State," and
except for alienable agricultural lands of the public domain, natural resources cannot be alienated.
Sections 2 and 3, Article XII of the 1987 Constitution state that –

"Section 2. All lands of the public domain, waters, minerals, coal, petroleum and other mineral
oils, all forces of potential energy, fisheries, forests or timber, wildlife, flora and fauna, and
other natural resources are owned by the State. With the exception of agricultural lands,
all other natural resources shall not be alienated. The exploration, development, and
utilization of natural resources shall be under the full control and supervision of the State. x x x.

Section 3. Lands of the public domain are classified into agricultural, forest or timber, mineral
lands, and national parks. Agricultural lands of the public domain may be further classified by
law according to the uses which they may be devoted. Alienable lands of the public domain
shall be limited to agricultural lands. Private corporations or associations may not hold
such alienable lands of the public domain except by lease, for a period not exceeding
twenty-five years, renewable for not more than twenty-five years, and not to exceed one
thousand hectares in area. Citizens of the Philippines may lease not more than five hundred
hectares, or acquire not more than twelve hectares thereof by purchase, homestead, or grant.

Taking into account the requirements of conservation, ecology, and development, and subject
to the requirements of agrarian reform, the Congress shall determine, by law, the size of lands
of the public domain which may be acquired, developed, held, or leased and the conditions
therefor." (Emphasis supplied)

The 1987 Constitution continues the State policy in the 1973 Constitution banning private
corporations from acquiring any kind of alienable land of the public domain. Like the 1973
Constitution, the 1987 Constitution allows private corporations to hold alienable lands of the public
domain only through lease. As in the 1935 and 1973 Constitutions, the general law governing the
lease to private corporations of reclaimed, foreshore and marshy alienable lands of the public domain
is still CA No. 141.

The Rationale behind the Constitutional Ban

The rationale behind the constitutional ban on corporations from acquiring, except through lease,
alienable lands of the public domain is not well understood. During the deliberations of the 1986
Constitutional Commission, the commissioners probed the rationale behind this ban, thus:

"FR. BERNAS: Mr. Vice-President, my questions have reference to page 3, line 5 which says:
`No private corporation or association may hold alienable lands of the public domain except by
lease, not to exceed one thousand hectares in area.'

If we recall, this provision did not exist under the 1935 Constitution, but this was introduced in
the 1973 Constitution. In effect, it prohibits private corporations from acquiring alienable public
lands. But it has not been very clear in jurisprudence what the reason for this is. In some
of the cases decided in 1982 and 1983, it was indicated that the purpose of this is to
prevent large landholdings. Is that the intent of this provision?

MR. VILLEGAS: I think that is the spirit of the provision.

FR. BERNAS: In existing decisions involving the Iglesia ni Cristo, there were instances where
the Iglesia ni Cristo was not allowed to acquire a mere 313-square meter land where a chapel
stood because the Supreme Court said it would be in violation of this." (Emphasis supplied)

In Ayog v. Cusi,64 the Court explained the rationale behind this constitutional ban in this way:

"Indeed, one purpose of the constitutional prohibition against purchases of public agricultural
lands by private corporations is to equitably diffuse land ownership or to encourage 'owner-
cultivatorship and the economic family-size farm' and to prevent a recurrence of cases like the
instant case. Huge landholdings by corporations or private persons had spawned social
unrest."

However, if the constitutional intent is to prevent huge landholdings, the Constitution could have
simply limited the size of alienable lands of the public domain that corporations could acquire. The
Constitution could have followed the limitations on individuals, who could acquire not more than 24
hectares of alienable lands of the public domain under the 1973 Constitution, and not more than 12
hectares under the 1987 Constitution.

If the constitutional intent is to encourage economic family-size farms, placing the land in the name of
a corporation would be more effective in preventing the break-up of farmlands. If the farmland is
registered in the name of a corporation, upon the death of the owner, his heirs would inherit shares in
the corporation instead of subdivided parcels of the farmland. This would prevent the continuing
break-up of farmlands into smaller and smaller plots from one generation to the next.

In actual practice, the constitutional ban strengthens the constitutional limitation on individuals from
acquiring more than the allowed area of alienable lands of the public domain. Without the
constitutional ban, individuals who already acquired the maximum area of alienable lands of the
public domain could easily set up corporations to acquire more alienable public lands. An individual
could own as many corporations as his means would allow him. An individual could even hide his
ownership of a corporation by putting his nominees as stockholders of the corporation. The
corporation is a convenient vehicle to circumvent the constitutional limitation on acquisition by
individuals of alienable lands of the public domain.

The constitutional intent, under the 1973 and 1987 Constitutions, is to transfer ownership of only a
limited area of alienable land of the public domain to a qualified individual. This constitutional intent is
safeguarded by the provision prohibiting corporations from acquiring alienable lands of the public
domain, since the vehicle to circumvent the constitutional intent is removed. The available alienable
public lands are gradually decreasing in the face of an ever-growing population. The most effective
way to insure faithful adherence to this constitutional intent is to grant or sell alienable lands of the
public domain only to individuals. This, it would seem, is the practical benefit arising from the
constitutional ban.
The Amended Joint Venture Agreement

The subject matter of the Amended JVA, as stated in its second Whereas clause, consists of three
properties, namely:

1. "[T]hree partially reclaimed and substantially eroded islands along Emilio Aguinaldo
Boulevard in Paranaque and Las Pinas, Metro Manila, with a combined titled area of 1,578,441
square meters;"

2. "[A]nother area of 2,421,559 square meters contiguous to the three islands;" and

3. "[A]t AMARI's option as approved by PEA, an additional 350 hectares more or less to
regularize the configuration of the reclaimed area."65

PEA confirms that the Amended JVA involves "the development of the Freedom Islands and further
reclamation of about 250 hectares x x x," plus an option "granted to AMARI to subsequently reclaim
another 350 hectares x x x."66

In short, the Amended JVA covers a reclamation area of 750 hectares. Only 157.84 hectares of the
750-hectare reclamation project have been reclaimed, and the rest of the 592.15 hectares are
still submerged areas forming part of Manila Bay.

Under the Amended JVA, AMARI will reimburse PEA the sum of P1,894,129,200.00 for PEA's "actual
cost" in partially reclaiming the Freedom Islands. AMARI will also complete, at its own expense, the
reclamation of the Freedom Islands. AMARI will further shoulder all the reclamation costs of all the
other areas, totaling 592.15 hectares, still to be reclaimed. AMARI and PEA will share, in the
proportion of 70 percent and 30 percent, respectively, the total net usable area which is defined in the
Amended JVA as the total reclaimed area less 30 percent earmarked for common areas. Title to
AMARI's share in the net usable area, totaling 367.5 hectares, will be issued in the name of AMARI.
Section 5.2 (c) of the Amended JVA provides that –

"x x x, PEA shall have the duty to execute without delay the necessary deed of transfer or
conveyance of the title pertaining to AMARI's Land share based on the Land Allocation
Plan. PEA, when requested in writing by AMARI, shall then cause the issuance and
delivery of the proper certificates of title covering AMARI's Land Share in the name of
AMARI, x x x; provided, that if more than seventy percent (70%) of the titled area at any given
time pertains to AMARI, PEA shall deliver to AMARI only seventy percent (70%) of the titles
pertaining to AMARI, until such time when a corresponding proportionate area of additional
land pertaining to PEA has been titled." (Emphasis supplied)

Indisputably, under the Amended JVA AMARI will acquire and own a maximum of 367.5
hectares of reclaimed land which will be titled in its name.

To implement the Amended JVA, PEA delegated to the unincorporated PEA-AMARI joint venture
PEA's statutory authority, rights and privileges to reclaim foreshore and submerged areas in Manila
Bay. Section 3.2.a of the Amended JVA states that –

"PEA hereby contributes to the joint venture its rights and privileges to perform Rawland
Reclamation and Horizontal Development as well as own the Reclamation Area, thereby
granting the Joint Venture the full and exclusive right, authority and privilege to undertake the
Project in accordance with the Master Development Plan."
The Amended JVA is the product of a renegotiation of the original JVA dated April 25, 1995 and its
supplemental agreement dated August 9, 1995.

The Threshold Issue

The threshold issue is whether AMARI, a private corporation, can acquire and own under the
Amended JVA 367.5 hectares of reclaimed foreshore and submerged areas in Manila Bay in view of
Sections 2 and 3, Article XII of the 1987 Constitution which state that:

"Section 2. All lands of the public domain, waters, minerals, coal, petroleum, and other mineral
oils, all forces of potential energy, fisheries, forests or timber, wildlife, flora and fauna, and
other natural resources are owned by the State. With the exception of agricultural lands, all
other natural resources shall not be alienated. x x x.

xxx

Section 3. x x x Alienable lands of the public domain shall be limited to agricultural


lands. Private corporations or associations may not hold such alienable lands of the
public domain except by lease, x x x."(Emphasis supplied)

Classification of Reclaimed Foreshore and Submerged Areas

PEA readily concedes that lands reclaimed from foreshore or submerged areas of Manila Bay are
alienable or disposable lands of the public domain. In its Memorandum,67 PEA admits that –

"Under the Public Land Act (CA 141, as amended), reclaimed lands are classified as
alienable and disposable lands of the public domain:

'Sec. 59. The lands disposable under this title shall be classified as follows:

(a) Lands reclaimed by the government by dredging, filling, or other means;

x x x.'" (Emphasis supplied)

Likewise, the Legal Task Force68 constituted under Presidential Administrative Order No. 365
admitted in its Report and Recommendation to then President Fidel V. Ramos, "[R]eclaimed lands
are classified as alienable and disposable lands of the public domain."69 The Legal Task Force
concluded that –

"D. Conclusion

Reclaimed lands are lands of the public domain. However, by statutory authority, the rights of
ownership and disposition over reclaimed lands have been transferred to PEA, by virtue of
which PEA, as owner, may validly convey the same to any qualified person without violating
the Constitution or any statute.

The constitutional provision prohibiting private corporations from holding public land, except by
lease (Sec. 3, Art. XVII,70 1987 Constitution), does not apply to reclaimed lands whose
ownership has passed on to PEA by statutory grant."
Under Section 2, Article XII of the 1987 Constitution, the foreshore and submerged areas of Manila
Bay are part of the "lands of the public domain, waters x x x and other natural resources" and
consequently "owned by the State." As such, foreshore and submerged areas "shall not be
alienated," unless they are classified as "agricultural lands" of the public domain. The mere
reclamation of these areas by PEA does not convert these inalienable natural resources of the State
into alienable or disposable lands of the public domain. There must be a law or presidential
proclamation officially classifying these reclaimed lands as alienable or disposable and open to
disposition or concession. Moreover, these reclaimed lands cannot be classified as alienable or
disposable if the law has reserved them for some public or quasi-public use.71

Section 8 of CA No. 141 provides that "only those lands shall be declared open to disposition or
concession which have been officially delimited and classified."72 The President has the authority
to classify inalienable lands of the public domain into alienable or disposable lands of the public
domain, pursuant to Section 6 of CA No. 141. In Laurel vs. Garcia, 73 the Executive Department
attempted to sell the Roppongi property in Tokyo, Japan, which was acquired by the Philippine
Government for use as the Chancery of the Philippine Embassy. Although the Chancery had
transferred to another location thirteen years earlier, the Court still ruled that, under Article 422 74 of
the Civil Code, a property of public dominion retains such character until formally declared otherwise.
The Court ruled that –

"The fact that the Roppongi site has not been used for a long time for actual Embassy service
does not automatically convert it to patrimonial property. Any such conversion happens only if
the property is withdrawn from public use (Cebu Oxygen and Acetylene Co. v. Bercilles, 66
SCRA 481 [1975]. A property continues to be part of the public domain, not available for
private appropriation or ownership 'until there is a formal declaration on the part of the
government to withdraw it from being such' (Ignacio v. Director of Lands, 108 Phil. 335
[1960]." (Emphasis supplied)

PD No. 1085, issued on February 4, 1977, authorized the issuance of special land patents for lands
reclaimed by PEA from the foreshore or submerged areas of Manila Bay. On January 19, 1988 then
President Corazon C. Aquino issued Special Patent No. 3517 in the name of PEA for the 157.84
hectares comprising the partially reclaimed Freedom Islands. Subsequently, on April 9, 1999 the
Register of Deeds of the Municipality of Paranaque issued TCT Nos. 7309, 7311 and 7312 in the
name of PEA pursuant to Section 103 of PD No. 1529 authorizing the issuance of certificates of title
corresponding to land patents. To this day, these certificates of title are still in the name of PEA.

PD No. 1085, coupled with President Aquino's actual issuance of a special patent covering the
Freedom Islands, is equivalent to an official proclamation classifying the Freedom Islands as
alienable or disposable lands of the public domain. PD No. 1085 and President Aquino's issuance of
a land patent also constitute a declaration that the Freedom Islands are no longer needed for public
service. The Freedom Islands are thus alienable or disposable lands of the public domain,
open to disposition or concession to qualified parties.

At the time then President Aquino issued Special Patent No. 3517, PEA had already reclaimed the
Freedom Islands although subsequently there were partial erosions on some areas. The government
had also completed the necessary surveys on these islands. Thus, the Freedom Islands were no
longer part of Manila Bay but part of the land mass. Section 3, Article XII of the 1987 Constitution
classifies lands of the public domain into "agricultural, forest or timber, mineral lands, and national
parks." Being neither timber, mineral, nor national park lands, the reclaimed Freedom Islands
necessarily fall under the classification of agricultural lands of the public domain. Under the 1987
Constitution, agricultural lands of the public domain are the only natural resources that the State may
alienate to qualified private parties. All other natural resources, such as the seas or bays, are "waters
x x x owned by the State" forming part of the public domain, and are inalienable pursuant to Section
2, Article XII of the 1987 Constitution.

AMARI claims that the Freedom Islands are private lands because CDCP, then a private corporation,
reclaimed the islands under a contract dated November 20, 1973 with the Commissioner of Public
Highways. AMARI, citing Article 5 of the Spanish Law of Waters of 1866, argues that "if the ownership
of reclaimed lands may be given to the party constructing the works, then it cannot be said that
reclaimed lands are lands of the public domain which the State may not alienate."75 Article 5 of the
Spanish Law of Waters reads as follows:

"Article 5. Lands reclaimed from the sea in consequence of works constructed by the State, or
by the provinces, pueblos or private persons, with proper permission, shall become the
property of the party constructing such works, unless otherwise provided by the terms of
the grant of authority." (Emphasis supplied)

Under Article 5 of the Spanish Law of Waters of 1866, private parties could reclaim from the sea only
with "proper permission" from the State. Private parties could own the reclaimed land only if not
"otherwise provided by the terms of the grant of authority." This clearly meant that no one could
reclaim from the sea without permission from the State because the sea is property of public
dominion. It also meant that the State could grant or withhold ownership of the reclaimed land
because any reclaimed land, like the sea from which it emerged, belonged to the State. Thus, a
private person reclaiming from the sea without permission from the State could not acquire ownership
of the reclaimed land which would remain property of public dominion like the sea it replaced. 76 Article
5 of the Spanish Law of Waters of 1866 adopted the time-honored principle of land ownership that "all
lands that were not acquired from the government, either by purchase or by grant, belong to the
public domain."77

Article 5 of the Spanish Law of Waters must be read together with laws subsequently enacted on the
disposition of public lands. In particular, CA No. 141 requires that lands of the public domain must first
be classified as alienable or disposable before the government can alienate them. These lands must
not be reserved for public or quasi-public purposes.78 Moreover, the contract between CDCP and the
government was executed after the effectivity of the 1973 Constitution which barred private
corporations from acquiring any kind of alienable land of the public domain. This contract could not
have converted the Freedom Islands into private lands of a private corporation.

Presidential Decree No. 3-A, issued on January 11, 1973, revoked all laws authorizing the
reclamation of areas under water and revested solely in the National Government the power to
reclaim lands. Section 1 of PD No. 3-A declared that –

"The provisions of any law to the contrary notwithstanding, the reclamation of areas under
water, whether foreshore or inland, shall be limited to the National Government or any
person authorized by it under a proper contract. (Emphasis supplied)

x x x."

PD No. 3-A repealed Section 5 of the Spanish Law of Waters of 1866 because reclamation of areas
under water could now be undertaken only by the National Government or by a person contracted by
the National Government. Private parties may reclaim from the sea only under a contract with the
National Government, and no longer by grant or permission as provided in Section 5 of the Spanish
Law of Waters of 1866.
Executive Order No. 525, issued on February 14, 1979, designated PEA as the National
Government's implementing arm to undertake "all reclamation projects of the government," which
"shall be undertaken by the PEA or through a proper contract executed by it with any person
or entity." Under such contract, a private party receives compensation for reclamation services
rendered to PEA. Payment to the contractor may be in cash, or in kind consisting of portions of the
reclaimed land, subject to the constitutional ban on private corporations from acquiring alienable
lands of the public domain. The reclaimed land can be used as payment in kind only if the reclaimed
land is first classified as alienable or disposable land open to disposition, and then declared no longer
needed for public service.

The Amended JVA covers not only the Freedom Islands, but also an additional 592.15 hectares
which are still submerged and forming part of Manila Bay. There is no legislative or Presidential
act classifying these submerged areas as alienable or disposable lands of the public domain
open to disposition. These submerged areas are not covered by any patent or certificate of title.
There can be no dispute that these submerged areas form part of the public domain, and in their
present state are inalienable and outside the commerce of man. Until reclaimed from the sea,
these submerged areas are, under the Constitution, "waters x x x owned by the State," forming part of
the public domain and consequently inalienable. Only when actually reclaimed from the sea can
these submerged areas be classified as public agricultural lands, which under the Constitution are the
only natural resources that the State may alienate. Once reclaimed and transformed into public
agricultural lands, the government may then officially classify these lands as alienable or disposable
lands open to disposition. Thereafter, the government may declare these lands no longer needed for
public service. Only then can these reclaimed lands be considered alienable or disposable lands of
the public domain and within the commerce of man.

The classification of PEA's reclaimed foreshore and submerged lands into alienable or disposable
lands open to disposition is necessary because PEA is tasked under its charter to undertake public
services that require the use of lands of the public domain. Under Section 5 of PD No. 1084, the
functions of PEA include the following: "[T]o own or operate railroads, tramways and other kinds of
land transportation, x x x; [T]o construct, maintain and operate such systems of sanitary sewers as
may be necessary; [T]o construct, maintain and operate such storm drains as may be necessary."
PEA is empowered to issue "rules and regulations as may be necessary for the proper use by private
parties of any or all of the highways, roads, utilities, buildings and/or any of its properties and
to impose or collect fees or tolls for their use." Thus, part of the reclaimed foreshore and submerged
lands held by the PEA would actually be needed for public use or service since many of the functions
imposed on PEA by its charter constitute essential public services.

Moreover, Section 1 of Executive Order No. 525 provides that PEA "shall be primarily responsible for
integrating, directing, and coordinating all reclamation projects for and on behalf of the National
Government." The same section also states that "[A]ll reclamation projects shall be approved by the
President upon recommendation of the PEA, and shall be undertaken by the PEA or through a proper
contract executed by it with any person or entity; x x x." Thus, under EO No. 525, in relation to PD No.
3-A and PD No.1084, PEA became the primary implementing agency of the National Government to
reclaim foreshore and submerged lands of the public domain. EO No. 525 recognized PEA as the
government entity "to undertake the reclamation of lands and ensure their maximum utilization
in promoting public welfare and interests."79 Since large portions of these reclaimed lands would
obviously be needed for public service, there must be a formal declaration segregating reclaimed
lands no longer needed for public service from those still needed for public service.1âwphi1.nêt

Section 3 of EO No. 525, by declaring that all lands reclaimed by PEA "shall belong to or be owned
by the PEA," could not automatically operate to classify inalienable lands into alienable or disposable
lands of the public domain. Otherwise, reclaimed foreshore and submerged lands of the public
domain would automatically become alienable once reclaimed by PEA, whether or not classified as
alienable or disposable.

The Revised Administrative Code of 1987, a later law than either PD No. 1084 or EO No. 525, vests
in the Department of Environment and Natural Resources ("DENR" for brevity) the following powers
and functions:

"Sec. 4. Powers and Functions. The Department shall:

(1) x x x

xxx

(4) Exercise supervision and control over forest lands, alienable and disposable public
lands, mineral resources and, in the process of exercising such control, impose appropriate
taxes, fees, charges, rentals and any such form of levy and collect such revenues for the
exploration, development, utilization or gathering of such resources;

xxx

(14) Promulgate rules, regulations and guidelines on the issuance of licenses, permits,
concessions, lease agreements and such other privileges concerning the development,
exploration and utilization of the country's marine, freshwater, and brackish water and
over all aquatic resources of the country and shall continue to oversee, supervise and
police our natural resources; cancel or cause to cancel such privileges upon failure, non-
compliance or violations of any regulation, order, and for all other causes which are in
furtherance of the conservation of natural resources and supportive of the national interest;

(15) Exercise exclusive jurisdiction on the management and disposition of all lands of
the public domain and serve as the sole agency responsible for classification, sub-
classification, surveying and titling of lands in consultation with appropriate
agencies."80 (Emphasis supplied)

As manager, conservator and overseer of the natural resources of the State, DENR exercises
"supervision and control over alienable and disposable public lands." DENR also exercises "exclusive
jurisdiction on the management and disposition of all lands of the public domain." Thus, DENR
decides whether areas under water, like foreshore or submerged areas of Manila Bay, should be
reclaimed or not. This means that PEA needs authorization from DENR before PEA can undertake
reclamation projects in Manila Bay, or in any part of the country.

DENR also exercises exclusive jurisdiction over the disposition of all lands of the public domain.
Hence, DENR decides whether reclaimed lands of PEA should be classified as alienable under
Sections 681 and 782 of CA No. 141. Once DENR decides that the reclaimed lands should be so
classified, it then recommends to the President the issuance of a proclamation classifying the lands
as alienable or disposable lands of the public domain open to disposition. We note that then DENR
Secretary Fulgencio S. Factoran, Jr. countersigned Special Patent No. 3517 in compliance with the
Revised Administrative Code and Sections 6 and 7 of CA No. 141.

In short, DENR is vested with the power to authorize the reclamation of areas under water, while PEA
is vested with the power to undertake the physical reclamation of areas under water, whether directly
or through private contractors. DENR is also empowered to classify lands of the public domain into
alienable or disposable lands subject to the approval of the President. On the other hand, PEA is
tasked to develop, sell or lease the reclaimed alienable lands of the public domain.

Clearly, the mere physical act of reclamation by PEA of foreshore or submerged areas does not make
the reclaimed lands alienable or disposable lands of the public domain, much less patrimonial lands
of PEA. Likewise, the mere transfer by the National Government of lands of the public domain to PEA
does not make the lands alienable or disposable lands of the public domain, much less patrimonial
lands of PEA.

Absent two official acts – a classification that these lands are alienable or disposable and open to
disposition and a declaration that these lands are not needed for public service, lands reclaimed by
PEA remain inalienable lands of the public domain. Only such an official classification and formal
declaration can convert reclaimed lands into alienable or disposable lands of the public domain, open
to disposition under the Constitution, Title I and Title III83of CA No. 141 and other applicable laws.84

PEA's Authority to Sell Reclaimed Lands

PEA, like the Legal Task Force, argues that as alienable or disposable lands of the public domain, the
reclaimed lands shall be disposed of in accordance with CA No. 141, the Public Land Act. PEA, citing
Section 60 of CA No. 141, admits that reclaimed lands transferred to a branch or subdivision of the
government "shall not be alienated, encumbered, or otherwise disposed of in a manner affecting its
title, except when authorized by Congress: x x x."85 (Emphasis by PEA)

In Laurel vs. Garcia,86 the Court cited Section 48 of the Revised Administrative Code of 1987, which
states that –

"Sec. 48. Official Authorized to Convey Real Property. Whenever real property of the
Government is authorized by law to be conveyed, the deed of conveyance shall be
executed in behalf of the government by the following: x x x."

Thus, the Court concluded that a law is needed to convey any real property belonging to the
Government. The Court declared that -

"It is not for the President to convey real property of the government on his or her own sole
will. Any such conveyance must be authorized and approved by a law enacted by the
Congress. It requires executive and legislative concurrence." (Emphasis supplied)

PEA contends that PD No. 1085 and EO No. 525 constitute the legislative authority allowing PEA to
sell its reclaimed lands. PD No. 1085, issued on February 4, 1977, provides that –

"The land reclaimed in the foreshore and offshore area of Manila Bay pursuant to the
contract for the reclamation and construction of the Manila-Cavite Coastal Road Project
between the Republic of the Philippines and the Construction and Development Corporation of
the Philippines dated November 20, 1973 and/or any other contract or reclamation covering
the same area is hereby transferred, conveyed and assigned to the ownership and
administration of the Public Estates Authority established pursuant to PD No. 1084;
Provided, however, That the rights and interests of the Construction and Development
Corporation of the Philippines pursuant to the aforesaid contract shall be recognized and
respected.

Henceforth, the Public Estates Authority shall exercise the rights and assume the obligations of
the Republic of the Philippines (Department of Public Highways) arising from, or incident to,
the aforesaid contract between the Republic of the Philippines and the Construction and
Development Corporation of the Philippines.

In consideration of the foregoing transfer and assignment, the Public Estates Authority shall
issue in favor of the Republic of the Philippines the corresponding shares of stock in said entity
with an issued value of said shares of stock (which) shall be deemed fully paid and non-
assessable.

The Secretary of Public Highways and the General Manager of the Public Estates Authority
shall execute such contracts or agreements, including appropriate agreements with the
Construction and Development Corporation of the Philippines, as may be necessary to
implement the above.

Special land patent/patents shall be issued by the Secretary of Natural Resources in


favor of the Public Estates Authority without prejudice to the subsequent transfer to the
contractor or his assignees of such portion or portions of the land reclaimed or to be
reclaimed as provided for in the above-mentioned contract. On the basis of such
patents, the Land Registration Commission shall issue the corresponding certificate of
title." (Emphasis supplied)

On the other hand, Section 3 of EO No. 525, issued on February 14, 1979, provides that -

"Sec. 3. All lands reclaimed by PEA shall belong to or be owned by the PEA which shall
be responsible for its administration, development, utilization or disposition in accordance with
the provisions of Presidential Decree No. 1084. Any and all income that the PEA may derive
from the sale, lease or use of reclaimed lands shall be used in accordance with the provisions
of Presidential Decree No. 1084."

There is no express authority under either PD No. 1085 or EO No. 525 for PEA to sell its reclaimed
lands. PD No. 1085 merely transferred "ownership and administration" of lands reclaimed from Manila
Bay to PEA, while EO No. 525 declared that lands reclaimed by PEA "shall belong to or be owned by
PEA." EO No. 525 expressly states that PEA should dispose of its reclaimed lands "in accordance
with the provisions of Presidential Decree No. 1084," the charter of PEA.

PEA's charter, however, expressly tasks PEA "to develop, improve, acquire, administer, deal in,
subdivide, dispose, lease and sell any and all kinds of lands x x x owned, managed, controlled
and/or operated by the government."87 (Emphasis supplied) There is, therefore, legislative
authority granted to PEA to sell its lands, whether patrimonial or alienable lands of the public
domain. PEA may sell to private parties its patrimonial properties in accordance with the PEA
charter free from constitutional limitations. The constitutional ban on private corporations from
acquiring alienable lands of the public domain does not apply to the sale of PEA's patrimonial lands.

PEA may also sell its alienable or disposable lands of the public domain to private individuals
since, with the legislative authority, there is no longer any statutory prohibition against such sales and
the constitutional ban does not apply to individuals. PEA, however, cannot sell any of its alienable or
disposable lands of the public domain to private corporations since Section 3, Article XII of the 1987
Constitution expressly prohibits such sales. The legislative authority benefits only individuals. Private
corporations remain barred from acquiring any kind of alienable land of the public domain, including
government reclaimed lands.

The provision in PD No. 1085 stating that portions of the reclaimed lands could be transferred by PEA
to the "contractor or his assignees" (Emphasis supplied) would not apply to private corporations but
only to individuals because of the constitutional ban. Otherwise, the provisions of PD No. 1085 would
violate both the 1973 and 1987 Constitutions.

The requirement of public auction in the sale of reclaimed lands

Assuming the reclaimed lands of PEA are classified as alienable or disposable lands open to
disposition, and further declared no longer needed for public service, PEA would have to conduct a
public bidding in selling or leasing these lands. PEA must observe the provisions of Sections 63 and
67 of CA No. 141 requiring public auction, in the absence of a law exempting PEA from holding a
public auction.88 Special Patent No. 3517 expressly states that the patent is issued by authority of the
Constitution and PD No. 1084, "supplemented by Commonwealth Act No. 141, as amended." This is
an acknowledgment that the provisions of CA No. 141 apply to the disposition of reclaimed alienable
lands of the public domain unless otherwise provided by law. Executive Order No. 654, 89 which
authorizes PEA "to determine the kind and manner of payment for the transfer" of its assets and
properties, does not exempt PEA from the requirement of public auction. EO No. 654 merely
authorizes PEA to decide the mode of payment, whether in kind and in installment, but does not
authorize PEA to dispense with public auction.

Moreover, under Section 79 of PD No. 1445, otherwise known as the Government Auditing Code, the
government is required to sell valuable government property through public bidding. Section 79 of PD
No. 1445 mandates that –

"Section 79. When government property has become unserviceable for any cause, or is no
longer needed, it shall, upon application of the officer accountable therefor, be inspected by
the head of the agency or his duly authorized representative in the presence of the auditor
concerned and, if found to be valueless or unsaleable, it may be destroyed in their presence. If
found to be valuable, it may be sold at public auction to the highest bidder under the
supervision of the proper committee on award or similar body in the presence of the auditor
concerned or other authorized representative of the Commission, after advertising by printed
notice in the Official Gazette, or for not less than three consecutive days in any
newspaper of general circulation, or where the value of the property does not warrant the
expense of publication, by notices posted for a like period in at least three public places in the
locality where the property is to be sold.In the event that the public auction fails, the
property may be sold at a private sale at such price as may be fixed by the same
committee or body concerned and approved by the Commission."

It is only when the public auction fails that a negotiated sale is allowed, in which case the Commission
on Audit must approve the selling price.90 The Commission on Audit implements Section 79 of the
Government Auditing Code through Circular No. 89-29691 dated January 27, 1989. This circular
emphasizes that government assets must be disposed of only through public auction, and a
negotiated sale can be resorted to only in case of "failure of public auction."

At the public auction sale, only Philippine citizens are qualified to bid for PEA's reclaimed foreshore
and submerged alienable lands of the public domain. Private corporations are barred from bidding at
the auction sale of any kind of alienable land of the public domain.

PEA originally scheduled a public bidding for the Freedom Islands on December 10, 1991. PEA
imposed a condition that the winning bidder should reclaim another 250 hectares of submerged areas
to regularize the shape of the Freedom Islands, under a 60-40 sharing of the additional reclaimed
areas in favor of the winning bidder.92No one, however, submitted a bid. On December 23, 1994, the
Government Corporate Counsel advised PEA it could sell the Freedom Islands through negotiation,
without need of another public bidding, because of the failure of the public bidding on December 10,
1991.93

However, the original JVA dated April 25, 1995 covered not only the Freedom Islands and the
additional 250 hectares still to be reclaimed, it also granted an option to AMARI to reclaim another
350 hectares. The original JVA, a negotiated contract, enlarged the reclamation area to 750
hectares.94 The failure of public bidding on December 10, 1991, involving only 407.84 hectares, 95 is
not a valid justification for a negotiated sale of 750 hectares, almost double the area publicly
auctioned. Besides, the failure of public bidding happened on December 10, 1991, more than three
years before the signing of the original JVA on April 25, 1995. The economic situation in the country
had greatly improved during the intervening period.

Reclamation under the BOT Law and the Local Government Code

The constitutional prohibition in Section 3, Article XII of the 1987 Constitution is absolute and clear:
"Private corporations or associations may not hold such alienable lands of the public domain except
by lease, x x x." Even Republic Act No. 6957 ("BOT Law," for brevity), cited by PEA and AMARI as
legislative authority to sell reclaimed lands to private parties, recognizes the constitutional ban.
Section 6 of RA No. 6957 states –

"Sec. 6. Repayment Scheme. - For the financing, construction, operation and maintenance of
any infrastructure projects undertaken through the build-operate-and-transfer arrangement or
any of its variations pursuant to the provisions of this Act, the project proponent x x x may
likewise be repaid in the form of a share in the revenue of the project or other non-monetary
payments, such as, but not limited to, the grant of a portion or percentage of the reclaimed
land, subject to the constitutional requirements with respect to the ownership of the
land: x x x." (Emphasis supplied)

A private corporation, even one that undertakes the physical reclamation of a government BOT
project, cannot acquire reclaimed alienable lands of the public domain in view of the constitutional
ban.

Section 302 of the Local Government Code, also mentioned by PEA and AMARI, authorizes local
governments in land reclamation projects to pay the contractor or developer in kind consisting of a
percentage of the reclaimed land, to wit:

"Section 302. Financing, Construction, Maintenance, Operation, and Management of


Infrastructure Projects by the Private Sector. x x x

xxx

In case of land reclamation or construction of industrial estates, the repayment plan may
consist of the grant of a portion or percentage of the reclaimed land or the industrial estate
constructed."

Although Section 302 of the Local Government Code does not contain a proviso similar to that of the
BOT Law, the constitutional restrictions on land ownership automatically apply even though not
expressly mentioned in the Local Government Code.

Thus, under either the BOT Law or the Local Government Code, the contractor or developer, if a
corporate entity, can only be paid with leaseholds on portions of the reclaimed land. If the contractor
or developer is an individual, portions of the reclaimed land, not exceeding 12 hectares 96 of non-
agricultural lands, may be conveyed to him in ownership in view of the legislative authority allowing
such conveyance. This is the only way these provisions of the BOT Law and the Local Government
Code can avoid a direct collision with Section 3, Article XII of the 1987 Constitution.

Registration of lands of the public domain

Finally, PEA theorizes that the "act of conveying the ownership of the reclaimed lands to public
respondent PEA transformed such lands of the public domain to private lands." This theory is echoed
by AMARI which maintains that the "issuance of the special patent leading to the eventual issuance of
title takes the subject land away from the land of public domain and converts the property into
patrimonial or private property." In short, PEA and AMARI contend that with the issuance of Special
Patent No. 3517 and the corresponding certificates of titles, the 157.84 hectares comprising the
Freedom Islands have become private lands of PEA. In support of their theory, PEA and AMARI cite
the following rulings of the Court:

1. Sumail v. Judge of CFI of Cotabato,97 where the Court held –

"Once the patent was granted and the corresponding certificate of title was issued, the land
ceased to be part of the public domain and became private property over which the Director of
Lands has neither control nor jurisdiction."

2. Lee Hong Hok v. David,98 where the Court declared -

"After the registration and issuance of the certificate and duplicate certificate of title based on a
public land patent, the land covered thereby automatically comes under the operation of
Republic Act 496 subject to all the safeguards provided therein."3. Heirs of Gregorio Tengco v.
Heirs of Jose Aliwalas,99 where the Court ruled -

"While the Director of Lands has the power to review homestead patents, he may do so only
so long as the land remains part of the public domain and continues to be under his exclusive
control; but once the patent is registered and a certificate of title is issued, the land ceases to
be part of the public domain and becomes private property over which the Director of Lands
has neither control nor jurisdiction."

4. Manalo v. Intermediate Appellate Court,100 where the Court held –

"When the lots in dispute were certified as disposable on May 19, 1971, and free patents were
issued covering the same in favor of the private respondents, the said lots ceased to be part of
the public domain and, therefore, the Director of Lands lost jurisdiction over the same."

5.Republic v. Court of Appeals,101 where the Court stated –

"Proclamation No. 350, dated October 9, 1956, of President Magsaysay legally effected a land
grant to the Mindanao Medical Center, Bureau of Medical Services, Department of Health, of
the whole lot, validly sufficient for initial registration under the Land Registration Act. Such land
grant is constitutive of a 'fee simple' title or absolute title in favor of petitioner Mindanao
Medical Center. Thus, Section 122 of the Act, which governs the registration of grants or
patents involving public lands, provides that 'Whenever public lands in the Philippine Islands
belonging to the Government of the United States or to the Government of the Philippines are
alienated, granted or conveyed to persons or to public or private corporations, the same shall
be brought forthwith under the operation of this Act (Land Registration Act, Act 496) and shall
become registered lands.'"
The first four cases cited involve petitions to cancel the land patents and the corresponding
certificates of titles issued to private parties. These four cases uniformly hold that the Director of
Lands has no jurisdiction over private lands or that upon issuance of the certificate of title the land
automatically comes under the Torrens System. The fifth case cited involves the registration under
the Torrens System of a 12.8-hectare public land granted by the National Government to Mindanao
Medical Center, a government unit under the Department of Health. The National Government
transferred the 12.8-hectare public land to serve as the site for the hospital buildings and other
facilities of Mindanao Medical Center, which performed a public service. The Court affirmed the
registration of the 12.8-hectare public land in the name of Mindanao Medical Center under Section
122 of Act No. 496. This fifth case is an example of a public land being registered under Act No. 496
without the land losing its character as a property of public dominion.

In the instant case, the only patent and certificates of title issued are those in the name of PEA, a
wholly government owned corporation performing public as well as proprietary functions. No patent or
certificate of title has been issued to any private party. No one is asking the Director of Lands to
cancel PEA's patent or certificates of title. In fact, the thrust of the instant petition is that PEA's
certificates of title should remain with PEA, and the land covered by these certificates, being alienable
lands of the public domain, should not be sold to a private corporation.

Registration of land under Act No. 496 or PD No. 1529 does not vest in the registrant private or public
ownership of the land. Registration is not a mode of acquiring ownership but is merely evidence of
ownership previously conferred by any of the recognized modes of acquiring ownership. Registration
does not give the registrant a better right than what the registrant had prior to the registration. 102 The
registration of lands of the public domain under the Torrens system, by itself, cannot convert public
lands into private lands.103

Jurisprudence holding that upon the grant of the patent or issuance of the certificate of title the
alienable land of the public domain automatically becomes private land cannot apply to government
units and entities like PEA. The transfer of the Freedom Islands to PEA was made subject to the
provisions of CA No. 141 as expressly stated in Special Patent No. 3517 issued by then President
Aquino, to wit:

"NOW, THEREFORE, KNOW YE, that by authority of the Constitution of the Philippines and in
conformity with the provisions of Presidential Decree No. 1084, supplemented by
Commonwealth Act No. 141, as amended, there are hereby granted and conveyed unto the
Public Estates Authority the aforesaid tracts of land containing a total area of one million nine
hundred fifteen thousand eight hundred ninety four (1,915,894) square meters; the technical
description of which are hereto attached and made an integral part hereof." (Emphasis
supplied)

Thus, the provisions of CA No. 141 apply to the Freedom Islands on matters not covered by PD No.
1084. Section 60 of CA No. 141 prohibits, "except when authorized by Congress," the sale of
alienable lands of the public domain that are transferred to government units or entities. Section 60 of
CA No. 141 constitutes, under Section 44 of PD No. 1529, a "statutory lien affecting title" of the
registered land even if not annotated on the certificate of title. 104 Alienable lands of the public domain
held by government entities under Section 60 of CA No. 141 remain public lands because they
cannot be alienated or encumbered unless Congress passes a law authorizing their disposition.
Congress, however, cannot authorize the sale to private corporations of reclaimed alienable lands of
the public domain because of the constitutional ban. Only individuals can benefit from such law.

The grant of legislative authority to sell public lands in accordance with Section 60 of CA No. 141
does not automatically convert alienable lands of the public domain into private or patrimonial lands.
The alienable lands of the public domain must be transferred to qualified private parties, or to
government entities not tasked to dispose of public lands, before these lands can become private or
patrimonial lands. Otherwise, the constitutional ban will become illusory if Congress can declare lands
of the public domain as private or patrimonial lands in the hands of a government agency tasked to
dispose of public lands. This will allow private corporations to acquire directly from government
agencies limitless areas of lands which, prior to such law, are concededly public lands.

Under EO No. 525, PEA became the central implementing agency of the National Government to
reclaim foreshore and submerged areas of the public domain. Thus, EO No. 525 declares that –

"EXECUTIVE ORDER NO. 525

Designating the Public Estates Authority as the Agency Primarily Responsible for all
Reclamation Projects

Whereas, there are several reclamation projects which are ongoing or being proposed to be
undertaken in various parts of the country which need to be evaluated for consistency with
national programs;

Whereas, there is a need to give further institutional support to the Government's declared
policy to provide for a coordinated, economical and efficient reclamation of lands;

Whereas, Presidential Decree No. 3-A requires that all reclamation of areas shall be limited to
the National Government or any person authorized by it under proper contract;

Whereas, a central authority is needed to act on behalf of the National Government


which shall ensure a coordinated and integrated approach in the reclamation of lands;

Whereas, Presidential Decree No. 1084 creates the Public Estates Authority as a
government corporation to undertake reclamation of lands and ensure their maximum
utilization in promoting public welfare and interests; and

Whereas, Presidential Decree No. 1416 provides the President with continuing authority to
reorganize the national government including the transfer, abolition, or merger of functions and
offices.

NOW, THEREFORE, I, FERDINAND E. MARCOS, President of the Philippines, by virtue of


the powers vested in me by the Constitution and pursuant to Presidential Decree No. 1416, do
hereby order and direct the following:

Section 1. The Public Estates Authority (PEA) shall be primarily responsible for
integrating, directing, and coordinating all reclamation projects for and on behalf of the
National Government. All reclamation projects shall be approved by the President upon
recommendation of the PEA, and shall be undertaken by the PEA or through a proper contract
executed by it with any person or entity; Provided, that, reclamation projects of any national
government agency or entity authorized under its charter shall be undertaken in consultation
with the PEA upon approval of the President.

x x x ."
As the central implementing agency tasked to undertake reclamation projects nationwide, with
authority to sell reclaimed lands, PEA took the place of DENR as the government agency charged
with leasing or selling reclaimed lands of the public domain. The reclaimed lands being leased or sold
by PEA are not private lands, in the same manner that DENR, when it disposes of other alienable
lands, does not dispose of private lands but alienable lands of the public domain. Only when qualified
private parties acquire these lands will the lands become private lands. In the hands of the
government agency tasked and authorized to dispose of alienable of disposable lands of the
public domain, these lands are still public, not private lands.

Furthermore, PEA's charter expressly states that PEA "shall hold lands of the public domain" as
well as "any and all kinds of lands." PEA can hold both lands of the public domain and private lands.
Thus, the mere fact that alienable lands of the public domain like the Freedom Islands are transferred
to PEA and issued land patents or certificates of title in PEA's name does not automatically make
such lands private.

To allow vast areas of reclaimed lands of the public domain to be transferred to PEA as private lands
will sanction a gross violation of the constitutional ban on private corporations from acquiring any kind
of alienable land of the public domain. PEA will simply turn around, as PEA has now done under
the Amended JVA, and transfer several hundreds of hectares of these reclaimed and still to be
reclaimed lands to a single private corporation in only one transaction. This scheme will effectively
nullify the constitutional ban in Section 3, Article XII of the 1987 Constitution which was intended to
diffuse equitably the ownership of alienable lands of the public domain among Filipinos, now
numbering over 80 million strong.

This scheme, if allowed, can even be applied to alienable agricultural lands of the public domain since
PEA can "acquire x x x any and all kinds of lands." This will open the floodgates to corporations and
even individuals acquiring hundreds of hectares of alienable lands of the public domain under the
guise that in the hands of PEA these lands are private lands. This will result in corporations amassing
huge landholdings never before seen in this country - creating the very evil that the constitutional ban
was designed to prevent. This will completely reverse the clear direction of constitutional
development in this country. The 1935 Constitution allowed private corporations to acquire not more
than 1,024 hectares of public lands.105 The 1973 Constitution prohibited private corporations from
acquiring any kind of public land, and the 1987 Constitution has unequivocally reiterated this
prohibition.

The contention of PEA and AMARI that public lands, once registered under Act No. 496 or PD No.
1529, automatically become private lands is contrary to existing laws. Several laws authorize lands of
the public domain to be registered under the Torrens System or Act No. 496, now PD No. 1529,
without losing their character as public lands. Section 122 of Act No. 496, and Section 103 of PD No.
1529, respectively, provide as follows:

Act No. 496

"Sec. 122. Whenever public lands in the Philippine Islands belonging to the x x x Government
of the Philippine Islands are alienated, granted, or conveyed to persons or the public or
private corporations, the same shall be brought forthwith under the operation of this Act and
shall become registered lands."

PD No. 1529
"Sec. 103. Certificate of Title to Patents. Whenever public land is by the Government alienated,
granted or conveyed to any person, the same shall be brought forthwith under the operation
of this Decree." (Emphasis supplied)

Based on its legislative history, the phrase "conveyed to any person" in Section 103 of PD No. 1529
includes conveyances of public lands to public corporations.

Alienable lands of the public domain "granted, donated, or transferred to a province, municipality, or
branch or subdivision of the Government," as provided in Section 60 of CA No. 141, may be
registered under the Torrens System pursuant to Section 103 of PD No. 1529. Such registration,
however, is expressly subject to the condition in Section 60 of CA No. 141 that the land "shall not be
alienated, encumbered or otherwise disposed of in a manner affecting its title, except when
authorized by Congress." This provision refers to government reclaimed, foreshore and marshy
lands of the public domain that have been titled but still cannot be alienated or encumbered unless
expressly authorized by Congress. The need for legislative authority prevents the registered land of
the public domain from becoming private land that can be disposed of to qualified private parties.

The Revised Administrative Code of 1987 also recognizes that lands of the public domain may be
registered under the Torrens System. Section 48, Chapter 12, Book I of the Code states –

"Sec. 48. Official Authorized to Convey Real Property. Whenever real property of the
Government is authorized by law to be conveyed, the deed of conveyance shall be executed in
behalf of the government by the following:

(1) x x x

(2) For property belonging to the Republic of the Philippines, but titled in the name of
any political subdivision or of any corporate agency or instrumentality, by the executive
head of the agency or instrumentality." (Emphasis supplied)

Thus, private property purchased by the National Government for expansion of a public wharf may be
titled in the name of a government corporation regulating port operations in the country. Private
property purchased by the National Government for expansion of an airport may also be titled in the
name of the government agency tasked to administer the airport. Private property donated to a
municipality for use as a town plaza or public school site may likewise be titled in the name of the
municipality.106 All these properties become properties of the public domain, and if already registered
under Act No. 496 or PD No. 1529, remain registered land. There is no requirement or provision in
any existing law for the de-registration of land from the Torrens System.

Private lands taken by the Government for public use under its power of eminent domain become
unquestionably part of the public domain. Nevertheless, Section 85 of PD No. 1529 authorizes the
Register of Deeds to issue in the name of the National Government new certificates of title covering
such expropriated lands. Section 85 of PD No. 1529 states –

"Sec. 85. Land taken by eminent domain. Whenever any registered land, or interest therein, is
expropriated or taken by eminent domain, the National Government, province, city or
municipality, or any other agency or instrumentality exercising such right shall file for
registration in the proper Registry a certified copy of the judgment which shall state definitely
by an adequate description, the particular property or interest expropriated, the number of the
certificate of title, and the nature of the public use. A memorandum of the right or interest taken
shall be made on each certificate of title by the Register of Deeds, and where the fee simple is
taken, a new certificate shall be issued in favor of the National Government, province,
city, municipality, or any other agency or instrumentality exercising such right for the land so
taken. The legal expenses incident to the memorandum of registration or issuance of a new
certificate of title shall be for the account of the authority taking the land or interest therein."
(Emphasis supplied)

Consequently, lands registered under Act No. 496 or PD No. 1529 are not exclusively private or
patrimonial lands. Lands of the public domain may also be registered pursuant to existing laws.

AMARI makes a parting shot that the Amended JVA is not a sale to AMARI of the Freedom Islands or
of the lands to be reclaimed from submerged areas of Manila Bay. In the words of AMARI, the
Amended JVA "is not a sale but a joint venture with a stipulation for reimbursement of the original
cost incurred by PEA for the earlier reclamation and construction works performed by the CDCP
under its 1973 contract with the Republic." Whether the Amended JVA is a sale or a joint venture, the
fact remains that the Amended JVA requires PEA to "cause the issuance and delivery of the
certificates of title conveying AMARI's Land Share in the name of AMARI." 107

This stipulation still contravenes Section 3, Article XII of the 1987 Constitution which provides that
private corporations "shall not hold such alienable lands of the public domain except by lease." The
transfer of title and ownership to AMARI clearly means that AMARI will "hold" the reclaimed lands
other than by lease. The transfer of title and ownership is a "disposition" of the reclaimed lands, a
transaction considered a sale or alienation under CA No. 141,108 the Government Auditing
Code,109 and Section 3, Article XII of the 1987 Constitution.

The Regalian doctrine is deeply implanted in our legal system. Foreshore and submerged areas form
part of the public domain and are inalienable. Lands reclaimed from foreshore and submerged areas
also form part of the public domain and are also inalienable, unless converted pursuant to law into
alienable or disposable lands of the public domain. Historically, lands reclaimed by the government
are sui generis, not available for sale to private parties unlike other alienable public lands. Reclaimed
lands retain their inherent potential as areas for public use or public service. Alienable lands of the
public domain, increasingly becoming scarce natural resources, are to be distributed equitably among
our ever-growing population. To insure such equitable distribution, the 1973 and 1987 Constitutions
have barred private corporations from acquiring any kind of alienable land of the public domain.
Those who attempt to dispose of inalienable natural resources of the State, or seek to circumvent the
constitutional ban on alienation of lands of the public domain to private corporations, do so at their
own risk.

We can now summarize our conclusions as follows:

1. The 157.84 hectares of reclaimed lands comprising the Freedom Islands, now covered by
certificates of title in the name of PEA, are alienable lands of the public domain. PEA may
lease these lands to private corporations but may not sell or transfer ownership of these lands
to private corporations. PEA may only sell these lands to Philippine citizens, subject to the
ownership limitations in the 1987 Constitution and existing laws.

2. The 592.15 hectares of submerged areas of Manila Bay remain inalienable natural
resources of the public domain until classified as alienable or disposable lands open to
disposition and declared no longer needed for public service. The government can make such
classification and declaration only after PEA has reclaimed these submerged areas. Only then
can these lands qualify as agricultural lands of the public domain, which are the only natural
resources the government can alienate. In their present state, the 592.15 hectares of
submerged areas are inalienable and outside the commerce of man.
3. Since the Amended JVA seeks to transfer to AMARI, a private corporation, ownership of
77.34 hectares110 of the Freedom Islands, such transfer is void for being contrary to Section 3,
Article XII of the 1987 Constitution which prohibits private corporations from acquiring any kind
of alienable land of the public domain.

4. Since the Amended JVA also seeks to transfer to AMARI ownership of 290.156
hectares111 of still submerged areas of Manila Bay, such transfer is void for being contrary to
Section 2, Article XII of the 1987 Constitution which prohibits the alienation of natural
resources other than agricultural lands of the public domain. PEA may reclaim these
submerged areas. Thereafter, the government can classify the reclaimed lands as alienable or
disposable, and further declare them no longer needed for public service. Still, the transfer of
such reclaimed alienable lands of the public domain to AMARI will be void in view of Section 3,
Article XII of the 1987 Constitution which prohibits private corporations from acquiring any kind
of alienable land of the public domain.

Clearly, the Amended JVA violates glaringly Sections 2 and 3, Article XII of the 1987 Constitution.
Under Article 1409112 of the Civil Code, contracts whose "object or purpose is contrary to law," or
whose "object is outside the commerce of men," are "inexistent and void from the beginning." The
Court must perform its duty to defend and uphold the Constitution, and therefore declares the
Amended JVA null and void ab initio.

Seventh issue: whether the Court is the proper forum to raise the issue of whether the
Amended JVA is grossly disadvantageous to the government.

Considering that the Amended JVA is null and void ab initio, there is no necessity to rule on this last
issue. Besides, the Court is not a trier of facts, and this last issue involves a determination of factual
matters.

WHEREFORE, the petition is GRANTED. The Public Estates Authority and Amari Coastal Bay
Development Corporation are PERMANENTLY ENJOINED from implementing the Amended Joint
Venture Agreement which is hereby declared NULL and VOID ab initio.

SO ORDERED.

[G.R. No. 14167. August 14, 1919. ]

THE GOVERNMENT OF THE PHILIPPINE ISLANDS, petitioner and appellee. ANTIPAS


VAZQUEZ and BASILIO GAYARES, Petitioners-Appellants, v. RUFINA ABURAL ET. AL.,
objectors-appellees.

Cohn & Fisher, for Appellants.

Hilado & Hilado, for Appellees.

SYLLABUS

1. LAND REGISTRATION; TORRENS SYSTEM; PURPOSE. — The prime purpose of the Torrens
System, as established in the Philippine Islands by the Land Registration Law (Act No. 496), is to
decree land titles that shall be final, irrevocable, and indisputable.

2. ID.; CADASTRAL SYSTEM; PURPOSE. — The purpose of the offspring of the Torrens System
here known as the Cadastral System, as established in the Philippine Islands by the Cadastral Act
(No. 2259), is, like the purpose of the Torrens System, proper incontestability of title. As stated in
Section 1 of the Cadastral Act, the purpose is to serve the public interest, by requiring that the titles to
any lands "be settled and adjudicated."cralaw virtua1aw library

3. ID.; ID.; PROCEEDINGS. — Many precautions are taken to guard against injustice.

4. ID.; ID.; ID. — After trial in a cadastral case, three actions are taken. The first adjudicates
ownership in favor of one of the claimants. This constitutes the decision — the judgment — the
decree of the court. The second action is the declaration by the court that the decree is final and its
order for the issuance of the certificates of title by the Chief of the Land Registration Office. Such
order is made if within thirty days from the date of receipt of a copy of the decision no appeal is taken
from the decision. The third and last action devolves upon the General Land Registration Office.

5. ID.; ID.; ID.; FINALITY OF DECREE. — For a decree to exist in legal contemplation, it is not
necessary to await the preparation of a so-called decree by the Land Registration Office.

6. ID.; ID.; ID.; ID. — Cadastral proceedings commenced. Notice published in the Official Gazette.
Trial judge also issued general notice. S asks for the registration in his name of lot No. 1608. Hearing
had. On September 21, 1916, the court in a decree awarded the lot to S. On November 23, 1916, the
time for an appeal having passed, the court declares the decree final. On July 23, 1917, before the
issuance by the Land Registration Office of the so-called technical decree, V and G ask that the case
be reopened to receive proof relative to the ownership of the lot. Motion denied by the trial court.
Held: That since the judgment of the Court of First Instance of September 21, 1916, has become
final, and since no action was taken within the time provided by law for the prosecution of
an appeal by bill of exceptions, the Supreme Court is without jurisdiction, and the appeal must be
dismissed.

7. ID.; ID.; RELIEF FROM JUDGMENT. — Whether Sections 113 and 513 of the Code of Civil
Procedure apply to cadastral proceedings, quare.

8. GENERAL LAND REGISTRATION OFFICE. — The General Land Registration Office has been
instituted "for the due effectuation and accomplishment of the laws relative to the registration of land."
(Administrative Code of 1917, Sec. 174.)

DECISION

MALCOLM, J. :

The principal question which this appeal presents is — When does the registration of title, under the
Torrens System of Land Registration, especially under the different Philippine laws establishing the
Cadastral System, become final, conclusive, and indisputable? The supplementary questions are —
At what stage of the cadastral proceedings does a decree exist in legal contemplation? Does it exist
from the moment that the court, after hearing the evidence, adjudicates the land in favor of a person
and then, or later decrees the land in favor of this person, or does it exist when the Chief of the Land
Registration Office transcribes the adjudication in the prescribed form?

STATEMENT OF THE CASE.


Cadastral proceedings were commenced in the municipality of Hinigaran, Province of Occidental
Negros, upon an application of the Director of Lands, on June 16, 1916. Notice of the proceedings
were published in the Official Gazette as provided by law. The trial judge also issued general notice to
all interested parties. Among others, Victoriano Siguenza presented an answer asking for registration
in his name of lot No. 1608. The instant petitioners, Antipas Vazquez and Basilio Gayares, although
said to reside in this municipality, and although said to have participated in other cadastral cases, did
not enter any opposition as to this lot. Hearing was had during September, 1916. On September 21 of
this year, the court issued the following decree:jgc:chanrobles.com.ph

"It is hereby decreed that, upon a previous declaration of general default, the following lots be
adjudged and registered in the names of those persons whose names appear next after the lots, and
in accordance with the following conditions: . . .

"Lot No. 1608 with the improvements thereon to the conjugal partnership of Victoriano Siguenza and
Marcela Guanzon."cralaw virtua1aw library

On November 23 of the same year, the court declared final the foregoing decree in the following
language:jgc:chanrobles.com.ph

"The decision rendered by the court in the above-entitled case having become final on September 21,
1916, it is hereby ordered that the Chief of the General Land Registration Office issue the decrees
corresponding to the lots adjudged by said decision.

"An appeal having however been interposed as to the lots enumerated as follows, the decrees
thereon, must be suspended until further order by this court:jgc:chanrobles.com.ph

"Lot No. 521.

Eight months later, that is, on July 23, 1917, but before the issuance by the Land Registration Office
of the so-called technical decree, Antipas Vazquez and Basilio Gayares, the latter as guardian of the
minor Estrella Vazquez, came into the case for the first time. The petitioners, after setting forth their
right of ownership in lot No. 1608, and that it was included in their "Hacienda Santa Filomena," and
after stating that they were in complete ignorance of the proceedings, asked that the judgment of the
court be annulled and that the case be reopened to receive proof relative to the ownership of the lot.
Counsel for Victoriano Siguenza answered by counter-motion, asking the court to dismiss the motion
presented on behalf of Vazquez and Gayares. The court denied the motion for a new trial on the
theory that there being a decree already rendered and no allegation of fraud having been made, the
court lacked jurisdiction. It may also be stated parenthetically that counsel for Vazquez and Gayares
made an unsuccessful attempt in the Supreme Court, through mandamus, to have the record
completed by the taking of evidence.

In order that the matter may not be confused, let it again be made clear that counsel for petitioners
have not raised the question of fraud as provided for in Section 38 of the Land Registration Law, nor
have they asked to be relieved from a judgment or order, pursuant to Section 113 of the Code of Civil
Procedure, because of mistake, inadvertence, surprise, or excusable neglect. As a matter of fact,
they could not well claim fraud because all the proceedings were public and free from any suspicion
of chicanery. As a matter of fact, also, any special reliance on Section 113 of the Code of Civil
Procedure would not get them anywhere because more than six months had elapsed after the
issuance of a judgment in this case. The issue fundamentally becomes one of whether or not
the Supreme Court has jurisdiction over the appeal, since if the judgment and the supplemental
decree issued by the Judge of the Court of First Instance on September 21, 1916, and November 23,
1916, respectively, have become final, petitioners may no bring their appeal before this court,
because the time for the filing of their bill of exceptions has expired; while, if the cadastral
proceedings did not become final until the formal decree was issued by the Land Registration Office,
then it was proper for them to ask for a reopening of the case, and it would, consequently, be just as
proper for this court to order the trial court to permit the same.

OPINION.

The prime purpose of the Torrens System is, as has been repeatedly stated, to decree land titles that
shall be final, irrevocable, and indisputable. Incontestability of title is the goal. All due precaution must
accordingly be taken to guard against injustice to interested individuals who, for some good reason,
may not be able to protect their rights. Nevertheless, even at the cost of possible cruelty which may
result in exceptional cases, it does become necessary in the interest of the public weal to enforce
registration laws. No stronger words can be found than those appearing in Section 38 of the Land
Registration Law (Act No. 496) wherein it is said that: "Every decree of registration shall bind the land,
and quiet title thereto. . . . It shall be conclusive upon and against all persons, including the Insular
Government and all the branches thereof, whether mentioned by name in the application, notice, or
citation, or included in the general description ’To all whom it may concern,’ Such decree shall not be
opened by reason of the absence, infancy, or other disability of any person affected thereby, nor by
any proceeding in any court for reversing judgments or decrees; subject, however, to the right of any
person deprived of land or of any estate or interest therein by decree of registration obtained by fraud
to file in the Court of Land Registration (Court of First Instance) a petition for review within one year
after entry of the decree, provided no innocent purchaser for value has acquired an
interest."cralaw virtua1aw library

While such statements can be made of the Torrens System proper, they become even more incisive
and peremptory when we come to consider the offspring of this system, here known as the Cadastral
System. Under the Torrens System proper, whether action shall or shall not be taken is optional with
the solicitant. Under the Cadastral System, pursuant to initiative on the part of the Government, titles
for all the land within a stated area, are adjudicated whether or not the people living within this district
desire to have titles issued. The purpose, as stated in section one of the Cadastral Act (NO. 2259), is
to serve the public interests, by requiring that the titles to any lands "be settled and
adjudicated."cralaw virtua1aw library

Admitting that such compulsory registration of land and such excessive interference with private
property constitutes due process of law and that the Acts providing for the same are constitutional, a
question not here raised, yet a study of the law indicates that many precautions are taken to guard
against injustice. The proceedings are initiated by a notice of survey. When the lands have been
surveyed and plotted, the Director of Lands, represented by the Attorney General, files a petition in
court praying that the titles to the lands named be settled and adjudicated. Notice of the filing of
the petition is then published twice in successive issues of the Official Gazette in both the English and
Spanish languages. All persons interested are given the benefit of assistance by competent officials
and are informed of their rights. A trial is had. "All conflicting interests shall be adjudicated by the
court and decrees awarded in favor of the persons entitled to the lands or the various parts thereof,
and such decrees, when final, shall be the bases of original certificates of title in favor of said
persons." (Act No. 2259, Sec. 11.) Aside from this, the commotion caused by the survey and a trial
affecting ordinarily many people, together with the presence of strangers in the community, should
serve to put all those affected on their guard.

After trial in a cadastral case, three actions are taken. The first adjudicates ownership in favor of one
of the claimants. This constitutes the decision — the judgment — the decree of the court, and speaks
in a judicial manner. The second action is the declaration by the court that the decree is final and its
order for the issuance of the certificates of title by the Chief of the Land Registration Office. Such
order is made if within thirty days from the date of receipt of a copy of the decision no appeal is taken
from the decision. This again is judicial action, although to a less degree than the first.

The third and last action devolves upon the General Land Registration Office. This office has been
instituted "for the due effectuation and accomplishment of the laws relative to the registration of land."
(Administrative Code of 1917, Sec. 174.) An official found in the office, known as the chief surveyor,
has as one of his duties "to prepare final decrees in all adjudicated cases." (Administrative Code of
1917, Sec. 177.) This latter decree contains the technical description of the land and may not be
issued until a considerable time after the promulgation of the judgment. The form for the decree used
by the General Land Registration Office concludes with the words: "Witness, the Honorable (name of
the judge), on this the (date)." The date that is used as authority for the issuance of the decree is the
date when, after hearing the evidence, the trial court decreed the adjudication and registration of the
land.

The judgment in a cadastral survey, including the rendition of the decree, is a judicial act. As the law
says, the judicial decree when final is the base of the certificate of title. The issuance of the decree by
the Land Registration Office is ministerial act. The date of the judgment, or more correctly stated, the
date on which the defeated party receives a copy of the decision, begins the running of the time for
the interposition of a motion for a new trial or for the perfection of an appeal to the Supreme Court.
The date of the title prepared by the Chief Surveyor is unimportant, for the adjudication has taken
place and all that is left to be performed is the mere formulation of the technical description. If an
unknown individual could wait possibly years until the day before a surveyor gets around to
transcribing a technical description of a piece of land, the defeated party could just as reasonably
expect the same consideration for his appeal. As a matter of fact, the so-called unknown is a party
just as much as the known oppositor for notice is to all the world, and the decree binds all the world.

Both counsel for petitioners and respondents rely upon the decision of this court in the case of
Tambunting v. Manuel ([1916], 35 Phil.; 699) . That case and the instant case are not the same. In
the Tambunting case the contest was really between two parties each claiming to have a Torrens
title; here one party has the title and the other is seeking to oust him from his fortress. In the
Tambunting case the declaration of ownership but not the decree of registration had issued; here
both declaration and decree have issued. The doctrines announced in the decision in Grey Alba v. De
la Cruz ([1910], 17 Phil., 49) relating to general notice and the indefensibility of land titles under the
Torrens system are much more applicable and can, with as much reason, be applied to the cadastral
system.

As a general rule, registration of title under the cadastral system is final, conclusive and indisputable,
after the passage of the thirty-day period allowed for an appeal from the date of receipt by the party of
a copy of the judgment of the court adjudicating ownership without any step having been taken to
perfect an appeal. The prevailing party may then have execution of the judgment as of right and is
entitled to the certificate of title issued by the chief of the Land Registration Office. The exception is
the special provision providing for fraud.

Counsel for appellants and appellees have favored the court with able arguments relative to the
applicability of Sections 113 and 513 of the Code of Civil Procedure to cadastral proceedings. The
view we take of the case would make unprofitable any discussion of this question.

It appearing that the judgment of the Court of First Instance of Occidental Negros of September 21,
1916, has become final, and that no action was taken within the time provided by law for the
prosecution of an appeal by bill of exceptions, this court is without jurisdiction. Accordingly
the appeal is dismissed with costs against the appellants. So ordered.
Arellano, C.J., Torres, Johnson, Street, Avanceña and Moir, JJ., concur.

G.R. No. 123586 August 12, 2004

SPOUSES BEDER MORANDARTE and MARINA FEBRERA, petitioners,


vs.
COURT OF APPEALS, REPUBLIC OF THE PHILIPPINES, and SPOUSES VIRGINIO B. LACAYA
and NENITA LACAYA, respondents.

DECISION

AUSTRIA-MARTINEZ, J.:

Before us is a petition for review on certiorari under Rule 45 of the Rules of Court which seeks the
reversal of the Decision,1 dated August 23, 1995, of the Court of Appeals (CA for brevity) in CA-G.R.
CV No. 36258, affirming the Decision, dated November 5, 1991, rendered by the Regional Trial Court
(Branch 7), Dipolog City, Zamboanga del Norte (RTC for brevity) in Civil Case No. 3890, declaring
Free Patent No. (IX-8) 7852 and Original Certificate of Title No. P-21972, in the name of petitioner
Beder Morandarte (Morandarte for brevity), and all its derivative titles, null and void ab initio.

The factual antecedents are as follows:

Morandarte filed an application for free patent, dated December 5, 1972, before the Bureau of Lands,
Dipolog City District Land Office (BOL for brevity), covering a parcel of land located at Sta. Filomena,
Dipolog City with an area of 4.5499 hectares and described as a portion of Lot 1038 of Dipolog
Cadastre No. 85.3

On July 27, 1976, the District Land Officer of the BOL approved the free patent application of
Morandarte and directed the issuance of a free patent in his favor. 4 Accordingly, Free Patent No. (IX-
8) 785 for Lot No. 7, Csd-09-05-00078-D was issued in the name of Morandarte. On September 20,
1976, the Register of Deeds of Zamboanga del Norte issued the corresponding Original Certificate of
Title No. (P-21972) 5954.5

Subsequently, Morandarte caused a subdivision survey of the lot, dividing the same into Lot No.
6781-A, with an area of 13,939 square meters, and Lot No. 6781-B, with an area of 32,819 square
meters. As a result of the subdivision survey, Transfer Certificates of Title Nos. T-1835 and T-1836
covering Lots 6781-A and 6781-B, respectively, were issued in favor of Morandarte on May 12, 1980
by the Registry of Deeds of Dipolog City.6

On May 22, 1981, Morandarte and his wife, Marina Febrera, executed a real estate mortgage over
Lot 6781-B, subject of TCT No. 1836, in favor of the Development Bank of the Philippines, Dipolog
City branch (DBP for brevity), in consideration of a loan in the amount of P52,160.00.7
More than ten years after the issuance of the OCT in Morandarte's name, or on March 19, 1987,
respondent Republic of the Philippines (Republic for brevity), represented by the Director of Lands,
filed before the RTC a Complaint for Annulment of Title and Reversion against the Morandarte
spouses, the Register of Deeds of Zamboanga del Norte, the Register of Deeds of Dipolog City, and
DBP, docketed as Civil Case No. 3890.8

The Republic alleged that the BOL found that the subject land includes a portion of the Miputak River
which cannot be validly awarded as it is outside the commerce of man and beyond the authority of
the BOL to dispose of. It claimed that the Morandarte spouses deliberately and intentionally
concealed such fact in the application to ensure approval thereof. Considering that the Morandarte
spouses are guilty of fraud and misrepresentation in the procurement of their title, the Republic
stressed that their title is void.9

The Register of Deeds of Dipolog City filed a Motion to Dismiss, dated April 7, 1987, praying for the
dismissal of the complaint as against her since the complaint failed to state a claim against her. 10

In their Answer dated April 13, 1987, the Morandarte spouses denied the allegations of the complaint
and claimed that they were able to secure the title in accordance and in compliance with the
requirements of the law. They alleged that the land is a portion of inherited property from Antonio L.
Morandarte whose ownership thereof is covered by Tax Declaration No. 2296.

As regards the Miputak River, they argued that the river changed its course brought about by the fact
that a portion of the Miputak River was leased by the Bureau of Fisheries (BOF for brevity) to a
certain Aguido Realiza whose rights were subsequently transferred to Virginio Lacaya. They alleged
that they indicated in their survey plan the actual location of the Miputak River in relation to the
property but the BOL returned the survey with the directive that the existence of the river should not
be indicated as the original survey did not show its existence, to which they complied with by
submitting a new survey plan which did not indicate the existence of the river.

In the alternative, they alleged that inclusion of the Miputak River should not render the title void; only
the portion of the property covered by the Miputak River should be nullified but their title to the
remaining portion should be maintained.11

For its part, DBP filed its Answer dated April 13, 1987 praying for the dismissal of the complaint as
against it since it had nothing to do with the issuance of the title to the spouses. 12 DBP interposed a
cross-claim against the spouses for the payment of their outstanding obligations. 13 The Morandarte
spouses filed an Answer to the Crossclaim dated April 29, 1987.14

No answer was filed by the Register of Deeds of Zamboanga del Norte.

On March 4, 1988, upon prior leave of court, herein respondent spouses Virginio B. Lacaya and
Nenita Lacaya filed their Complaint-In-Intervention which alleged that they are holders of a fishpond
lease agreement covering a fishpond area of about 5.0335 hectares, 1.2681 hectares of which have
been included in the title issued to the Morandarte spouses. Considering that the land of the
Morandarte spouses encroaches on the area leased to them, the Lacaya spouses submit that the
former's title thereto is void.15

In their Answer to the complaint-in-intervention, dated March 19, 1988, the Morandarte spouses
denied the allegations of the Lacaya spouses.16 They maintained that the portion of the fishpond
originally belonged to Antonio L. Morandarte, their predecessor-in-interest, and the Lacaya spouses
have never been in possession thereof but are actually squatters therein.
On the other hand, the Republic, in its Answer to the complaint-in-intervention, dated March 21, 1988,
adopted the allegations of the complaint-in-intervention to further support its claim that the title of the
Morandarte spouses is void.17 The Lacaya spouses filed their Reply and Answer on March 30, 1988,
denying the arguments of the Morandarte spouses and reiterating the allegations in their complaint-
in-intervention.18

Following trial on the merits, on November 5, 1992, the RTC rendered a Decision 19 in favor of the
Republic and the Lacaya spouses. The RTC declared that while fraud in the procurement of the title
was not established by the State, Morandarte's title is, nonetheless, void because it includes a portion
of the Miputak River which is outside the commerce of man and beyond the authority of the BOL to
dispose of. In addition, the RTC sustained the fishpond rights of the Lacaya spouses over a portion
included in Morandarte's title based on a Deed of Transfer of Fishpond Rights from Felipe B. Lacaya
and a Fishpond Lease Agreement with the BOF.

The dispositive portion of the decision of the trial court reads:

WHEREFORE, judgment is hereby rendered:

1. Declaring null and void ab initio Free Patent No. (IX-5) (sic) 785 and Original Certificate of
Title No. P-21972 in the name of Beder Morandarte, as well as all derivative titles issued
thereafter;

2. Ordering defendants spouses Beder Morandarte and Marina Febrera to surrender their
owner's duplicate copies of Transfer Certificate of Title Nos. T-1835 and T-1836, which were
the derivative titles of Original Certificate of Title No. P-21972;

3. Directing the Register of Deeds of Zamboanga del Norte to cancel Original Certificate of
Title No. P-21972 in the name of Beder Morandarte, and the Register of Deeds of Dipolog City
to cancel Transfer Certificate of Title Nos. T-1835 and T-1836 in the name of the same
defendant;

4. Ordering the reversion of the land in question to the state, free from liens and
encumbrances;

5. Enjoining defendants spouses Beder Morandarte and Marina Febrera from exercising any
act of ownership or possession of the subject property;

6. Dismissing the Cross-Claim of defendant Development Bank of the Philippines against


Cross Defendants Spouses Beder Morandarte and Marina Febrera, for being premature, but
ordering the latter cross defendants to give a substitute security in favor of DBP as indicated in
this decision;

7. Declaring valid and enforceable the Lease Agreement for a period of twenty five years over
the fishpond area of Intervenors;

8. Denying Intervenors' prayer for damages against defendants-spouses Morandarte; and

9. Dismissing, for lack of merit, the counterclaim and prayer for damages of defendants
spouses Morandarte against the Intervenors.

No costs against defendant-spouses Morandarte.


IT IS SO ORDERED.20

Dissatisfied, the Morandarte spouses appealed to the CA.21 In a Decision dated August 23, 1995, the
CA affirmed the decision of the RTC,22 ratiocinating, as follows:

The present controversial Miputak River used to occupy the area adjacent to the northern and
western boundaries of Lot No. 6781 Cad-85 (Exh. J). As time passed, it changed its course
and occupies (sic) Lot No. 6781 Cad-85 (identical to Lot 7, Exh. H). This will explain Beder
Morandarte's argument that when he applied for the Sales Patent Lot 7 (identical to Lot 6781),
the original technical description did not show the Miputak River. But it is inescapable though,
that while originally, Lot 6781 is not occupied by the river, at the time that the Sales Application
was filed by Beder Morandarte, the Miputak River was actually occupying said Lot 6781 or Lot
7 covered by his Sales Application and the titles sought to be annulled in this case.

Rivers and their natural beds are undoubtedly properties of public dominion (Art. 502 par. 1,
Civil Code of the Philippines). Whether navigable or not, rivers belong to the public and cannot
be acquired by prescription (Com vs. Meneses, 38 O.G. 2839, Paras, Civil Code, p. 328, Vol.
II, 12th Edition). In fact, a stream located within private land is still property of public dominion,
even if the Torrens Title of the land does not show the existence of said stream (Talion vs.
Sec. of Public Works and Highways, L-24281, May 16, 1967; Paras, supra).

Correspondingly, Art. 462 of the same Civil Code provides:

Art. 462. Whenever a river, changing its course by natural causes, opens a new bed through a
private estate, this bed shall become of public dominion.

The rule is the same that even if the new bed is on private property. The bed becomes
property of public dominion. Just as the old bed had been of public dominion before the
abandonment, the new riverbed shall likewise be of public dominion (Hilario vs. City of Manila,
L-19570, April 27, 1967).23

On October 10, 1995, the Morandarte spouses filed a motion for reconsideration. 24 In its Resolution
dated January 19, 1996, the CA found no justifiable cause or reason to modify or reverse its
decision.25

Hence, the instant petition for review anchored on the following assigned errors:

A.

RESPONDENT COURT COMMITTED A GRAVE ERROR OF LAW IN APPLYING ARTICLE


462 OF THE CIVIL CODE TO THIS CASE WHEN THE CHANGE IN COURSE OF THE OLD
MIPUTAK RIVER WAS NOT DUE TO NATURAL CAUSES BUT WAS ACCIDENTAL.

B.

ASSUMING ARGUENDO THAT THE CHANGE OF COURSE OF THE OLD MIPUTAK RIVER
WAS DUE TO NATURAL CAUSE ONLY A PORTION OF THE SUBJECT PROPERTY OF
PETITIONERS WAS AFFECTED THEREBY SO THAT THE TITLE OF PETITIONERS TO
THE REMAINING PORTION IS VALID AND CANNOT BE NULLIFIED AS IT REMAINED
PRIVATE PROPERTY.
C.

RESPONDENT COURT GRAVELY ERRED IN ORDERING THE REVERSION OF LOT 7,


CSD-09-05-00078-D TO THE PUBLIC DOMAIN.

D.

RESPONDENT COURT GRAVELY ERRED IN NOT DECLARING AS NULL AND VOID THE
LEASE AGREEMENT EXECUTED IN FAVOR OF INTERVENORS.

E.

RESPONDENT COURT GRAVELY ERRED IN NOT DISMISSING THE COMPLAINT


CONSIDERING THAT NO FRAUD OR MISREPRESENTATION WAS EMPLOYED BY THE
SPOUSES MORANDARTE IN OBTAINING THE TITLE.26

The Morandarte spouses emphatically argue that the CA failed to take into consideration the true
state of the present Miputak River in relation to Lot 7. They contend that the Miputak River changed
its course due to the closure of the river bed through the construction of dikes by the Lacaya spouses,
forcing the river to be diverted into Lot 6781-B. Thus, they submit that the applicable provision is
Article 77 of the Law of Waters, which provides that "[l]ands accidentally inundated by the waters of
lakes, or by creeks, rivers and other streams shall continue to be the property of their respective
owners."

Furthermore, they staunchly claim that the Miputak River does not actually correspond to Lot 7. The
Miputak River occupies only 12,162 square meters of Lot 7 which has an area of 45,499 square
meters. Also, they insist that the lower courts made capital, albeit erroneously, of their agreement to a
reversion. The reversion agreed to refers only to the 12,162 square meters portion covered by the
Miputak River, which should be voided, while the portion unaffected by the Miputak River is valid and
their title thereto should be maintained and respected.

Moreover, they vigorously contend that the CA erred in sustaining the validity of fishpond rights of the
Lacaya spouses. They aver that the Lacaya spouses violated the terms of the lease agreement by
constructing dikes for the fishponds which caused the Miputak River to traverse the property of the
Morandarte spouses.

Prefatorily, it must be stated that in petitions for review on certiorari, only questions of law may be
raised by the parties and passed upon by this Court.27 Factual findings of the trial court, when
adopted and confirmed by the CA, are binding and conclusive upon the Supreme Court and generally
will not be reviewed on appeal.28 Inquiry upon the veracity of the CA's factual findings and conclusion
is not the function of the Supreme Court for the Court is not a trier of facts. 29

While this Court has recognized several exceptions to this rule, to wit: (1) when the findings are
grounded entirely on speculation, surmises, or conjectures; (2) when the inference made is manifestly
mistaken, absurd, or impossible; (3) when there is grave abuse of discretion; (4) when the judgment
is based on a misapprehension of facts; (5) when the findings of facts are conflicting; (6) when in
making its findings, the CA went beyond the issues of the case, or its findings are contrary to the
admissions of both the appellant and the appellee; (7) when the findings are contrary to the trial court;
(8) when the findings are conclusions without citation of specific evidence on which they are based;
(9) when the facts set forth in the petition as well as in the petitioner's main and reply briefs are not
disputed by the respondent; (10) when the findings of fact are premised on the supposed absence of
evidence and contradicted by the evidence on record; and (11) when the CA manifestly overlooked
certain relevant facts not disputed by the parties, which, if properly considered, would justify a
different conclusion,30 none of these exceptions find application here.

A complaint for reversion involves a serious controversy, involving a question of fraud and
misrepresentation committed against the government and it seeks the return of the disputed portion
of the public domain. It seeks to cancel the original certificate of registration, and nullify the original
certificate of title, including the transfer certificate of title of the successors-in-interest because the
same were all procured through fraud and misrepresentation.31

The State, as the party alleging that fraud and misrepresentation attended the application for free
patent, bears the burden of proof. The circumstances evidencing fraud and misrepresentation are as
varied as the people who perpetrate it in each case. It assumes different shapes and forms and may
be committed in as many different ways.32 Therefore, fraud and misrepresentation are never
presumed but must be proved by clear and convincing evidence;33 mere preponderance of evidence
not even being adequate.34

In this case, the State failed to prove that fraud and misrepresentation attended the application for
free patent. The RTC, in fact, recognized that no fraud attended the application for free patent35 but
declared reversion based on the judicial admission of the Morandarte spouses that reversion is
warranted due to the inalienability of the Miputak River. Ordinarily, a judicial admission requires no
proof and a party is precluded from denying it except when it is shown that such admission was made
through palpable mistake or that no such admission was made.36 In this case, the exception finds
application since the records lay bare that such admission was made through mistake and not in the
context it was considered. As reflected in the Order dated May 25, 1998,37 the Morandarte spouses
essentially agreed only to a reconveyance of the portion covering the Miputak River. Undoubtedly,
such acquiescence to return the portion covering the Miputak River is not, and cannot be considered,
an admission that fraud and misrepresentation attended the application for free patent. This fact,
standing alone, does not prove fraud and misrepresentation.

Besides, it is undisputed that the original survey plan submitted by Morandarte to the BOL reflected
the true state of the Miputak River in Lot 1038 but the BOL did not approve the plan because a 1916
survey did not so indicate the existence of a river traversing Lot 1038 such that Morandarte was
directed to submit an amended plan deleting the existence of the Miputak River. This mothered the
subsequent error of the BOL of approving the amended plan as CAS-09-05-000078-D.

This error could have been discovered through a thorough ocular inspection of the property claimed
under the free patent application. However, Aurelio F. Bureros, Hearing Officer I of the BOL,
surprisingly failed to notice the existence of the river traversing Lot 1038 in the field investigation he
conducted on January 10, 1976.38

Neither did Bureros note the 13,339 square meter portion already covered by an existing fishpond
lease agreement granted by the BOF in favor of Felipe B. Lacaya, the predecessor-in-interest of the
Lacaya spouses.39

The records reveal that as early as 1948, 4.6784 hectares40 of the public land have been leased for
fishpond purposes. Aguido S. Realiza was the initial grantee of a fishpond lease agreement. 41 Amor
A. Realiza, Aguido's son, acquired his fishpond permit on May 29, 1953. 42 Amor A. Realiza
transferred his fishpond rights to Felipe B. Lacaya on May 14, 1956.43 By 1960, the public land leased
for fishpond purposes had increased to 5.0335 hectares.44 Felipe B. Lacaya transferred his fishpond
rights to Virgilio B. Lacaya on October 25, 1977.45 Thus, the fishpond rights have been in existence
since 1948, prior to the 1972 free patent application of Morandarte.
Regardless of the foregoing, Aurelio F. Bureros, concluded that Morandarte is a qualified applicant
and recommended that a free patent be granted to him. This error culminated in the erroneous grant
of a free patent on July 27, 1976 covering the Miputak River and land subject of the fishpond rights of
Felipe B. Lacaya.46

Be that as it may, the mistake or error of the officials or agents of the BOL in this regard cannot be
invoked against the government with regard to property of the public domain. It has been said that the
State cannot be estopped by the omission, mistake or error of its officials or agents. 47

It is well-recognized that if a person obtains a title under the Public Land Act which includes, by
oversight, lands which cannot be registered under the Torrens system, or when the Director of Lands
did not have jurisdiction over the same because it is a public domain, the grantee does not, by virtue
of the said certificate of title alone, become the owner of the land or property illegally
included.48 Otherwise stated, property of the public domain is incapable of registration and its
inclusion in a title nullifies that title.49

The present controversy involves a portion of the public domain that was merely erroneously included
in the free patent. A different rule would apply where fraud is convincingly shown. The absence of
clear evidence of fraud will not invalidate the entire title of the Morandarte spouses.

Accordingly, the 12,162-square meter portion traversed by the Miputak River and the 13,339-square
meter portion covered by the fishpond lease agreement of the Lacaya spouses which were
erroneously included in Free Patent No. (IX-8) 785 and Original Certificate of Title No. P-21972
should be reconveyed back to the State.

The Morandarte spouses cannot seek refuge in their claim that Antonio A. Morandarte, their
predecessor-in-interest, was already the owner of that portion of Lot 1038 when the fishpond
application of Aguido S. Realiza was approved in 1948 because Lot 1038 was still part of the public
domain then. It was only in 1972, through Forestry Administrative Order No. 4-1257, which was
approved August 14, 1972, when Lot 1038 was declared alienable or disposable property of the
State.50

It is a settled rule that unless a public land is shown to have been reclassified as alienable or actually
alienated by the State to a private person, that piece of land remains part of the public domain.
Hence, Antonio A. Morandarte's occupation thereof, however long, cannot ripen into private
ownership.51

The Morandarte spouses also unsuccessfully harp on the inapplicability of Article 462 of the Civil
Code by claiming that the change of course of the Miputak River was due to a man-made cause and
not by natural means. They offered no iota of evidence to substantiate this claim, other than the bare
testimony of Beder Morandarte. Neither is there proof that the movement of the river was caused by
accident or calamity, such as a typhoon, and not by the natural movements thereof. General
statements, which are mere conclusions of law and not proofs, are unavailing and cannot suffice.

Besides, at the time of the filing of the application for free patent in 1972, a portion of the Miputak
River was already in its present course, traversing Lot 1038, particularly Lot 7 of the amended plan
submitted by Morandarte.

We need not delve on the question of whether the Lacaya spouses violated the terms of the fishpond
lease agreement. It is not material in this case in the sense that it was not made an issue by the
parties. Neither is there evidence to corroborate the bare allegation of petitioners that the Lacaya
spouses constructed dikes for the fishponds which caused the Miputak River to traverse Lot 7. What
is significant here is the established fact that there was an existing fishpond lease agreement
between Felipe Lacaya and the Bureau of Fisheries at the time of Morandarte's application for free
patent; in effect, proving that the area covering the fishpond belongs to the Government and
petitioners have no rights thereto.

In closing, we cannot but decry the carelessness of the BOL in having issued the Free Patent in
Morandarte's favor which covered the Miputak River and the fishpond rights of Felipe B. Lacaya.
Surely, a more diligent search into their records and thorough ocular inspection of Lot 7 would have
revealed the presence of the Miputak River traversing therein and an existing fishpond right thereon.
Had more vigilance been exercised by the BOL, the government agency entrusted specifically with
the task of administering and disposing of public lands, the present litigation could have been averted.

WHEREFORE, the petition is partly GRANTED. The assailed Decision of the Court of Appeals, dated
August 23, 1995, in CA G.R. No. 36258 is REVERSED insofar only as it affirmed the nullity of Free
Patent No. (IX-8) 785 and Original Certificate of Title No. P-21972, in the name of petitioner Beder
Morandarte. In its stead, petitioners Spouses Beder Morandarte and Marina Febrera are directed to
reconvey to the respondent Republic of the Philippines within thirty (30) days from the finality of this
Decision the 12,162-square meter portion traversed by the Miputak River and the 13,339-square
meter portion covered by the fishpond lease agreement of the Lacaya spouses. No pronouncement
as to costs.

SO ORDERED.

You might also like